MKSAP 17 Rheumatology PDF

Download as pdf or txt
Download as pdf or txt
You are on page 1of 186

®

ACP IM KSAP �
Medical Knowledge Self-Assessment Program ®

Rheumatology

6 A 1c pAmerican College of Physicians


� Leading Internal Medicine, Improving Lives
®
Welcome to the Rheumatology
Section of MKSAP 17!
In these pages, you will find updated information on approaches to the patient with rheumatologic disease, principles of thera­
peutics, rheumatoid arthritis, osteoarthritis, systemic lupus erythematosus, infectious arthritis, idiopathic inflammatory myop­
athies, systemic vasculitis, systemic sclerosis, autoinflammatory diseases, true connective tissue diseases, and other clinical
challenges. All of these topics are uniquely focused on the needs of generalists and subspecialists outside of rheumatology.

The publication of the 17111 edition of Medical Knowledge Self-Assessment Program (MKSAP) represents nearly a half-century
of serving as the gold-standard resource for internal medicine education. It also marks its evolution into an innovative learn­
ing system to better meet the changing educational needs and learning styles of all internists.

The core content of MKSAP has been developed as in previous editions-newly generated, essential information in 11 topic
areas of internal medicine created by dozens of leading generalists and subspecialists and guided by certification and recer­
tification requirements, emerging knowledge in the field, and user feedback. MKSAP 17 also contains 1200 all-new, psycho­
metrically validated, and peer-reviewed multiple-choice questions (MCQs) for self-assessment and study, including 96 in
Rheumatology. MKSAP 17 continues to include High Value Care (HVC) recommendations, based on the concept of balancing
clinical benefit with costs and harms, with links to MCQs that illustrate these principles. In addition, HVC Key Points are
highlighted in the text. Also highlighted, with blue text, are Hospitalist-focused content and MCQs that directly address the
learning needs of internists who work in the hospital setting.

MKSAP 17 Digital provides access to additional tools allowing you to customize your learning experience, including regular text
updates with practice-changing, new information and 200 new self-assessment questions; a board-style pretest to help direct your
learning; and enhanced custom-quiz options. And, with MKSAP Complete, learners can access 1200 electronic flashcards for quick
review of important concepts or review the updated and enhanced version of Virtual Dx, an image-based self-assessment tool.

As before, MKSAP 17 is optimized for use on your mobile devices, with iOS- and Android-based apps allowing you to sync
your work between your apps and online account and submit for CME credits and MOC points online.

Please visit us at the MKSAP Resource Site (mksap.acponline.org) to find out how we can help you study, earn CME credit
and MOC points, and stay up to date.

Whether you prefer to use the traditional print version or take advantage of the features available through the digital version,
we hope you enjoy MKSAP 17 and that it meets and exceeds your personal learning needs.

On behalf of the many internists who have offered their time and expertise to create the content for MKSAP 17 and the editmial staff
who work to bring this mate1ial to you in the best possible way, we are honored that you have chosen to use MKSAP 17 and appreci­
ate any feedback about the program you may have. Please feel free to send us any comments to [email protected].

Sincerely,

Philip A. Masters, MD, FACP


Editor-in-Chief
Senior Physician Educator
Director, Content Development
Medical Education Division
American College of Physicians

ii
Rheumatology

Committee Vikas Majithia, MD, MPH, FACP 2


Professor of Medicine
Michael H. Pillinger, MD, Section Editor 2
Chief, Division of Rheumatology
Professor of Medicine, Biochemistry and Molecular
Department of Medicine
Pharmacology
University of Mississippi Medical Center
Department of Medicine
Jackson, Mississippi
NYU School of Medicine
Section Chief, Rheumatology
VA New York Harbor Health Care System, New York Editor-in-Chief
Campus Philip A. Masters, MD, FACP 1
New York, New York Director, Clinical Content Development
Virginia U. Collier, MD, MACP, Associate Editor 2 American College of Physicians
Hugh R. Sharp, Jr. Chair of Medicine Philadelphia, Pennsylvania
Christiana Care Health System
Newark, Delaware
Director, Clinical Program Development
Professor of Medicine
Sidney Kimmel Medical College at Thomas Jefferson Cynthia D. Smith, MD, FACP 2
University American College of Physicians
Philadelphia, Pennsylvania Philadelphia, Pennsylvania

Daria B. Crittenden, MD 2
Adjunct Assistant Professor of Medicine Rheumatology Reviewers
New York University School of Medicine Stewart F. Babbott, MD, FACP1
Clinical Research Senior Medical Scientist Pieter A. Cohen, MD2
Bone Therapeutic Area Lia S. Logio, MD, FACP2
Amgen Inc. George F. Moxley, MD2
Thousand Oaks, California
Gregory C. Gardner, MD, FACP 1 Rheumatology ACP Editorial Staff
Gilliland-Henderson Professor of Medicine Megan Zborowski 1, Senior Staff Editor
Fellowship Program Director Margaret Wells 1 , Director, Self-Assessment and Educational
Division of Rheumatology Programs
University of Washington Becky Krumm', Managing Editor
Seattle, Washington
Sharon L. Kolasinski, MD, FACP 2 ACP Principal Staff
Professor of Clinical Medicine
Patrick C. Alguire, MD, FACP 2
Division of Rheumatology
Senior Vice President, Medical Education
Perelman School of Medicine at the University of
Pennsylvania Sean McKinney•
Philadelphia, Pennsylvania Vice President, Medical Education
Margaret Wells 1
Bonita S. Libman, MD, FACP 2 Director, Self-Assessment and Educational Programs
Professor of Medicine, Director of the Rheumatology
Fellowship Training Program Becky Krumm 1
Division of Rheumatology and Clinical Immunology Managing Editor
The University of Vermont Medical Center Valerie A. Dangovetsky 1
Burlington, Vermont Administrator

iii
Ellen McDonald, PhD' Daria B. Crittenden, MD
Senior Staff Editor Research Grants/Contracts
Savient Pharmaceuticals
Katie Idell'
Employment - Clinical Research Senior Medical Scientist
Digital Content Associate/Editor
Amgen Inc.
Megan Zborowski 1
Sharon L. Kolasinski, MD, FACP
Senior Staff Editor
Honoraria
Randy Hendrickson 1 Curatio CME, Georgetown University Rheumatology
Production Administrator/Editor Division, American College of Rheumatology, Rush
University Medical Center, New York University, Congress
Linnea Donnarumma1
of Clinical Rheumatology, American College of Physicians
Staff Editor
Delaware Chapter
Susan Galeone1 Consultantship
Staff Editor Vindico Medical Education
Research Grants/Contracts
Jackie Twomey'
Human Genome Sciences, UCB, Bristol-Myers Squibb,
Staff Editor
Amgen, Abbott
Kimberly Kerns'
Bonita S. Libman, MD, FACP
Administrative Coordinator
Research Grants/Contracts
Human Genome Sciences, GlaxoSmithKline
1. Has no relationships with any entity producing, marketing, reselling, or
distributing health care goods or services consumed by, or used on, patients.
Lia S. Logio, MD, FACP
2. Has disclosed relationship{s) with any entity producing, marketing, reselling, or Royalties
distributing health care goods or services consumed by, or used on, patients.
McGraw Hill

Disclosure of Relationships with any entity producing, Vikas Majithia, MD, MPH, FACP
marketing, reselling, or distributing health care goods Speakers Bureau
or services consumed by, or used on, patients: GlaxoSmithKline
Research Grants/Contracts
Patrick C. Alguire, MD, FACP GlaxoSmithKline
Board Member
Teva Pharmaceuticals George F. Moxley, MD
Consultantship Employment
National Board of Medical Examiners Virginia Commonwealth University
Royalties Michael Pillinger, MD
UpToDate Research Grants/Contracts
Stock Options/Holdings Takeda Incorporated, Savient Pharmaceuticals
Amgen Inc, Bristol-Myers Squibb, GlaxoSmithKline, Consultant
Covidien, Stryker Corporation, Zimmer Orthopedics, AstraZeneca, Crealta
Teva Pharmaceuticals, Express Scripts, Medtronic
Cynthia D. Smith, MD, FACP
Pieter A. Cohen, MD Stock Options/Holdings
Stock Options/Holdings (spouse) Merck and Co.; spousal employment at Merck
Bio Reference Labs, Idexx Laboratories, Johnson &
Johnson, Mettler Toledo International Inc., Stryker Acknowledgments
Corp., Biota Pharmaceuticals, Pfizer, ResMed Inc.,
Vertex Pharmaceuticals The American College of Physicians (ACP) gratefully
Honoraria acknowledges the special contributions to the develop­
Consumer Union, Wall Street Journal ment and production of the 17th edition of the Medical
Knowledge Self-Assessment Program" (MKSAP"' 17) made
Virginia U. Collier, MD, MACP by the following people:
Stock Options/Holdings
Celgene, Pfizer, Merck, Abbott, Abbevie, Johnson and Graphic Design: Michael Ripca (Graphics Technical
Johnson, Medtronic, McKesson, Amgen Inc., Wellpoint, Administrator) and WFGD Studio (Graphic Designers).
Roche, Sanofi, Novartis, Covidien, Stryker, Amerisource Production/Systems: Dan Hoffmann (Director, Web
Bergen, Schering Plough Services & Systems Development), Neil Kohl (Senior

iv
Architect), Chris Patterson (Senior Architect), and Scott Program, visit the MKSAP Resource Site at mksap.
Hurd (Manager, Web Projects & CMS Services). acponline.org.
MKSAP 17 Digital: Under the direction of Steven Spadt,
Vice President, Digital Products & Services, the digital The Royal Australasian College
version of MKSAP 17 was developed within the ACP's of Physicians CPD Program
Digital Product Development Department, led by Brian
In Australia, MKSAP 17 is a Category 3 program that may
Sweigard (Director). Other members of the team included
be used by Fellows ofThe Royal Australasian College
Dan Barron (Senior Web Application Developer/ Architect),
of Physicians (RACP) to meet mandatory Continuing
Chris Forrest (Senior Software Developer/Design Lead),
Professional Development (CPD) points.Two CPD cred­
Kara Kronenwetter (Senior Web Developer), Brad Lord
its are awarded for each of the 200 AMA PRA Category 1
(Senior Web Application Developer), John McKnight
CreditsTM available in MKSAP 17. More information about
(Senior Web Developer), and Nate Pershall (Senior Web
using MKSAP 17 for this purpose is available at the MKSAP
Developer).
Resource Site at mksap.acponline.org and at www.racp.
The College also wishes to acknowledge that many other edu.au. CPD credits earned through MKSAP 17 should be
persons, too numerous to mention, have contributed to reported at the MyCPD site at www.racp.edu.au/mycpd.
the production of this program. Without their dedicated
f
eforts, this program would not have been possible.
Continuing Medical Education
The American College of Physicians (ACP) is accredited
MKSAP Resource Site by the Accreditation Council for Continuing Medical
(mksap.acponline.org) Education (ACCME) to provide continuing medical educa­
The MKSAP Resource Site (mksap.acponline.org) is a tion for physicians.
continually updated site that provides links to MKSAP The ACP designates this enduring material, MKSAP 17,
17 online answer sheets for print subscribers; the lat- for a maximum of 200 AMA PRA Category 1 CreditsrM .
est details on Continuing Medical Education (CME) and Physicians should claim only the credit commensurate
Maintenance of Certification (MOC) in the United States, with the extent of their participation in the activity.
Canada, and Australia; errata; and other new information.
Up to 16 AMA PRA Category 1 Credits1M are available
from July 31, 2015, to July 31, 2018, for the MKSAP 17
ABIM Maintenance of Certification Rheumatology section.
Check the MKSAP Resource Site (mksap.acponline.org)
for the latest information on how MKSAP tests can be
Learning Objectives
used to apply to the American Board of Internal Medicine
The learning objectives of MKSAP 17 are to:
for Maintenance of Certification (MOC) points.
• Close gaps between actual care in your practice and
preferred standards of care, based on best evidence
• Diagnose disease states that are less common and
Royal College Maintenance
sometimes overlooked or confusing
of Certification
• Improve management of comorbid conditions that can
In Canada, MKSAP 17 is an Accredited Self-Assessment complicate patient care
Program (Section 3) as defined by the Maintenance of • Determine when to reter patients for surgery or care by
Certification (MOC) Program of The Royal College of subspecialists
Physicians and Surgeons of Canada and approved by the • Pass the ABIM Certification Examination
Canadian Society oflntemal Medicine on December 9, 2014. • Pass the ABIM Maintenance of Certification Examination
Approval extends from July 31, 2015 until July 31, 2018 for
the Part A sections. Approval extends from December 31,
2015 to December 31, 2018 for the Part B sections. Target Audience
Fellows of the Royal College may earn three credits per • General internists and primary care physicians
hour for participating in MKSAP 17 under Section 3. • Subspecialists who need to remain up-to-date in internal
MKSAP 17 also meets multiple CanMEDS Roles, includ- medicine and in areas outside of their own subspecialty area
ing that of Medical Expert, Communicator, Collaborator, • Residents preparing for the certification examination in
Manager, Health Advocate, Scholar, and Professional. internal medicine
For information on how to apply MKSAP 17 Continuing • Physicians preparing for maintenance of certification in
Medical Education (CME) credits to the Royal College MOC internal medicine (recertification)

V
Earn "Instantaneous" CME therapeutic options and are required to identify any unap­
Credits Online proved, off-label, or investigative use of commercial prod­
Print subscribers can enter their answers online to earn ucts or devices. Where a trade name is used, all available
instantaneous Continuing Medical Education (CME) cred­ trade names for the same product type are also included.
its. You can submit your answers using online answer lf trade-name products manufactured by companies with
sheets that are provided at mksap.acponline.org, where a whom contributors have relationships are discussed, con­
record of your MKSAP 17 credits will be available. To earn tributors are asked to provide evidence-based citations
CME credits, you need to answer all of the questions in in support of the discussion. The information is reviewed
a test and earn a score of at least 50% correct (number of by the committee responsible for producing this text. If
correct answers divided by the total number of questions). necessary, adjustments to topics or contributors' roles in
Take any of the following approaches: content development are made to balance the discussion.
Further, all readers of this text are asked to evaluate the
l. Use the printed answer sheet at the back of this book to content for evidence of commercial bias and send any rel­
record your answers. Go to mksap.acponline.org, access evant comments to [email protected] so that
the appropriate online answer sheet, transcribe your future decisions about content and contributors can be
answers, and submit your test for instantaneous CME made in light of this information.
credits. There is no additional fee for this service.
2. Go to mksap.acponline.org, access the appropriate Resolution of Conflicts
online answer sheet, directly enter your answers, and
To resolve all conflicts of interest and influences of vested
submit your test for instantaneous CME credits. There is
interests, the American College of Physicians (ACP) pre­
no additional fee for this service.
cluded members of the content-creation committee from
3. Pay a $15 processing fee per answer sheet and submit deciding on any content issues that involved generic or
the printed answer sheet at the back of this book by trade-name products associated with proprietary entities
mail or fax, as instructed on the answer sheet. Make with which these committee members had relationships.
sure you calculate your score and fax the answer sheet In addition, content was based on best evidence and
to 215-351-2799 or mail the answer sheet to Member updated clinical care guidelines, when such evidence and
and Customer Service, American College of Physicians, guidelines were available. Contributors' disclosure infor­
190 N. Independence Mall West, Philadelphia, PA mation can be found with the list of contributors' names
19106-1572, using the courtesy envelope provided in and those of ACP principal staff listed in the beginning of
your MKSAP 17 slipcase. You will need your 10-digit this book.
order number and 8-digit ACP ID number, which are
printed on your packing slip. Please allow 4 to 6 weeks
for your score report to be emailed back to you. Be sure Hospital-Based Medicine
to include your email address for a response. For the convenience of subscribers who provide care in
If you do not have a 10-digit order number and 8-digit hospital settings, content that is specific to the hospital
ACP ID number or if you need help creating a user name setting has been highlighted in blue. Hospital icons (Cl)
and password to access the MKSAP 17 online answer highlight where the hospital-based content begins.
sheets, go to mksap.acponline.org or email custserv@ continues over more than one page, and ends.
acponline.org.

High Value Care Key Points


Disclosure Policy Key Points in the text that relate to High Value Care
It is the policy of the American College of Physicians concepts (that is, concepts that discuss balancing clinical
(ACP) to ensure balance, independence, objectivity, and benefit with costs and harms) are designated by the
scientific rigor in all of its educational activities. To this HVC icon (HVC).
end, and consistent with the policies of the ACP and the
Accreditation Council for Continuing Medical Education
(ACCME), contributors to all ACP continuing medical Educational Disclaimer
education activities are required to disclose all relevant The editors and publisher of MKSAP 17 recognize that the
financial relationships with any entity producing, mar­ development of new material offers many opportunities
keting, reselling, or distributing health care goods or for error. Despite our best efforts, some errors may persist
services consumed by, or used on, patients. Contributors in print. Drug dosage schedules are, we believe, accurate
are required to use generic names in the discussion of and in accordance with current standards. Readers are

vi
advised, however, to ensure that the recommended dos­ earning Continuing Medical Education (CME) credits and
ages in MKSAP 17 concur with the information provided Maintenance of Certification (MOC) points/credits and for
in the product information material. This is especially other authorized uses of MKSAP 17.
important in cases of new, infrequently used, or highly
toxic drugs. Application of the information in MKSAP 17
remains the professional responsibility of the practitioner.
Unauthorized Use of This Book
The primary purpose of MKSAP 17 is educational. Is Against the Law
Information presented, as well as publications, technol­ Unauthorized reproduction of this publication is unlaw­
ogies, products, and/or services discussed, is intended to ful. The American College of Physicians (ACP) prohibits
inform subscribers about the knowledge, techniques, and reproduction of this publication or any of its parts in any
experiences of the contributors. A diversity of professional form either for individual use or for distribution.
opinion exists, and the views of the contributors are their
own and not those of the American College of Physicians The ACP will consider granting an individual permission
(ACP). Inclusion of any material in the program does not to reproduce only limited portions of this publication for
constitute endorsement or recommendation by the ACP. his or her own exclusive use. Send requests in writing to
The ACP does not warrant the safety, reliability, accuracy, MKSAP" Permissions, American College of Physicians, 190
completeness, or usefulness of and disclaims any and all N. Independence Mall West, Philadelphia, PA 19106-1572,
liability for damages and claims that may result from the or email your request to [email protected].
use of information, publications, technologies, products, MKSAP 17 lSBN: 978-1-938245-18-3
and/or services discussed in this program. (Rheumatology) ISBN: 978-1-938245-24-4
Printed in the United States of America.
Publisher's Information For order information in the United States or Canada
Copyright© 2015 American College of Physicians. All call 800-523-1546, extension 2600. All other countries
rights reserved. call 215-351-2600, (M-F, 9 AM - 5 PM ET). Fax inquiries
to 215-351-2799 or email to [email protected].
This publication is protected by copyright. No part of
this publication may be reproduced, stored in a retrieval
system, or transmitted in any form or by any means, elec­
tronic or mechanical, including photocopy, without the Errata
express consent of the American College of Physicians. Errata for MKSAP 17 will be available through the MKSAP
MKSAP 17 is for individual use only. Only one account Resource Site at mksap.acponline.org as new information
per subscription will be permitted for the purpose of becomes known to the editors.

vii
Table of Contents

Approach to the Patient with Rheumatologic Disease Rheumatologic Medications and Pregnancy ......... 18
Introduction..................................... 1 Nonphannacologic and Nontraditional Management ...18
Inflammatory Versus Noninflammatory Pain ......... 1 Physical and Occupational Therapy ............ 18
The Musculoskeletal Examination .................. 1 Complementary and Alternative Medicine ...... 18
Arthritis ........................................ 2 Role of Surgery ............................. 18
Monoarticular Arthritis ....................... 2
Oligoarthritis ................................ 2 Rheumatoid Arthritis
Poiyarthritis ................................. 2 Introduction.................................... 20
Extra-Articular Manifestations Epidemiology................................... 20
of Rheumatologic Disease ......................... 2 Pathophysiology and Risk Factors.................. 20
Constitutional Symptoms...................... 2 Genetic Factors ............................. 20
Dermatologic Manifestations ................... 2 Autoantibodies ............................. 20
Inflammatory Eye Disease ..................... 2 Environmental Factors .......................20
Internal Organ Involvement.................... 3 Infection................................... 20
Rheumatologic and Musculoskeletal Manifestations Hormones ................................. 20
in Systemic Disease............................... 3
Diagnosis ...................................... 21
Laboratory Studies ............................... 3
Clinical Manifestations ....................... 21
Tests That Measure Inflammation ............... 4
Laboratory Studies .......................... 22
Autoantibody Tests ........................... 6
Imaging Studies............................. 23
Imaging Studies.................................. 7
Complications and Extra-A11icular Manifestations.... 24
Radiography................................. 7
Joints...................................... 24
CT ......................................... 7
Skin....................................... 24
MRI ........................................ 7
Eyes....................................... 24
Ultrasonography ............................. 7
Pulmonary Involvement...................... 24
Joint Aspiration .................................. 9
Cardiac Involvement ......................... 24
Other Complications......................... 24
Principles of Therapeutics Management ................................... 25
Introduction..................................... 9 General Considerations....................... 25
Anti-Inflammatory Agents........................ 10 Disease-Modifying Antirheumatic Drugs........ 27
NSAIDs .................................... 10 Glucocorticoids ............................. 27
Glucocm1icoids ............................. 10 NSAIDs .................................... 27
Colchicine ................................. 10 Surgical Therapy ............................ 27
Analgesics ..................................... 11 Pregnancy and Rheumatoid Arthritis............... 28
Disease-Modifying Antirheumatic Drugs............ 11
Nonbiologic Disease-Modifying
Antirheumatic Drugs ........................ 11 Osteoarthritis
Biologic Disease-Modifying Introduction.................................... 28
Antirheumatic Drugs ........................ 13 Epidemiology................................... 28
Vaccination and Screening in Immunosuppression ... 16 Pathophysiology ................................ 28
Urate-Lowering Therapy ......................... 17 Risk Factors .................................... 29
Allopurinol................................. 17 Classification ................................... 29
Febuxostat ................................. 17 Primary Osteoarthritis ....................... 29
Probenecid ................................. 17 Secondary Osteoarthritis ..................... 29
f
Pegloticase ................................. 17 Dif use Idiopathic Skeletal Hyperostosis ........ 29

ix
Diagnosis ...................................... 30 Hematologic Involvement.....................46
Clinical Manifestations ....................... 30 Gastrointestinal Involvement .................. 46
Laboratory and Imaging Studies ............... 31 Malignancy ................................. 46
Differential Diagnosis ........................ 31 Diagnosis ...................................... 46
Management ................................... 32 Laboratory Studies .......................... 47
Nonpharmacologic Therapy ................... 32 Differential Diagnosis ........................ 48
Pharmacologic Therapy ...................... 32 Management ................................... 49
Intra-articular Injection ...................... 33 Pregnancy and Childbirth Issues ................... 50
Surgical Therapy ............................ 33 Prognosis ...................................... 50

Fibromyalgia Sjogren Syndrome


Introduction.................................... 33 Introduction....................................50
Epidemiology................................... 33 Pathophysiology ................................ 50
Pathophysiology ................................ 34 Epidemiology................................... 51
Diagnosis ...................................... 34 Clinical Manifestations ........................... 51
Management ................................... 34 Diagnosis ...................................... 51
Management ................................... 51
Spondyloarthritis Prognosis ...................................... 52
Introduction.................................... 35
Pathophysiology ................................ 35 Mixed Connective Tissue Disease
Genetic Factors ............................. 35 Introduction.................................... 52
Environmental Factors ....................... 35 Epidemiology ................................... 52
Classification ................................... 35 Clinical Manifestations
Ankylosing Spondylitis ....................... 35 and Diagnosis .................................. 52
Psoriatic Arthritis ........................... 37 Management ................................... 52
Inflammatory Bowel Disease-Associated Prognosis ...................................... 53
Arthritis ................................... 37
Reactive Arthritis ............................ 38 Crystal Arthropathies
Diagnosis ...................................... 38 Introduction.................................... 53
Laboratory Studies .......................... 39 Gout .......................................... 53
Imaging Studies ............................. 39 Epidemiology ............................... 53
Management ................................... 41 Pathophysiology ............................ 53
General Considerations....................... 41 Clinical Manifestations ....................... 54
Ankylosing Spondylitis ....................... 41 Diagnosis .................................. 55
Psoriatic Arthritis ........................... 42 Management ............................... 55
Inflammatory Bowel Disease-Associated Calcium Pyrophosphate Deposition ................ 57
Arthritis ................................... 42 Pathophysiology ............................ 57
Reactive Arthritis............................ 42 Clinical Manifestations ....................... 57
Management ............................... 58
Systemic Lupus Erythematosus Basic Calcium Phosphate Deposition ............... 58
Introduction.................................... 42
Pathophysiology ................................ 42 Infectious Arthritis
Epidemiology................................... 43 Introduction.................................... 59
Clinical Manifestations ........................... 43 Pathophysiology ................................ 59
Mucocutaneous Involvement .................. 43 Diagnosis ...................................... 59
Musculoskeletal Involvement.................. 44 Clinical Manifestations ....................... 59
Kidney Involvement ......................... 44 Laboratory Studies and Imaging ............... 60
Neuropsychiatric Involvement. ................ 44 Causes ......................................... 60
Cardiovascular Involvement. .................. 45 Infection with Gram-Positive Organisms ........60
Pulmonary Involvement...................... 45 Infection with Gram-Negative Organisms .......60

X
Lyme Arthritis .............................. 61 Kawasaki Disease ........................... 73
Mycobacterial Infections...................... 61 Small-Vessel Vasculitis ........................... 73
Fungal Infections............................ 61 ANCA-Associated Vasculitis................... 73
Viral Infections ............................. 61 Immune Complex-Mediated Vasculitis ......... 75
Prosthetic Joint Infections .................... 62
Infections in Previously Damaged Joints ........ 62 Systemic Sclerosis
Management ................................... 63 Introduction.................................... 77
Pharmacologic Therapy ...................... 63 Pathophysiology ................................ 78
Surgical Therapy ............................ 64 Epidemiology................................... 78
Classification ................................... 78
Idiopathic Inflammatory Myopathies Diagnosis ...................................... 78
Introduction.................................... 64 Clinical Manifestations and Management ........... 78
Pathophysiology ................................ 64 Cutaneous Involvement ...................... 78
Epidemiology................................... 64 Musculoskeletal lnvolvement.................. 80
Clinical Manifestations ........................... 64 Vascular Involvement ........................ 80
Muscular Involvement ....................... 64 Gastrointestinal Involvement .................. 81
Cutaneous Involvement ......................64 Kidney Involvement ......................... 81
Cardiopulmonary Involvement ................ 65 Pulmonary Involvement...................... 81
Gastrointestinal Involvement..................66 Cardiac Involvement ......................... 82
Association with Malignancy...................... 66 Pregnancy and Systemic Sclerosis.................. 82
Diagnosis ...................................... 66
Muscle-Related Enzymes ..................... 66 Other Rheumatologic Diseases
Autoantibodies ............................. 66 Behc;et Syndrome ............................... 83
Imaging Studies............................. 67 Relapsing Polychondritis ......................... 83
Electromyography ........................... 67 Adult-Onset Still Disease ......................... 84
Muscle Biopsy .............................. 67 Autoinflammatory Diseases....................... 85
Differential Diagnosis ............................ 68 Sarcoidosis ..................................... 86
Management ................................... 68 Trne Connective Tissue Diseases................... 86
Prognosis ...................................... 69 Ehlers-Danlos Syndrome ..................... 86
Marfan Syndrome ........................... 86
Systemic Vasculitis Osteogenesis Imperfecta...................... 87
Introduction.................................... 69 IgG4-Related Disease ............................ 87
Large-Vessel Vasculitis ........................... 69
Giant Cell Arteritis .......................... 70 Bibliography .................................. 88
Polymyalgia Rheumatica ..................... 71
Takayasu Arteritis ........................... 71
Self-Assessment Test. ........................... 91
Medium-Vessel Vasculitis......................... 72
Polyarteritis Nodosa ......................... 72
Primary Angiitis of the Central Nervous System .. 72 Index ........................................ 163

xi
Rheumatology High Value Care
Recommendations

The American College of Physicians, in collaboration with • CT is useful in assessing bony abnormalities but is more
multiple other organizations, is engaged in a worldwide expensive than plain radiography and exposes the patient
initiative to promote the practice of High Value Care to more radiation.
(HVC). The goals of the HVC initiative are to improve • MRI is useful in detecting soft-tissue abnormalities,
health care outcomes by providing care of proven benefit inflammation, and fluid collections.
and reducing costs by avoiding unnecessary and even • Musculoskeletal ultrasonography is a low risk and
harmful interventions. The initiative comprises several relatively inexpensive way to detect soft-tissue abnor­
programs that integrate the important concept of health malities such as synovitis, tendinitis, bursitis, and joint
care value (balancing clinical benefit with costs and fluid.
harms) for a given intervention into a broad range of • Topical NSAIDs provide similar pain relief for inflamma­
educational materials to address the needs of trainees, tory conditions as oral medications with fewer gastro­
practicing physicians, and patients. intestinal effects and are preferred for patients 75 years
or older; however, they are associated with more skin
HVC content has been integrated into MKSAP 17 in several
reactions and are significantly more expensive than oral
important ways. MKSAP 17 now includes HVC-identified
NSAIDs.
key points in the text, HVC-focused multiple choice
• Rheumatoid factor, anti-cyclic citrullinated peptide anti­
questions, and, for subscribers to MKSAP Digital, an HVC
bodies, and inflammatory markers assist in confirming
custom quiz. From the text and questions, we have gen­
a diagnosis of rheumatoid arthritis; however, serologies
erated the following Ii.st of HVC recommendations that
should never be used as the sole criterion for diagnosis
meet the definition below of high value care and bring us
and should be avoided in patients with low pretest prob­
closer to our goal of improving patient outcomes while
ability for disease due to the high rate of false-positive
conserving finite resources.
results.
High Value Care Recommendation: A recommendation to • Plain radiography of the hands, wrists, and/or feet are
choose diagnostic and management strategies for patients indicated to aid in the diagnosis and to follow progres­
in specific clinical situations that balance clinical benefit sion of rheumatoid arthritis; in contrast, MRI of periph­
with cost and harms with the goal of improving patient eral joints should not be routinely performed to monitor
outcomes. disease progression.
• Methotrexate is the initial treatment of choice for patients
Below are the High Value Care Recommendations for the
with new-onset. rapidly progressive, or erosive rheuma­
Rheumatology section of MKSAP 17.
toid arthritis (see Item 54).
• A physical examination is essential when diagnosing • Leflunomide-induced liver chemistry elevation is com­
musculoskeletal pain and can help to avoid unnecessary mon (seen in up to 20% of patients taking the medica­
laboratory and radiographic testing. tion) and is usually reversible with dose reduction or
• The most appropriate and cost-effective means of assess­ drug discontinuation; thus, further evaluation, including
ing the cause of acute monoarthritis is by aspiration and liver biopsy, is not necessary (see Item 75).
analysis of the synovial fluid for leukocytes, Gram stain • Laboratory studies are indicated only when needed to
with culture, and crystals. rnle out other diagnoses in patients with primary osteo­
• Laboratory studies such as erythrocyte sedimentation arthritis (OA); the diagnostic role for MRI and ultraso­
rate, rheumatoid factor. and antinuclear antibodies have nography in OA has not been established.
low specificity for diagnosing rheumatologic disease in • In patients with suspected osteoarthritis, confirmatory
patients with low pretest probability, thus limiting their plain radiographs with standing views are appropriate to
utility in this population. solidify the diagnosis and rule out less common findings
• The dose of glucocorticoid therapy should not be such as avascular necrosis, fractures, and malignancies
prolonged or increased in patients with polymyalgia (see Item 37).
rheumatica who have clinically improved based on an • Additional testing such as autoantibody measurements
elevated erythrocyte sedimentation rate, as this is a non­ or radiography is unnecessary in patients with clinically
specific test (see Item 94). diagnosed hand osteoarthritis (see Item 43).

xiii
• In patients with osteoarthritis, initial treatment with • Patients with ankylosing spondylitis who are responding
acetaminophen for pain control is generally recom­ well to treatment should be monitored clinically and do
mended (see Item 72). not require periodic imaging studies less than every
• An NSAID should be initiated in patients with osteoar­ 2 years unless absolutely necessary (see Item 33).
thritis if first-line therapy with acetaminophen does not • In patients with strongly suspected spondyloarthritis,
provide adequate relief (see Item 89). MRI of the sacroiliac joints and/or spine should only be
• Fibromyalgia is a clinical diagnosis characterized by considered if conventional radiographs are negative.
chronic widespread pain, tenderness of the skin and • Age-appropriate cancer screening is recommended for
muscles to pressure, fatigue, sleep disturbance, and exer­ patients with dermatomyositis or polymyositis, with
cise intolerance. consideration of additional testing for ovarian cancer;
• Initial laboratory evaluation of fibromyalgia includes a additional CT or PET scanning to look for underlying
complete blood count, chemistry panel, thyroid-stim­ malignancy is not cost effective unless the patient has
ulating hormone, and erythrocyte sedimentation rate additional risk factors.
or C-reactive protein; routine testing for antinuclear • Immediate treatment with prednisone, 60 mg/d (or 1
antibodies, rheumatoid factor, anti-cyclic citrullinated mg/kg/d), is indicated for patients with suspected giant
peptide antibodies, or muscle enzymes should be cell arteritis to prevent visual complications (see Item 17).
avoided. • Sarcoidosis can manifest as Lofgren syndrome, which is
• Nonpharmacologic therapy, including regular aerobic characterized by acute arthritis, bilateral hilar lymphade­
exercise, is the cornerstone of fibromyalgia treatment nopathy, and erythema nodosum; when all three occur
and should be initiated in all affected patients (see together, there is a 95% specificity for the diagnosis, and
Item 67). further diagnostic tests are unnecessary.
• Conventional radiography of the spine and sacroiliac • Primary Raynaud phenomenon is common and carries
joints is generally adequate to demonstrate synovitis, a low risk for progression; thus, a serologic evaluation
axial erosion, or new bone formation in patients with for underlying connective tissue disease is low yield and
spondyloarthritis; CT should be reserved for identifying not cost effective unless severe and prolonged vasospas­
occult spine fractures and bony erosions in patients at tic episodes, asymmetric involvement of the digits, and
high risk due to expense and higher level of radiation abnormal nailfold capillary examination or digital pitting
exposure. are present (see Item 73).

xiv
Rheumatology

Approach to the Patient KEY POINT

• Inflammatory pain involves redness, swelling, warmth,


with Rheumatologic and tenderness; prolonged morning stiffness and con­
Disease stitutional symptoms may also occur.

Introduction
In the United States, the estimated prevalence of chronic
arthritis among adults is 33%; older persons (>65 years of age)
The Musculoskeletal Examination
have a higher prevalence than younger persons (<44 years A physical examination is essential when diagnosing muscu­
of age). loskeletal pain and can help to avoid unnecessary laboratory
and radiographic testing. Pain may be articular (from the
joint), periarticular (from soft-tissue structures around the
joint such as tendons, bursae, or muscles), or referred (from
Inflammatory Versus structures proximal or distal to the joint, or neurogenic in ori­
Noninflammatory Pain gin). Inspection, palpation, and range of motion are generally
In patients with musculoskeletal pain, it is critical to distin­ sufficient to establish whether there is an articular or a periar­
guish between inflammatory and noninflammatory pain, ticular condition. Peripheral joints are usually straightforward
which typically have different causes. Inflammatory pain to examine, although adiposity or edema can make inspection
involves classic signs and symptoms of redness, swelling, and palpation difficult. Axial joints such as the spine, shoul­
warmth, and tenderness. Inflammation is often associated ders, and hips are more difficult to assess for warmth and
with prolonged morning stiffness and, potentially, constitu­ swelling; tenderness, range of motion, and special maneuvers
tional symptoms. Although noninflammatory pain may also are often required to adequately assess these areas.
manifest with tenderness, other signs of inflammation are Differentiating between active (patient moves the joint
generally mild or absent. Table I compares the features of using his/her own power) and passive (examiner moves the
inflammatory and noninflammatory pain. joint for the patient) range of motion is important because

TABLE 1. Features of Inflammatory Versus Noninflammatory Pain


Feature Inflammatory Pain Noninflammatory Pain

Physical examination findings Erythema; warmth; soft-tissue swelling No soft-tissue swelling; minimal or no warmth;
bony enlargement and joint effusions may
occur in osteoarthritis

Morning stiffness >60 min <30 min

Constitutional symptoms Fever; fatigue; malaise Generally absent

Synovial fluid Leukocyte count >2000/µL (2.0 x 10 /L),


9
Leukocyte count between 200/µL and 2000/µL
predominantly neutrophils in acute (0.2 x 109 /L and 2.0 x 109 /L), predominantly
inflammation and monocytes in chronic monocytes
inflammation

Other laboratory findings Elevated inflammatory markers (ESR, CRP); Inflammatory markers usually normal or
anemia of chronic disease minimally elevated
Arthritis imaging studies Symmetric/diffuse joint-space narrowing; Asymmetric/compartmental joint-space
periarticular osteopenia; erosions; bony narrowing; osteophytes; subchondral
proliferation in secondary osteoarthritis or sclerosis; limited or no synovitis on MRI
spondyloarthritis; synovitis on MRI or or ultrasound
ultrasound

CRP = (-reactive protein; ESR = erythrocyte sedimentation rate.

1
Approach to the Patient with Rheumatologic Disease

pain with both passive and active range of motion implies an noninllammatory oligoarthritis. particularly if the involved
intrinsic joint condition, whereas pain with only active range joints sh,1re a history or trauma or overuse.
of motion may be due to a periarticular condition.
See Musculoskeletal Pain in MKSAP 17 General Internal Polyarthritis
Medicine for more information. Polyarthrit is in\'olves fh·e or more joints. The 111ost common
causes olchronic inlla111111atory poly,1nhrilis include rheuma­
KEY POINTS
toid arthritis. systemic lupus erythematosus (SLl:l. and psori­
H\IC • .,\ physiGtl ex,1111ination is essential when diagnosing
atic arthritis (see Table 2). In contrast. acute onset of
111usculoskeletal pain and can help to avoid unnecessary
inllammatory polyarthritis is more commonly caused by viral
laboratory ancl radiographic testing.
infections such as hepatitis. parvovirus. rubella. herpes. HIV.
• Pain with both passive and active range of motion aclenovirus. mumps. or enternvirus. Polyarthritis may also be
implies an intrinsic joint condition, whereas pain with caused by a drug-induced serum sickness reaction. an i111111une
only active range of motion may be due to a periarticu­ complc.\ reaction to hacteri,11 inlections such as enclocarditis.
lar condition. or other icJrms of" crystal or autoimmune diseases. Cl
KEY POINT

CJ Arthritis • The most appropriate and cost-effective means of


assessing the cause of acute monoat1hritis is by aspira­
HVC

In ,iddition to the p1Tsence or ,1bsence ol inlb111111ation. other


tion and analysis of the synovial fluid for leukocytes,
10,llures help to reline the dil'l<.:rcnti,11 di,1gnosis olmusculoskel
Gram stain with culture, and crystals.
et,1l 1x1in: p,lltern ol joint in\'OJ,ement. number ol"j(>ints involved.
duration ol",ymptom,. ,111cl presence or absence of"symmetry.

Extra-Articular Manifestations C]
Monoarticular Arthritis
l\lonoarticul,1r arthritis (monoarthritis) itll'nil·cs ,1 single joint.
of Rheumatologic Disease
The dil'lcrcnti,11 diagnosis depends upon \\' hether there is Many rhcumatologic diseases are systemic and therefore cause
inl"Jamm,1tion. the ,tcuity ol symptoms. and the patient·s extra ,irticular manifestations in various sites and organs.
dcmogrnphic and cpidemiologic risk foctors.
lnllam111,1tor,· 111on<Jcll"thritis 1rn1y be due to inlection: bac Constitutional Symptoms
tcrial in acute ec1ses. and atypirnl organisms such ,ls lungi. myco Constitutional symptoms are common in systemic inllamrna­
h,1Cteria. or spirochetes (Lyme) in chmnic cases. Noninlectious tor\ diseases. Fever is especially typic.il ol' adult onset Still
u1uses include nyst,11 rcl,lled ,rnd ,1utoimmune dise,1ses. disease ancl Lhe autointlarnmatory dise,1ses (also kno,�-n as the
Cryst,11 rcl,lled clise,1scs typiec1lly present ,icutel,· hut ocrnsion­ periodic fever syndromes) but may also occur in SLE. rheuma­
,illy can he chronic. cspecialil those c.1used hy calcium pyroph­ toid ,1rthritis. and ,·asculitis. in which it is commonly IO'vl'
osph,1te deposition. \011inlL1mm,1tory monoarthrilis is usually grade. M,liaise and unintended weight loss also occur. In the
due to tlstrn,irthrith or mechanical dernngcmcnt (such ,is a torn ,1bsence of a clear cause. such symptoms may also suggest
meniscus or ligament). ..\ history ol t rnuma prior lo onset or occult malignancy and or infection.
symptoms points to\\·,ird noninllamm,Hory ,irthritis.
The most ,1ppropri,1le and cost ei'lc.'Cli\'C' means 01",1ssessing Dermatologic Manifestations
the cause 0J' ,1cute monoarthritis h hy aspiration ,rncl analysis ol' Skin involvement is common in 111,111y rheumatologic diseases
the syno,,ial lluid fill" leukocytes. Gram st<1in 1\ ith culture. nncl (Table 3. on 1x1ge -1). Rec,1use ol' easy access by physical examina­
nystals. JJ'there is high suspicion !or inlcction (espcci,1lly ,1typi tion and or biopsy skin manilestations can be helpl"ul in estab­
cal organisms) and syno,·i,11 lluid cultures arc unrevealing. syn lishing a diagnosis. especially SLE. systemic sclerosis. and
o,·ial hiops,· or othn speci.11 IL'sts m,1y be needed. Hlood in the v,1sculilis. Some cutaneous l1ndings may be subtle or unnoticed
s,·nm·i,11 fluid (hem,irthrusis) h typic,111\" ,issocidlccl with trauma by the p,1Licnt (k)r e.\ample. nail pitting): therefcire. a high index of
e.\cept in p.itients \\·ith bleeding di,Hhcscs such ,ls hemophilia. ,uspicion ,md a thorough physic,11 ex,1111ination 111ay he required.

Oligoarthritis Inflammatory Eye Disease


Oligoarthritis imoh-es t\\·o to li1ur joints. commonly in the F.ye involvement is common in systemic rheumatologic dis­
lower C.\t remit ics. and is of"tcn ,1sy111111ct ric. ,\Ithough m,111y eases and provides clirect visual access to both mucosa! and
disc,1scs c,rn m,rnifcst \\ith oligoarthritis. the most eommon central nervous system tissues. The location ancl type of ocu­
,1utoi111mune inll,1111m,1tory forms or oligoanhritis are the lar im·ol,·ement may help narrow the differential diagnosis
spomlyloarthri I is discases (Table 2). Disscmina led gonococec1 I (Table 4. on page-I). Some disease's ,Jr<" particularly notable lor
infection. rheumatic lc,·er. and Lyme disease 111ay also present ocular in\'Olvement. including rheumatoid arthritis (episcle­
as inffam111,1ton· oligo,irthritis. Osteo,1rthritis m,1y c,1u,e a ritis and scleritis): sponclyloarthritis and s,ncoidosis (uveitis):

2
Approach to the Patient with Rheumatologic Disease

TABLE 2. Autoimmune Inflammatory Arthritis


Condition Pattern of Joint Extra-articular Diagnostic Studies
Involvement Features

Rheumatoid arthritis Symmetric polyarthritis; Rheumatoid nodules; d ry Rheumatoid factor; anti-CCP;


involves small joints (wrist, eyes and mouth; interstitial acute phase reactants; erosive
MCP, PIP, MTP) but also can lung disease; Felty changes on radiograph
involve hips, knees, elbows, syndrome (splenomegaly,
shoulders, and cervical leukopenia, leg ulcers)
spine; spares thoracic and
lumbar spine and DIP joints

Systemic lupus Symmetric polyarthritis Constitutional (fever, ANA; anti-double-stranded


erythematosus with large and small joint fatigue); multi-organ DNA; anti-Smith; anti-U1-RNP;
involvement; minimal to involvement (rash, oral anti-Ro/SSA; anti-La/SSB; no
no swelling ulcers, alopecia, serositis, erosions on radiograph
kidney disease, neurologic
disease, cytopenias)
Spondyloarthritis

Ankylosing spondylitis Sacroiliac and spinal Uveitis Calcification of anterior


involvement; symmetric; longitudinal ligament of spine
large joints (shoulders, on radiograph; sacroiliitis;
hips); spares small joints usually HLA-B27 positive

Psoriatic arthritis Asymmetric oligoarthritis or Psoriasis; uveitis "Pencil-in-cup" deformities;


symmetric polyarthritis; DIP erosions and osteophytes
joint preference; dactylitis on radiograph; sometimes
(sausage digits); enthesitis HLA-B27 positive
(insertion of tendon to
bone); axial disease with
sacroiliitis

Reactive arthritis Asymmetric oligoarthritis; Uveitis; keratoderma Sacroiliitis; sometimes


(formerly known as knee and ankle blennorrhagicum; HLA-B27 positive
Reiter syndrome) involvement; enthesitis; preceding infection
Achilles tendinitis; plantar (Chlamydia; enteropathic)
fasciitis; sacroiliitis
Inflammatory bowel Asymmetric; sacroiliitis; Crohn disease; ulcerative Sacroiliitis; sometimes
disease-associated knee and feet involvement colitis HLA-B27 positive
arthritis

ANA= antinuclear antibodies; CCP = cyclic citrullinated peptide; DIP= distal interphalangeal; MCP = metacarpophalangeal; MTP = metatarsophalangeal; PIP= proximal
interphalangeal; RNP = ribonucleoprotein.

CJ and vasculitis. especially granulomatosis with polyangiitis Rheumatologic and


(formerly known as Wegener granulomatosis). which can
CONT.
affect all parts of the eye. Dry eyes (keratoconjunctivitis sicca)
Musculoskeletal Manifestations
are a major feature of' Sjogren syndrome, whether primary or in Systemic Disease
in association with other rheumatologic diseases. Nonrheumatologic systemic diseases may have rheumatologic
Scleritis can result in permanent loss of vision. especially or musculoskeletal manifestations (Table 6. on page 6). Cl
ifscleral perforation occurs. Severe uveitis can also cause per­
KEY POINT
manent visual loss. Recognition and treatment of' these enti­
ties are therefore an urgent matter. • Systemic rheumatologic diseases can cause constitu­
tional symptoms and extra-articular manifestations in
Internal Organ Involvement various sites, including the skin, eyes, and internal
Internal organs are potential targets fc)r rheumatologic dis­ organs.
eases (Table 5. on page 5). Vasculitis may affect one or more
internal organs and should be considered in the differential
diagnosis ofa multisystem disease. Certain patterns of'involve­ Laboratory Studies
ment may be helpful in developing a differential diagnosis. In rheumatologic diseases. laboratory studies are used to aid in
such as pulmonary-renal syndrome (pulmonary vasculitis1 diagnosis, follow disease activity, assess the extent of disease­
hemorrhage and glomerulonephritis) that may occur with related internal organ involvement, and monitor patients tak­
ANCA-associated vasculitis. SLE. or Goodpasture syndrome. ing chronic immunomodulatory medications. However, some

3
Approach to the Pati ent with Rheumato l ogic Disease

TABLE 3. Dermatologic Manifestations of Rheumatologic Disease


Disease Manifestations

Systemic lupus erythematosus Butterfly (malar) rash; photosensitive rash; discoid lupus erythematosus; subacute cutaneous
lupus erythematosus; oral ulcerations (on the tongue or hard palate that are usually painless);
alopecia; lupus panniculitis (painful, indurated subcutaneous swelling with overlying erythema
of the skin)

Dermatomyositis Gottron papules (erythematous plaques on extensor surfaces of MCP and PIP joints);
photodistributed poikiloderma, including shawl sign (over the back and shoulders) and V sign
(over the posterior neck/back or neck/upper chest); heliotrope rash (violaceous rash on the upper
eyelid); mechanic's hands (hyperkeratotic, fissured skin on the palmar and lateral aspects of
fingers); nailfold capillary abnormalities; holster sign (poikilodermic rash along lateral thigh)
Amyopathic dermatomyositis Skin findings listed above under Dermatomyositis but without myositis findings
Systemic sclerosis Skin thickening and hardening (limited disease involves face and skin distal to elbows/knees;
diffuse disease involves skin proximal to distal forearms/knees); nailfold capillary changes

Vasculitis Palpable purpura; cutaneous nodules; ulcers

Beh c;:et syndrome Painful oral and genital ulcers; erythema nodosum; acne/folliculitis; pathergy (skin inflammation/
ulceration from minor trauma)
Sarcoidosis Erythema nodosum
Psoriatic arthritis Plaque psoriasis typically on extensor surfaces, umbilicus, gluteal fold, scalp, and behind ears;
pustular psoriasis on palms and soles; arthritis may precede rash by up to 10 years; nail pitting;
onycholysis
Reactive arthritis (formerly known Keratoderma blennorrhagicum (psoriasiform rash on soles, toes, palms); circinate balanitis
as Reiter syndrome) (psoriasiform rash on penis)

Adult-onset Still disease Evanescent, salmon-colored rash on trunk and proximal extremities
Rheumatic fever (secondary to Erythema marginatum (annular pink to red nonpruritic rash with central clearing)
streptoccocal infection)

Lyme disease Erythema chronicum migrans (slowly expanding, often annual lesion with central clearing)

MCP = metacarpophalangeal; PIP= proximal interphalangeal.

TABLE 4. Ocular Manifestations of Systemic Tests That Measure Inflammation


Inflammatory Disease Several tests may be abnormal when rheumatologic or other
Ocular Associated Systemic inflanunation is present. These include erythrocyte sedimenta­
Manifestation Inflammatory Disease tion rate and acute phase reactants, particularly C-reactive
Uveitis Spondyloarthritis (ankylosing
protein but also fibrinogen and ferritin. Platelet counts also tend
spondylitis, reactive arthritis [formerly to rise with inflammation. Complement levels usually increase
known as Reiter syndrome], with inflammation but fall with immune complex formation.
inflammatory bowel disease) (anterior
chamber); sarcoidosis (anterior and/or
posterior chamber); Behc;:et syndrome Erythrocyte Sedimentation Rate
(anterior and/or posterior chamber); Erythrocyte sedimentation rate (ESR) measures the rate at
granulomatosis with polyangiitis
(formerly known as Wegener
which erythrocytes settle with gravity in an upright tube of
granulomatosis) (posterior chamber) anticoagulated whole blood. ESR is dictated by characteristics
Episcleritis Rheumatoid arthritis; spondyloarthritis; of the erythrocytes themselves (size, shape, surface charge)
systemic vasculitis and by the presence of specific plasma proteins that alter the
Scleritis Rheumatoid arthritis; relapsing normal repulsive forces (that is, neutralize surface charge)
polychondritis; inflammatory bowel between erythrocytes and promote their ability to aggregate
disease; systemic vasculitis
and form rouleaux and sediment more quickly. Such plasma
Retinal disease Systemic vasculitis; antiphospholipid proteins include several acute phase reactants (especially
antibody syndrome
fibrinogen) produced by the liver in response to proinflamma­
tory cytokines arising in rheumatologic, infectious, or malig­
nant conditions. The normal ESR rises with age and tends to
commonly used laboratory studies have low specificity for be higher in women. An estimate of the maximal expected
diagnosing rheumatologic disease when the pretest probabil­ normal ESR based on age and gender is age in years/2 for men
ity of disease is low, and rigor must be applied when interpret­ and (age in years+ 10)/2 for women. Thus, a mildly elevated
ing such results. ESR in an older patient must be interpreted with caution.

4
Approach to the Patien t with Rheumatologic Dis e a s e

TABLE 5. Internal Organ Involvement in Rheum atologic Disease


Or gan Disease Type of Involvement
Heart

Kawasaki disease Coronary artery vasculitis


Systemic sclerosis Arrhythmia; cardiomyopathy; pulmonary
hypertension
SLE Pericarditis; valvular disease; cardiomyopathy
RA Pericarditis; cardiomyopathy
Rheumaticfever; antiphospholipid antibody Valvular disease
syndrome

GCA; Takayasu arteritis Aortitis; heart failure; large-vessel vasculitis

Lung

RA Serositis; ILD; rheumatoid nodules


SLE; CTDs; HSP Serositis; pneumonitis; pulmonary hemorrhage
from vasculitis
AAV; systemic vasculitis Pulmonary hemorrhage; cavitary nodules
Systemic sclerosis ILD; pulmonary hypertension
Antiphospholipid antibody syndrome Pulmonary embolism
Sarcoidosis Hilar lymphadenopathy; ILD

Goodpasture syndrome Pulmonary hemorrhage

Kidney

SLE; CTDs; AAV; systemic vasculitis (except PAN) Glomerulonephritis


PAN Renal artery vasculitis
Antiphospholipid antibody syndrome Renal infarct; renal vein thrombosis

Sjogren syndrome Acute interstitial nephritis/RTA

Goodpasture syndrome Glomerulonephritis

Gastrointestinal

PAN Mesenteric vasculitis

Spondyloarthritis Inflammatory bowel disease


HSP Intestinal vasculitis; ulcerations

Beh,;:et syndrome Ulcerations; inflammatory bowel disease

FMF Peritonitis

Neurologic

SLE; CTDs; AAV; systemic vasculitis Mononeuritis multiplex; peripheral neuropathy

PACNS CNS vasculitis

AAV= ANCA-associated vasculitis; CNS= central nervous system; CTD = connective tissue disease; FMF = familial Mediterranean fever; GCA= giant cell arteritis; HSP = Henoch-
Sch0n1ein purpura; ILD= interstitial lung disease; PACNS= primary angiitis of the central nervous system; PAN= polyarteritis nodosa; RA= rheumatoid arthritis; RTA= renal
tubular acidosis; SLE = systemic lupus erythematosus.

ESR elevations are used to identify and monitor disease independently causing elevated fibrinogen and/or other
activity in rheumatologic diseases, especially in polymyalgia plasma proteins.
rheumatica and giant cell arteritis; however. because the Although elevations in ESR most commonly indicate
population with these diseases is older, ESR may be less reli­ inflammation, some noninflammatory conditions (kidney
able. ESR is an important component of several measures disease, diabetes mellitus, pregnancy, and obesity) are also
used to assess disease activity in rheumatoid arthritis and is associated with elevated fibrinogen levels and can produce an
included in the American College ofRheumatology's recom­ elevated ESR. S ome patients make paraproteins (for example,
mendations for use in clinical practice. Interpretation of an patients with multiple myeloma) that can also cause ESR
elevated ESR can be difficult in the presence of conditions elevations. Conditions associated with low ESR (due to

5
Approach to the Patient with Rheumatologic Disease

! TABLE 6. Rheumatologic and Musculoskeletal Manifestations in Systemic Disease . .


Systemic Disease Rheumatologic/Musculoskeletal Manifestations

Diabetes mellitus Dupuytren contracture; adhesive capsulitis of the shoulder; diabetic amyotrophy (ischemic lumbosacral
plexopathy); carpal tunnel syndrome; diffuse idiopathic skeletal hyperostosis; trigger finger;
cheiroarthropathy (scleroderma-like thickening of hands); diabetic osteoarthropathy/Charcot foot

Hypothyroidism Arthralgia/myalgia; myopathy (with elevated serum creatine kinase)

Hyperthyroidism Osteoporosis; myopathy (serum creatine kinase not elevated)

Hyperparathyroidism Calcium pyrophosphate deposition; osteoporosis

Acromegaly Arthralgia/osteoarthritis; bone pain; calcium pyrophosphate deposition

Sickle cell disease Sickle crisis; osteonecrosis; bone pain

Hemophilia Hemarthroses

Carcinoma Inflammatory polyarthritis or tendinitis; dermatomyositis; hypertrophic osteoarthropathy

Myeloma/lymphoma/ Cryoglobulinemia; amyloidosis; arthritis (particularly in children)


leukemia
Hemochromatosis Osteoarthritis (especially atypical joints such as metacarpophalangeal joints); calcium pyrophosphate
deposition

reduced fibrinogen or abnormal erythrocyte shape and attrac­ complexes and apoptotic cells; genetic C4 deficiency may also be
tion) include heart and liver failure, sickle cell disease, poly­ seen in lupus-like autoimmunity.
cythemia, and spherocytosis. Reductions in C3 and C4 levels usually are measured to
assess for immune complex-mediated consumption. CHSO
C-Reactive Protein tests measure the ability of serum complement to lyse immu­
C-reactive protein (CRP) is an acute phase reactant synthe­ noglobulin-coated erythrocytes. CHSO thus assesses overall
sized by the liver during inflammation in response to proin­ activation of the classical complement pathway and is abnor­
flammatory cytokines. CRP can adhere to bacteria and activate mal when any component of the classical system is depleted.
complement, promoting phagocytosis. Values greater than However, CHSO is no longer routinely used because it is labor
0.8 mg/dL (8.0 mg/L) are considered inflammatory, whereas intensive, expensive, and usually no more useful in diagnosis
levels between 0.2 mg/dL and 0.8 mg/dL (2.0 mg/L and and assessment than C3 and C4.
8.0 mg/L) are indeterminate.
CRP responds rapidly to inflammation, both rising and fall­ Autoantibody Tests
ing more quickly than ESR. CRP is more stable and less affected The presence of autoantibodies is characteristic of many rheu­
by other sernm components compared with ESR. Like ESR, CRP matologic diseases. Rheumatoid factor and antinuclear anti­
is elevated with disease activity in most rheumatologic diseases bodies (ANA) are the most commonly ordered autoantibody
and during other forms of inflammation (for example, infection tests. Rheumatoid factor is an immunoglobulin directed
or malignancy). However, in some patients with SLE, CRP will against the Fe portion of lgG, and ANA is directed against
increase during infection but not during disease flare. nuclear antigens. The specificity of these autoantibodies for a
particular rheumatologic disease is relatively low because they
Complement may occur in other rheumatologic and nonrheumatologic
The complement system assists in bacterial opsonization and diseases and even in healthy persons.
lysis during humoral immune responses; it also promotes Rheumatoid factor presence is characteristic of rheumatoid
inflammatory reactions. There are three complement cascades arthritis, but a significant minority of patients with rheumatoid
(classical, alternative, and mannose-binding lectin pathways); arthritis lack rheumatoid factor. Anti-cyclic citrnllinated peptide
each is triggered differently, but all generate pro-opsonic and antibodies are more specifically characteristic of RA and often
proinflammatory components. appear earlier in the disease; using the two antibody tests together
Complement levels are generally increased in inflammatory may be helpful. Rheumatoid factor is frequently present in
states (that is, complement components are acute phase reac­ healthy persons, especially at older ages, but usually in low titer.
tants). However, when immune complexes are present (for The immunofluorescence assay is considered more relia­
example, SLE or certain types of vascuJitis), complement is con­ ble than the enzyme-linked immunosorbent assay (ELISA) for
sumed, leading to lower than nom1al levels. Rare genetic defi­ detecting ANA, but ELISA is less expensive and less labor inten­
ciencies of complement, particularly of the early complement sive. About one third of the healthy population have a positive
components Clq, C2, and C4, paradoxically increase the risk of low-titer (1:40) ANA, and 3% to 5% have a titer of 1:160 or
autoimmune disease, perhaps by impairing clearance of immune higher. Asymptomatic ANA positivity is more common in

6
Approach to the Patient with Rheumatologic Disease

women and healthy first-degree relatives of patients with auto­ Osteoarthritis causes compartmental joint-space narrowing
immune disease. Higher ANA titers are more likely to be asso­ and bony hypertrophy.
ciated with rheumatologic disease. More than 95% of patients Plain radiography does not visualize soft tissues nearly as
with SLE have a positive ANA; conversely, a negative ANA is rare well as bone. and clue to the two-dimensional nature of the
in those with SLE. ANA occurs in other autoimmune diseases, images. not all bone findings are visible on every view (for
including rheumatoid arthritis, other connective tissue dis­ example, visible on oblique but not anteroposterior imaging).
eases, infection, and malignancy. ANA can also be drug Plain radiography may not detect early (within the f1rst
induced. ANA titer does not correlate with disease activity and 6-12 months) or mild erosive arthritic changes. Despite these
should not be checked repeatedly during disease management. limitations. serial plain radiography can be useful for monitor­
A positive ANA in a patient with nonspecific symptoms is ing arthritis disease progression.
difficult to interpret. As with all tests, the positive predictive Plain radiography is relatively inexpensive and readily
value of ANA rests upon the pretest probability of disease. The available. Despite low levels of ionizing radiation. plain radiog­
American College of Rheumatology's Choosing Wisely list cur­ raphy is considered safe except for pregnant women.
rently recommends against testing ANA subserologies without
the combination of a positive ANA and clinical suspicion of CT
immune-mediated disease. In contrast to plain radiography. CT scanning permits multiple
Table 7 provides details on these and other autoanti­ views and orientations from a single study. CT is more useful
bodies. for bony abnormalities than for soft-tissue inflammation or
fluid collections . Calcium pyrophosphate deposition is readily
KEY POINTS
detected on CT. even when calcium deposits are overlooked on
HVC • Laboratory studies such as erythrocyte sedin1entation other modalities . CT is more sensitive in detecting bone ero­
rate, rheumatoid factor, and antinuclear antibodies have sions than plain radiographs or MRI.
low specificity for diagnosing rheumatologic disease in CT is more expensive than plain radiography and exposes
patients with low pretest probability, thus limiting their the patient to more radiation.
utility in this population.
• Although elevations in erythrocyte sedimentation rate MRI
(ESR) most commonly reflect the presence of inflam­ MRI is useful in detecting soft-tissue abnormalities. inflam­
mation, increasing age and some noninflammatory mation. and fluid collections. but is less effective than CT in
conditions are also associated with elevated ESR. demonstrating bony abnormalities and erosive changes. MRI is
more sensitive than plain radiography in detecting early spine
• C-reactive protein responds rapidly to inflammation, both
and sacroiliac joint inflammation.
rising and falling more quickly than erythrocyte sedi­
MRI is more expensive than plain radiography and CT and
mentation rate (ESR) and is more stable and less affected
is generally ordered when assessment of soft-tissue imaging is
by other serum components compared with ESR.
required. Although MRI does not expose the patient to radiation,
MRI contrast (gadolinium) must be avoided in patients with

C] Imaging Studies kidney disease clue to the risk or nephrogenic systemic fibrosis.
Patient intolerance due to claustrophobia or inability of the
Imaging studies can be useful to diagnose and follow patients patient to fit in the scanner due to large body habitus may limit
with rheumatologic diseases. For example. inllammatory arthri­ the ability to obtain MRI data. Becau e data are currently inade­
tis can be evaluated with studies ranging from plain radiography quate to justify its use. the American College of Rheumatology
to MRI. and medium- and large-vessel vasculitis can be detected Choosing Wisely list questions the utility of routinely ordering
using angiography. For all studies. it is important to consider MRI or peripheral joints to monitor rheumatoid arthritis.
whether the ordering oCimaging tests will aid in the diagnosis or
management of a suspected condition. and whether the benelits Ultrasonography
of such testing clearly outweigh the potential risks and costs. Musculoskeletal ultrasonography is increasingly utilized in
rheumatologic disease. Ultrasonography can detect soft­
Radiography tissue abnormalities such as synovitis. tendinitis, bursitis. and
Plain radiography is used to assess inflammatory and degenera­ joint fluid. and Doppler can assess for increased tissue blood
tive arthritis (Table 8. on page 9). Autoimmune inflammatory flow consistent with synovitis. Ultrasonography can diagnose
arthritis may produce joint-space narrowing. erosions. and and monitor disease. and can be used to guide arthrocentesis.
osteopenia. and in the case of spondylo,1rlhritis. productive Dynamic pathologies such as impingement can be visualized.
bony overgrowth. Crystal-related inflammatory diseases have Unlike other modalities. ultrasonography is portable
characteristic radiographic findings. including punched-out and can be used in the outpatient setting at the bedside for
bone lesions in gout or cartilage calcification (also known as point-of-care evaluation and procedures. However. it is
chondrocalcinosis) in calcium pyrophosphate deposition. operator dependent. and training and practice are required

7
Approach to the Pa ti ent with Rheumatologic Disease

TABLE 7. Autoantibodies in Rheumatologic Disease


Autoantibody Rheumatologic Condition Sensitivity/Specificity Comments

ANA SLE; also SSc, Sjogren, SLE: 95% sensitivity; poor Does not correlate with disease
and MCTD specificity activity

Anti-double-stranded DNA SLE SLE: 60% sensitivity, >95% Found in more severe disease,
specificity; Crithidia IFA or especially kidney disease;
Farr assays more specific antibody levels commonly
than ELISA follow disease activity and
are useful to monitor

Anti-Smith SLE SLE: 30% sensitivity, 99% Most specific test for SLE; does
specificity not correlate with disease
activity

Anti-U1-RNP MCTD; SLE 100% sensitivity High titer seen in MCTD


(>1:10,000); does not correlate
with disease activity

Anti-Ro/SSA; anti-La/SSB Sjogren; SLE; RA; SSc Sjogren: 70% sensitivity; Sicca symptoms; in SLE,
SLE: 30% sensitivity associated with photosensitive
rash and neonatal lupus
erythematosus (rash and
conduction block)

Anti-Scl-70 DcSSc 10%-30% sensitivity Seen more often in patients


(antitopoisomerase) with SSc who have pulmonary
fibrosis

Anticentromere LcSSc (CREST) 10%-30% sensitivity Patients with SSc with this
antibody are more likely to
develop pulmonary
hypertension
c-ANCA (antiproteinase-3) GPA 90% sensitivity when Correlation with disease
disease is active; high activity is unclear
specificity in classic
presentations

p-ANCA MPA; EGPA MPA: 80% sensitivity; EGPA: Atypical p-ANCA


(antimyeloperoxidase) 60% sensitivity; less specific (antimyeloperoxidase
than c-ANCA negative) can be seen in
inflammatory bowel disease
and with positive ANA
Anti-Jo-1 Myositis 20%-30% sensitivity Associated with antisynthetase
syndrome

Rheumatoid factor RA; Sjogren; RA: 70% sensitivity; limited RF is an antibody to lg and
cryoglobulinemia specificity, especially in hence many false positives
patients without a classic (hepatitis C, SLE); 30% with RA
disease presentation are RF negative; may convert
to positive later in RA course
Anti-cyclic citrullinated RA RA: 70% sensitivity; more Can be positive in RF-negative
peptide specific than RF for RA RA patients; often present
before RF becomes positive;
associated with erosions;
predicts disease progression
in undifferentiated arthritis
Antihistone DILE 95% sensitivity; poor Also seen in patients with
specificity native lupus
Cryoglobulins Vasculitis; hepatitis C; Type II or Ill cryoglobulins May be present in connective
myeloma; SLE; RA seen in patients with tissue diseases in the absence
cryoglobulinemic vasculitis of vasculitis

ANA= antinuclear antibodies; CREST= calcinosis, Raynaud phenomenon, esophageal dysmotility, sclerodactyly, and telangiectasia; DcSSc = diffuse cutaneous systemic scle·
rosis; DILE= drug-induced lupus erythematosus; EGPA= eosinophilic granulomatosis with polyangiitis (formerly known as Churg-Strauss syndrome); ELISA= enzyme-linked
immunosorbent assay; GPA= granulomatosis with polyangiitis (formerly known as Wegener granulomatosis); IFA = immunofluorescent assay; LcSSc= limited cutaneous sys­
temic sclerosis; MCTO = mixed connective tissue disease; MPA = microscopic polyangiitis; RA= rheumatoid arthritis; RF= rheumatoid factor; RNP = ribonucleoprotein;
SLE= systemic lupus erythematosus; SSc= systemic sclerosis.

8
Principles of Therapeutics

TABLE 8. Radiographic Findings of Common Rheumatologic Diseases


Rheumatologic Disease Radiographic Findings

Rheumatoid arthritis Bony erosions; periarticular osteopenia; subluxations; soft-tissue swelling; MCP and PIP involvement
on hand radiograph
Osteoarthritis Asymmetric joint-space narrowing; osteophytes; subchondral sclerosis and cystic changes;
degenerative disk disease with collapse of disks; degenerative joint disease with facet joint
osteophytes; these findings lead to spondylolisthesis (anterior/posterior misalignment of the spine)
and kyphosis
Diffuse idiopathic skeletal Calcification of the anterior longitudinal ligament; bridging horizontal syndesmophytes; usually more
hyperostosis prominent on right side of spine than left
Ankylosing spondylitis Sacroiliitis; squaring of the vertebral bodies; bridging vertical enthesophytes; shiny corners
Psoriatic arthritis Destructive arthritis with erosions and osteophytes; DIP involvement; "pencil-in-cup" deformities on
hand radiograph; arthritis mutilans
Gout Punched-out erosions with sclerotic border and overhanging edge; periarticular soft-tissue swelling
with calcifications in tophaceous deposits

Calcium pyrophosphate Chondrocalcinosis, most commonly of the knees, shoulders, wrists, pubic symphysis; leads to
deposition osteoarthritis

DIP= distal interphalangeal; MCP = metacarpophalangeal; PIP= proximal interphalangeal.

Cl
CONT.
to achieve competency at performing and interpreting
ultrasonography.
noninflammatory conditions, and greater than 2000/µL
(2.0 x 109/L) are associated with inflammatory states. The
Ultrasonography is relatively inexpensive, and there is no higher the count is. the more inflammatory the fluid and the
ionizing radiation. CJ greater the suspicion for crystal-related or infectious disease.
There is no absolute cutorr value that distinguishes
KEY POINTS
infection from crystal-related disease. because some infec­
HVC • CT is useful in assessing bony abnormalities but is more tions may have lower counts than expected and crystal­
expensive than plain radiography and exposes the related disease may have counts greater than 100,000/µL
patient to more radiation. (100 x 10" L). Thus. the proper application or synovial fluid
HVC • MRI is useful in detecting soft-tissue abnormalities, leukocyte counts requires conser vative interpretation.
inflammation, and fluid collections but should not be Generally. counts greater than 50.000/µL (50 x 109 /L) should
routinely used to monitor rheumatoid arthritis disease be managed as inrectious until explicitly proven otherwise:
activity in peripheral joints. if there is clinical suspicion for infection. fluid should be
HVC • Musculoskeletal ultrasonography is a low risk and rela- sent for stains and cultures even in the setting of counts less
tively inexpensive way to detect soft-tissue abnormali­ than 50.000 µL (50 x 109 L).
ties such as synovitis, tendinitis, bursitis, and joint fluid. See Crystal Arthropathies for a discussion on synovial
fluid crystal evaluation. Cl

CJ Joint Aspiration KEY POINT


• Synovial fluid leukocyte counts less than 200/µL
Synovial fluid aspiration is essential when evaluating for infec­
(0.2 x 109/L) are considered normal, between 200/µL and
tion and crystal-related disease and can distinguish bet,veen
2000/µL (0.2 x 109/L and 2.0 x 109/L) are associated with
inflammatory and noninflammatory conditions. The most
noninflammatory conditions, and greater than 2000/µL
useful tests of synovial fluid for infection are leukocyte count,
(2.0 x 109/L) are associated with inflammatory states.
stains. and cultures. as well as evaluation of synovial fluid for
crystals under polarized light. Synovial fluid autoantibodies
and protein and glucose levels are of limited utility and used
primarily in research investigations. The gross appearance of Principles of Therapeutics
synovial fluid can predict results in the laboratory. Normal
fluid is clear and highly viscous. lntlammatory fluid is cloudy Introduction
and watery. Infected fluid is cloudy to opaque and. while This section discusses the uses, mechanisms of action, targets,
'·thick·' like pea soup. lacks the viscosity of normal fluid . potential toxicities, and common monitoring parameters of
Synovial fluid leukocyte counts less than 200/µL medications used in rheumatologic disease. Specific applica­
(0 .2 x 109 /L) are considered normal. between 200/�tL and tions of individual drugs are further discussed in the disease­
2000/µL (0.2 x 109/L and 2.0 x 109 /L) are associated with specific sections.

9
Principles of Therapeutics

Anti-Inflammatory Agents Oral NSAIDs vary in potency, kinetics, metabolism,


cost, and selectivity for COX-1 versus COX-2. Selection of a
Anti-inflammatory agents reduce pain and improve swelling,
particular NSAID depends mainly upon convenience of use
warmth, and redness but generally do not prevent disease
and individual tolerance, including consideration of the
progression.
patient's comorbidities. All NSAIDs should be used at the
lowest effective dose for the shortest time; however, effective
NSAIDs use of NSA!Ds in rheumatologic disease often requires high
NSA!Ds inhibit cyclooxygenase (COX) enzymes to block the gen­ and prolonged dosing. NSA!Ds are not generally disease
eration of the lipid prostaglandin E2 (PGE). PGE2 stimulates modifying; they improve arthritis symptoms but not
inflammation, vasodilation, smooth muscle contraction, pain, long-term outcomes.
and fever; NSAlDs therefore convey anti-inflammatory, analge­ Topical NSA!Ds such as diclofenac (available as a solution,
sic, and antipyretic effects. However, PGE 2 also maintains gastric spray, gel, or patch) provide similar pain relief for inf1amma­
mucosa and promotes kidney sodium excretion and glomerular tory conditions as oral medications with fewer gastrointestinal
filtration. Other COX products include thromboxane A2 , a pro­ effects. The American College of Rheumatology currently rec­
thrombotic regulator of platelets, and prostacyclin, an antithrom­ ommends topical NSA!Ds rather than oral NSA1Ds for patients
botic and vasodilatory lipid. Because NSA!Ds inhibit ail of these, aged 75 years or older. However, they are associated with more
the consequences of COX inhibition are complex and accompa­ skin reactions than placebo and are significantly more expen­
nied by multiple potential side effects (Table 9). Side-effect risk sive than oral NSA!Ds.
is increased in older patients and those with preexisting comor­
bidities. Regular monitoring of blood pressure, kidney function, Glucocorticoids
and blood counts during chronic NSAID use is recommended. Glucocorticoids are potent anti-inf1ammatories and are
The identification of two distinct COX isoforms permitted effective in many rheumatologic diseases; however, they
the development of selective COX-2 inhibitors. Because the are associated with multiple potential toxicities and should
COX-1 isoform is responsible for platelet function and gastropro­ be used at the lowest effective dose for the shortest period
tection, COX-2 inhibitors cause less gastrotoxicity and bleeding possible. Intermediate to long-term use is associated with
than nonselective NSA!Ds. However, COX-2 inhibitors are not an increased risk of diabetes mellitus, osteoporosis, oste­
less nephrotoxic and cause similar amounts of hypertension. onecrosis, weight gain, fluid retention, hypertension, car­
The most selective COX-2 inhibitors also cause increased cardio­ diovascular disease, striae and bruising, and glaucoma and
vascular risk and have been removed from the market. However, cataracts; monitoring for adverse events can help limit
nearly all NSAlDs may convey some cardiovascular risk. morbidity. Virtually all patients receiving chronic gluco­
In contrast to other NSA!Ds, aspirin (acetylsalicylate) per­ corticoids or frequent tapering doses should take calcium
manently inactivates COX enzymes. Platelets cannot generate and vitamin D supplementation; patients receiving gluco­
replacement COX and are particularly vulnerable to this effect. corticoids for more than 4 weeks at doses greater than
Therefore, low-dose aspirin administration for cardiovascular s mg of prednisone daily should be considered for
disease permits a platelet-selective effect that reduces, but bisphosphonate treatment.
does not entirely alleviate, potential gastrointestinal toxicity.
Higher doses promote more general COX inhibition and con­ Colchicine
vey the same risks as other NSAlDs. Colchicine disrupts microtubules to interfere with leukocyte
adhesion and migration. Other potential effects include inhi­
bition of interleukin (IL)-1 generation. Colchicine is used to
TABLE 9, Potential Toxicities of NS�ID Use treat gout, familial Mediterranean fever, and hypersensitivity
.
Category Toxicity vasculitis. At low doses, colchicine is generally safe and well
Cardiovascular Hypertension; myocardial infarction;
tolerated; higher doses routinely cause diarrhea. Acute colchi­
exacerbation of heart failure cine overdose can be fatal; chronic low-level colchicine over­
Hemostatic Bleeding diathesis dose can cause neuromyopathy. Colchicine is renally cleared;
patients with kidney disease require dose adjustment.
Gastrointestinal Dyspepsia; reflux; peptic ulcer disease;
gastrointestinal bleeding Colchicine is metabolized in the liver by the CYP3A4
Obstetric/ Bleeding; delayed labor; premature
cytochrome, and its absorption from the stomach is limited by
Gynecologic ductus arteriosus closure the P-glycoprotein export pump; consequently, patients must
Pulmonary Asthma exacerbation reduce or avoid colchicine when also taking moderate/strong
CYP3A4 inhibitors (for example, clarithromycin, most antiret­
Renal Hypertension; decreased glomerular
filtration; increased salt and water roviral drugs) or P-glycoprotein inhibitors (for example, cyclo­
retention; increased renin production; sporine). When used in combination with statins (especially
uncommonly, allergic interstitial nephritis
those metabolized by CYP3A4), colchicine may increase the
or acute tubular necrosis
risk of drug-induced myopathy.

10
Principles of Therapeutics

KEY POINTS KEY POINTS

HVC • Topical NSAIDs provide similar pain relieff or innam- • Simple analgesics are neither anti-inflammatory nor
matory conditions as oral medications with fewer gas­ disease modifying but may help alleviate arthritis pain.
trointestinal effects and are preferred for patients 75 • Acetaminophen is generally well tolerated and benefl­
years or older; however, they are associated with more cial, but excessive doses carry risk for liver failure and
skin reactions and are significantly more expensive even death.
than oral NSA!Ds.
• Virtually all patients receiving chronic glucocorticoids
or frequent tapering doses should take calcium and
vitamin D supplementation; patients receiving gluco­ Disease-Modifying
corticoids for more than 4 weeks at doses greater than Antirheumatic Drugs
5 mg of prednisone daily should be considered for bis­ Disease-modifying anti.rheumatic drugs (DMARDs) are
phosphonate treatment. immunosuppressive agents used to achieve control and/or
remission in rheumatologic disease. Most DMARDs increase
the risk of infection, and each agent has its own specific
Analgesics potential toxicities.
Simple analgesics are neither anti-inflammatory nor disease
modifying but can help relieve pain in patients with arthritis. Nonbiologic Disease-Modifying
Acetaminophen is generally well tolerated and has both Antirheumatic Drugs
analgesic and antipyretic effects; its mechanism of action is Table 10 summarizes the mechanisms of action, indications,
not well established. The maximum recommended daily dose and common monitoring parameters of various nonbiologic
by the FDA is 4 g/d, and doses greater than 4 g/d carry an DMARDs. See Rheumatologic Medications and Pregnancy
increased risk for liver failure and even death. Because aceta­ for information on these medications in women of child­
minophen content in over-the-counter products (for example, bearing potential.
for allergies, headaches, and colds) may not be taken into
account by some patients when calculating the total daily Methotrexate
dose, some guidelines limit maximum daily intake to 3 g/d in Methotrexate inhibits folic acid metabolism and increases
most patients and 2 g/d in patients with liver disease. In extracellular adenosine levels. It is the recommended initial
chronic acetaminophen use, periodic assessment of liver and DMARD for most patients with rheumatoid arthritis and is
kidney function may be prudent. useful in other disorders, including psoriatic arthritis, vascu­
Opiates act on neurons to block pain signaling. Opiates litis, and sarcoidosis. Methotrexate is administered weekly
are less effective for inflammatory than for malignancy pain, along with daily folic acid supplementation, which limits
but milder opiates (for example, codeine or oxycodone) have toxicity without aftecting efficacy. Potential toxicities include
an occasional role in treating patients with arthritis who can­ hepatitis and bone marrow suppression (leukopenia, anemia).
not otherwise obtain relief. Opiates are potentially habit­ Patients with liver disease should not receive methotrexate,
forming, can cause constipation, and may cause respiratory and limitation of alcohol intake is strongly advised.
depression at excessive doses. Tramadol is a centrally acting
synthetic opioid analgesic that also weakly inhibits the reup­ Hydroxychloroquine
take of norepinephrine and serotonin; potential for addiction Hydroxychloroquine is an antimalarial medication that
is lower than for traditional opioids. appears to inhibit antigen processing. In systemic lupus ery­
Agents that modulate pain signals in the spinal cord (for thematosus (SLE), hydroxychloroquine is used to control skin
example, pregabalin, gabapentin, tricyclic antidepressants such and joint disease, prevent systemic and organ-specific disease
as amitriptyline, and dual serotonin-norepinephrine reuptake flare-ups, and reduce overall mortality. It is also used (alone or
inhibitors such as duloxetine and milnacipran) are useful in in combination with methotrexate and sulfasalazine) to treat
patients with central chronic pain syndromes (for example, fibro­ undifferentiated connective tissue disease, rheumatoid arthri­
myalgia) as well as musculoskeletal pain. Side effects include tis, and other forms of inflammatory arthritis. Although its
sedation and dry mouth. Concomitant use of serotonin reuptake efficacy in arthritis is modest, its excellent side-effect profile
inhibitors, tricyclic antidepressants, or monoamine oxidase makes it a useful adjunctive therapy.
inhibitors may lower seizure threshold and/or raise the risk of
serotonin syndrome (mental status changes, autonomic instabil­ Sulfasalazine
ity, neuromuscular aberrations, and gastrointestinal symptoms). Sulfasalazine exerts systemic effects through its metabolite
Topical analgesics (such as capsaicin and lidocaine) may sulfapyridine and intracolonic effects via the salicylate moi­
be useful and can limit systemic drug exposure when only a ety. Sulfasalazine is moderately effective in rheumatoid
single area is painful. arthritis with or without methotrexate. It is also used to treat

11
Pr inciples of Th e rapeutics

TABLE 10. Nonbiologic Disease-Modifying Antirheumatic Drugs


Agent Mechanism Indications Common Monitoring Parameters

Methotrexate DHFR inhibition; RA; psoriasis; psoriatic Baseline: chest radiography,


extracellular adenosine arthritis; DM; PM; vasculitis hepatitis screening, CBC, LCTs,
level modification serum creatinine

Thereafter: CBC, LCTs, serum


creatinine every 3 months•

Hydroxychloroquine Uncertain; appears to SLE; RA Baseline: CBC, LCTs, serum


involve stabilization of creatinine
lysosomal vacuoles, leading
Baseline/periodic ophthalmologic
to inhibition of antigen
examinations approximately every
processing and
12 months to evaluate for
costimulatory activation
hydroxychloroquine deposition,
which rarely can lead to visual loss

Sulfasalazine Anti metabolite; a pro-drug RA; SpA; IBD Baseline: CBC, LCTs, serum
broken down into 5-amino creatinine
salicylic acid (active
Thereafter: CBC, LCTs, serum
metabolite in the
creatinine every 3-6 months
gastrointestinal tract)
and sulfapyridine (exerts
systemic action)

Leflunomide Blocks dihydroorotase, RA Baseline: chest radiography,


an enzyme involved in hepatitis screening, CBC, LCTs,
pyrimidine biosynthesis serum creatinine
that targets replicating
Thereafter: CBC, serum creatinine
lymphocytes, which
every 3 months; LCTs every 8-12
lack pyrimidine salvage
weeks, and leflunomide temporarily
pathways; antiproliferative
or permanently discontinued for
significant elevations >2 times
normal and additional therapy
with cholestyramine for elevations
>3 times normal•
Azathioprine Purine analogue; inhibits SLE; DM; PM; vasculitis; IBD Baseline: CBC, LCTs, serum
DNA synthesis essential creatinine
for proliferating T- and
Thereafter: CBC, LCTs, serum
B-lymphocytes
creatinine every 3 months•
Cyclophosphamide Alkylating agent; blocks Severe and life-threatening Close monitoring clinically and
DNA synthesis and causes disease in SLE, DM, PM, and measuring CBC, chemistries, LCTs,
cell death vasculitis urinalysis every 4-8 weeks

Mycophenolate lnosine monophosphate SLE (especially lupus Baseline: CBC, LCTs, serum
mofetil inhibition; antiproliferative; nephritis); vasculitis; DM; creatinine
mycophenolate is PM
Thereafter: CBC, LCTs, serum
converted into the active
creatinine every 3 months•
metabolite, mycophenolic
acid, which inhibits
inosine monophosphate
dehydrogenase (an enzyme
in the purine synthetic
pathway) and preferentially
inhibits T- and B-lymphocytes
Cyclosporine Inhibits calcineurin (a SLE; psoriasis; RA Baseline: CBC, LCTs, serum
transcription activating creatinine
factor); preferentially
targets T cells Thereafter: CBC, LCTs, serum
creatinine every 2-3 months•
Apremilast Inhibits phosphodiesterase Psoriasis; psoriatic arthritis Baseline: weight
4
Thereafter: weight, neuropsychiatric
effects

CBC= complete blood count; DHFR = dihydrofolate reductase; DM= dermatomyositis; 180 = inflammato,y bowel disease; LCTs = liver chemistry tests; PM= polymyositis;
RA= rheumatoid arthritis; SLE = systemic lupus erythematosus; SpA= spondyloarthritis.

a Recommended monitoring interval is for a stable dose but may be shorter after initiation or in the case of abnormal results and must be individualized to the patient's risk of
toxicity.

12
Principles of Therapeutics

spondyloarthritis and inflammatory bowel disease. Toxicities Although experience is limited, studies suggest that apremilast
include gastrointestinal upset, headache, agranulocytosis, is well tolerated and of moderate efficacy.
hepatitis, and reversible oligospermia.
KEY POINTS

Leflunomide • Methotrexate is the recommended initial disease­


Leflunomide inhibits lymphocyte activation by blocking the modifying antirheumatic drug for most patients with
pyrimidine synthesis pathway. It is approved to treat rheumatoid rheumatoid arthritis.
arthritis, in which its efficacy is comparable to methotrexate. • Hydroxychloroquine is used in patients with systemic
Toxicities include gastrointestinal upset, diarrhea, aminotransam­ lupus erythematosus to control skin and joint disease,
inase elevations, cytopenias, infection, and teratogenesis. prevent systemic and organ-specific disease flare-ups,
and reduce overall mortality.
Azathioprine • Cyclophosphamide is used to treat severe and/or life­
Azathioprine is a purine analogue that inhibits nucleotide threatening manifestations of systemic lupus erythema­
synthesis. It is used to treat and/or maintain control of SLE, tosus, systemic sclerosis, the inflammatory myopathies,
vasculitis, the inflammatory myopathies, and other autoim­ interstitial lung disease, and vasculitis.
mune diseases. Toxicities include gastrointestinal intolerance,
• Mycophenolate mofetil may be at least as effective as
bone marrow suppression, hepatitis, and pancreatitis.
cyclophosphamide for systemic lupus erythematosus
Coadministration with allopurinol or febuxostat should be
but with fewer, and milder, side effects.
avoided because these three drugs compete for the same met­
abolic pathway, and toxic levels may ensue.
Biologic Disease-Modifying Antirheumatic Drugs

Cl Cyclophosphamide
Biologic DMARDs are protein-based products that alter the
Cyclophosphamide is a DNA alkylating agent with potent body's natural processes to block immune responses (Figure 1).
immunosuppressive properties. It is used to treat severe and/ Biologic DMARDs are more specific and typically more effec­
or life-threatening manifestations or SLE (including nephri­ tive than nonbiologic DMARDs; however, they are significantly
tis), systemic sclerosis, the inflammatory myopathies, inter­ more expensive. Biologic DMARDS are generally administered
stitial lung disease, and vasculitis. Toxicities include bone parenterally; the suffixes of their names indicate their general
marrow suppression. leukopenia, anemia. infections. infertil­ structure (for example. "mab" for monoclonal antibody and
ity. hemorrhagic cystitis and bladder cancer, and lymphoma "cept" for receptor derived).
and other malignancies. Evaluation of patients with painless Toxicities of biologic agents relate mainly to the pathways
hematuria and history of past cyclophosphamide treatment they block. Infection risk is elevated with most biologic
should include cystoscopy to evaluate for bladder cancer. agents; therapy should be temporarily interrupted during any
significant infection. Biologic agents are frequently used in
Mycophenolate Mofetil combination with a nonbiologic DMARD. However, concur­
Mycophenolate mofetil inhibits the purine pathway of nucleo­ rent use of two or more biologic agents is contraindicated
tide synthesis and may be at least as effective as cyclophospha­ because infection rates are increased with minimal, if
mide for SLE (including nephritis) but with fewer. and milder. any, added efficacy. See Vaccination and Screening in
side effects. It is also used to treat autoimmune myositis and as Immunosuppression for more details.
a glucocorticoid-sparing agent in systemic vasculitis. Toxicities Table 11 on page 15 summarizes the structures, targets,
include diarrhea. cytopenias. and infection. Cl indications. and common monitoring parameters of various
biologic DMARDs. See Rheumatologic Medications and
Cyclosporine Pregnancy for information on these medications in women
Cyclosporine is both a calcineurin and T -lymphocyte inhibi­ of childbearing potential.
tor that is efficacious in several autoimmune diseases, includ­

C]
ing rheumatoid arthritis, SLE, autoimmune myositis, psoria­ Tumor Necrosis Factor a Inhibitors
sis, and inflammatory bowel disease. Cyclosporine is mainly Tumor necrosis factor (TNF)-o: inhibitors are usually the treat-
used as a third-line agent in rheumatologic disease because ment of first choice for patients with rheumatoid or psoriatic
its potential toxicities (for example. nephrotoxicity, hyper­ arthritis after inadequate response to nonbiologic DMARDs.
tension. tremors, and hirsutism) require close monitoring. TNF-o: inhibitors also treat ankylosing spondylitis after failure
of NSAIDs. TNF-a. inhibitors are effective in SO'Y.. to 70% of
Apremilast patients with these diseases. Five TNF-o: inhibitors (inflixi-
Apremilast was recently approved by the FDA for treatment of mab. adalimumab. etanercept, certolizumab pegol, and goli­
psoriasis and psoriatic arthritis. Apremilast inhibits phospho­ mumab) are FDA approved to treat rheumatoid arthritis,
diesterase 4, resulting in increases in cyclic adenosine psoriatic arthritis. and/or ankylosing spondylitis. These agents
monophosphate that inhibit inflammatory responses. decrease disease activity and inhibit the progression of

13
Principles of Therapeutics

Neutrophil

MMPsb(Q) �TNF-a

Osteoclast T
lnfliximab RF, anti-CCP
( Etanercept antibodies
Adalimumab
Golimumab
Certilizumab

FIGURE 1 . Biologic targets in rheumatoid arthritis. Various processes within the rheumatoid joint may be targeted by biologic (and nonbiologic) DMARDs. This figure
illustrates the targets of specific biologic agents, described more fully in the text. CCP = cyclic citrullinated peptide; DMARD = disease-modifying antirheumatic drug;
IL= interleukin; LTB4 = leukotriene B,; MMP = matrix metalloproteinase; 0 2 = oxygen; PGE 2 = prostaglandin E 2; RF= rheumatoid factor; TNF = tumor necrosis factor.
�Activated synovial fibroblasts seC1ete MMPs and othe, enzymes that cont,ibute to the degradation ot articular cartilage.

bActivated osteoclasts secrete MMPs and other enzymes that contribute to marginal erosions of bone.

CJ
COf T.
structural damage in rheumatoid ,1rthritis. most ef'fectively in
combination with methotrcx,llc. In psoriasis psoriatic ,irthri
Abatacept
Abatacept interferes with antigen presentation to T cells and is
tis. they suppress both cut,meous and articular disease. In indicated lor moderate to se,,ere rheumatoid arthritis in
anky losing sponclylitis. TNF-a inhibitors improve both axial patients with inadequate response to methotrexate and1or
and peripheral joint symptoms. although radiographic pro TNF-a inhibition. Abatacept may be administered intrave­
gression or spinal disease may continue. They are also used nously or as a subcutaneous injection, Abatacept is associated
olf-label in uveitis. Behc;et syndrome. sarcoidosis. inflamrna with increased risk of infection as well as COPD exacerbation.
Lory bowel dise,1se. and pyoderma syndromes. It may also be associated with an increased risk or lymphoma
Common toxicities include risk or tuberculosis or heJXll it is and lung cancer.
B reactivation as well as fungal (aspergillosis. histoplasmosis.
coccidioidomycosis) ,111d b,1cterial infect ions. Other potential Riwximab
toxicities include injection site and infusion reactions. leuko Rituximab depletes B cells and is used in combination with
penia. induction or autoimmunity (such as drug-induced lupus methotrexate to treat rheumatoid arthritis in patients who have
erythematosus). and hc,1rt L1ilure. Rarer toxicities include pso­ not adequately responded to a TNF-a inhibitor. Rituximab has
riasilorm skin eruption and demyelinating syndromes. CJ recently been shown to be effective in ANCA-associated vasculi­
Despite early concerns, overall cancer incidence with use tis and has been used olT label for SLE and sarcoiclosis. Rituximab
of TNF-a inhibitors does not appear to be increased, with the is administered every 6 months in rheumatoid arthritis or as four
exception of skin cancer. Nonetheless, TNF-a. inhibitors should weekly infusions for induction of remission in vasculitis.
usually be discontinued if the patient develops any malig­ Toxicities include potentially severe infusion reactions: rare
nancy. In patients with a remote history of malignancy, TNF-a cases of progressive multi local leukoencephalopathy have been
inhibitors have been used cautiously without recurrence in a reported. Despite depleting B-cell populations. rituximab has
limited number of patients. not been associated with signilicant increases in infections.

C:I Other Biologic Disease-Modifying Antirheumatic Drugs


Other biologic agents are typically started alter l;.1ilure or one
Tocilizumab
Tocilizumab blocks I L-6 receptors and is used lo treat rhcuma
or two TNF a. inhibitors. although some ,ire also appro,,ed as toid arthritis in patients who have experienced an inadequate
first-line therapies. response to TNF-a, inhibitors. Tocilizumab may be associated

14
Principles of Ther apeutics

TABLE 11. Biologic Disease-Modifying Antirheumatic Drugs


Agent Agent Structure Target Indications Common Monitoring Parameters

Adalimumab Fully humanized monoclonal TNF-o: RA; psoriatic arthritis; TB, fungal, and other infections;
antibody ankylosing spondylitis; CBC, serum creatinine, and LCTs
IBD at baseline; thereafter every 3-6
months
Etanercept Soluble p75 TNF-o: receptor/lgG TNF-o: RA; psoriatic arthritis; TB, fungal, and other infections;
Fe segment chimer ankylosing spondylitis CBC, serum creatinine, and LCTs
at baseline; thereafter every 3-6
months

Certolizumab Fab' segment of monoclonal TNF-o: RA; psoriatic arthritis; TB, fungal, and other infections;
pegol antibody modified by ankylosing spondylitis CBC, serum creatinine, and LCTs
polyethylene glycol strands to at baseline; thereafter every 3-6
reduce immunogenicity months

Golimumab Fully humanized monoclonal TNF-o: RA; psoriatic arthritis; TB, fungal, and other infections;
antibody ankylosing spondylitis CBC, serum creatinine, and LCTs
at baseline; thereafter every 3-6
months

lnfliximab Partially humanized mouse TNF-o: RA; psoriatic arthritis; TB, fungal, and other infections;
monoclonal antibody ankylosing spondylitis; CBC, serum creatinine, and LCTs
IBD at baseline; thereafter every 2-3
months

Abatacept Soluble CTLA4 receptor/lgG Fe T -cell RA TB, fungal, and other infections;
segment chimer costimulation CBC, serum creatinine, and LCTs
at baseline; thereafter every 3-6
months

Rituximab Chimeric (mouse+ human) CD20+ B cells RA; ANCA-associated Infections; lgG levels; CBC,
monoclonal antibody vasculitis chemistries, and LCTs at baseline
and at 2 weeks; thereafter every
3-6 months

Tocilizumab Humanized monoclonal antibody IL-6 receptor RA; JIA; Castleman TB and other infections; CBC,
disease chemistries, and LCTs at baseline
and with each infusion or every
2-3 months; lipid profile every
3-6 months

Belimumab Human monoclonal antibody BLyS/BAFF SLE lgG levels; CBC, chemistries, and
LCTs at baseline and 2 weeks;
thereafter every 3 months

Tofacitinib Orally available small molecule JAK RA TB and other infections; CBC,
agent chemistries, and LCTs at baseline
and every 3 months

Ustekinumab Human monoclonal antibody IL-12/IL-23 Psoriasis; psoriatic Close monitoring for TB and other
arthritis infections; CBC, chemistries,
and LCTs at baseline and every
3 months

Anakinra Recombinant receptor antagonist IL-1 preceptor RA; AOSD; cryopyrin- CBC at baseline and every 3
associated syndromes months

Rilonacept Dual IL-1 preceptors chimerically IL-1 Cryopyrin-associated CBC at baseline and every 3
attached to lgG Fe segment syndromes; refractory months
gout

Canakinumab Fully humanized antibody IL-1 p Cryopyrin-associated CBC at baseline and every 3
syndromes months

AOSD = adult-onset Still disease; BAFF= B-cell-activating factor; BLyS = B-lymphocyte stimulator; CBC= complete blood count; IBD = inflammatory bowel disease; IL= interleu-
kin; JAK = Janus kinase; JIA = juvenile idiopathic arthritis; LCTs = liver chemistry tests; RA= rheumatoid arthritis; SLE = systemic lupus erythematosus; TB= tuberculosis; TNF = tumor
necrosis factor.

15
Principles of Therapeutics

Cl with elevated liver chemistries, leukopenia. thrombocytope­


nia, and elevated serum lipid levels. Reactivation of tuberculo­
COIJl . . .
KEY PO INTS (continued)

• Other biologic agents are typically started after failure


sis anct mvaswe funga I .111,ect1ons
C • can occur, an ct rare cases of
of one or two tumor necrosis factor a inhibitors,
colon or small bowel perforation have been reported in
although some are also approved as first-line therapies.
patients with a history of diverticulitis.

Tofacitinib
Tofacitinib is an oral agent that is FDA approved to treat moder­ Vaccination and Screening
ate to severe rheumatoid arthritis in patients who have experi­ in lmmunosuppression
enced an inadequate response to methotrexate. In contrast to Patients with autoimmune diseases are at increased risk for infec­
protein-based biologics. tofacitinib is the first small molecule, tion, and risk may be significantly increased with the institution
specific signal transduction inhibitor to be used in rheumato­ of immunosuppressive therapies. Accordingly, patients with
logic disease and works by inhibiting Janus kinase (JAK) path­ rheurnatologic disease should be fully vaccinated according to
way signaling. Tofacitinib may be associated with bone marrow general guidelines; specific recommendations for immunocom­
suppression and elevated liver chemistries. Tuberculosis, inva­ promised patients may also apply. Whenever possible, patients
sive fungal infections. and bacterial and viral infections can should be brought up to date on vaccinations prior to starting
occur . Like tocilizumab, tofacitinib can induce lipid abnor­ immunosuppressive agents because vaccine responses may be
malities and carries a risk of intestinal perforation. diminished when on treatment. Nonetheless, immunosuppressed
patients do respond immunologically to vaccines and should not
Ustekinumab
generally be deprived of the opportunity to be vaccinated.
Ustekinumab is an anti-lL-12/IL-23 antibody that is FDA
Patients on nonbiologic DMARDs can receive vaccines of
approved to treat active psoriatic arthritis and moderate to
all types. Because of a theoretical risk of infection in a higher
severe plaque psoriasis. Ustekinumab is administered subcu­
state of immunosuppression, the use of live attenuated vaccines
taneously every 12 weeks. Serious infections are uncommon
(for example, herpes zoster vaccine, yellow fever vaccine) is cur­
but have been reported.
rently contraindicated for patients on biologic therapies.
However, such patients may be administered live attenuated
!11terleuki11-lf3 lnhibitors
vaccines approximately 4 weeks before starting biologic therapy.
Anal<.inra is FDA approved to treat rheumatoid arthritis but is
Prior to initiating aggressive immunosuppressive ther­
infrequently used due to limited efficacy. However. anal<.inra is
apy, the following recommendations for screening have been
approved for a cryopyrin-associated periodic fever syndrome
suggested:
(neonatal-onset multisystem inflammatory disease) and is used
off-label in similar autoinflammatory syndromes such as • Tuberculosis screening with tuberculin skin testing or
Muckle-Wells syndrome, familial cold autoinflammatory syn­ interferon-y release assay, particularly for patients initiat­
drome, and adult-onset Still disease. Ri.lonacept and canal<.i­ ing most biologic therapies
numab are also beneficial in the treatment ofcryopyrin-associated • Hepatitis B and C serologies, particularly for patients
periodic fever syndromes. Anal<.inra and canakinurnab may also initiating potentially hepatotoxic agents (methotrexate,
f
be considered for of-label use in severe cases of gout refractory leflunomide) or TNF-a inhibitor therapy
to other therapies. • HIV screening
Belimumab • Strongyloidiasis screening if patient is from an endemic area
Belimumab is an anti-BLyS (B lymphocyte stimulator) antibody Patients with latent or active tuberculosis, active hepatitis
that is FDA approved to treat active SLE in patients on standard B, or untreated HIV infection require initiation of infection
therapy. Belimumab is generally well tolerated, but cytopenias therapy prior to initiating immunosuppressive therapy. Repeat
and infection may occur. In one trial. belimumab failed to dem­ screening for tuberculosis should be performed annually, with
onstrate efficacy among black patients with SLE. C] similar repeat screening for hepatitis, HIV, and strongyloidiasis
KEY POINTS
in the presence of ongoing risk.

• Infection risk is elevated with most biologic agents; KEY POINTS

therapy should be temporarily interrupted during any • Whenever possible, patients should be brought up to
significant infection. date on vaccinations prior to starting immunosuppres­
• Tumor necrosis factor a inhibitors are usually the sive therapy.
treatment of first choice for patients with rheumatoid • Live attenuated vaccines are currently contraindicated
or psoriatic arthritis after inadequate response to non­ for patients on biologic therapies; however, live attenu­
biologic disease-modifying antirheumatic drugs. ated vaccines may be administered approximately
(Continued) 4 weeks before starting biologic therapy. (Continued)

16
Principles of Therapeutics

KEV PO IN TS (continued) a starting dose of 100 mg/d (SO mg/d in patients with stage 4
• Prior to initiating aggressive immunosuppressive ther­ or 5 chronic kidney disease) with a urate check and titration
apy, screening for infections (tuberculosis, hepatitis B upward every 2 to S weeks, until a target level of 6.0 mg/dL
and C, HIV) is indicated; if needed, infection therapy (0.35 mrnol/L) or less is achieved.
must be started before initiating immunosuppressive
therapy. Febuxostat
Febuxostat is an alternative first-line therapy for urate lower­
ing. In contrast to the purine analogue allopurinol, febuxostat
Urate-Lowering Therapy is a non-purine, non-competitive xanthine oxidase inhibitor
that is more specific than allopurinol. Febuxostat is newer and
The American College of Rheumatology guidelines currently
more expensive than allopurinol. It is less likely to cause
indicate that urate-lowering therapy (ULT) should be initiated
hypersensitivity reactions and may be used in patients who
in patients with gout who have had two or more attacks within
have had adverse reactions to allopurinol. Febuxostat is
a 1-year period, one attack in the setting of chronic kidney
excreted via the gastrointestinal tract and kidneys and needs
disease of stage 2 or worse, one attack with the presence of
no dose adjustment for patients with mild to moderate
tophi visible on examination or imaging, or one attack with a
chronic kidney disease. Febuxostat should be avoided in
history of urolithiasis. ULT reduces total body uric acid bur­
patients taking other purine analogues such as azathioprine
den; the treatment target is a serum urate level of 6.0 mg/dL
or 6-mercaptopurine because toxic levels of either or both
(0.35 mmol/L) or less. Over time, urate lowering reduces risk
drug may ensue.
of future gout attacks and promotes regression of tophi.
Febuxostat is approved at doses of 40 mg/d or 80 mg/d in
Treatment with ULT is usually life-long.
the United States and 120 mg/d in Europe. The 40-mg dose is
Anti-inflammatory prophylaxis to prevent gout attacks is
roughly equivalent to 300-mg allopurinol in efficacy, whereas
recommended when ULT is initiated because of the para­
the 80-mg dose is superior to 300-mg allopurinol. To date,
doxical increased risk of acute gout attacks when serum urate
studies comparing the efficacy of febuxostat versus higher
levels are rapidly decreased by medication. See Crystal
doses of allopurinol are lacking. Patients should be monitored
Arthropathies for more information on gout prophylaxis.
for liver function during therapy with this agent.
Allopurinol
Allopurinol is a first-line agent for serum urate reduction in Probenecid
patients with gout. Allopurinol is a purine analogue that Probenecid is a second-line agent that promotes kidney uric
inhibits xanthine oxidase, the final enzyme in the pathway of acid excretion (uricosuric effect). Probenecid may also be used
urate synthesis from purine precursors. Allopurinol is metab­ in conjunction with a xanthine oxidase inhibitor for synergis­
olized by the liver and renally excreted; its active metabolite tic effect. Probenecid has limited efficacy in patients with a
oxypurinol has a half-life ofl2 to 17 hours, or longer in patients creatinine clearance less than SO mL/min and is relatively
with kidney disease. contraindicated in patients with kidney stones or hyperurico­
Allopurinol is generally well tolerated and should be ini­ suria (indicating uric acid overproduction). Probenecid
tial treatment for patients requiring ULT due to its extensive requires frequent daily dosing and adequate hydration, and it
use history and cost-benefit profile. However, it can very rarely has multiple drug-drug interactions. Kidney function and
provoke a severe hypersensitivity reaction with fever, lym­ complete blood count levels should be monitored.
phadenopathy, widespread erythema, purpura, and skin
necrosis and should therefore be discontinued at the first sign Pegloticase
of a rash. Allopurinol should be avoided in patients taking Humans lack the gene for uricase, which converts uric acid to
other purine analogues such as azathioprine or 6-mercap­ the more soluble allantoin; therefore, repletion of uricase is a
topurine because toxic levels of either or both drug may ensue. potential strategy for lowering uric acid. Pegloticase is a
Patients should be periodically monitored for liver function, recombinant pegylated uricase that dramatically lowers
kidney function, and complete blood count levels during serum urate. Pegloticase is indicated for patients with persis­
aJlopurinol treatment. tent moderate to severe gout who have failed standard ULT.
Allopurinol is FDA approved for dosing up to 800 mg/d. Pegloticase is administered intravenously every 2 weeks; gout
However, the dose most commonly used in routine practice is flares are common, and infusion reactions may occur. Patients
300 mg/d, a dose that is sufficient to achieve the serum urate on pegloticase must eschew other ULTs and should have
target in only half the patients treated. Although concern was serum urate levels checked before each infusion. Increase of
expressed regarding allopurinol dosing and the risk of hyper­ serum urate to greater than 6.0 mg/dL (0.35 mmol/L) sug­
sensitivity, it appears that gradual dose escalation with careful gests failure of the drug due to immunogenicity, which indi­
monitoring is a safe approach, even in patients with kidney cates that the patient is at risk for infusion reactions, and
disease. The American College of Rheumatology recommends requires discontinuation.

17
Principles of Therapeutics

KEY POINTS medicine taught in U.S. medical schools. Nearly 40% of


Americans use CAM, usually as an adjunct to traditional med­
• Orate-lowering therapy is indicated for patients with
ical care. Patients with rheumatologic cliseases and chronic
gout who have had two or more attacks within a 1-year
pain syndromes are more likely to use CAM.
period, one attack in the setting of chronic kidney dis­
ease of stage 2 or worse, one attack with the presence of
Natural Products
tophi visible on examination or imaging, or one attack
Vitamins, minerals, herbal preparations, and probiotics are
with a history of urolithiasis.
labeled as dietary supplements, which permits sale without
• Allopurinol and febuxostat are first-line agents for proof of specific effects or content verification. Although labels
sernm urate reduction; allopurinol is less expensive but on these products commonly make broad statements about
can rarely cause a severe hypersensitivity reaction and particular health benefits, most have not been studied in well­
should be discontinued at the first sign of a rash. controlled trials.
Glucosamine sulfate and chondroitin sulfate are the most
commonly used dietary supplements for arthritis. Earlier tri­
Rheumatologic Medications als suggested analgesic benefit, but recent studies have shown
and Pregnancy little effect on symptoms and/or radiographic disease pro­
gression in osteoarthritis, and current guidelines do not rec­
Some rheumatologic medications have potentially adverse
ommend their use.
effects on pregnancy. Table 12 discusses these agents and their
Omega-3 fatty acids present in fish oil inhibit prostaglan­
relative risks.
din and leukotriene production and reduce inflammation.
KEY POINTS Clinical trials in rheumatoid arthritis show modest benefit.
• Methotrexate is highly teratogenic and abortifacient Numerous herbal products for arthritis are available over
and must be discontinued at least 3 months prior to the counter, including ginger, curcumin , bromelain, evening
conception. primrose and borage oils, feverfew, willow bark. cat's claw. and
Boswellia. Although many of these have been shown to have
• Hydroxychloroquine is relatively safe in pregnancy and
in vitro anti-inflammatory effects. clinical trials are lacking.
should not be cliscontinued.
• Leflunornide is extremely teratogenic and must not be Mind-Body Practices
used before/during pregnancy; if leflunomide is inad­ Mind-body practices include tai chi, yoga, acupuncture. med­
vertently administered, cholestyramine treatment is itation, deep-breathing exercises, guided imagery, hypno­
required to remove the drng from the body before therapy, progressive relaxation, and qi gong. Randomized
pregnancy. controlled trials support the use of tai chi for reducing pain
and enhancing function in patients with osteoarthritis. Some
data suggest that yoga may also be beneficial. Acupuncture
Nonpharmacologic and
has been the subject of numerous studies in osteoarthritis,
Nontraditional Management and modest, short-term reductions in pain and improved
Physical and Occupational Therapy function have been demonstrated. Few studies have assessed
Physical and occupational therapy are integral to the compre­ the other modalities.
hensive management of many types of arthritis. Physical
therapy can help manage pain and functional deficits in Role of Surgery
arthritis. Overuse injuries related to occupational or recrea­ When medications fail to adequately control pain or prevent
tional activities or Joss of strength and flexibility after surgical disability, surgery can be considered. Surgery may be needed
intervention also respond well to graded exercise. Modalities acutely or planned electively in chronic circumstances.
such as local heat or cold, electrical stimulation, and massage Osteoarthritis is the most frequent reason for total joint
can provide analgesia and may enhance the ability of the arthroplasty. The knee, hjp, and, occasionally, the shoulder are
patient to exercise. replaced when end-stage joints cause pain at rest or at night or
Occupational therapy refers not only to rehabilitative when function is compromised to an unacceptable level. Joint
efforts for enhancing workplace functioning but also to ther­ replacements in other locations have less predictable benefit.
apy for the hands and upper extremities, including splinting, Arthroscopy is performed primarily for specific indications
range of motion exercises. and instruction in joint-sparing such as the removal of a loose body or repair of a torn menis­
techniques. cus. Arthroscopy in routine degenerative disease has not been
shown to be superior to sham surgery in controlled trials.
Complementary and Alternative Medicine Arthroscopic synovectomy may be helpful in rheumatoid
Complementary and alternative medicine (CAM) refers to a arthritis when excessive pannus accumulates and does not
diverse group of interventions outside the mainstream of respond to medical management.

18
Principles of Therapeutics

TABLE 12. Rheumatologic Medications and Pregnancy


Medication/Class FDA Pregnancy Comments
Category•
Anti-Inflammatory Agents

NSAIDs Various May impede implantation and may be associated with a small increased risk
of miscarriage when used before 20 weeks' gestation. Use of NSAIDs after 30
weeks' gestation can lead to premature closure of the ductus arteriosus.
Glucocorticoids Various In the first trimester, they can increase the risk of cleft palate in the fetus and
of gestational diabetes in the mother throughout pregnancy. Nonetheless,
they can be useful in the management of active autoimmune disease in
pregnancy. Non-fluorinated glucocorticoids (e.g., prednisone, prednisolone,
methylprednisolone) have limited ability to cross the placenta and may be
preferred, except when treating the fetus (e.g., neonatal lupus erythematosus).
Colchicine C Should be used only if the potential benefit justifies the potential risk to the fetus.

Analgesics

Acetaminophen B Generally considered safe at standard dosing, but does cross the placenta.
Opiates Various Some opiates/opioids cross the placenta; may cause fetal opioid withdrawal at birth.
Tramadol C Should be used only if the potential benefit justifies the potential risk to the
fetus; post-marketing reports suggest the possibility of neonatal seizures,
withdrawal syndrome, and still birth.
Topical agents Varies by agent, Topical use may limit serum levels; individual agents should be reviewed for
concentration, pregnancy impact prior to use.
and vehicle

Nonbiologic DMARDs

Methotrexate X Highly teratogenic and abortifacient; must be discontinued at least 3 months


before pregnancy.

Hydroxychloroquine C Despite a C rating, relatively safe in pregnancy and should not be discontinued.
Sulfasalazine B Relatively safe during pregnancy.
Leflunomide X Extremely teratogenic; must not be used before/during pregnancy;
cholestyramine administration is required to remove the drug from the body in
all women of childbearing potential upon discontinuation and specifically in
those wishing to become pregnant; should be followed up with measurement
of leflunomide and its metabolite levels to ensure removal of the drug.
Azathioprine D Routine use in pregnancy is not recommended; however, despite a D rating,
azathioprine may be safer than some other DMARDs and may be used if an
immunosuppressive agent is imperatively needed.
Cyclophosphamide D Not used in pregnancy unless absolutely necessary.
Mycophenolate mofetil D Teratogenic; should not be used in pregnancy; discontinue for 3 months
before attempting pregnancy.
Cyclosporine C May be used in pregnancy only if benefits outweigh the risks.

Biologic DMARDs

TNF-a inhibitors B Accumulating retrospective data suggest low risk in pregnancy, but evidence is
limited; can be continued if absolutely needed; different agents may have
different considerations regarding crossing the placenta.

Ustekinumab, anakinra B Should be used only if the potential benefit justifies the undefined risk to the fetus.

Abatacept, rituximab, tocilizumab, C Should be used only if the potential benefit justifies the potential risk to the
belimumab, tofacitinib, rilonacept, fetus; tofacitinib may be teratogenic at high doses.
canakinumab

Urate-Lowering Therapy

Allopurinol C Should be used only if the potential benefit justifies the potential risk to the fetus.
Febuxostat C Should be used only if the potential benefit justifies the potential risk to the fetus.
Probenecid B No current evidence for adverse impact on pregnancy.

Pegloticase C Should be used only if the potential benefit justifies the potential risk to the fetus.

DMARD = disease-modifying antirheumatic drug; TNF = tumor necrosis factor.


asee MKSAP 17 General Internal Medicine for information on the FDA pregnancy categories.

19
Rheumatoid Arthritis

KEY POINTS peripheral blood and synovial fluid of patients with RA. These
autoantibodies typically precede clinical disease by years;
• Physical and occupational therapy are integral compo­
their possible role in pathogenesis remains an area of investi­
nents of a comprehensive management plan for many
gation. Although they aid in diagnosis, autoantibodies are
types of arthritis.
neither necessary nor sufficient for diagnosing RA.
• When medications fail to adequately control pain or Rheumatoid factor, an immunoglobulin directed against
prevent disability, surgery can be considered. the Fe portion of lgG, is associated with increased risk of RA
diagnosis as well as erosive and/or more widespread joint dis­
ease. However, some patients with RA lack rheumatoid factor,
Rheumatoid Arthritis and it is also found in other diseases as well as in healthy per­
sons, limiting its diagnostic specificity.
Introduction Anti-CCP antibodies occur less frequently than rheuma­
Rheumatoid arthritis (RA) is a systemic autoimmune disorder of toid factor but have more diagnostic specificity. Anti-CCP
unknown cause that typically presents as a symmetric inflam­ antibodies are directed against citrullinated proteins, includ­
matory polyarthritis. Characteristically affected joints include ing proteins present in inflamed joint tissues, suggesting that
the proximal interphalangeal and metacarpophalangeal joints anti-CCP antibodies may play a pathogenic role. Clinically,
of the hands and feet and the wrists, but other joints can also be the presence of anti-CCP antibodies predicts a greater risk of
involved. Prolonged morning stiffness is common. RA is also erosive disease and radiographic progression.
associated with extra-articular manifestations, including
inflammation of the skin, eyes, pleura, and pericardium. Early Environmental Factors
diagnosis and immunomodulation offer the best opportunity to
Several environmental exposures increase the risk of RA in
avoid permanent joint damage and multisystem complications.
genetically susceptible persons. For individuals carrying the
shared epitope, smoking conveys up to a fivefold increase in
Epidemiology RA risk (more in the presence of rheumatoid factor or anti­
RA affects approximately 1% of the population worldwide. CCP antibodies). Smoking risk may be mediated, in part, by
Women are affected two to three times more often than men. the ability of cigarette smoke to induce protein citrullination
Onset can occur at any age, with peak incidence between ages in the lungs, which may serve as antigens to drive anti-CCP
30 and 60 years. antibody production.
Occupational exposure to silica or asbestos, as well as
Pathophysiology and Risk Factors occupations involving electrical or carpentry work, has also
been associated with increased RA risk.
Genetic Factors
The risk of RA is increased in relatives of affected persons, pri­
Infection
marily as a result of shared genetic factors. Siblings of those
Although it has long been hypothesized that infection could trig­
with RA have at least twice the risk of developing RA as unre­
ger RA, no definitive proof of the role of any individual organism
lated individuals, and offspring of affected persons have about
three times the risk. Based on twin studies, the heritability of exists. Nonetheless, observations about Porphyromonas gingi­
RA is estimated at 60%, suggesting that genetic factors account valis, the pathogenic agent in periodontitis, continue to stimu­
for the majority of disease susceptibility within the population. late investigations. The presence of periodontal disease is
More than 20 risk alleles have been identified, with the epidemiologically linked to RA. Furthermore, P. gingivalis has
largest contribution coming from genes in the major histocom­ the ability to generate citrullinated peptides, suggesting a possi­
patibility complex (MHC) region. Alleles encoding the "shared ble antigenic effect.
epitope" of the HLA-DRBl molecule have the strongest associa­ More recent studies have focused on a possible role for the
tion with RA (especially severe articular disease, multisystem intestinal microbiome in promoting immune responses lead­
disease, and rheumatoid vasculitis). The shared epitope corre­ ing to RA.
sponds to a specific amino acid sequence in the antigen-bind­
ing site of the MHC molecule, suggesting a possible (but Hormones
unproven) role in facilitating the presentation of specific but The increased incidence of RA in women is most evident
currently unidentified antigens. This association of RA with a prior to menopause, suggesting a role for sex hormones in
gene determining a specific antigen response capacity under­ the modulation of disease. However, the relationship
scores the relationship of disease to immune dysregulation. between RA and hormones is complex. Estrogen levels
alone cannot adequately explain the link between gender
Autoantibodies and RA risk because oral contraceptives and postmenopau­
Autoantibodies, including rheumatoid factor and anti-cyclic sal hormone replacement have no predictable effect on
citrullinated peptide (CCP) antibodies, are often present in the disease risk.

20
Rheumatoid Arthritis

KEY POINTS Clinical Manifestations


• Rheumatoid arthritis typically presents as a symmetric Patients with RA characteristically present with pain and
inflammatory polyarthritis affecting small joints and is swelling in multiple (>3) small joints of the hands and/or feet,
associated with prolonged morning stiffness and extra­ along with morning stiffness lasting at least 1 hour. Distal
articular manifestations. interphalangeal joint involvement is distinctly rare. Many
patients have 12 or more affected joints. Rarely, only a single
• Possible risk factors for rheumatoid arthritis include
joint may be initially involved.
genetic and environmental factors, autoantibodies,
Initial symptoms often worsen gradually over weeks to
infection, and hormones.
months; fewer than 10% of patients have an abrupt onset of
disease. RA frequently interferes with activities of daily living,
Diagnosis including occupational and recreational activities. Constitutional
A careful history, physical examination, and assessment of symptoms such as increased fatigue and malaise are common.
clinical manifestations are the first steps in the evaluation of Depression and myalgia may occur and, less often, fever, ano­
a patient with suspected RA. Laboratory and imaging stud­ rexia, and weight loss. Musculoskeletal manifestations are listed
ies are indicated to properly establish the diagnosis. in Table 14.
Differential diagnoses, including infection, should be Physical examination reveals tenderness and swelling of
excluded. The American College of Rheumatology classifi­ the joints, sometimes with warmth and erythema; symmetric
cation criteria provide guidelines for identifying patients joint involvement is common. Joint symmetry refers to
with RA, with the goal of permitting early diagnosis and involvement of the same rank of joints on both sides (for
management (Table 13). example, hand metacarpophalangeal joints), rather than exact

TABLE 13. The 2010 American College of Rheumatology/European League Against Rheumatism Classification
Criteria for Rheumatoid Arthritis
Target Population: newly presenting patients

1) who have at least 1 joint with definite clinical synovitis (swelling)

2) with synovitis not better explained by another disease (rheumatology consult may be indicated)

Classification Criteria for RA (score-based algorithm: add score of categories A-D; a score of i?:6/10 is needed for
classification of a patient as having definite RA; score may change over time)
Score
A) Joint Involvement (swollen or tender)
1 large joint (shoulders, elbows, hips, knees, ankles) 0
2-10 large joints 1
1-3 small joints (with or without involvement of large joints)• 2
4-10 small joints (with or without involvement of large joints) 3
>10 joints (at least 1 small joint) 5
B) Serology (at least 1 test result is needed for classification)
Negative RF and negative anti-CCP antibodies 0
Low-positive RF or low-positive anti-CCP antibodies (>ULN but :53 times ULN) 2
High-positive RF or high-positive anti-CCP antibodies (>3 times ULN) 3
C) Acute Phase Reactants (at least 1 test result is needed for classification)

Normal CRP and normal ESR 0


Abnormal CRP or abnormal ESR 1
D) Duration of Joint Symptoms (by patient self-report)

<6 weeks 0
<!6 weeks 1
CCP= cyclic citrullinated peptide; CRP= (.reactive protein; ESR = erythrocyte sedimentation rate; RA= rheumatoid arthritis; RF= rheumatoid factor; ULN = upper limit of normal
for the laboratory and assay .

.i"Small joints" refers to the metacarpophalangeal joints, proximal interphalangeal joints, second through fifth metatarsophalangeal joints, thumb interphalangeal joints, and wrists.

Adapted with permission from John Wiley & Sons, from Aletaha D, Neogi T, Silman AJ, et al. 2010 Rheumatoid arthritis classification criteria: an American College of Rheumatology/
European League Against Rheumatism collaborative initiative. Arthritis Rheum. 2010 Sep;62(9):2S69-81. [PMID: 20872595] Copyright 2010 American College of Rheumatology.

21
Rheumatoid Arthritis

TABLE 14. Musculoskeletal Manifestations of Rheumatoid Arthritis


Feature Findings Comments

Joint inflammation Morning stiffness; joint tenderness; soft-tissue Assess duration of morning stiffness by
swelling; palpable joint effusion; local asking "How long does it take from when
warmth; pain on active and passive range of you wake up in the morning until you feel
motion as good as you are going to feel for the
rest of the day?"

Distribution of joint involvement Symmetric; initially small joints; progresses DIP joint involvement is uncommon (seen
proximally to larger joints; commonly involves in psoriatic arthritis, osteoarthritis).
MCP, PIP, MTP, and wrist joints

Joint damage Decreased range of motion; contractures; Marginal erosions may be evident earliest
ulnar deviation; subluxation; cervical at the 5th MTP joint. Cartilage degradation
instability; basilar invagination causes joint-space narrowing. Ankylosis
can occur in long-standing disease.

Periarticular involvement Bursitis; tenosynovitis; tendinopathy; swan Olecranon bursitis and rotator cuff
neck and boutonniere deformities; flexion tendinopathy are common. Tenosynovitis
contractures; popliteal (Baker) and ganglion can cause trigger finger but is less
cysts prominent than in spondyloarthritis.
Popliteal (Baker) cysts are contiguous with
the knee joint.
Muscular weakness Disuse atrophy; drug-induced myopathy lnterosseous and quadriceps muscles are
(glucocorticoids and other drugs) common sites of atrophy from disuse.
Decreased bone quality Periarticular osteopenia; generalized loss of Risk of fracture may be underestimated by
bone mineral density; increased risk of bone mineral density alone.
fracture

DIP = distal interphalangeal; MCP= metacarpophalangeal; MTP = metatarsophalangeal; PIP= proximal interphalangeal.

mirroring of involved digits. See Figure 2 for examples of Laboratory Studies


involvement of the hands in RA. Laboratory studies, including rheumatoid factor, anti-CCP anti­
Because viral and other infections can cause transient bodies, and inflammatory markers, can assist in confirming a
symmetric arthritis of small joints, active signs of inflamma­ diagnosis of RA; however. serologies should never be used as the
tion for at least 6 weeks should be documented before diagno­ sole criterion for diagnosis because of limitations in sensitivity
sis of RA is established. and specificity.

FIGURE 2. Involvement of the hands in rheumatoid arthritis. Early rheumatoid arthritis with mild fusiform soft-tissue swelling of the proximal interphalangeal joints (left
panel). Moderate to severe rheumatoid arthritis with synovitis of the metacarpophalangeal joints and swan neck deformities of the second and third digits (center panel).
Severe deforming rheumatoid arthritis with ulnar deviation, multiple rheumatoid nodules, and proximal interphalangeal joint subluxations (right panel).

22
Rheumatoid Arthritis

Rheumatoid factor is approximately 70% sensitive for the


diagnosis of RA. Approximately 50% of patients with RA have
detectable rheumatoid factor at onset, increasing to 60% to
80% in established disease. Conversely, up to 20% of patients
with RA lack rheumatoid factor. Moreover, rheumatoid factor
occurs in other rheumatologic diseases (Sjogren syndrome,
systemic lupus erythematosus, polymyositis, dermatomyosi­
tis); cryoglobulinemia due to hepatitis B or C virus infection;
primary biliary cirrhosis; subacute bacterial endocarditis; and
certain lung diseases (sarcoidosis, B-cell lymphomas).
Rheumatoid factor can also be present in healthy persons.
Thus, the positive predictive value of rheumatoid factor is poor
in populations with a low pretest probability of RA. Testing
patients with fibromyalgia, osteoarthritis, or nonspecific aches
and pains is therefore not recommended because a positive
result is more likely to represent a false positive. Higher titers
of rheumatoid factor are more likely to be associated with RA
as well as with more severe RA disease, multisystem manifes­
tations, and involvement of more joints. However, fluctuations
in rheumatoid factor do not mirror disease activity, and serial
testing lacks clinical utility in established disease.
Anti-CCP antibody testing has similar sensitivity but
superior specificity compared with rheumatoid factor.
Although anti-CCP antibody specificity is reported to be
around 95%, anti-CCP antibodies occasionally occur in
other rheumatologic diseases, active tuberculosis, and FIGURE 3. Hand radiograph showing rheumatoid arthritis. Periarticular osteo­
chronic lung disease. The dual presence of rheumatoid fac­ penia is present at the metacarpophalangeal joints. Marginal erosions are present
tor and anti-CCP antibodies makes a diagnosis of RA sub­ at the second proximal interphalangeal and metacarpophalangeal joints, as well as
stantially more likely. Seronegative RA has an identical the ulnar styloid. Both are characteristic of rheumatoid arthritis and findings that
can aid in diagnosis. Joint-space narrowing (a nonspecific finding) is seen at the
clinical appearance as seropositive RA but is more likely to
second and fifth proximal interphalangeal joints.
occur in men.
Elevation of inflammatory markers such as erythrocyte
sedimentation rate (ESR) or C-reactive protein (CRP) suggests
than radiography for identifying synovitis and erosions.
RA disease activity; however, normal levels do not absolutely However, it may not be more specific for RA diagnosis than
rule out RA activity.
the standard approach.
A normochromic, normocytic anemia and/or thrombo­ MRI is sensitive for identifying erosions, synovitis, and
cytosis may reflect chronic inflammatory RA disease. tenosynovitis, but its value in RA diagnosis and management
Imaging Studies is not established. The American College of Rheumatology
Choosing Wisely list questions the utility of routinely order­
More than half of inadequately treated patients with RA
ing MRI of peripheral joints to monitor rheumatoid arthritis.
develop bone erosions within the first 2 years of disease;
However, MRI can be used in the evaluation of cervical spine
therefore, baseline and subsequent radiographs are indi­
involvement if subluxation or myelopathy is suspected.
cated to aid in the diagnosis and to follow disease progres­
sion. Plain radiographs of the hands, wrists, and/or feet may KEY POINTS
show characteristic findings of periarticular osteopenia and • Patients with rheumatoid arthritis characteristically pre­
marginal (near the edges of the joint) erosions (Figure 3); sent with pain and swelling in multiple (>3) small joints
however, erosive changes may not be evident early in the of the hands and/or feet and prolonged morning stiffness.
disease course. The earliest site of RA erosion of the foot is
• Rheumatoid factor, anti-cyclic citrullinated peptide HVC
classically at the fifth metatarsophalangeal joint (Figure 4).
antibodies, and inflammatory markers assist in con-
In the presence of long-standing inflammation, relatively
firming a diagnosis of rheumatoid arthritis; however,
uniform joint-space narrowing may occur across the entire
serologies should never be used as the sole criterion for
affected joint.
diagnosis and should be avoided in patients with low
Musculoskeletal ultrasonography is increasingly uti­
pretest probability for disease due to the high rate of
lized for RA diagnosis and management. It requires special­
false-positive results. (Continued)
ized training and is operator dependent but is more sensitive

23
Rheumatoid Arthritis

most frequent manifestation. These firm, subcutaneous


masses measure from a few millimeters to several centimeters
and may be mobile or adhere to the underlying periosteum.
Rheumatoid nodules occur alone or in clusters, often on pres­
sure areas such as the elbows, finger joints, ischial and sacral
prominences, occiput, and Achilles tendons (see Figure 2).
Although usually in periarticular locations on extensor sur­
faces, they may appear in any location, including the lungs,
heart, and muscle.
P yoderma gangrenosum is also seen in RA. It usually
occurs as a single painful lesion on the lower extremities,
beginning as a tender erythematous or violaceous papule and
rapidly expanding into a purulent, necrotic, nonhealing ulcer
(see MKSAP 17 Dermatology, Cutaneous Manifestations of
Internal Disease).
Rheumatoid vasculitis is a late complication of RA that
affects small and medium vessels and may involve the skin
and other organs. It is most common in seropositive male
patients with long-standing disease. Small-vessel involve­
ment presents as purpura, petechiae, splinter hemorrhages,
nailfold infarctions, and peripheral neuropathy. In medium­
vessel disease, nodules, ulcerations, livedo reticularis, and
digital infarcts can occur.

Eyes
The most common eye manifestation of RA is keratoconjunctivi­
tis sicca, as is also seen in Sjogren syndrome and systemic lupus
FIGURE 4. Foot radiograph showing rheumatoid arthritis. Characteristic erythematosus. Episcleritis occurs with more severe RA disease
changes of rheumatoid arthritis are frequently seen in the small joints of the feet, activity, appears acutely, and causes eye redness and pain; changes
as seen here with severe erosive change at the fifth metatarsophalangeal joint and in vision rarely occur. Scleritis, uveitis, ulcerative keratitis, and
marginal erosions at the first and second metatarsophalangeal joints and the first corneal filamentary keratitis also occur during more severe dis­
interphalangeal joint. Nonspecific cystic changes in bone may be seen in many dif·
ease and may lead to visual impairment. See Eye Disorders in
ferent forms of arthritis.
MKSAP 17 General Internal Medicine for more information.

KEV PO IN TS (continued) Pulmonary Involvement


HIIC • Plain radiography of the hands, wrists, and/or feet is Pleuritis is the most common RA pulmonary manifestation
indicated to aid in the diagnosis and to follow progression but is frequently asymptomatic. Exudative pleural effusions
of rheumatoid arthritis; in contrast, MRI of peripheral may occur. Rheumatoid nodules in the lungs can be difficult
joints should not be routinely performed to monitor to diagnose because they are often peripheral in location and
disease progression. usually measure less than 1 cm in diameter. Interstitial lung
disease occurs in up to 10% of patients, particularly in male
smokers with long-standing, seropositive disease.
Complications and
Extra-Articular Manifestations Cardiac Involvement
Joints RA is an independent risk factor for both coronary artery dis­
In the absence of appropriate immunosuppressive therapy, ease and heart failure; patients with severe extra-articular
patients can develop increasing numbers of swollen and ten­ disease are at particularly increased risk of cardiovascular
der joints, subluxation and malalignment, tenosynovitis and death. Pericarditis is common but is often asymptomatic.
ligamentous laxity, reduced grip strength, loss of range of Rarely, pericarditis is severe, unresponsive to glucocorticoids,
motion, and loss of function. Permanent joint damage and and may be restrictive. Successful treatment with disease­
radiographic abnormalities can occur within the first year. modifying agents appears to reduce cardiac comorbidity.

Skin Other Complications


RA may affect various organs in addition to joints. The skin is Felty syndrome is a rare complication occurring in patients
most commonly affected, with rheumatoid nodules being the with severe, erosive, seropositive, long-standing RA. Felty

24
Rheumatoid Arthritis

syndrome is characterized by neutropenia (absolute neutro­ Once a diagnosis of RA is established, management with
phil count <2000/µL [2.0 x 109 /L]) and splenomegaly, often disease-modifying antirheumatic drugs (DMARDs) is man­
accompanied by fever, anemia, thrombocytopenia, and/or dated to minimize damage and reduce disability. The initial
vasculitis, and can predispose to recurrent bacterial infec­ goal of treatment is to rapidly reduce disease activity, but the
tions. Felty syndrome has limited responsiveness to disease­ ultimate goal is remission (absence of signs and symptoms
modifying drugs and may require use of granulocyte colony­ of significant inflammatory activity). This "treat-to-target"
stimulating factor. approach requires treatment protocols and regular follow-up
Unusual complications of long-standing, severe RA visits to reassess clinical status and response to changes in
include mesangioproliferative glomerulonephritis, amyloido­ medication. Therapy is advanced at each follow-up visit until
sis, atlantoaxial subluxation due to erosion of the odontoid the therapeutic target is reached. The usual interval of follow­
process, and peripheral neuropathy. up is every 2 to 3 months to reassess whether goals are being
met and toxicity is present. Patients may be seen by rheuma­
KEY POINTS
tologists more frequently if disease activity is high or less often
• Extra-articular manifestations of rheumatoid arthritis (for example, every 6 to 12 months) if remission has been
include rheumatoid nodules, rheumatoid vasculitis, achieved. In long-standing refractory disease, low disease
keratoconjunctivitis sicca, pleuritis, and pericarditis. activity may be an acceptable alternative target, and response­
• Rheumatoid arthritis is an independent risk factor for limiting factors such as the presence of structural damage,
coronary artery disease and heart failure; patients with functional impairment, medical comorbidities, drug side­
severe extra-articular disease are at particularly effect risks, and other individual patient-related factors must
increased risk of cardiovascular death. be taken into account.
Disease activity is assessed based on history, physical
examination, and the use of composite disease activity scores
Management that incorporate findings such as the number of tender or swol­
General Considerations len joints and laboratory studies such as ESR or CRP. Composite
RA management often requires a team approach to comprehen­ scores provide a more comprehensive and standardized assess­
sively provide optimal care of the joints and other organs and to ment that can account for variability between patients and
address psychosocial needs. The American College of within the course of disease for a single patient. Furthermore,
Rheumatology treatment recommendations for early and estab­ treatment targets can be expressed as a single number facilitat­
lished RA are presented in Figure 5 and Figure 6, respectively. ing comparisons over time. In clinical trials, the most

Early RA
Low High
Disease activity•

Target low
disease
activity or Without Features of poor With Without Features of poor With
remission prognosis b prognosis b

Combination DMARD Anti-TNF with or without MTX


DMARD DMARD therapy monotherapy OR
monotherapy (double and triple OR Combination DMARD therapy
therapy) HCQ and MTX (double and triple therapy)

FIGURE 5. 2012 American College of Rheumatology recommendations update for the treatment of early rheumatoid arthritis, defined as a disease duration <6 months.
DMARDs include hydroxychloroquine, leflunomide, methotrexate, minocycline, and sulfasalazine. DMARD= disease-modifying antirheumatic drug; HCO = hydroxychloroquine;
MTX= methotrexate; RA= rheumatoid arthritis; TNF = tumor necrosis factor.
'Definitions of disease activity are available at http://onlinelibrary.wiley.com/journal/10.1001/(ISSN)2151-4658) and were categorized as low, moderate. or high.

hPatients were categorized based on the presence or absence of 1 or more of the following poor prognostic features: functional limitatlon (e.g., Health Assessment Questionnaire score or simila, valid tools), extra-articular disease (e.g., pres­
ence of rheumatoid nodules, RA vasculitis, Felty syndrome), positive rheumatoid factor or anti-cyclic citrullinated peptide antibodies, and bony erosions by radiograph.
Adapted with permission from John Wiley & Sons, from Singh JA, Furst DE. Bharat A, et al. 2012 update of the 2008 American College of Rheumatology recommendations for the use of disease-modifying antirheumatic drugs and biologic
agents in the treatment of rheumatoid arthritis. Mhritis Care Res (Hoboken). 2012;64(5):625-639. IPMID: 22473917) Copyright 2011 American College of Rheumatology.

25
Rheumatoid Arthritis

Established RA
Low disease
Low disease activity with poor
activity without poor prognosis
prognosis• OR
moderate/high
disease
activity•

MTX monotherapy OR combination DMARD therapy


DMARD monotherapy
(including double or triple therapy)

Add MTX, HCQ, or


LEF (as appropriate)

Target low
disease
activity or Add or switch to
remission Add OR switch to anti-TNF biologic
abatacept OR rituximab

Reassess Reassess
If
OR OR
serious adverse
if nonserious adverse if any adverse
event
event event

Switch to a non-TNF biologic Switch to anti-TNF biologic OR non-TNF biologic

Switch to another type or category of anti-TNF or non-TNF biologic

FIGURE 6. 2012 American College of Rheumatology (ACR) recommendations update for the treatment of established rheumatoid arthritis (RA), defined as a disease dura·
tion >6 months or meeting the 1987 ACR classification criteria. Depending on a patient's current medication regimen, the management algorithm may begin at an appropri­
ate rectangle in the figure, rather than only at the top of the figure. DMARDs include HCO, LEF, MTX, minocycline, and sulfasalazine (therapies are listed alphabetically; azathi­
oprine and cyclosporine were considered but not included). DMARD monotherapy refers to treatment in most instances with HCQ, LEF, MTX, or sulfasalazine; in few instances,
where appropriate, minocycline may also be used. Anti-TNF biologics include adalimumab, certolizumab pegol, etanercept, infliximab, and golimumab. Non-TNF biologics
include abatacept, rituximab, or tocilizumab (therapies are listed alphabetically). DMARD = disease-modifying anti rheumatic drug; HCO = hydroxychloroquine; LEF = leflunomide;
MTX= methotrexate; RA= rheumatoid arthritis; TNF = tumor necrosis factor.
'Definitions of disease activity are available at http://onlinelibrary.wiley.com/journal/10.1002/(ISSN)2151-4658) and were categorized as low, moderate, or high. Features of poor prognosis included the presence of 1 or more of the follow·
ing: functional limitation (e.g., Health Assessment Questionnaire score or similar valid tools), extra·articular disease (e.g., presence of rheumatoid nodules, RA vasculitis, Felty syndrome), positive rheumatoid factor or anti-cyclic citrullinated
peptide antibodies, and bony erosions by radiograph.

Adapted with permission from John Wiley & Sons, from Singh JA, Furst OE, Bharat A, et al. 2012 update of the 2008 American College of Rheumatology recommendations for the use of disease-modifying antirheumatic drugs and biologic
agents in the treatment of rheumatoid arthritis. Arthritis Care Res (Hoboken). 2012;64(5):625-639. [PMID: 224739171 Copyright 2012 American College of Rheumatology.

frequently used disease assessment instrument has been the to <3.2 (see Figure 5), and high disease activity based on DAS is
Disease Activity Score 28 (DAS28), which incorporates the >5.1 (see Figure 6). Other disease activity scales to determine
number of tender and swollen joints out of a predetermined set the appropriate level of treatment are available.
of 28 joints, a weighted ESR or CRP, and a global health assess­ Patient education is fundamental to managing RA and
ment. An example of low disease activity based on DAS is �2.6 includes discussing medications and side effects, advising on

26
Rheumatoid Arthritis

lifestyle choices (diet, weight loss, exercise), and assessing leading to cartilage degradation. In addition, TNF-a increases
psychosocial needs. Physical and occupational therapy may be bone resorption, inhibits proteoglycan synthesis, and increases
appropriate throughout the course of RA. Appropriate exercise expression of adhesion molecules, thus enhancing inflamma­
can strengthen and protect joints and is an important adjunct tory cell recruitment as well as production of additional pro­
in combating fatigue and optimizing function. Smoking cessa­ inflammatory cytokines and arachidonic acid metabolites.
tion is important not only because of its link to disease risk, TNF-a inhibitors are highly effective for treating RA, lead­
but also because continued smoking may impair the response ing to rapid (weeks), significant improvement in signs and
to therapy and exacerbate rheumatoid lung disease. symptoms for most patients. Their use is associated with
increased likelihood of achieving remission in both new-onset
Disease-Modifying Antirheumatic Drugs and established disease, reduction in radiographic progres­
Principles, toxicities, baseline evaluation and monitoring, use sion, normalization of acute phase reactants, and reduced
of vaccinations, and pregnancy issues associated with medica­ cardiovascular risk. Their efficacy is enhanced when used in
tions used in RA are discussed in Principles of Therapeutics. combination with methotrexate.
Other biologic agents target other proinflammatory
Nonbiologic Disease-Modifying Antirheumatic Drugs cytokines and pathways. Tocilzumab is a monoclonal antibody
Methotrexate is the recommended initial DMARD for most that neutralizes IL-6, a cytokine that activates T cells, B cells,
patients with RA and is appropriate at disease onset as well as macrophages, osteoclasts, and the hepatic acute phase
in patients whose disease is well established. It is generally response. Abatacept blocks necessary second signals between
continued indefinitely and can be used alone or in combina­ antigen-presenting cells and T cells during antigen presenta­
tion with biologic DMARDs. Methotrexate is usually well toler­ tion, thereby blocking T -cell activation. Rituximab is a mono­
ated and has good efficacy, high long-term compliance rates, clonal antibody that depletes B-cell populations, leading to a
and relatively low cost but requires regular monitoring. reduction in B-cell cytokine production, B-cell help for T -cell
Methotrexate or another nonbiologic DMARD should be tried activation, and the production of a number of autoantibodies,
before initiating therapy with a considerably more expensive including rheumatoid factor and anti-CCP antibodies. All of
biologic DMARD. Generally, the dose of methotrexate is these agents have efficacy similar to TNF-a inhibitors.
titrated up until the treatment target is achieved. If the Tofacitinib is a new oral agent and the first small molecule
response is below target at an oral dose of 25 mg weekly, clini­ therapy for RA that specifically targets an intracellular signal­
cians may switch to parenteral administration since absorp­ ing molecule in immune/inflammatory pathways. Tofacitinib
tion and efficacy may improve. However, if toxicity or lack of inhibits Janus-associated kinases, which signal in response to
efficacy is encountered at this dose, the use of a biologic agent membrane cytokine receptors to activate STATs (signal trans­
is indicated. ducers and activators of transcription). Tofacitinib appears to
Hydroxychloroquine and sulfasalazine have long-acting have efficacy similar to biologic agents.
effects and may be used alone, together, or in combination with
methotrexate. Triple therapy with these three agents has a rea­ Glucocorticoids
sonable side-effect profile and can be highly efficacious in Oral or intra-articular glucocorticoids are used as adjunctive
symptom control and in reducing the risk of structural damage. therapy in RA. Glucocorticoids act rapidly to control inflamma­
Leflunomide may be used with or as a substitute for tion and joint symptoms and can be useful until slower-acting
methotrexate. It is appropriate as a first choice in the treat­ DMARDs achieve full effect. Glucocorticoids are also used to
ment of RA or as an alternative to methotrexate if side effects manage intermittent flares in patients already taking other
or other considerations limit methotrexate use. agents. Regular, frequent use of glucocorticoids should signal
the need for increasing the DMARD dose or adding/switching
Biologic Disease-Modifying Antirheumatic Drugs DMARDs. Given their many side effects, glucocorticoids should
The decision to add a biologic agent should be made based on be employed at the lowest dose and shortest period possible
inadequate response to a nonbiologic DMARD (especially and should never be used as standing monotherapy.
methotrexate) while balancing the patient-specific risks of
biologic therapy. Biologic agents are commonly added to NSAIDs
methotrexate but are not used in combination with other bio­ NSA!Ds can ameliorate RA joint symptoms. However, NSA!Ds
logics because of an unacceptable increase in infection with­ lack disease-modifying activity, do not alter the destructive
out added efficacy. course of untreated RA, and should never be used as mono­
Tumor necrosis factor (TNF)-a inhibitors are the most therapy.
widely used biologic agents for RA and are available in both
intravenous and subcutaneously administered injections. All Surgical Therapy
TNF-a inhibitors interfere with the actions of TNF-a, a major Surgical interventions are available to patients with RA com­
proinflammatory cytokine in RA pathogenesis. TNF-a stimu­ plications or adv�rnced disease unresponsive to pharmacologic
lates synovial cell proliferation and synthesis of collagenase, therapy. Large joint (hip. knee) replacement is an option for

27
Osteoarthritis

Cl
COIIT.
intractable disease unresponsive to medical management.
Patients with pain at resl or night pain are particularly appro­
KEY POINTS

• Methotrexate and leflunomide are absolutely contrain­


priate for total joint replacement. Patients with RA have a dicated in pregnancy and must be discontinued prior to
higher risk of prosthetic joint infection than patients having
conception.
total joinl arthroplasty for other indications. Repair proce­
f
dures for tendon rnptures and rotator cuf disease are occa­ • Both hydroxychloroquine and sulfasalazine are consid­
sionally needed. Degenerative disk disease may be accelerated ered relatively safe in pregnancy.
in the presence of RA, and spinal procedures such as laminec­
tomy may be indicated. Improvements in pharmacologic man­
agement have made surgical approaches to RA increasingly
uncommon. Osteoarthritis
Patients with RA undergoing general anesthesia for any
kind of surgery should have cervical spine radiography with
Introduction
flexion and extension views to assess for atlantoaxial subluxa­ Osteoarthritis (OA) is characterized by loss of cartilage
tion. which rarely can lead to neurologic compromise when accompanied by reactive bony changes, including osteo­
the neck is extended during intubation. Cl phyte formation, subchondral bony sclerosis, and sub­
chondral cysts. OA typically affects the knees, hips, hands,
KEY POINTS
spine, and feet. Pain and loss of function are the hallmark
• Methotrexate is the recommended initial disease­ features; however, OA is variable in its clinical presenta­
modifying antirheumatic drug for most patients with tion, which can range from asymptomatic radiographic
rheumatoid arthritis. changes to severe and disabling pain and permanent
• The use of tumor necrosis factor a inhibitors or other impairment and deformity.
biologic agents in patients with rheumatoid arthritis can
achieve remission in new-onset and established disease,
reduce risk of radiographic progression, normalize acute Epidemiology
phase reactants, and reduce cardiovascular risk. OA is the most common form of arthritis, affecting at least
30 million persons in the United States. Prevalence increases
with age; OA among younger populations is frequently
related to a history of recreational or occupational injury in a
Pregnancy and specific joint or to a strong genetic predisposition. Whereas
Rheumatoid Arthritis early OA is slightly more common in men, at older ages
Nulliparity has been suggested as a risk factor for RA, but data women are slightly more frequently affected. Overall, radio­
are conflicting. During pregnancy, patients with RA may expe­ graphic knee OA is present in about 35% of adults over the
rience relative disease quiescence, but disease may flare post­ age of 60, and radiographic hand OA is present in 55% to
partum. There is also evidence that the incidence of RA is 65%. OA is the most common cause of disability in patients
increased in the year after delivery, particularly after the first over age 65.
pregnancy. Breastfeeding may decrease the risk.
Methotrexate and leflunomide are absolutely contraindi­
cated in pregnancy and must be discontinued prior to concep­ Pathophysiology
tion. Limited case studies suggest that use of TNF-a inhibitors OA is the consequence of biomechanical and biochemical
during pregnancy may be safe, but a relationship to rare birth processes involving the cartilage, synovium , bone, and soft
defects has been raised by a single report. Decisions regarding tissues surrounding the joint. These processes can be initi­
the use of any biologic agent in pregnancy should incorporate ated by a single traumatic event, repeated microtrauma, or
risk-benefit analysis. Both hydroxychloroquine and sulfasala­ genetic, metabolic, or systemic factors affecting the integrity
zine are considered relatively safe in pregnancy. Non-fluorinated of cartilage. During OA progression, matrix metalloprotein­
glucocorticoids such as prednisone, prednisolone, or methyl­ ases (MMPs), including collagenase, stromelysin, and gelati­
prednisolone have limited ability to cross the placenta and may nase, are secreted by chondrocytes and degrade cartilage
be preferred. NSA!Ds may impede implantation and may be collagen. Inappropriately low levels of tissue inhibitors of
associated with a small increased risk of miscarriage when used metalloproteinases (TIMPs) may reduce the ability of carti­
prior to 20 weeks' gestation. Use of NSA!Ds after 30 weeks' ges­ lage to resist MMPs.
tation can lead to premature closure of the ductus arteriosus. Inflammatory cytokines such as interleukin (IL)-1� may
See Rheumatologic Disease and Pregnancy in Principles induce MMP production and suppress collagen production.
of Therapeutics for more information on these and other Ineffective repair responses within the joint may be mediated
medications and their role in pregnancy. by insulin-like growth factor and transforming growth factor�,

28
Osteoarthritis

leading to abnormal bone growth, including osteophyte for­ bone in the affected joints. Patients with erosive OA are
mation and subchondral sclerosis. more likely to have pain and disability than those without
erosive features. In contrast to rheumatoid arthritis, erosive
OA is common in the distal interphalangeal joints, does not
Risk Factors typically affect the wrists or elbows, and is not associated
Risk factors for OA include advanced age, female gender, with rheumatoid factor, anti-cyclic citrullinated peptide
obesity, and joint injury caused by repetitive use, trauma, or antibodies, or an elevated erythrocyte sedimentation rate or
certain occupations. Joint malalignment, ligamentous lax­ C-reactive protein level.
ity, meniscal injury, or surgical meniscectomy can acceler­ Prevalence of erosive OA is approximately 3% in the gen­
ate the onset of OA, as can quadriceps weakness and defects eral population over the age of 55 years and 10% among those
in proprioception, all resulting in abnormal joint forces. with any symptomatic hand OA. Men and women are equally
Family history may indicate risk; a multitude of genetic affected.
variants has been associated with various OA subsets, but
no single gene has emerged as definitively predicting onset
Secondary Osteoarthritis
or severity.
Secondary OA indicates joint degeneration in the setting of
OA risk varies depending on the joint involved. The life­
preexisting joint abnormality. It may occur in the setting of
time risk for symptomatic knee OA is approximately 45%, with
trauma or congenital anatomic abnormalities (such as hip
a higher risk after job- or sports-related injury or overuse or
dysplasia). Secondary OA may follow inflammatory arthritis
among those who are obese. Lifetime risk of symptomatic hip
(such as rheumatoid or gouty arthritis or calcium pyrophos­
OA is approximately 27%, is higher in women, and is less
phate deposition), avascular necrosis, infectious arthritis,
strongly related to obesity and trauma compared with knee
Paget disease, osteopetrosis (congenitally increased bone
OA. Hand OA risk is increased on the basis of genetic predis­
mass and skeletal fragility), or osteochondritis dissecans (in
position and is more common in women. Manual labor and
which a portion of bone and cartilage separates from the sur­
obesity confer additional risk for hand OA.
rounding bone). Finally, secondary OA may occur in the set­
KEY POINTS ting of metabolic or systemic diseases such as hemochroma­
• Osteoarthritis is characterized by loss of cartilage and tosis (iron overload, which is associated with a characteristic
the presence of osteophytes, subchondral bony sclero­ OA pattern involving the second and third metacarpophalan­
sis, and subchondral cysts. geal joints and is diagnosed with the aid of transferrin satura­
• Risk factors for osteoarthritis include advanced age, tion measurement), ochronosis (excessive accumulation of
female gender, obesity, and acute or chronic joint injury. homogentisic acid), Gaucher disease, hemoglobinopathy, or
Ehlers-Danlos syndrome.

Classification Diffuse Idiopathic Skeletal Hyperostosis


Primary Osteoarthritis Diffuse idiopathic skeletal hyperostosis (DISH) is often consid­
Primary (idiopathic) OA constitutes most cases of OA, ered in the same context as OA by virtue of its lack of systemic
which are designated as such when no specific anteced­ inflammation, the presence of characteristic bony remodeling
ent event or predisposing disease is present. The designa­ changes, a similar epidemiology, and its frequent coexpression
tion of primary OA does not exclude the impact of rou­ with OA. DISH is diagnosed on plain radiograph by the pres­
tine factors such as obesity, aging, or a chronic history of ence of flowing osteophytes involving the anterolateral aspect
significant but non-injurious joint use. Primary OA typi­ of four or more contiguous vertebrae most easily detected in
cally affects the knees, hips, hands, spine, and feet and the thoracic spine (Figure 7). Intervertebral disk spaces are
may be localized (a single or few joints) or generalized typically preserved; apophyseal or sacroiliac joint inflamma­
(multiple joints). It typically becomes clinically evident tory changes are absent. Peripheral enthesitis and ossification
around the age of 55 years. of ligaments in nonvertebral locations may occur. DISH is dis­
tinguished from primary OA of the spine (with which it may
Erosive Osteoarthritis co-occur) by the manner in which the flowing osteophytes
Erosive OA is a subset of primary OA in which radiographic bridge the vertebrae, by its anatomic location, and by the rela­
erosions are seen. Erosive OA typically involves the inter­ tive preservation of the disk spaces when OA is absent.
phalangeal joints of the fingers and is associated with inter­ Complications include dysphagia, unstable spinal fractures,
mittent flares of swelling and redness of the affected joints. spinal stenosis, postsurgical heterotropic ossifications, difficult
Diagnosis is radiographic, based on the presence of central intubation, difficult gastroscopy, aspiration pneumonia, and
erosions (contrasting with the marginal erosions of rheuma­ myelopathy. Radiographically, DISH may be difficult to differ­
toid and psoriatic arthritis) and collapse of the subchondral entiate from ankylosing spondylitis (see Spondyloarthritis).

29
Osteoarthritis

Diagnosis
Clinical Manifestations
A thorough history and physical examination are warranted in
the initial evaluation of the patient with suspected OA. Patients
with symptomatic OA describe pain and stiffness in the
affected joint(s). Pain may occur intermittently and be associ­
ated with joint use. Patients often report a decline in function
associated with the pain. Stiffness may worsen with rest and
improve with activity, but typically lasts for only brief periods
(less than 30 minutes).
On physical examination, affected joints may be tender,
or the patient may report discomfort only when the joint is put
through a range of motion (often reduced due to pain and/or
deformity). Joint effusions may occur; erythema and warmth
are less common than in inflammatory arthritis. More
advanced disease may be accompanied by change in the phys­
ical appearance of the joints, manifesting as bony enlargement
and/or changes in alignment. The most commonly affected
joints are the first carpometacarpal and distal and proximal
interphalangeal joints of the hands, knees, hips, and apophy­
seal joints of the spine.
Hand OA typically presents with pain, aching, or stiffness
for most days of the prior month. Bony enlargement of a distal
interphalangeal joint is termed a Heberden node (Figure 8);
similar enlargement in a proximal interphalangeal joint is
termed a Bouchard node. Occasionally, an individual joint in
FIGURE 7. Diffuse idiopathic skeletal hyperostosis. This disorder is character­ the hand may be swollen or erythematous (inflammatory or
ized by calcification of the enthesis regions (where the tendons or ligaments insert erosive OA), but the presence of multiple swollen joints, par­
into bone) and the spinal ligaments. The diagnosis is confirmed on radiograph by ticularly proximal joints, should lead to consideration of con­
the presence of flowing osteophytes along the anterolateral aspect of at least four comitant calcium pyrophosphate deposition or rheumatoid or
contiguous vertebral bodies (most easily detected in the thoracic spine), preserved
vertebral height, and absent findings typical for ankylosing spondylitis.
psoriatic arthritis.
Diagnosis of knee or hip OA is established in the presence
of knee or groin pain for most days of the prior month. Hip
The prevalence of DISH increases with age and is approx­ involvement can occasionally result in referred pain that is
imately 15% in patients over 50 years of age. In contrast to OA, perceived by the patient as knee pain. Crepitus on active joint
DISH is twice as common in men than in women.
KEY POINTS

• Primary osteoarthritis (OA) constitutes most cases of


OA, which are designated as such when no clear ante­
cedent event or predisposing disease is present.
• Erosive hand osteoarthritis is associated with intermit­
tent flares of swelling and redness of the affected joints
as well as the presence of central erosions and collapse
of subchondral bone in the interphalangeal joints as
seen on plain radiographs.
• Secondary osteoarthritis can occur in settings of trauma
and metabolic or systemic disease or following joint
damage from inflammatory or infectious arthritis.
• Diffuse idiopathic skeletal hyperostosis is characterized
by flowing osteophytes involving the anterolateral
aspect of four or more contiguous vertebrae most easily
detected in the thoracic spine. FIGURE 8. Heberden nodes in osteoarthritis are bony spurs at the dorsolateral
and medial aspects of the distal interphalangeal joints.

30
Osteoarthritis

motion, limited range of motion, and morning stiffness lasting


less than 30 minutes are typical. Bony enlargement of the knee
is common. Knee effusions, which are usually cool, are also
common. Chronic effusions may be associated with the for­
mation of popliteal fossa fluid collections (Baker cysts). Knee
malalignment is common and may result in a "knocked knee"
or "bow-legged" appearance. Long-standing kneeOA is asso­
ciated with quadriceps muscle atrophy.
OA of the spine is a common source of back pain and is
often seen in association with degenerative disk disease.
Physical examination may demonstrate reduced range of
motion and associated muscle spasm; spinal tenderness is
rarely present.

Laboratory and Imaging Studies


No specific laboratory abnormalities are associated with pri­
maryOA. In most cases, laboratory studies are indicated only
when needed to rule out other diagnoses. If NSAID use is
contemplated, it is appropriate to first evaluate kidney func­
tion. Although arthrocentesis is not mandatory for diagnosing
OA, synovial fluid examination should be considered if FIGURE 1 0. Plain radiograph of knee medial compartment osteoarthritis
showing joint-space narrowing and subchondral sclerosis (arrow).
needed to help rule out inflammatory arthritis (rheumatoid
arthritis, crystal-induced disease), hemarthrosis, or infection.
Noninflammatory synovial fluid (leukocyte count <2000/µL radiographs as increased bone density subjacent to joint­
[2.0 x 109/L]) is typical ofOA. space narrowing. Both subchondral sclerosis and osteophytes
Joint-space narrowing (articular cartilage loss), sub­ reflect reaction of bone to growth factors stimulated by the
chondral sclerosis, and marginal osteophyte formation are the mechanical changes driving OA. In erosive hand OA, central
radiographic hallmarks of OA (Figure 9); periarticular osteo­ erosions are visible on radiograph. However, plain radiogra­
penia and marginal erosions (as seen in rheumatoid arthritis) phy is insensitive for the detection of earlyOA, andOA sever­
are absent. InOA of the knee and hip, joint-space narrowing ity on plain radiographs is inconsistently associated with pain
characteristically occurs mainly on the weight-bearing areas and other symptoms.
of the joint, distinguishing it from inflammatory arthritis MRI and ultrasonography are currently being investigated
(Figure 10). Subchondral sclerosis is seen on plain as potential tools inOA diagnosis and management.

Differential Diagnosis
Polyarthritis of the proximal hand digits may warrant exclu­
sion of rheumatoid arthritis because bothOA and rheumatoid
arthritis can affect these joints. Whereas rheumatoid arthritis
rarely, if ever, affects the distal interphalangeal joints, psoriatic
arthritis may affect both proximal and distal joints, but tends
to affect these joints more globally thanOA. The presence of
extensive inflammation and/or morning stiffness, a positive
rheumatoid factor, anti-cyclic citrullinated peptide antibod­
ies, elevated erythrocyte sedimentation rate or C-reactive pro­
tein, anemia of chronic disease or thrombocytosis without
other explanation, or inflammatory synovial fluid also favor
the diagnosis of rheumatoid arthritis.
Calcium pyrophosphate deposition can co-occur with
OA, may be related to severe presentations of OA, and can
promote inflammation that is difficult to distinguish from
rheumatoid arthritis when metacarpophalangeal joint and
wrist involvement is present. It can be a cause of acute mono­
FIGURE 9. Plain radiograph of hand osteoarthritis showing joint-space narrow­
articular or oligoarticular arthritis, particularly in the knee.
ing, subchondral sclerosis, and osteophyte formation involving the distal inter­ Joints may be chronically painful or abruptly warm and swol­
phalangeal and proximal interphalangeal joints. len, and large knee effusions can result.

31
Osteoarthritis

KEY POINTS equipment in bathrooms, can relieve symptoms and enhance


functioning. Chronic bracing can be used for severely affected
HUC • Laboratory studies are indicated only when needed to
joints that may be unstable, particularly when patients are
rule out other diagnoses in patients with primary osteo­
poor candidates for surgery.
arthritis (OA); the diagnostic role for MRI and ultra­
sonography in OA has not been established. Pharmacologic Therapy
• Joint-space narrowing, subchondral sclerosis, and mar­ Because no disease-modifying agents are currently available
ginal osteophyte formation are the radiographic hall­ for the treatment of OA, pharmacologic options address the
marks of osteoarthritis; periarticular osteopenia and symptoms of OA rather than its underlying cause. Most treat­
marginal erosions (as seen in rheumatoid arthritis) are ment guidelines suggest the initial use of acetaminophen for
absent. pain control, with recognition of potential hepatotoxicity
• Polyarthritis of the proximal hand digits may warrant leading to some recommendations limiting the total daily
exclusion of rheumatoid arthritis because both osteoar­ dose to approximately �3 g/d. If acetaminophen proves inad­
thritis and rheumatoid arthritis can affect these joints. equate, an over-the-counter or prescriptionNSAID should be
used. NSAIDs are available in oral, topical, and intravenous
forms. Oral preparations are usually first-line NSAID therapy
Management in patients without a contraindication to treatment due to
efficacy and cost-effectiveness. TopicalNSAIDs are considered
OA management requires a comprehensive, individualized
to provide similar pain relief OA as oral medications with
evaluation of symptoms, level of functioning, and patient
fewer gastrointestinal effects. Furthermore, the American
expectations. Because no single intervention addresses all of
College of Rheumatology currently recommends topical
the ways in which OA can impact quality of life, a multidisci­
NSAIDs rather than oralNSAIDs for patients aged 75 years or
plinary perspective promotes the best patient outcomes.
older. However, they are associated with more skin reactions
and are significantly more expensive than oral NSAIDs.
Nonpharmacologic Therapy NSAIDs can be also be used in combination with acetami­
Exercise (aquatic or land based) is among the most important nophen. Treatment guidelines suggest using the lowest effec­
nonpharmacologic treatments for most patients with OA. tiveNSAID dose for the shortest time period in order to reduce
Exercise prescriptions should first address acutely sympto­ risk of side effects. However, many patients with OA require
matic joints, targeting rehabilitation within the physical capa­ years ofNSAID use given the prolonged timeframe over which
bilities of the patient. Exercise should be repeated regularly the disease is symptomatic and the limited number of phar­
and should not overuse the joints or exacerbate symptoms. macologic alternatives.
Exercise should be advanced as the patient improves and can Tramadol is a centrally acting opioid analgesic that weakly
include strengthening, stretching, range of motion, and aero­ inhibits norepinephrine-serotonin reuptake; potential for
bic activity. Although a physical or occupational therapist is addiction is lower than for traditional opioids. Tramadol may
often useful initially and can also provide thermal agents and be useful for managing OA pain when acetaminophen or
manual therapy, the goal is to assist patients in incorporating NSAIDs are ineffective or contraindicated (for example,
exercise into their daily self-management routine. patients with a history of bleeding peptic ulcer, heart failure,
Evidence for the efficacy of exercise is most convincing or chronic kidney disease).
for knee OA. Because excessive weight contributes to both Duloxetine is a dual serotonin-norepinephrine reuptake
knee OA risk and symptomatology, measures to promote inhibitor indicated for chronic musculoskeletal pain, includ­
weight loss (exercise, food diaries, support groups, meal plan­ ing pain due to OA. Duloxetine may be used as an adjunct to
ning, nutritional counseling, portion size regulation, evalua­ more rapidly acting analgesics or as a single agent in patients
tion of psychosocial factors and coping strategies) are usually with chronic OA pain.
appropriate for overweight patients with knee OA. Obese Traditional opiates may rarely be warranted to control
patients undergoing bariatric surgery may experience reduc­ pain in patients with OA who have not responded to other
tion in knee OA symptoms. agents or are poor candidates for other interventions to treat
Properly fitting footwear and/or orthotics can improve painful joints, such as surgery. Patient selection, informed
ambulation among patients with OA of the feet, ankles, knees, consent, and monitoring of response and for side effects
hips, and/or spine. Assistive devices such as canes and walkers should proceed according to treatment guidelines established
can reduce lower extremity symptoms and may permit delay for opiate treatment of any chronic pain.
or deferral of surgical intervention. Canes should be held on Although early trials suggested an analgesic benefit from
the side opposite from the affected joint. Short-term use of glucosamine sulfate and chondroitin sulfate, subsequent stud­
splints (for example, thumb spica), cushions, and ergonomic ies have shown a minimal effect on symptoms and no mod­
supports (for example, in vehicles and at desks and worksta­ eration in the progression of disease. Therefore, guidelines do
tions), as well as the use of grab bars and other specialized not recommend their use.

32
Fibromyalgia

See Principles of Therapeutics for a discussion on medi­ surfaces followed by replacement with metal and polyethylene
cation toxicities and side effects. prosthetic components). In properly selected patients. arthro­
plasty can reduce pain. improve function, and enhance quality
Intra-articular Injection of life. Patients are transiently anticoagulated to reduce the
When a single symptomatic joint is present, injection directly rate of postoperative deep venous thrombosis to about 1 %.
into the joint may deliver medication to the affected site while Prosthetic joint infection occurs in 1 % to 2% of patients under­
minimizing the potential for systemic effects. Intra-articular going total joint arthroplasty; patients with diabetes mellitus
injections may be used along with or in place of oral or topical are at increased risk. After 10 years, 85% to 90% of total joint
analgesics. replacements for OA remain functional. However. about 20%
of total knee and 10% of total hip arthroplasty patients con­
Glucocorticoid Injection tinue to have some level of persistent pain . CJ
Numerous glucocorticoid preparations are available for Arthroscopy may be helpful in knee OA when retrieval of
intra-articular injection, including triamcinolone acetonide, a loose body is required to prevent locking of the joint.
triamcinolone hexacetonide, and methylprednisolone. However, the routine use of arthroscopic lavage and debride­
Glucocorticoids can reduce OA knee pain within days to ment for OA has fallen out of favor because several studies have
weeks. When a joint effusion is present, glucocorticoid injec­ demonstrated no benefit of this procedure compared with
tions can be particularly helpful after drainage of the excessive blinded sham surgery.
joint fluid. Intra-articular facet joint injections may provide
KEY POINTS
pain relief from cervical or lumbar spinal OA, although studies
are limited. Local side effects include skin hypopigmentation, • A regular exercise program, physical therapy, nutri­
subcutaneous tissue atrophy, and joint infection. The fre­ tional counseling, and appropriate footwear, splints,
canes, or other assistive devices can benefit patients
quency of glucocorticoid injections is usually limited to every
with osteoarthritis.
3 months due to theoretical concerns about adverse effects on
the joint. • Acetaminophen is the initial choice for osteoarthritis
pain control in most instances; if acetaminophen pro­
Hyaluronic Acid Injection vides inadequate relief, NSAIDs should usually be tried.
Although multiple individual trials of hyaluronic acid injec­ • Intra-articular glucocorticoids reduce osteoarthritis
tion have suggested analgesic benefit for knee OA, recent knee pain within days to weeks.
meta-analyses suggest that any such benefit may be clinically
• The most effective surgical intervention for knee or
trivial. Functional benefit has also not been demonstrated.
hip osteoarthritis is total joint arthroplasty, which can
However, the possibility that specific patient subsets may
reduce pain, improve function, and enhance quality
obtain benefit from hyaluronic acid injection is not yet deter­
of life.
mined. Side effects are uncommon but include transient
postinjection joint inflammation. The risk of joint infection
after hyaluronic acid injection is not currently known.

Surgical Therapy Fibromyalgia


Cl When pain or functional impairment cannot be controlled by
other means, orthopedic intervention for OA deserves consid­
Introduction
Fibromyalgia is characterized by chronic widespread pain,
eration. The decision to proceed with surgery should take into
consideration the patient's medical comorbidities and level of tenderness of skin and muscles to pressure, fatigue, sleep dis­
preoperative strength and mobility. Radiographic severity turbance, and exercise intolerance. Patients with fibromyalgia
alone is an imperfect predictor of symptoms and an inade­ often have coexisting somatic conditions such as pelvic pain,
quate gauge of the need for surgery; decision making should headaches, and temporal mandibular joint pain. Effective
incorporate an appreciation for the likely benefit in terms of treatment addresses both the biologic and psychologic aspects
quality of life and patient expectations, particularly regarding of chronic pain.
employment and recreational activities. Pain at rest or that
awakens the patient in the middle of the night despite analge­
sic use is often present in those considering surgery. Epidemiology
Preoperative anxiety and depression are associated with worse Using the 1990 American College ofRheumatology fibromyal­
postoperative outcomes. Morbidly obese patients may experi­ gia classification criteria (widespread body pain, tender
ence earlier failure of prostheses and are often advised to lose points), prevalence in the general population ranges from
weight prior to surgery. 0.06% to 10%. Fibromyalgia increases with age up to the sixth
For knee and hip OA, the most effective surgical interven­ decade and declines thereafter. The female-male ratio is 9:1,
tion is total joint arthroplasty (removal of the affected a11icular but this may change with use of newer diagnostic criteria.

33
Fibromyalgia

Fibromyalgia may co-occur in patients with inflamma­ Initial laboratory studies include a complete blood count,
tory diseases such as rheumatoid arthritis (25%), systemic chemistry panel, thyroid-stimulating hormone, and erythro­
lupus erythematosus (30%), and primary Sjogren syndrome cyte sedimentation rate (ESR) or C-reactive protein (CRP). ESR
(50%). Psychologic distress, mood disorder, victimization, and and CRP should be normal; elevated levels should prompt
disordered sleep have also been reported to play contributory additional evaluation. Antinuclear antibodies, rheumatoid
roles. The role of physical trauma as an initiator of fibromyal­ factor, or anti-cyclic citrullinated peptide antibodies should
gia is controversial. Importantly, not all patients with fibromy­ not be obtained unless examination or initial laboratory test­
algia have evidence of psychologic distress or physical trauma, ing suggests that these are indicated. Muscle enzymes are fre­
underlining that such factors may contribute to, but do not quently checked but are useful only in patients who are
define, fibromyalgia. actually weak on examination.

Pathophysiology Management
Patients with fibromyalgia typically benefit from validation
Studies suggest that the pain state of fibromyalgia is mediated
of their symptoms by a physician because many patients
by aberrant chronic pain reflex arcs centered on the dorsal
have previously had their complaints disregarded by care­
ganglia of the spine. These arcs function independently but
givers or family. These patients should be reassured that the
may be exacerbated by acute or chronic pain input such as that
condition is not an indication of a more serious problem and
seen in arthritis or by the input contributed by psychologic
that additional diagnostic testing is generally unhelpful.
distress. Functional MRI brain studies indicate that pain per­
Discussing the current understanding of fibromyalgia ("a
ception among patients with fibromyalgia is indistinguishable
problem of central pain processing") can be useful when the
from, but more readily triggered than, pain perception in
treatment program is described. The goal is to have the
unaffected individuals. Patients with fibromyalgia demon­
patient actively involved in a program that improves pain
strate elevation in pain-facilitating neurotransmitters (gluta­
processing centrally and increases pain thresholds. Four
mate and substance P) and reduction of neurotransmitters
issues need to be addressed: psychologic distress and/or
that down-regulate pain (norepinephrine, serotonin, and
mood disorder, sleep disturbance (may be tied to the dis­
y-aminobutyric acid). Endogenous opioid levels are paradoxi­
tress), pain, and deactivation. Although patients often avoid
cally elevated along with decreased opioid receptor availabil­
aerobic exercise, it is critical for functional improvement. A
ity, perhaps accounting for the poor response to narcotics seen
physical therapist can be useful to discuss daily stretching
in those with fibromyalgia.
and a slowly progressive home aerobic exercise program.
Sleep hygiene should be reviewed. When another rheumato­
Diagnosis logic disease or significant psychologic distress is present,
The 2010 American College of Rheumatology fibromyalgia referral to a specialist is suggested.
diagnostic criteria eliminated the tender point examination, Three drugs are FDA approved for fibromyalgia: the anti­
owing to sensitivity biases that resulted in overdiagnosis convulsant pregabalin and the dual serotonin-norepinephrine
among women. Current criteria include documentation of reuptake inhibitors duloxetine and milnacipran. Each pro­
self-reported pain for at least 3 months at 19 different body vides a modest benefit over placebo, with 30% to SO% of
locations (Widespread Pain Index [WPI]) along with a severity patients experiencing more than 30% reduction in pain. The
score (scale of 0-3) for three reported symptoms: fatigue, wak­ latter two agents can also address coexisting mood disorder.
ing unrefreshed, and cognitive symptoms (Symptoms Severity Off-label therapies that have provided benefit include the tri­
Scale [SSS]). The combination of a WP! of�7 plus an SSS of�S, cyclic antidepressants (which have some demonstrated sleep
or a WP! between 3 and 6 plus an SSS of�9, establishes a diag­ utility), and, to a lesser extent, the selective serotonin reuptake
nosis of fibromyalgia for the purposes of study enrollment. inhibitors, particularly when administered in conjunction
In routine clinical practice, the combination of widespread with a tricyclic agent.
pain ("hurt all over"), waking unrefreshed ("always tired and The combination of medications, reactivation, and a pro­
fatigued"), cognitive fatigue (forgets words or loses track of con­ ductive lifestyle offers the best chance for significant improve­
versations in mid-sentence), and exercise intolerance ("if I ment. Patients should be reassured that the chances of further
overdo it, I pay for it for several days") are clues to the diagnosis. deterioration are low but should also be aware that there cur­
Previous lack of response to multiple medications, including rently is no medical cure for this condition.
NSA!Ds, is also a diagnostic clue. Cross-sectional data suggest KEY POINTS
that obstructive sleep apnea may be common. Examination is • Fibromyalgia is a clinical d�agnosis characterized by HVC
generally unremarkable except for diffuse tenderness to palpa­ chronic widespread pain, tenderness of the skin and
tion. The association of fibromyalgia with some inflammatory muscles to pressure, fatigue, sleep disturbance, and
diseases requires that these disorders be considered when per­ exercise intolerance.
forming a history and physical examination. (Continued)

34
Spondyloarthritis

KEY PO INTS (continued) Although less strongly associated with other forms of
HVC • Initial laboratory evaluation of fibromyalgia includes a spondyloarthritis, HLA-B27 is present in up to 70% of patients
complete blood count, chemistry panel, thyroid-stimu­ with reactive and/or IBD-associated arthritis. HLA-B27 is also
lating hormone, and erythrocyte sedimentation rate or present in 60% to 70% of patients with axial psoriatic arthritis
C-reactive protein; routine testing for antinuclear anti­ and in 25% of those with peripheral psoriatic arthritis without
bodies, rheumatoid factor, anti-cyclic citrullinated pep­ axial involvement.
tide antibodies, or muscle enzymes should be avoided.
Environmental Factors
• Management of fibromyalgia addresses psychologic dis­
The strongest evidence that infectious triggers can play a role
tress and/or mood disorder, sleep disturbance, pain,
in spondyloarthritis is found in reactive arthritis, which typi­
and deactivation.
cally develops after specific gastrointestinal or genitourinary
• For patients with fibromyalgia, aerobic exercise is criti­ infections. However, the immunobiology underlying infec­
cal for functional improvement. tious triggers remains uncertain, and infectious agents have
• Pregabalin, duloxetine, and milnacipran are FDA not been shown to trigger ankylosing spondylitis. A disrupted
approved and modestly effective for fibromyalgia. mucosa! barrier between the gut and bloodstream, potentially
fostering unopposed interaction of microorganisms with
immune tissues, has been hypothesized to promote IBD­

Spondyloa rth ritis associated arthritis.


Reactive arthritis and psoriatic arthritis may be more
severe among patients with HIV infection, particularly when
Introduction
HLA-B27 is present. Unexpectedly severe presentations of
Spondyloarthritis refers to a group of disorders that share an either disease may warrant evaluation for underlying HIV
overlapping set of features, including inflammation of the infection in appropriate cases.
axial skeleton, tendons, and entheses (insertion of tendon to
bone); tendon and enthesis calcification; an association with KEY POINTS

HLA-B27; and mucocutaneous, gastrointestinal, and ocular • The gene most strongly associated with spondyloarthri­
inflammation. tis is HLA-B27, which is present in approximately 90%
The four disorders of spondyloarthritis are ankylosing of patients with ankylosing spondylitis.
spondylitis, psoriatic arthritis, inflammatory bowel disease • The strongest evidence that infectious triggers can play
(IBD)-associated arthritis, and reactive arthritis (formerly a role in spondyloarthritis is found in reactive arthritis,
known as Reiter syndrome). Patients who have some of these which typically develops after specific gastrointestinal
features but do not meet criteria for one of the four disorders or genitourinary infections.
may be designated as having undifferentiated spondyloarthritis.
• Reactive arthritis and psoriatic arthritis may be more
KEY POINT severe among patients with HIV infection, particularly
• Spondyloarthritis refers to a group of disorders that when HLA-B27 is present; unexpectedly severe presen­
share an overlapping set of features, including inflam­ tations of either disease may warrant evaluation for
mation of the axial skeleton, tendons, and entheses; underlying HIV infection in appropriate cases.
tendon and enthesis calcification; an association with
HLA-B27; and mucocutaneous, gastrointestinal, and
ocular inflammation. Classification
Although the disorders comprising spondyloarthritis share
common clinical features, each disorder has a greater or lesser
Pathophysiology tendency to manifest specific features (Table 15). However, the
Genetic Factors features can overlap and the phenotype of each syndrome also
The gene most strongly associated with spondyloarthritis is varies from patient to patient, making the distinction some­
the class I major histocompatibility complex molecule HLA­ times challenging.
B27, which is present in approximately 90% of patients with
ankylosing spondylitis. Among first-degree relatives of patients Ankylosing Spondylitis
with ankylosing spondylitis, the presence of HLA-B27 confers Ankylosing spondylitis characteristically affects the axial skel­
an increased risk (approximately 10%-30%) of developing the eton and also has extra-articular manifestations (see Table 15).
disease. Moreover, ankylosing spondylitis is rare in low HLA­ Ankylosing spondylitis is more common in men than women
B27 prevalence populations. However, HLA-B27 alone is insuf­ (3:1 ratio), although some studies suggest it is underrecognized
ficient to produce spondyloarthritis because most individuals in the female population. Peak age of onset is in the second to
who are positive for HLA-B27 never develop the disease. third decade of life.

35
Spondyloarthritis

TABLE 15. Clinical Features of Spondyloarthritis


Ankylosing Psoriatic Arthritis IBD-Associated Reactive Arthritis
Spondylitis Arthritis

Musculoskeletal

Axial involvement Axial involvement May occur at any level; May be asymptomatic Less common than in
predominates; initially may start in the cervical but can follow a course other forms of
symmetrically involves spine; may skip regions; similar to ankylosing spondyloarthritis
the SI joints and lower may be asymmetric spondylitis; SI
spine, progressing involvement often
cranially; does not skip asymmetric; arthritis
regions does not parallel IBD
activity

Peripheral Enthesitis (e.g., Achilles Various patterns, most Two patterns: mono/ Enthesitis and
involvement tendinitis) with or commonly polyarticular; oligoarticular large joint asymmetric large-joint
without asymmetric DIP involvement is lower extremity oligoarthritis; usually
large-joint oligoarthritis; associated with nail (parallels IBD activity), self-limited; nonerosive;
hip involvement can involvement; dactylitis; and polyarticular small some patients
cause significant enthesitis; tenosynovitis; joint upper extremity experience recurrent or
functional limitation; arthritis mutilans (does not parallel IBD persistent arthritis; may
shoulders can be activity); dactylitis and develop features of
involved enthesitis may occur other forms of
spondyloarthritis

Dermatologic Psoriasis may coexist Psoriasis typically Pyoderma Keratoderma


precedes joint gangrenosum; blennorrhagicum;
involvement; nail erythema nodosum circinate balanitis
pitting; onycholysis

Ophthalmologic Uveitis (typically Conjunctivitis more Uveitis (anterior; can be Conjunctivitis is more
anterior, unilateral, common than uveitis bilateral, insidious, or common than uveitis
recurrent) (anterior; can be chronic); conjunctivitis,
bilateral, insidious, or keratitis, and episcleritis
chronic) are rare

Gastrointestinal Asymptomatic intestinal Crohn disease; Prior GI infection in


ulcerations ulcerative colitis some patients

Genitourinary Urethritis (rare) Nephrolithiasis Prior GU infection in


some patients; sterile
urethritis; prostatitis;
cervicitis; salpingitis

Cardiovascular Aortic valve disease; Association with Thromboembolism


aortitis; conduction traditional CAD risk
abnormalities; CAD factors

Pulmonary Restrictive lung disease


from costovertebra I
rigidity; apical fibrosis
(rare)

Bone quality Falsely elevated bone Increased risk of High risk for vitamin D Localized osteopenia
mineral density from fracture; multifactorial deficiency, low bone
syndesmophytes; density, and fracture
increased risk of spine
fracture

CAD= coronary artery disease; D1P = distal interphalangeal; GI= gastrointestinal; GU;:. genitourinary; IBD = inflammatory bowel disease; SI= sacroiliac.

Inflammatory low back pain of insidious onset is the refers to the bony bridging of the vertebrae resulting from
hallmark of ankylosing spondylitis, manifesting as pain and chronic inflammation; the calcified ligaments and disk cap­
stiffness that are worse after immobility and are better with sules in such cases are read on radiographs as syndesmo­
use. Symptoms are prominent in the morning (>l hour); night phytes. Fusion of the spine may occur over time, leading to
pain is characteristic and may awaken the patient. Buttock rigidity and kyphosis.
pain is common and bilateral, and it correlates with sacroili­ Risk factors for poor prognosis include male gender, early
itis. Early in its course, ankylosing spondylitis almost always age of onset, tobacco use, and the presence of hip or peripheral
affects the lumbar spine; longer and more severe disease may arthritis, psoriasis, IBO, iritis, or elevated erythrocyte sedi­
involve the thoracic and cervical regions as well. Ankylosis mentation rate (ESR). Mortality is increased, primarily relating

36
Spondyloarthritis

to increased rates of cardiovascular disease (coronary artery


disease, aortic valve regurgitation, aortic aneurysm, conduc­
tion disturbance), cancer, and infection.

Psoriatic Arthritis
Psoriatic arthritis is an inflammatory arthritis that is associ­
ated with psoriasis (see Table 15). Peak age of onset is between
40 and 60 years; men and women are equally affected.
Prevalence of psoriatic arthritis is approximately 1 % in the
general population. Although estimates of the prevalence of
psoriatic arthritis in patients with psoriasis vary, more recent
studies using standardized diagnostic criteria indicate that
psoriatic arthritis is present in approximately 15% to 20% of
those with psoriasis. Psoriasis most commonly precedes the FIGURE 1 2. This patient with psoriatic arthritis has onycholysis and onychodys·
onset of arthritis but may occur with or after arthritis onset. trophy (malformation of a fingernail or toenail).
The most common patterns of joint involvement are an
asymmetric lower extremity oligoarthritis (resembling reac­ TABLE 16. Classification Criteria for Psoriatic
tive arthritis) or a symmetric polyarthritis involving the distal Arthritis (CASPAR)
interphalangeal (DIP), proximal interphalangeal (PIP), and/or
Inflammatory articular disease (joint, spine, entheseal)
metacarpophalangeal (MCP) and metatarsophalangeal (MTP)
joints (distribution similar to rheumatoid arthritis but includes Plus three or more of the following:

the DIPs). Less common presentations include DIP involve­ Psoriasis (current, personal history of, family history of)
ment only as well as a chronic resorptive arthritis (arthritis Psoriatic nail dystrophy
mutilans) causing digital shortening and a "telescoping" Negative rheumatoid factor'
appearance. Spondylitis (spine or sacroiliac arthritis) may
Dactylitis/swelling of entire digit (current, personal history of)
occur; in contrast to ankylosing spondylitis, it is usually asym­
Radiologic evidence of juxta-articular new bone formation
metric and may skip regions. (joint of hand or foot)
Dactylitis (diffuse swelling of joints, tendons, and/or liga­
aFive to ten percent of patients may have a positive rheumatoid factor or anti-cyclic
ments of a digit, creating a sausage-like appearance) occurs in citrullinated peptide antibodies; this would not exclude the diagnosis as long as
approximately SO% of patients with psoriatic arthritis (Figure ll). the patient meets sufficient other criteria.

Nail involvement such as pitting or onycholysis (Figure 12) is


commonly observed concurrent with DIP joint involvement.
The recently developed Classification Criteria for Psoriatic psoriatic joint involvement (especially DIP arthritis and/or
Arthritis (CASPAR) convey a sensitivity and specificity of more dactylitis), but without apparent psoriasis, should undergo a
than 90% (Table 16). Patients with a characteristic pattern of thorough examination for occult psoriatic skin or nail changes.
Although patients with psoriatic arthritis are characteristically
seronegative, 5% to 10% may demonstrate rheumatoid factor or
anti-cyclic citrullinated peptide (CCP) antibodies, possibly
owing to overlapping genetic proclivities. Typical radiographic
changes of psoriatic arthritis include a combination of mar­
ginal joint erosions and new bone formation; classic findings
include pencil-in-cup changes (Figure 13).
Prognosis of psoriatic arthritis is variable, with studies
suggesting a relationship between disease severity and mortal­
ity. Severity of skin disease neither predicts nor correlates with
severity of arthritis. Elevated ESR and the presence of radio­
logic joint damage at presentation predict both arthritis pro­
gression and mortality; the presence of nail lesions may be
protective. Cardiovascular disease is the most common cause
of mortality.

Inflammatory Bowel Disease-Associated Arthritis


FIGURE 11 . Dactylitis ("sausage digit") associated with psoriatic arthritis.
Inflammatory arthritis is present in 20% to 30% of patients
Dactylitis manifests as swelling of the entire digit and arises from inflammation of with ulcerative colitis or Crohn disease. IBO-associated arthri­
the flexor tendon and adjacent soft tissue. tis may be axial or peripheral. Peripheral joint involvement

37
Spondyloarthritis

and severity of initial infection may increase the risk of develop­


ing reactive arthritis.
Up to 50% of reactive arthritis cases resolve by 6 months,
but 20% of patients progress to chronic disease. Recurrent
infectious episodes may convey an increased risk of recurrent
reactive arthritis.
KEY POINTS

• Inflammatory low back pain of insidious onset is the


hallmark of ankylosing spondylitis, manifesting as pain
and stiffness that are worse after immobility and are
better with use.
• Psoriatic arthritis typically presents as an asymmetric
lower extremity oligoarthritis or a symmetric polyar­
thritis involving the distal interphalangeal, proximal
interphalangeal, and/or metacarpophalangeal and met­
atarsophalangeal joints, often with dactylitis.
• Arthritis of varying patterns may occur in patients with
FIGURE 1 3. Pencil-in-cup deformity in psoriatic arthritis. Note the erosion and inflammatory bowel disease (IBD); however, only oli­
remodeling of the index proximal interphalangeal joint with a cup-like appearance goarticular peripheral arthritis parallels IBD activity.
of the distal bone on the ulnar aspect of the joint; the bone proximal to the joint
becomes so eroded as to look like a pencil point within the cup. • Reactive arthritis is a noninfectious, autoimmune
inflammatory arthritis that can occur after specific gas­
trointestinal or genitourinary infections and typically
may be oligoarticular (type 1) or polyarticular (type 2). Only presents as asymmetric monoarthritis or oligoarthritis
the oligoarticular peripheral arthritis parallels IBD activity. See in the lower extremities; enthesopathy, dactylitis, and
Table 15 for more information. sacroiliitis may also occur.

Reactive Arthritis
Reactive arthritis is a noninfectious inflammatory arthritis Diagnosis
that can occur after specific gastrointestinal or genitourinary Diagnosis of a specific spondyloarthritis disorder is based on
infections. Asymmetric monoarthritis or oligoarthritis in the the characteristic history and physical examination, as well
lower extremities is the most common presentation, but up to as radiographic findings such as sacroiliitis and enthesitis
20% of patients have polyarthritis. Enthesopathy (including at (Table 17). The clinical setting may provide a clue to diagno­
the Achilles tendon insertion to the calcaneus), dactylitis, and sis. For example, ankylosing spondylitis should be consid­
sacroiliitis may occur. Erosive disease is uncommon. See Table ered in patients with chronic inflammatory back pain who
15 for clinical features. are younger than 45 years old. Peripheral arthritis or tend­
Despite the preceding infection, reactive arthritis is an initis is less likely to occur in ankylosing spondylitis and
autoimmune, rather than infectious, arthritis. It is hypothe­ should raise suspicion for a different form of spondyloarthri­
sized that exposure of a susceptible patient to a defined infec­ tis. Psoriatic arthritis should be considered if there is a prior
tious antigen may induce a cross-reaction to a similarly or current diagnosis of psoriasis, especially along with the
structured, previously tolerated self-antigen (molecular mim­ characteristic arthritis patterns previously described. IBD­
icry), resulting in autoimmunity and self-perpetuating inflam­ associated arthritis should be considered if there is a prior
mation. Therefore, use of antibiotics specifically to treat diagnosis of IBD or current symptoms and signs of possible
reactive arthritis has failed to demonstrate benefit. IBD. Reactive arthritis should be considered if there is a his­
Gastrointestinal pathogens associated with reactive tory of preceding infection, especially gastrointestinal or
arthritis include Yersinia, Salmonella, Shigella, genitourinary infection. Considerable overlap exists between
Campylobacter, and, rarely, Escherichia coli and Clostridium these diseases; for example, a patient with ankylosing spon­
difficile. The most common genitourinary pathogen is dylitis may also have psoriasis and/or IBD.
Chlamydia trachomatis; cases due to Ureaplasma urealyti­ Diagnosis may be supported by the presence of HLA-B27
cum have been reported. and/or elevated inflammatory markers. Patients lacking labo­
Reactive arthritis typically occurs approximately 3 to ratory and radiographic evidence of disease may nonetheless
6 weeks after the infectious trigger, with a latency range of have sufficient historical and physical manifestations (charac­
2 weeks to 6 months. In a significant minority of cases, the trig­ teristic inflammatory arthritis, along with dactylitis, enthesi­
gering infection may go unrecognized. The presence ofHLA-B27 tis, inflammatory eye disease, or psoriasis) to support a

38
Spondyloarthritis

TABLE 17. Evaluation of Patients with Suspected Spondyloarthritis


History

Inflammatory back and/or joint pain•


Family history of SpA or psoriasis

Positive response to NSAIDs

Symptoms consistent with, or existing diagnosis of, psoriasis, IBD, and/or ocular inflammation (iritis, uveitis, conjunctivitis)

Preceding infection

Urethritis symptoms in the absence of ongoing infection

Physical Examination Findings

Spine/SI joint tenderness; limited ROM of spineb; tenderness and/or swelling of joints, tendons (dactylitis), and/or tendon insertions to
bone (enthesitis, especially Achilles tendon insertion)
Rash and/or nail changes indicative of psoriasis

Ocular inflammation (iritis, uveitis, conjunctivitis)

Laboratory Studies

Positive HLA-B27

Elevated C-reactive protein and/or erythrocyte sedimentation rate

Radiography

Plain radiography of SI joints and/or symptomatic area of spine (lumbar, thoracic, cervical) showing characteristic changes; if plain
radiograph is negative, consider MRI of SI joints+/- spine
Plain radiograph of symptomatic peripheral joints to look for erosions and new bone formation suggestive of inflammatory arthritis/
enthesitis

IBD = inflammatory bowel disease; ROM= range of motion; SI= sacroiliac; SpA = spondyloarthritis.

a1nflammatory joint pain is worse overnight, in the morning, and after immobility; improves with joint use; and is associated with morning stiffness lasting longer than 1 hour.

bSchober test, chest expansion, and occiput-wall distance measurements can be useful.

diagnosis. Classification criteria for psoriatic arthritis Imaging Studies


(CASPAR) and for peripheral and axial spondyloarthritis Conventional radiography of the spine and sacroiliac joints is
(Assessment of SpondyloArthritis international Society) have inexpensive and generally adequate to demonstrate synovi­
been developed; although these are primarily intended for tis, axial erosion, or new bone formation in patients with
research purposes, they provide informative paradigms for spondyloarthritis, although results may be normal in early
clinicians diagnosing these disorders. disease. With peripheral spondyloarthritis, conventional
See Figure 14 and Figure 15 for more information on diag­ radiography may detect erosive changes with bony prolifera­
nosing spondyloarthritis. tion at the entheses.
Sacroiliitis is usually bilateral and symmetric in ankylos­
Laboratory Studies ing spondylitis but may be unilateral or absent in other forms
No specific serologic tests are available to diagnose spondy­ of spondyloarthritis. On plain radiographic imaging, sacroiliac
loarthritis. Rheumatoid factor, anti-CCP antibodies, and anti­ erosion initially appears as irregular widening of the joint
nuclear antibodies are usually negative, although low-titer space, accompanied by sclerotic changes. Later, the joint space
positivity may occasionally be present. Elevated inflammatory narrows, and eventually the sacroiliac joint may fuse. Vertebral
markers such as ESR, C-reactive protein, and serum amyloid A plain radiographic findings in ankylosing spondylitis include
protein often correlate with disease activity but may also be sclerosis at the attachment of annulus fibrosis to the anterior
normal, especially in patients with ankylosing spondylitis. corner of the vertebral endplate ("shiny corner"), and erosion
HLA-B27 testing can define a probability for spondyloarthritis at the point of contact between the disk and the vertebra. In
but cannot independently confirm or exclude any specific later disease, vertebrae may lose their normal anterior concav­
diagnosis. Although infection has usually resolved by the time ity due to periosteal bone proliferation, resulting in squaring of
of arthritis onset in patients with reactive arthritis, DNA the vertebral bodies. Calcification of the anterior longitudinal
amplification urine testing for Chlamydia trachomatis should ligament and bridging syndesmophyte formation are late fea­
be performed because some individuals may have asympto­ tures, leading to ankylosis and a "bamboo" spine appearance
matic persistent infection or carriage of this organism. (Figure 16, on page 41). This may be difficult to differentiate

39
Spondyloarthritis

Inflammatory back pain•

Conventional radiography
of SI joints± spine

Normal Inflammation

Negative Positive b

If clinical suspicion
remains high,
consider MRI of SI
joints± spine

lr---�---------------------,I
Normal Inflammation
Likely SpA
I
Consider alternative
diagnosis/observation

FIGURE 1 4. Diagnostic evaluation for suspected axial spondyloarthritis. SI= sacroiliac; SpA = spondyloarthritis.
"Characterized by pain and stiffness in the spine that is worse aher immobility and better with use; prominent morning pain lasting> 1 hour; buttock pain is common and iypically bilateral.

bParticularly if at least one other feature typical of SpA is present: uveitis, enthesitis, dactylitis, psoriasis, inflammatory bowel disease. positive family history of SpA, or elevated inflammatory markers.

Inflammatory arthritis, enthesitis, dactylitis

No predisposing condition Presence of predisposing condition

HLA-B27 Preceding
Psoriasis
Conventional SI joint± spine radiography infection
Evaluation for uveitis

,-----�
None present At least one present Likely Likely
Likely IBD­
psoriatic reactive
related SpA
arthritis arthritis

Undifferentiated Undifferentiated SpA


a rthrit is/enthesiti s/dactyIitis
• Consider MRI of SI
joints ± spine if
sacroiliitis suspected
• Continue to monitor
for development of
other typical features

FIGURE 1 5. Diagnostic evaluation for suspected peripheral spondyloarthritis. lBD = inflammatory boviel disease; SI= sacroiliac; SpA = spondyloarthritis.

from diffuse idiopathic skeletal hyperostosis (DISH); however, CT is more sensitive than conventional radiography for
the changes in ankylosing spondylitis are usually on both sides detecting bony erosions and may be helpful when disease is
of the spine, whereas in DISH they are characteristically right suspected but plain radiographs are negative. CT is also useful
sided. Furthermore, DISH is most commonly thoracic, whereas for detecting subtle vertebral body fractures in patients with
ankylosing spondylitis starts with sacroiliitis and lumbar ankylosing spondylitis who are at increased risk of spine frac­
arthritis and usually does not skip regions as it ascends. tures. However, CT is expensive and conveys increased levels of

40
Spondyloarthritis

motion. Glucocorticoid injections may be helpful in providing


symptomatic relief, particularly if there are one or two active
isolated joints in which a local injection may reduce the need
for systemic medication. Pharmacologic therapy is needed for
control of symptoms in most forms of spondyloarthritis,
although the most effective medication regimen may vary by
type of spondyloarthritis present.
A mainstay of pharmacologic therapy is NSA!Ds. The
various NSA!Ds studied are equally effective, but differences
exist in individual response to, and tolerance of, a given NSA!D;
thus, it can be helpful to try a second NSAID if the first has
produced an inadequate response. Maximal daily NSAID dos­
ing is generally required to produce a positive response in
spondyloarthritis, but the dose may subsequently be reduced
depending on patient response and tolerance.
FIGURE 16. The initial radiographic findings of ankylosing spondylitis include For patients with more serious disease not adequately
irregularities along the margins of the sacroiliac joints leading to eventual ankylo­
controlled with NSA!Ds or who cannot tolerate NSA!Ds, non­
sis and fusion. Inflammation of the ligamentous attachments erodes the corners of
the vertebral bodies, which produces a squared-off appearance. Over time, ossifica­ biologic and biologic disease-modifying antirheumatic drugs
tion of these ligaments leads to the development of a rigid "bamboo spine," (DMARDs) can be helpful. However, these agents may increase
named because the shape of the vertebrae resemble bamboo on radiograph. the risk of infection, and certain drugs are extremely expen­
sive or may require parenteral administration.
ionizing radiation exposure. Accordingly, CT is not routinely Patients with severe end-stage joint or soft-tissue damage
used to monitor spondyloarthritis disease progression. may need surgery such as tendon repair or joint replacement.
MRI can detect early inflammation in the spine and sac­ Surgery to correct severe spine flexion deformities in ankylos­
roiliac joints, even before bony changes are detected on radio­ ing spondylitis may be considered but can be challenging and
graph or CT. MRI of the sacroiliac joints and/or spine should be risky due to difficulties with intubation for anesthesia, spine
obtained if there is high suspicion for spondyloarthritis and fragility, and stabilization.
conventional radiographs are negative. However, MRI is more
expensive and less generally available.
Ankylosing Spondylitis
Musculoskeletal ultrasonography can detect peripheral
Exercise is a particularly important component of ankylosing
enthesitis and arthritis but has no current role in detecting
spondylitis therapy and can help preserve mobility and pre­
sacroiliitis or axial involvement.
vent kyphosis. A physical therapist can assist patients in devel­
KEY POINTS oping a home exercise routine that can be maintained over
• In patients with spondyloarthritis, rheumatoid factor, time. Glucocorticoid injections to the sacroiliac joints can
anti-cyclic citrullinated peptide antibodies, and antinu­ relieve symptoms and facilitate exercise.
clear antibodies are usually negative. NSA!Ds are considered first-line therapy for ankylosing
• HLA-B27 testing can support, but cannot independently spondylitis. Studies of full-dose NSAIDs demonstrate sympto­
confirm or exclude, a diagnosis of spondyloarthritis. matic relief as well as reduced sacroiliac and spine inflamma­
tion as seen on MRI in some patients. In contrast to their
HVC • Conventional radiography of the spine and sacroiliac
limited effects in modifying the course of almost all other
joints is generally adequate to demonstrate synovitis, axial
rheumatologic diseases, some studies suggest that daily NSAID
erosion, or new bone formation in patients with spondy­
use may reduce progression of spine damage caused by anky­
loarthritis; CT should be reserved for identifying occult
losing spondylitis.
spine fractures and bony erosions in patients at 11.igh risk
Tumor necrosis factor (TNF)-a inhibitors are indicated for
due to expense and 11.igher level of radiation exposure.
patients who have had inadequate responses to NSA!Ds. If the
HVC • In patients with strongly suspected spondyloarthritis, first TNF-a inhibitor produces an inadequate response, a dif­
MRI of the sacroiliac joints and/or spine should only be ferent TNF-a inhibitor may be more successful. TNF-a inhibi­
considered if conventional radiographs are negative. tors demonstrably improve pain and mobility as well as reduce
inflammation in axial joints. Although recent reports suggest
that they may help slow radiographic disease progression,
Management more studies are needed for confirmation.
General Considerations Nonbiologic DMARDs such as methotrexate and sul­
Management of spondyloarthritis should include exercise to fasalazine are not helpful in treating axial disease but may be
preserve spine and peripheral joint strength and range of helpful in treating accompanying peripheral arthritis.

41
Systemic Lupus Erythematosus

Monitoring of patients with ankylosing spondylitis for Reactive Arthritis


response to therapy or progression of disease can include Reactive arthritis is usually self-limited, with symptoms
patient history, physical examination, and laboratory testing resolving within 6 months; in most cases, symptom-based
(such as erythrocyte sedimentation rate and C-reactive pro­ treatment is sufficient. NSA!Ds provide symptomatic relief,
tein). Serial imaging can also be used to help monitor patients and intra-articular glucocorticoid injections may provide local
with ankylosing spondylitis, but the 2010 Assessment of benefit. In patients who progress to severe and/or chronic
SpondyloArthritis international Society /European League disease, DMARDs such as sulfasalazine, methotrexate, or leflu­
Against Rheumatism (ASAS/EULAR) guidelines recommend nomide should be tried, although data for this indication are
against repeating spinal radiography more frequently than limited. A few case reports of benefit from TNF-a inhibitors
every 2 years unless absolutely necessary in specific cases. have been published, but more studies are needed.
Antibiotic therapy has not been shown to be effective in
Psoriatic Arthritis reactive arthritis except in rare cases of demonstrated persis­
NSA!Ds can improve symptoms of inflammation and are tent infection.
therefore commonly used for pain control but have not been
KEY POINTS
shown to prevent progression of erosions in psoriatic arthritis.
Glucocorticoids (systemically or by intra-articular injection) • Management of spondyloarthritis generally includes
can provide short-term relief, although flare-ups of psoriasis exercise to preserve strength and range of motion, glu­
may occur after discontinuation of systemic glucocorticoids. cocorticoid injections for symptomatic relief, and phar­
Nonbiologic DMARDs can treat peripheral arthritis and macologic therapy.
enthesitis but are unlikely to improve axial disease. • NSA!Ds are first-line therapy for ankylosing spondylitis;
Methotrexate has demonstrated efficacy in psoriatic arthritis tumor necrosis factor a inhibitors are used when
and also helps psoriasis. Sulfasalazine and hydroxychloro­ patients have had inadequate response to NSA!Ds.
quine can be useful for the arthritis, although there are reports
• In patients with psoriatic arthritis, NSA!Ds are typically
of hydroxychloroquine-induced exacerbation of psoriasis in
used to control inflammatory symptoms; nonbiologic
some patients.
disease-modifying antirheumatic drugs can treat
Several biologic agents are highly efficacious in psoriatic
peripheral arthritis and enthesitis, and biologic agents
arthritis and should be considered for more severe disease,
should be considered for more severe disease.
when nonbiologic DMARDs have provided inadequate benefit,
and/or in the presence of radiographic evidence of bony dam­ • Methotrexate or sulfasalazine can be used to treat both
age. TNF-a inhibitors can prevent progression of joint damage the arthritis and bowel disease associated with inflam­
and are also effective for treating the accompanying psoriasis. matory bowel disease-associated arthritis; tumor
Combination therapy with methotrexate and a TNF-a inhibi­ necrosis factor a inhibitors other than etanercept can
tor may be more helpful than monotherapy for some patients. be tried when nonbiologic agents fail.
Paradoxical worsening of psoriasis has been reported in some • Antibiotic therapy has not been shown to be effective in
patients receiving TNF-a inhibitors. Ustekinumab, a biologic reactive arthritis except in rare cases of demonstrated
agent that targets interleukin (IL)-12 and IL-23, is FDA persistent infection.
approved for use in both severe psoriasis and psoriatic arthri­
tis. Ustekinumab can slow progression of radiographic disease
and can be used in combination with methotrexate. Systemic Lupus
Inflammatory Bowel Disease-Associated Arthritis
Erythematosus
NSA!Ds may be contraindicated in patients with IBO due to Introduction
risk of bowel flare-up, limiting their usefulness in IBO­
Systemic lupus erythematosus (SLE) is characterized by mul­
associated arthritis. Glucocorticoids, systemically or by local
tiorgan involvement and the presence of autoantibodies,
injection, may provide short-term relief.
including antibodies directed at intranuclear antigens.
Methotrexate may ameliorate IBO-associated arthritis
and can also treat the underlying bowel disease, especially
Crohn disease. Similarly, sulfasalazine may also be effective for Pathophysiology
both IBO and IBO-related arthritis. SLE demonstrates a polygenic inheritance pattern. Clinical
With the exception of etanercept (a receptor-based rather disease results from the interaction of genes, environment,
than antibody-based biologic agent), TNF-a inhibitors are and random effects, resulting in loss of tolerance to self-anti­
effective treatments for IBO-associated arthritis that can be gens and active autoimmunity.
used when nonbiologic agents fail. However, paradoxical Patients with SLE have abnormalities in how dying cells
worsening of IBO has been reported in some patients receiving are handled by the immune system. The nuclear material of
these agents. dying cells may be inadequately cleared, engendering an

42
Systemic Lupus Erythematosus

immune response, and promoting up-regulation of autoreac­ bridge of the nose and potentially the forehead and chin; it
tive T and B cells and autoantibodies directed against nuclear characteristically spares the nasolabial folds. A more general­
and other antigens. Recent studies suggest a role for type 1 ized form of ACLE can also involve the dorsum of the arms and
interferons in SLE induction. Single gene mutations causing hands, including the areas between the fingers but sparing the
deficiencies of the complement components Clq, C2, or C4 can knuckle pads. Skin sequelae such as atrophy are not seen.
promote SLE, possibly by impairing clearance of immune Subacute cutaneous lupus erythematosus (SCLE) is a photo­
complexes and/or apoptotic cell debris. sensitive rash that occurs over the arms, neck, and face (Figure 18).
It consists of erythematous, annular, or polycyclic lesions, often
with a fine scale. SCLE may leave postinflarnmatory changes
Epidemiology (hypo- or hyperpigmentation) but does not cause atrophy. Anti-Ro/
Approximately 90% of patients with SLE are women, with SSA autoantibodies are present in 70% of patients with SCLE.
disease risk dramatically increasing with the appearance of The most common chronic cutaneous manifestation is
female sex hormones; the female-to-male ratio is 1:1 in child­ discoid lupus erythematosus (DLE) (Figure 19). DLE occurs in
hood versus 9:1 in adulthood. The disease is more common,
and often more severe, among women of African American,
Chinese, and Hispanic backgrounds.
KEY POINTS

• Systemic lupus erythematosus is characterized by mul­


tiorgan involvement and the presence of autoantibod­
ies, many directed at intranuclear antigens.
• Approximately 90% of patients with systemic lupus ery­
thematosus are women, with disease risk dramatically
increasing with the appearance of female sex hormones.

Clinical Manifestations
Mucocutaneous Involvement
FIGURE 1 8. Rash in subacute cutaneous lupus erythematosus. This patient has
Skin and/or mucous membranes are affected in 80% to 90% of
an annular polycyclic rash characterized by scaly erythematous circular plaques
patients with SLE; skin manifestations are classified as acute, with central hypopigmentation.
subacute, or chronic.
The characteristic SLE skin manifestation is acute cutane­
ous lupus erythematosus (ACLE), also known as malar or but­
terfly rash, which affects 40% to 50% of patients (Figure 17).
ACLE consists of erythema and edema over the cheeks and

FIGURE 19. Chronic cutaneous lupus erythematosus consists of chronic,


FIGURE 1 7. Malar (butterfly) rash. This patient has a classic acute cutaneous slowly progressive, scaly, infiltrative papules and plaques or atrophic red plaques
lupus erythematosus rash in a butterfly distribution that spares the nasolabial on sun-exposed skin surfaces, most commonly the face, neck, and scalp. Healed
folds. It is erythematous and raised, with a very slight scale. This rash will resolve lesions result in depressed scars, atrophy, telangiectasias, and hyperpigmentation
completely with treatment without leaving any skin atrophy. or hypopigmentation.

43
Systemic Lupus E rythematosus

20% of patients with SLE but more commonly occurs as an to need dialysis or transplant. Signs and symptoms defining
isolated, nonsystemic finding; patients with isolated OLE usu­ more severe lupus nephritis include hypertension. lower
ally do not go on to develop SLE. OLE usually affects the scalp extremity edema. active urine sediment (proteinuria. hematu­
and face and presents as hypo- or hyperpigmented, possibly ria. cellular casts). and elevated serum creatinine. Kidney biopsy
erythematous, patches or thin plaques that may be variably is frequently warranted to define the histological subtype and
atrophic or hyperkeratotic. In contrast to ACLE and SCLE, OLE the degree of disease activity and chronicity essential for plan­
can cause scarring, atrophy, and permanent alopecia. ning appropriate therapy. Indications for kidney biopsy are as
Painless oral or nasopharyngeal ulceration occurs in 5% of follows: increasing serum creatinine without explanation. pro­
patients with SLE. Nonscarring alopecia is a common feature teinuria >1000 mg 24 h. proteinuria >500 mg124 h with hema­
of active SLE, with hair regrowth a sign of disease control. turia. and proteinuria >500 mg,24 h with cellular casts.
Raynaud phenomenon occurs in 60%, reflecting arterial SLE patients with hypercoagulable states (for example.
vasospasm of the digits. the antiphospholipid antibody syndrome. the nephrotic syn­
drome) may be at risk for renal artery or vein thrombosis.
Musculoskeletal Involvement See Glomerular Diseases in MKSAP 17 Nephrology for
Joint involvement occurs in 90% of patients with SLE, with inflam­ more information on the six classes and treatment of lupus
matory polyarthralgia the most common presentation. Frank nephritis. Cl
arthritis occurs in 40% of patients with SLE. Both small and large
peripheral joints can be affected; the synovial fluid is only mildly Neuropsychiatric Involvement
inflammatory. SLE arthritis is nonerosive. However, persistent The American College of Rheumatology 1999 SLE classifica­
periarticular inflammation can damage joints supporting soft-tis­ tion criteria recognize 19 possible manifestations attributable
sue structures, resulting in reducible subluxation of the digits, to neuropsychiatric systemic lupus erythematosus (NPSLE)
swan neck deformities, and ulnar deviation (Jaccoud arthropathy). (Table 18). The most common manifestations are headache,
Pain or limitation of motion of the large joints, especially
the hips, should raise concern for possible osteonecrosis. Up to
TABLE 18. Neuropsychiatric Manifestations of Systemic
37% of patients with SLE develop osteonecrosis by serial MRI Lupus Erythematosus
scan of the hips, but less than 10% become symptomatic. Large
Central Nervous System
necrotic areas (for example, >20% of the femoral head) may
progress to bony collapse, whereas smaller lesions often Aseptic meningitis
resolve without structural perturbation. The occurrence of Cerebrovascular disease
osteonecrosis is attributable to glucocorticoid use and other Demyelinating syndrome
factors, including Raynaud phenomenon and lupus vasculitis ,
Headache
which may disrupt the blood supply to vulnerable bone vol­
Movement disorder (such as chorea)
umes. Prednisone doses greater than 20 mg/d and cushingoid
facial features are markers of risk. Patients with osteonecrosis Seizure disorder

have night pain and use pain. Early diagnosis is by MRI, with Myelopathy
more advanced disease visible on radiograph. Acute confusional state
Myalgia is reported by up to 85% of patients with SLE, but Anxiety disorder
frank myositis occurs in only about 10%. Histologically and clini­
Cognitive dysfunction
cally, SLE myositis resembles polymyositis. Medications such as
Mood disorder
antimalarials and glucocorticoids can cause drug myopathies that
must be ruled out in SLE patients with weakness. Fibromyalgia Psychosis
co-occurs in 30% of patients with SLE and is important to diagnose Peripheral Nervous System
to avoid wmecessary use ofirnrnunosuppressive medications.
Acute inflammatory demyelinating polyradiculoneuropathy

Cl
(such as Guillain-Barre syndrome)
Kidney Involvement
Autonomic neuropathy
Lupus nephritis occurs in up to 70% of patients with SLE: the
presence or anti-double-stranded DNA antibodies is a marker Mononeuropathy (single or multiplex)
llir risk. All patients with SLE should be evaluated for possible Myasthenia gravis
nephritis with baseline serum creatinine. urine protein-creati­ Cranial neuropathy
nine ratio for proteinuria. and urinalysis with microscopic eval­ Plexopathy
uation. Minor abnormalities (proteinuria <500 mg/24 h.
Polyneuropathy
minimal hematuria. no casts) should be monitored regularly
Cl-month intervals). whereas significant initial abnormalities Adapted with permission from John Wiley & Sons, from American College of
Rheumatology nomenclature and case definitions for neuropsychiatric lupus syn­
(especially cellular casts) require immediate rurther e,·aluation. dromes. Arthritis Rheum. 1999;42(4):599-608. [PMID: 10211873] Copyright 1999
American College of Rheumatology.
Patients with inadequately treated lupus nephritis may progress

44
Systemic Lupus Erythematosus

cognitive dysfunction, and mood disorder. NPSLE prevalence pathogenic role for SLE inflammation/autoimmunjty. Cardiac
is as high as 75%; prevalence of more acute presentations (for MRI is increasingly used to detect myocardial inflammation.
example, seizures and psychosis) is significantly lower. In Cardiac valves are abnormal on transesophageal echocar­
addition to central manifestations, peripheral neuropathy cliogram in many patients with SLE, most characteristically
occurs in up to 14% of SLE patients, with the majority due to those with APLA/LAC. Thickening of the mitral and aortic
the disease itself and the remainder due to non-SLE causes valve leaflets are the most common abnormalities. but vegeta­
(medications, other diseases such as diabetes mellitus, spinal tions, regurgitation, and stenosis also occur. Libman-Sacks
radiculopathies). endocarditis. the classic designation for noninfectious verru­
NPSLE pathophysiology may include vascular inflamma­ cous vegetations, favors the mitral valve. These verrucous
tion and/or occlusion, the effects of cytokines on neuronal masses can embolize. leading to downstream occlusion.
function, or antibodies directed against various neuronal or Patients with SLE have a 2- to 10-fold increased preva­
glial components. The blood-brain barrier generally prevents lence of ischemic heart disease, the most common cause of
passage of antibodies into the central nervous system (CNS) death among older patients with SLE. Severe SLE disease
but with inflammation becomes permeable. Some of the anti­ activity and prednisone closes of more than 20 mg/cl are inde­
bodies/cytokines in NPSLE may also be produced within the pendent risk factors for early myocardial infarction. Traditional
CNS itself. risk factors such as hypertension and total cholesterol levels
The general approach to suspected NPSLE is to first also play a role and require clinical management.
exclude an infection or medication effect. Evaluation of more
severe manifestations such as seizures or meningitis requires Pulmonary Involvement
CNS imaging, cerebrospinal fluid (CSF) analysis, and measure­ Pleuritis occurs in 45% to 60% of patients who have SLE. with
ment of NPSLE-associated autoantibodies. Autoantibodies or without pleural effusion. When present. effusions are
that may be found in the serum and/or CSF in patients with exudative, and lupus erythematosus cells and ANA may
NPSLE include antineuronal, anti-NMDA receptor, antiriboso­ be present.
mal P, and antiphospholipid antibodies/lupus anticoagulant Parenchymal lung involvement is relatively uncommon in
(APLA/LAC). For patients with suspected peripheral neuropa­ SLE: lung infiltrates are more likely to be infectious than
thies, electromyography (EMG) and nerve conduction studies directly associated with SLE. Nonetheless, interstitial lung
(NCS) are useful in the diagnosis and potential cause of the disease occurs in 3% to 8'X, of patients with SLE. Acute lupus
neuropathy. EMG/NCS should be able to distinguish a spinal pneumonitis is a potentially serious inflammatory airway dis­
radiculopathy (non-SLE cause) from a mononeuritis (likely ease characterized by fever. cough. shortness of breath, hypox­
associated with SLE), for example. emia, and pleuritic chest pain. Chest radiograph may show
Imaging of the patient with NPSLE should be influ­ unilateral or bilateral infiltrates. Mortality rates approach 50%,
enced by the clinical manifestations and is often done to rule and aggressive respiratory support combined with immuno­
out other possibilities. Imaging may include CT to rule out suppression is required. An even rarer manifestation is diffuse
CNS bleeding or stroke; MRI to identify infection, white alveolar hemorrhage, which typically presents with shortness
matter lesions, or myelopathy in a patient with suspected of breath, hypoxemia, diffuse alveolar infiltrates on chest radi­
transverse myelitis; and PET to evaluate functional ograph. a dropping hematocrit, and a high DLCO on pulmonary
abnormalities in patients with cognitive impairment. function tests, all in the setting of active SLE. Bronchoscopy
Neuropsychologic testing may help identify organic versus with bronchoalveolar lavage and biopsy is used to demonstrate
functional cognitive changes. hemorrhage and rule out infection. Chest MRI may detect
blood as changes on T2-weighted images. Aggressive immuno­
Cardiovascular Involvement suppressive therapy and respiratory support are required, but
Up to 40% of patients with SLE experience pericarditis. which mortality rates are as high as 50% to 90%.
is often asymptomatic. Pericarditis may be associated with a When evaluating a patient with SLE who has pulmonary
neutrophilic pericardia! effusion that can rarely lead to tam­ infiltrates, it is usually prudent to treat empirically for a pos­
ponade. Antinuclear antibodies (ANA). anti-double-stranded sible infectious cause. Antibiotics and immunosuppressive
DNA antibodies, and so-called lupus erythematosus cells therapy are generally started simultaneously and modified in
(neutrophils engulfing extruded cell nuclei) can be detected in response to additional diagnostic data. Opportunistic infec­
the fluid but rarely alter diagnosis or management. Constrictive tions should be considered in a patient who is immunosup­
pericarclitis may also occur. Pericardia! disease can be demon­ pressed and/or not responding appropriately to therapy.
strated by CT or MRI. Shrinking lung syndrome is characterized by pleuritic
Myocarditis occurs in 5% to 10% of patients with SLE, chest pain and shortness of breath, with progressive decrease
more commonly among black patients. Presentations can be in lung volumes on chest radiograph and pulmonary function
dramatic, with acute onset of hea1i failure. Echocardiogram­ tests. Aggressive immunosuppression may reverse the pro­
measured ejection fractions of less than 20% are not uncom­ cess. The cause is uncertain. but pleuropulmonary disease or
mon. Rapid response to high-dose glucocorticoids supports a diaphragmatic dysfuncLion is believed to contribute.

45
Systemic Lupus Erythematosus

CJ Hematologic Involvement P antibodies in SLE hepatitis. Very rarely, pancreatitis associ­


ated withSLE has been reported due to thrombosis, vasculitis,
COIIT.
All three bone marrow cell lines can be affected in SLE.
Leukopenia occurs in 50% of patients, with lymphopenia pre­ or medication complications. Cl
dominating. Whereas hemolytic anemia with direct antiglobulin

CJ
(Coombs) test positivity is seen in only 10%, up to 80% of patients Malignancy
with SLE have normocytic, normochrom.ic anemia of chronic In patients withSLE. there is a sevenfold increased risk of non-
disease. Thrombocytopenia occurs in 30% to 50% and is gener­ Hodgkin lymphoma, especially diffuse large B-cell lymphoma.
ally mild. Severe thrombocytopenia (<50.000/pL (50 x 109/L]) compared with the general population. This increased risk
occurs in 10% ofSLE patients in isolation or in conjunction with may relate to chronic B-cell activation and/or use of medica-
hemolytic anemia (Evans syndrome): isolated idiopathic throm­ tions (for example, azathioprine or cyclophosphamide) that
bocytopenic purpura may be a clue to underlying SLE. APLA/ promote hematologic malignancies. Hodgkin lymphomas and
LAC is also associated with thrombocytopenia. ln patients with leukemias are also more common in SLE. presumably for
quiescent SLE, mild cytopenias do not require intervention. similar reasons. Lung cancer rates are increased by 1.4 times
SLE cytopenias are driven by both immune and nonim­ the general population risk, and patients with SLE who
mune mechanisms. Immune-mediated hematologic conditions develop Jung cancer are invariably smokers. Cl
een in SLE include autoimmune hemolytic anemia. Evans Cervical cancer risk is increased and associated with both
syndrome. aplastic anemia. and pure red cell aplasia. immunosuppressive (cyclophosphamide) use and increased
Nonimmune causes include chronic SLE inflammation, infec­ prevalence of human papillomavirus in women with SLE.
tions. medications. myelodysplasia, and myelofibrosis. It may be Women with SLE are less likely to undergo cervical screening
difficult to distinguish disease activity from drug effect. and than women in the general population, possibly related to care
empiric withdrawal of a drng may be required. Indications for by specialists who are less likely to provide such screening.
bone marrow biopsy include the following: no obvious explana­ Certain solid organ tumors, notably breast, endometrial, pros­
tion for cytopenia. elevated serum iron. elevated mean corpus­ tate, and ovarian, may be less common in patients withSLE.
cular volume unexplained by medication effect. serum mono­ KEY POINTS
clonal spike, and the antiphospholipid antibody syndrome.
APLA/LAC antibodies are detected in 40% of patients • The characteristic skin manifestation of systemic lupus
erythematosus is acute cutaneous lupus erythematosus
with SLE. but fewer than 40% of these patients are clinically
( also known as malar or butterfly rash), which is char­
affected. The APLA/LACs are autoantibodies that interact with
acterized by erythema and edema over the cheeks and
endothelial cells, rnonocytes, platelets. and complement to
bridge of the nose.
cause a prothrombotic state. The common clinical manifesta­
tions include venous and arterial thrombosis. miscarriage • Musculoskeletal involvement, including joint manifesta­
(especially second trimester). livedo reticularis. cytopenias. tions (most commonly inflammatory polyarthralgia)
and cardiac valve thickening/vegetations. LAC carries a higher and myalgia, is common in patients with systerruc lupus
risk ofthrombosis than other APLAs: however. the highest risk erythematosus.
is in patients who are triple positive for LAC. anti-P 2- • All patients with systerruc lupus erythematosus should
glycoprotein I. and anticardiolipin antibodies, who have a 30% be evaluated for nephritis with baseline serum creati­
rate of thrombosis. Patients with SLE are at increased risk for nine, urine protein-creatinine ratio, and urinalysis with
thrombosis even in the absence of detectable APLA/LAC. microscopic evaluation.
See Thrombotic Disorders in MKSAP 17 Hematology and • The most common neuropsychiatric manifestations of
Oncology for a discussion on the antiphospholipid antibody systerruc lupus erythematosus are headache, cognitive
syndrome. dysfunction, and mood disorder.
Gastrointestinal Involvement • Pericarditis is the most common acute cardiac manifes­
Gastrointestinal disease occurs in 25% to 40% of patients with tation of systerruc lupus erythematosus (SLE); ischerruc
SLE and may be underrecognized. Esophageal dysmotility heart disease is the most common cause of death for
and/or reflux occurs in 5% of patients. especially those with older patients withSLE.
Raynaud phenomenon and anti-Ul-ribonucleoprotein anti­
bodies. Sterile peritonitis is a potential cause of abdominal
pain. Mesenteric vasculitis can occur. often accompanied by Diagnosis
cutaneous vasculitis. Mesenteric thrombosis can occur in SLE should be considered in any patient who presents with
patients with the antiphospholipid antibody syndrome. unexplained multisystem disease. The most common earlySLE
Hepatic involvement occurs in 10% and is distinguished from manifestations include constitutional symptoms (fever, weight
primary autoimmune hepatitis by lower aminotransferase loss, or severe fatigue), arthralgia/arthritis, and skin disease.
levels. lack of autoimmune hepatitis-associated autoantibod­ SLE should be included in the differential of a patient (especially
ies (anti-smooth muscle, anti-liver/kidney microsomal). dif­ a young woman) with any of the following manifestations: per­
ferent histology on biopsy, and the presence of antiribosomal sistent rash, particularly if it is photosensitive; polyarticular
46
Sys t emic Lupus Ery thematosus

arthritis; pleuritis; pericarditis; unexplained cytopenias, espe­ immunofluorescence ANA is highly sensitive (95%) for SLE
cially lymphopenia; a nephritic urine sediment; or thromboem­ and may convey additional information based on the immu­
bolic disease. The American College of Rheumatology has nofluorescence pattern observed, but its specificity is limited
defined ll criteria, with the presence of any 4 serially or simul­ because it can be positive in patients with other autoimmune
taneously sufficient to support the diagnosis of SLE, typically diseases or in those without clinical autoimmune disease
including a positive ANA (Table 19). Myalgia, arthralgia, and (usually at low titer). Therefore, ANA should always be inter­
fatigue are insufficient reasons to test for ANA unless accompa­ preted in the context of pretest probability.
nied by objective SLE findings. Patients with these nonspecific If ANA is positive, SLE-specific autoantibodies (anti­
symptoms should first be evaluated for more common condi­ double-stranded DNA, anti-Smith, anti-Ul-ribonucleopro­
tions such as anemia, thyroid disease, and fibromyalgia. lf unex­ tein, and anti-La/SSB), as well as tests for other autoimmune
plained findings such as cytopenias or organ involvement are diseases under consideration, should be obtained to further
found, SLE should then be added to the differential diagnosis. characterize the disease (Table 20). Anti-Ro/SSA antibody test­
ing may simultaneously occur because a small percentage of
Laboratory Studies patients with SLE are negative for ANA but positive for anti­
Initial laboratory studies for SLE include an ANA test to screen Ro/SSA antibodies. Conversely, a negative ANA plus a negative
for nuclear-directed autoantibodies. The traditional indirect anti-Ro/SSA essentially rules out SLE. In many commercial

TABLE 19. American College of Rheumatology Criteria for the Diagnosis of Systemic Lupus Erythematosus
Criteria• Definition

Malar rash Fixed erythema, flat or raised, over the malar eminences

Discoid rash Erythematous, circular, raised patches with keratotic scaling and follicular plugging; atrophic scarring
may occur

Photosensitivity Rash after exposure to ultraviolet light

Oral ulcers Oral and nasopharyngeal ulcers (observed by physician)

Arthritis Nonerosive arthritis of �2 peripheral joints, with tenderness, swelling, or effusion

Serositis Pleuritis or pericarditis (documented by electrocardiogram, rub, or evidence of effusion)

Kidney disorder Urinalysis: 3+ protein or urine protein >500 mg/24 h; cellular casts

Neurologic disorder Seizures or psychosis (without other cause)

Hematologic disorder Hemolytic anemia or leukopenia ( <4000/µL [4.0 x 109/L]) or lymphopenia ( <1500/µL [1.5 x 109/L]) or
thrombocytopenia (<100,000/µL [100 x 109/L]) in the absence of offending drugs

Immunologic disorder Anti-double-stranded DNA, anti-Smith, and/or antiphospholipid antibodies

ANA An abnormal titer of ANA by immunofluorescence or an equivalent assay at any point in the absence
of drugs known to induce ANA

ANA= antinuclear antibodies.

aAny combination of 4 or more of the 11 criteria, well documented at any time during a patient's history, makes it likely that the patient has systemic lupus erythematosus (specific­
ity and sensitivity are 95% and 75%, respectively).

Adapted with permission from John Wiley & Sons, from Hochberg MC. Updating the American College of Rheumatology revised criteria for the classification of systemic lupus
erythematosus. Arthritis Rheum. 1997;40(9): 1725. [PMID: 9324032] Copyright 1997 American College of Rheumatology.

TABLE 20. Common Autoantibodies in Systemic Lupus Erythematosus


Autoantibody Frequency in SLE Comments

Antinuclear 95% Useful as an initial screening test

Anti-double-stranded DNA 50% Found in more severe disease, especially kidney disease; antibody levels
commonly follow disease activity and are useful to monitor

Anti-Ro/SSA 40% Associated with photosensitive rashes, discoid lupus, and neonatal SLE,
including congenital heart block; also common in Sjogren syndrome

Anti-U 1-ribonucleoprotein 35% Associated with Raynaud phenomenon and esophageal dysmotility; also seen
in MCTD

Anti-Smith 25% Specific for SLE; often associated with more severe disease

Anti-La/SSB 15% Common in Sjogren syndrome; less common in SLE and neonatal SLE

Antiribosomal P 15% Associated with CNS lupus and lupus hepatitis

CNS= central nervous system; MCTD = mixed connective tissue disease; SLE = systemic lupus erythematosus.

47
Systemic Lupus E rythematosus

laboratories, the sequential testing of autoantibodies has antihistone antibodies. Diagnosis is confirmed when symp­
recently been automated. Although these new test sequences toms resolve after discontinuing the offending agent.
may simplify physician assessment, they often use a different Patients with SLE are at no more risk of DILE than the gen­
method for assessing the initial ANA (ELISA rather than indi­ eral population.
rect immunofluorescence) that may have reduced sensitivity High-risk medications for DILE include procainamide,
and specificity. methyldopa. quinidine, and hydralazine; all but hydralazine
A complete blood count, chemistry panel, and urinalysis are now rarely used in current practice. Some drugs are
(including microscopy) should be ordered, along with an lower risk (for example, minocycline), but because of fre­
erythrocyte sedimentation rate (ESR) and potentially a quent use, more cases develop. Table 21 lists medications
C-reactive protein level (CRP). ESR is preferred over CRP ini­ associated with DILE, the autoantibodies detected, and clin­
tially because some patients with SLE do not generate CRP ical manifestations. Cl
during SLE flares. However, monitoring CRP in patients with
SLE and defining those who do or do not generate CRP during Undifferentiated Connective Tissue Disease
flares may be helpful in distinguishing flares from infection. Undifferentiated connective tissue disease (UCTD) refers to
Complements should also be assessed (most commonly, C3 syndromes with objective abnormalities that do not meet suf­
and C4) because these levels are reduced during SLE activity, ficient criteria to be categorized as SLE or another specific
reflecting immune complex formation that provokes comple­ connective tissue disease. Patients who have two or three cri­
ment activation. These studies may help define the presence of teria for SLE can be diagnosed as having UCTD or alternatively
organ involvement and the level of inflammation as well as may be designated as having incomplete SLE.
provide a baseline for following disease activity.
KEY POINTS

Differential Diagnosis • The most common early manifestations of systemic


lupus erythematosus include constitutional symptoms,
The differential diagnosis of SLE includes other autoimmune
arthralgia/arthritis, and skin disease.
multisystem diseases, many of which are similarly more com­
mon in women: ANCA-associated vasculitis, rheumatoid • Initial testing for systemic lupus erythematous (SLE)
arthritis, adult-onset Still disease, dermatomyositis, Sjogren begins with antinuclear antibodies; if positive, SLE­
syndrome, and mixed connective tissue disease. Severe viral specific autoantibodies (anti-double-stranded DNA, anti­
infection can also mimic SLE (fever, myalgia, rash, cytopenias). Smith, anti-Ul-ribonucleoprotein, and anti-La/SSB)
should be obtained to further characterize the disease.

Cl Drug-Induced Lupus Erythematosus


Drug-induced lupus erythematosus (DILE) occurs when a
• In addition to antinuclear antibody and other autoanti­
body testing, laboratory studies for evaluating systemic
medication triggers immune-mediated disease with mani­ lupus erythematosus include a complete blood count,
festations mimicking SLE. Common symptoms include chemistry panel, complement levels, erythrocyte sedi-
malaise. fever. arthritis. and rash; kidney and CNS disease mentation rate, and urinalysis.
(Continued)
are uncommon. ANA is often transiently positive, along with

TABLE 21. Medications Commonly Associated with Drug-Induced Lupus Erythematosus


Medication Antibodies Detected Comments

Procainamide ANA; antihistone 75% ANA positive; 20% develop DILE; fever; arthritis;
serositis

Hydralazine ANA; antihistone 20% ANA positive; 5%-8% develop DILE; fever;
arthritis; rare vasculitis and kidney disease

Minocycline ANA; ANCA; anti-dsDNA rare Arthritis; vasculitis; autoimmune hepatitis

Antithyroid drugs ANA; ANCA; antihistone Vasculitic rash; rare pulmonary and kidney disease
Statins ANA; antihistone; anti-dsDNA SLE, SCLE, dermatomyositis, and polymyositis all
reported
Calcium channel blockers ANA; anti-Ro/SSA; antihistone rare SCLE
Thiazide diuretics ANA; anti-Ro/SSA; antihistone rare SCLE
ACE inhibitors ANA; anti-Ro/SSA; antihistone rare SCLE
TNF-a inhibitors ANA in 23%-57%; chromatin and anti­ DILE most common with infliximab, uncommon for
dsDNA common; antihistone rare etanercept; SLE, SCLE, DLE all reported

ANA= antinuclear antibodies; DILE= drug-induced lupus erythematosus; OLE= discoid lupus erythematosus; dsDNA= double-stranded DNA; SCLE = subacute cutaneous lupus
erythematosus; SLE= systemic lupus erythematosus; TNF= tumor necrosis factor.

48
Systemic Lupus Erythematosus

KEY PO I NT S (continued) For many acute manifestations ofSLE. glucocorticoids are


the initial Lreatmenl based on their ef'ficacy and rapid onset of
CJ
• Drug-induced lupus erythematosus occurs when a
medication triggers immune-mediated disease with action. Glucocorticoid dose should be appropriate to the dis-
manifestations mimicking systemic lupus erythemato­ ease severity and the organs involved. For SLE-related arthri-
sus; treatment includes discontinuing the offending tis. rash. pleuritis, pericarditis. or mild cytopenias, a dose of20
agent. lo 40 mg d of prednisone may be appropriate. Patients with
more severe disease (for example, class II-IV nephritis, more
• Undifferentiated connective tissue disease refers to syn­
severe cytopenias. or severe pleural or pericardia! disease) may
dromes with objective abnormalities that do not meet
require high-dose glucocorticoids (40 to 60 mg1d of pred­
sufficient criteria to be categorized as systemic lupus
nisone). The most severe manifestations (for example, rapidly
erythematosus or another specific connective tissue
progressive class Ill-IV nephritis. psychosis. seizures. diffuse
disease.
alveolar hemorrhage, or myocarditis) should generally be
treated in Lhe hospital with high-dose intravenous glucocorti-
coicls (1000 mg ct for 3 days) followed by 40 to 60 mg/cl of oral
Management preclnisone. Given the many adverse effects associated with
In virtually all cases,SLE management requires pharmacologic glucocorticoicl use, glucocorticoids alone are generally not
therapy (Table 22). Serial monitoring of disease-responsive used for chronic therapy, and additional therapy is quickly
laboratory studies (complete blood count, ESR, anti-double­ added for disease control and glucocorticoid sparing.
stranded DNA antibodies, complements [C3 and C4], urinaly­ Prednisone is generally tapered as tolerated after the f1rst
sis, and urine protein-creatinine ratio) is mandatory to track month of' therapy with the goal of reducing the dose to less
disease activity, identify warning signs of disease flare, and than 10 mgid by 3 months.
monitor response to therapy. Frequency of monitoring For severe SLE. the goal is to control autoimmunity and
depends upon current level of disease activity. Modification of inffammation and induce disease remission with potent med­
therapy should generally be done with input from the patient's ications. then substitute with less potent and less toxic therapy
rheumatologist and/or nephrologist. to maintain the remission. For active lupus nephritis,
Hydroxychloroquine is an anchor drug for SLE and mycophenolale mofetil is currently the preferred oral agent:
should be initiated in every patient withSLE who can toler­ dosage is 2000 to 3000 mg cl and may take 4 to 6 weeks to
ate the medication. This agent can prevent SLE flares, reach full efficacy. For the most severe disease (severe active
improve outcomes in high-risk pregnancies, reduce the risk nephritis, acute CNS lupus, pulmonary hemorrhage, myocar­
of thrombosis and myocardial infarction, reduce disease­ ditis). intravenous cyclophosphamide in a dosage of 500 mg
associated damage in general, and improve survival. It is every 2 weeks for 6 doses or 750 mg m 1 every month for
particularly useful for arthritis and the skin manifestations. 6 months is usually used lo induce remission, followed by
Hydroxychloroquine may be the only medication needed for mycophenolate mofetil or possibly azathioprine as mainte­
mild disease. nance therapy. The biologic agent belimumab is approved for

TABLE 22. Medications Commonly Used to Treat Systemic Lupus Erythematosus


Medication Common Uses in SLE Important Side Effects

NSAIDs Painful, non-organ-threatening manifestations Hypertension; GI bleeding; AKI


such as arthritis

Prednisone Used for all manifestations in varying doses Hypertension; glucose intolerance; weight gain;
infection; osteonecrosis

Hydroxychloroquine Used for mild to moderate disease; especially Rash; retinopathy; vacuolar myopathy
useful for skin involvement and to prevent
disease flares

Mycophenolate mofetil Moderate to severe disease; as effective as Bone marrow suppression; elevation of liver
cyclophosphamide for remission induction for enzymes; infection
nephritis

Azathioprine Moderate to severe disease Bone marrow suppression; elevation of liver


enzymes; hematologic malignancy

Cyclophosphamide Severe organ or life-threatening disease Bone marrow suppression; hemorrhagic cystitis;
infection; malignancy

Belimumab Add-on therapy for moderate to severe Infusion reactions; infections


disease

AKI= acute kidney injury; GI= gastrointestinal; SLE = systemic lupus erythematosus.

49
Sjogren Syndrome

Cl
(Off
patients with incomplete responses to convernional treat­
ments. See Glomerular Diseases in MKSAP 17 Nephrology for
details on the treatment or lupus nephritis.
Medications that are safe for use during SLE pregnancies
include hydroxychloroquine, prednisone, and azathioprine
(if an immunosuppressive agent is absolutely needed).
NSAIDs can be usef"ul for management or patients with Cyclophosphamide-induced infertility is age- and dose­
arthritis. pleurilis. or pcricarditis. These agents should be dependent and should be discussed with female patients of
avoided in patients with kidney disease and/or uncontrolled reproductive age before administering this medication. See
hypertension. C] Principles of Therapeutics for more information on medica­
See Principles of Therapeutics for additional information tions and pregnancy.
on SLE medication toxicities, monitoring parameters, and KEY POINTS
more.
• Women with systemic lupus erythematosus (SLE) expe­
KEY POINTS rience miscarriage, stillbirth, and premature delivery
• Serial monitoring of disease-responsive laboratory stud­ two to five times more often than women without the
ies (complete blood count, e1ythrocyte sedimentation disease; furthermore, SLE pregnancies are at an eight­
rate, anti-double-stranded DNA antibodies, comple­ fold risk for intrauterine growth retardation.
ments, urinalysis, and urine protein-creatinine ratio) is • Fetuses of mothers who are positive for anti-Ro/SSA
mandatory to track disease activity, identify warning or anti-La/SSB antibodies are at risk for developing
signs of disease flare, and monitor response to therapy. neonatal lupus erythematosus.
• Hydroxychloroquine should be initiated in every patient
with systemic lupus erythematosus who can tolerate
the medication. Prognosis
• Glucocorticoids are usually the initial therapy for acute SLE prognosis has improved significantly from the 1960s (SO%
manifestations of systemic lupus erythematosus, with S-year survival rate) to the present (>90% s-year survival rate).
dosing based on disease severity. Nonetheless, there continues to be a bimodal mortality for
• For severe systemic lupus erythematosus, the goal is to SLE, with early deaths related to SLE and infections and late
control autoimmunity and inflammation and induce mortality associated with cardiovascular disease. Factors
disease remission with potent medications, then substi­ adversely affecting survival include myocarditis, nephritis, low
tute with less potent and less toxic therapy to maintain socioeconomic status, male gender, and age over SO years
the remission. at diagnosis.
A complete S-year remission in SLE is uncommon, occur­
ring in only 2% of patients. Current data suggest 20% to 40% of
patients have an incomplete response to available therapies,
Pregnancy and Childbirth Issues
indicating the need for better therapeutic approaches.
Women with SLE expe1ience miscarriage, stillbirth, and prema­
ture delivery two to five times more often than women without KEY POINT

the disease. SLE pregnancies are at an eightfold risk for intrauter­ • Factors adversely affecting survival of patients with
ine growth retardation. Active disease, especially nephritis, and systemic lupus erythematosus include myocarditis,
the presence of anti-Ro/SSA and/or APL antibodies are risk fac­ nephritis, low socioeconomic status, male gender, and
tors for fetal morbidity and mortality Conception should ideally age over SO years at diagnosis.
be deferred until SLE disease is qui�scent.
Proteinuria may increase during pregnancy in patients
with SLE, rendering differentiation from preeclampsia/
eclampsia a challenge. Increases in anti-double-stranded DNA Sjogren Syndrome
antibody levels or the acute development of an active urine
sediment implicates SLE as the cause. In contrast, serum urate Introduction
rises in preeclampsia but not during lupus flares. Sjogren syndrome is an immune-mediated disease of
Fetuses of mothers who are positive for anti-Ro/SSA or unknown cause manifesting as infiltrative inflammation that
anti-La/SSB antibodies are at risk for developing neonatal damages exocrine glands, including the major and minor sali­
lupus erythematosus. Arthritis and rash are common manifes­ vary glands, lacrimal glands, and, less commonly, other exo­
tations; in 2% of these pregnancies, congenital heart block may crine glands such as the pancreas.
occur and cause mortality or require permanent pacing. Risk
of neonatal lupus erythematosus rises to 12% after a first child
born with the condition. Recent studies suggest that maternal Pathophysiology
hydroxychloroquine use may reduce the risk of neonatal lupus Biopsied glandular tissue from patients with Sjogren
erythematosus. syndrome reveals inflammatory infiltrates composed of

50
Sjogren Syndrome

CD4-positive T lymphocytes, accompanied by lesser popula­ (See MKSAP 17 Hematology and Oncology). Sjogren syndrome
tions of B and plasma cells. Immune system dysregulation, lymphoma risk is 16- to 44-fold that of the general population.
including B-cell hyperactivity and hypergammaglobulinemia, Hypocomplementemia and lymphopenia at the time of
is commonly reported. Sjogren diagnosis may predict lymphoma development. New
and persistent adenopathy or other symptoms suggestive of
lymphoma should prompt further evaluation with lymph
Epidemiology node biopsy.
The prevalence of Sjogren syndrome ranges from 0.19% to
1.39%, depending upon the classification criteria used.
However, Sjogren syndrome may be underdiagnosed, such Diagnosis
that its prevalence may be higher than reported. Incidence Diagnosis of Sjogren syndrome is based primarily on typical
peaks around the fifth decade, and there is a female predomi­ sicca symptoms as well as glandular (enlarged lacrimal and/or
nance (9:1 ratio). parotid glands) and extraglandular manifestations. It is helpful
to confirm eye and mouth dryness in an objective manner, for
example, by documenting reduced tear production utilizing
Clinical Manifestations the Schirmer test (decreased wetting of tear test strips) or with
The prominent clinical feature of Sjogren syndrome is sicca, or special stains and slit-lamp examination. Rarely, gallium scan­
dryness, particularly of the eyes (keratoconjunctivitis sicca) ning or sialography may be warranted to characterize the
and mouth (xerostomia). Keratoconjunctivitis sicca can result exocrine gland involvement.
in corneal damage and visual impairment. Xerostomia can Laboratory findings include positive autoimmune serolo­
result in dental caries due to loss of antibacterial features of gies (rheumatoid factor, antinuclear, anti-Ro/SSA, and anti­
saliva. Dryness of mucosal surfaces (such as the vagina, skin, La/SSB antibodies) and hypergammaglobulinemia. Anti-Ro/
or bronchi) and exocrine gland hypertrophy are common SSA and anti-La/SSB antibodies are characteristic for Sjogren
findings. Sjogren syndrome can also cause extraglandular syndrome but are also common in patients with systemic
manifestations as a result of autoimmune/inflammatory lupus erythematosus, mothers of infants with neonatal lupus,
mechanisms (Table 23). and, occasionally, healthy persons. Rheumatoid factor levels
Presumably because of preexisting lymphocyte activa­ are typically higher than those seen in rheumatoid arthritis. In
tion, patients with Sjogren syndrome have an increased risk of the presence of classic historical and physical findings, the
lymphoma, with diffuse large B-cell and mucosa-associated presence of anti-Ro/SSA and anti-La/SSB antibodies may be
lymphoid tissue (MALT) lymphomas being the most common sufficient to diagnose Sjogren syndrome. In unclear cases, a lip
biopsy demonstrating minor salivary gland inflammation is
considered the gold standard for diagnosis.
TABLE 23. Extraglandular Clinical Manifestations '>. · ' Other autoimmune diseases, including rheumatoid
of Sjogren Syndrome ,_ arthritis, systemic lupus erythematosus, and autoimmune
Site/Organ Manifestation/Frequency thyroiditis, are commonly associated with Sjogren syndrome.
General Fatigue (70%), fever (6%) This has traditionally been called secondary Sjogren syn­
drome, although recent classification criteria do not make this
Skin Rash, cutaneous vasculitis: 10%-16%
distinction. Conditions that mimic Sjogren syndrome include
Joint Arthralgia/arthritis: 36%
IgG4-related disease, graft versus host disease, amyloidosis,
Lung Interstitial pneumonitis: 5%-9% sarcoidosis, AIDS (diffuse infiltrative lymphocytosis syn­
Kidney Interstitial nephritis, distal (type 1) renal drome), hepatitis C virus infection, and history of head and
tubular acidosis, glomerulonephritis:
neck irradiation. Many of these conditions share with Sjogren
5%-6%
syndrome a predilection for infiltration of exocrine glands,
Neurologic Central nervous system (CNS):
demyelinating disease, myelopathy,
sicca symptoms, and/or positive antinuclear antibodies or
cranial nerve neuropathy rheumatoid factor, rendering diagnosis difficult.
Peripheral nervous system: small-fiber
neuropathy, mononeuritis multiplex,
peripheral neuropathy: 8%-27% (for CNS Management
and peripheral) Management of Sjogren syndrome consists of symptomatic,
Gastrointestinal Autoimmune hepatitis, primary biliary local, and systemic approaches (Table 24). Sicca symptoms are
cirrhosis: 3%-20%
treated with hydration and lubrication, although other local
Hematologic Lymphoma, cytopenia: 2% measures and medications may be helpful. Avoidance of med­
Other Systemic vasculitis (7%), cryoglobulinemia ications that worsen sicca (for example, anticholinergic
(4%-12%), Raynaud phenomenon (16%), agents) is recommended. Immunosuppressive therapy does
thyroid disease (10%-15%)
not alleviate sicca symptoms but may suppress extraglandular

51
Mixed Connective Tissue Disease

TABLE 24. Management of Sjogren Syndrome


Mixed Connective
Tissue Disease
Symptom Therapy

Ocular sicca Artificial tears; glasses with side panels


to reduce exposure; punctal plugs/
occlusion; topical cyclosporine
Introduction
Mixed connective tissue disease (MCTD) is an overlap syn­
Oral sicca Artificial saliva; sugar-free lozenges to
promote salivation; good dental drome that includes features of systemic lupus erythematosus
hygiene; muscarinic cholinergic receptor (SLE), systemic sclerosis, and/or polymyositis in the presence
stimulators (e.g., pilocarpine, cevimeline)
of anti-Ul-ribonucleoprotein (RNP) antibodies.
Vaginal/Skin sicca Topical lubrication

Extraglandular Mild symptoms (e.g., arthralgia or rash):


NSAIDs; hydroxychloroquine; topical or Epidemiology
low-dose glucocorticoids
MCTD is rare (l:1,000,000). Age at onset is between 30 and
Moderate to severe symptoms (e.g.,
50 years, with a 9:1 female predominance. Most patients have
lung, kidney, or nervous system
involvement; vasculitis): high-dose no known risk factors.
glucocorticoids; immunosuppressive
therapy such as methotrexate,
azathioprine, mycophenolate mofetil, Clinical Manifestations
cyclophosphamide, or rituximab
and Diagnosis
More than 50% of patients with MCTD have hand edema and
synovitis at disease onset. About one third develop myositis,
manifestations. Biologic agents have not been proven to treat and nearly half develop decreased esophageal motility and
Sjogren syndrome, although some reports suggest responsive­ fibrosing alveolitis. Pulmonary arterial hypertension occurs in
ness to the anti-B-cell antibody rituximab.
20%, with fatigue often the initial symptom. Patients sus­
pected of having MCTD should undergo high-resolution CT of
Prognosis the chest, echocardiography, and pulmonary function testing.
Clinical findings may evolve, such that a patient who initially
Most patients with Sjogren syndrome do not progressively appears to have a single disease (for example, SLE) may accrue
worsen over time, and their mortality rate is similar to the features to support an MCTD diagnosis.
general population. Increased mortality is associated with Skin manifestations include sclerodactyly, scleroderma,
lymphoproliferative malignancy and/or other associated auto­ calcinosis, telangiectasias, photosensitivity, malar rash, and
immune disease. Low complement levels, lymphocytopenia,
Gottron rash. Pleuropericarditis occurs in up to 60% of
and cryoglobulinemia at diagnosis are predictive of unfavora­ patients, and sicca symptoms in up to 50%. Trigeminal neural­
ble outcome due to lymphoma, severe disease manifestations
gia occurs in up to 25% of patients. Kidney involvement occurs
(such as vasculitis), and premature death. in 25% of patients, typically as membranous nephropathy.
KEY POINTS In addition to overlapping features of SLE, systemic scle­
• Sjogren syndrome manifests as infiltrative inflamma­ rosis, and/or polymyositis, MCTD diagnosis requires the pres­
tion of exocrine glands, characterized by dry eyes and ence of anti-Ul-RNP antibodies. Laboratory studies may
dry mouth. additionally show leukopenia, thrombocytopenia, and an
elevated erythrocyte sedimentation rate. Antinuclear antibod­
• Patients with Sjogren syndrome have a 16- to 44-fold
ies may be present, with a very high titer (2:1:1200) in a speck­
increased risk of lymphoma, most commonly from dif­
led pattern. The presence of anti-Smith or anti-double-stranded
fuse large B-cell and mucosa-associated lymphoid tis­
DNA antibodies should suggest SLE rather than MCTD.
sue lymphomas.
• In patients with classic historical and physical findings,
the presence of anti-Ro/SSA and anti-La/SSB antibodies Management
may be sufficient to diagnose Sjogren syndrome; in MCTD management is determined by the manifestations of the
unclear cases, a lip biopsy demonstrating minor salivary individual patient. Glucocorticoids, azathioprine, and metho­
gland inflammation is considered the gold standard for trexate can be used for arthritis and myositis. Symptoms asso­
diagnosis. ciated with Raynaud phenomenon may benefit from calcium
• Autoimmune diseases, including rheumatoid arthri­ channel blockers; patients with esophageal dysmotility should
tis, systemic lupus erythematosus, and autoimmune receive proton pump inhibitors. Cyclophosphamide may be
thyroiditis, are commonly associated with Sjogren tried for interstitial lung disease and phosphodiesterase inhib­
syndrome. itors and/or anti-endothelin therapies for pulmonary arterial
hypertension.

52
Crystal Arthropathies

CJ Prognosis Despite important advances in understanding and


treating gout, it remains a disease that is often suboptimally
The course of MCTD is variable; the likelihood of developing
managed.
interstitial lung disease, pulmonary arterial hypertension, and/
or cardiovascular disease increases with disease duration.
Pathophysiology
MCTD mortality is increased compared with SLE. mainly as a
Uric acid is the end product of purine metabolism in humans.
consequence of pulmonary arterial hype11ension. Renovascular
Free purines arise from nucleic acid breakdown during cell
disease as seen in systemic sclerosis may uncommonly contrib­
turnover, from adenosine triphosphate metabolism, and
ute to morbidity and mortality. CJ
through dietary intake. Xanthine oxidase is the terminal
KEY POINTS enzyme in human uric acid synthesis. In contrast to most
• Mixed connective tissue disease is an overlap syn­ mammals, humans lack the enzyme uricase, which processes
drome that includes features of systemic lupus ery­ urate to the more soluble allantoin (Figure 20).
thematosus, systemic sclerosis, and/or polymyositis At physiologic conditions (pH, 7.4), uric acid exists pri­
in the presence of anti-Ul-ribonucleoprotein marily as its ionized form, urate; therefore, hyperuricemia
antibodies. technically refers to elevated serum urate. The saturation con­
• Typical findings of mixed connective tissue disease centration of urate is 6.8 mg/dL (0.40 mmol/L}, with monoso­
include hand edema, synovitis, myositis, decreased dium urate crystals unlikely to form at concentrations below
esophageal motility, and fatigue. this level; hyperuricemia is therefore defined based on this
physiologic parameter. Urate is cleared by the kidneys via both
• Treatment for mixed connective tissue disease is deter­ glomerular filtration and active urate secretion in the proximal
mined by the manifestations of the individual patient. tubule; resorption by other proximal tubule transporters
occurs concurrently.
The most common risk factor for hyperuricemia is under­

Crystal Arthropathies excretion of urate by the kidneys due to impaired glomerular


filtration and/or defects of urate handling in the renal proxi­
mal tubule (Table 25). Alternatively, about 10% of patients with
Introduction
gout have genetic aberrancies causing excessive uric acid pro­
In crystal arthropathies, metabolic abnormalities promote the duction. Hyperuricemia in the adult population is common
formation and deposition of crystals that stimulate inflamma­ (up to 20%) but usually asymptomatic; measurement of
tion. This chapter discusses gout, calcium pyrophosphate dep­ serum urate is therefore not a useful screening tool for gout in
osition, and basic calcium phosphate deposition. the absence of appropriate clinical signs and symptoms.

Gout Ribose-5-P
Gout is characterized by intermittent painful inflammatory
joint attacks, resulting from crystallization of excessive levels '--j
I
PRPP synthetase
Feedback
of uric acid (hyperuricemia). 1
, inhibition �
•-------- Purines .., - - - ,
I
Epidemiology I HPRT
Gout is the most common inflammatory arthritis in the United : (purine salvage)
States (prevalence, 4%). Serum urate levels rise after puberty in
men and after menopause in women; premenopausal women Xanthine/hypoxanthine
are therefore protected from gout except in cases of underlying
Xanthine
disease or strong genetic factors. Gout becomes increasingly

I I I
oxidase
common among older individuals owing to accumulation of
additional hyperuricemia risk factors (for example, chronic Uric acid � - - �'�·�·- -> Allantoin
�. ---�· (not 1n humans . .
kidney disease). Approximately 13% of patients over the age of or other primates)

80 years are affected. FIGURE 2 0. Purine biosynthesis and metabolism to uric acid. De nova purine
Gout is typically associated with multiple comorbidities synthesis can occur in human cells. Rarely, hereditary overactivity of phosphoribo­
(hypertension, obesity, diabetes mellitus, chronic kidney dis­ sylpyrophosphate (PRPP) synthetase results in excessive purine production. Purine
ease, vascular disease, dyslipidemia, and nephrolithiasis) that salvage (to recover purines from the breakdown of nucleic acids for reuse) occurs
through hypoxanthine-guanine phosphoribosyltransferase (HPRT); HPRT deficiency
may complicate treatment, either by posing contraindications
results in purine depletion and thus increased uric acid synthesis due to lack of
to specific therapies or because treating some comorbid condi­ negative feedback (as in Lesch-Nyhan syndrome). Humans Jack the gene for
tions can worsen hyperuricemia (for example, using diuretics uricase; in most other mammals, this enzyme converts uric acid to the more solu­
to treat hypertension). ble allantoin and results in much lower urate levels.

53
Crystal Arthropathies

;TABLE 25. Ca�ses of Hyperuricemia. findings tend to correlate with the severity of the attack and
normalize rapidly with treatment.
Primary renal uric acid underexcretion (hereditary, renal tubular
Soft tissues adjacent to the joints can be affected during
basis)
acute flares, becoming red, painful, and edematous. Soft­
Chronic kidney disease of any cause (secondary uric acid
underexcretion)
tissue inflammation can mimic cellulitis, tenosynovitis, or
dactylitis, rendering the diagnosis of gout challenging,
Uric acid overproduction due to primary defect in purine
metabolism: PRPP synthetase overactivity; HPRT deficiency Although a chronically elevated sernm urate level is a hall­
mark of gout, sernm urate levels can occasionally be low dur­
Conditions of cell turnover leading to purine/urate generation:
leukemia/lymphoma; psoriasis; hemolytic anemia; ing acute attacks, possibly because cytokines promote renal
polycythemia vera urate excretion. Thus, when there is reasonable clinical suspi­
Drug-induced hyperuricemia (agents reducing renal cion for gout, treatment is indicated even with a normal sernm
glomerular filtration and/or tubular urate excretion): thiazide urate level. The sernm urate level shouJd be reassessed after the
and loop diuretics; cyclosporine; low-dose salicylates;
ethambutol; pyrazinamide; lead ingestion/toxicity
flare has resolved for a more accurate characterization.
Diet-induced hyperuricemia (agents high in purines or
inducing purine/urate biosynthesis): alcohol; shellfish; red Intercritical Gout
meat; high-fructose corn syrup-sweetened beverages and The period between gout attacks is referred to as the intercritical
foods phase, during which the patient is typica!Jy asymptomatic.
HPRT == hypoxanthine-guanine phosphoribosyltransferase; PRPP = phosphoribo­ Early in the course of disease, attacks tend to be infrequent, but
sylpyrophosphate.
most patients experience a recurrent attack within 2 years of
their first attack. In long-standing or poorly controlled gout, the
time between attacks tends to diminish.
Nonetheless, hypernricemia is the primary risk factor for gout,
with increasing incidence of gout in patients with higher Chronic Recurrent and Tophaceous Gout
sernm urate levels, The mechanisms leading to urate crystal­ Chronic recurrent gout refers to a disease state in which poorly
lization are incompletely elucidated but depend on tempera­ controlled gout results in frequent flares and/or chronic gouty
ture, pH, and other physicochemical factors, arthropathy, in which synovitis persists between acute attacks.
Once crystals form, resident tissue macrophages phago­ Chronic recurrent gout is usually the consequence of ineffec­
cytose them to initiate an inflammatory cascade, Interleukin tive therapy, medication noncompliance, or undertreatment.
(IL)-1� production fuels the process and promotes synthesis Chronic recurrent gout poses a significant economic burden in
of additional cytokines (for example, tumor necrosis factor a, terms of health care costs and lost time from work. Patients
IL-6), Additionally, complement activation on the crystal with this degree of gout severity should be referred to a rheu­
surface generates split products that stimulate and attract matologist for management.
neutrophils, Collectively, these signals promote neutrophil Tophi (stone-like deposits of monosodium urate sur­
infiltration, the hallmark of an established gout attack. rounded by a fibrous and inflammatory rind) may form in the
joints and soft tissues (Figure 21). Tophi can form in any joint,
Clinical Manifestations
Gout can be considered as having three phases: acute gouty
arthritis, intercritical gout, and chronic recurrent or tophaceous
gout (also known as chronic tophaceous gouty arthropathy),

Acute Gouty Arthritis


The classic gout presentation is podagra, in which the metatar­
sophalangeal joint of the great toe becomes painful, red, and
swollen over 12 to 24 hours, First gout attacks typically are
monoarticular and begin at night. Approximately 90% of first
attacks present in this manner. First attacks in women are less
likely to fit this classic picture.
Recurrent attacks can occur in nearly any joint, with
either monoarticular or polyarticular presentation. Attacks in
bursae also occur. In poorly controlled disease, flares become
more frequent and involve an increasing number of joints.
Systemic inflammation is common, including fever (typically
<38.9 °C (102.0 °F]), peripheral leukocytosis (up to 13,000/µL
FIGURE 21. Numerous, bulky deposits of monosodium urate crystals form
[13 x 10°/L]), and elevated inflammatory markers (C-reactive tophi, a sign of late-stage gout that has not been adequately treated with urate­
protein and erythrocyte sedimentation rate). Systemic lowering therapy.

54
Crystal Arthropathies
-- --- - - - -
-

leading to bone erosion, chronic joint damage, skin ulceration, However. a clinical diagnosis does not permit the definite
infection, disability, and impaired quality of life. exclusion or infection or other inflammatory arthritis. The
American College of Rheumatology preliminary classification
Diagnosis criteria for acute gout serve as a helpful. albeit imperfect, tool
Gout should be suspected in the setting of acute monoarticular for diagnosing gout (Table 27). Importantly. infection of the
or polyarticular inflammation in an at-risk patient (see first metatarsophalangeal joint is uncommon.
Epidemiology) . Infectious arthritis must be actively excluded. Radiography is of limited utility in early gout but may be
given its high rate of morbidity and mortality. The differential warranted to exclude fracture or other non-gout etiologies. In
diagnosis of acute gout is listed in Table 26. A high (-reactive severe or long standing gout. rac!iographs can demonstrate
protein and/or erythrocyte sedimentation rate support an classic gouty erosions described as punched-out lesions. often
inflammatory process but are nonspecific. A high serum urate with overhanging edges where the bones have been eaten
level supports the potential for gout. but most patients with away by tophaceous deposits (Figure 22) .
hyperuricemia do not have gout. and serum urate may be low
during some acute attacks. Management
The gold standard for diagnosing gout is joint aspiration Gout management centers on two fundamental principles:
and synovial fluid analysis. Synovial fluid analysis permits 1) control or infh1mmc1tion to treat or prevent acute gout
definitive diagnosis and can rule out other. clinically indistin­ attacks and 2) reduction or serum urate to treat the underlying
guishable conditions. Under polarized light. monosoc!ium hyperuricemia that leads to inflammatory disease. Both are
urate crystals are needle shaped and negatively birefringent essential for proper gout management.
(yellow vvhen parallel, blue when perpendicular to the polar­
izing axis). Synovial fluid leukocyte count should also be Treatment of Acute Gouty Arthritis
measured: gout fluid is typically inflammatory (2000 'µL to Acute gout f"lares should be treated with pharmacologic inter­
>100,000/µL (2.0-100 x 109/L], neutrophil predominance of vention. ideally within 24 hours or onset to maximize treat­
;:,:so%). Whereas extracellular crystals confirm a chronic gout ment emcacy. In patients with classic podagra or other mild to
diagnosis. crystals within neutrophils define active. gout­ moderate presentations. monotherapy with colchicine (1.2 mg
induced inflammation. Gram stain and cultures must be follovved by 0.6 mg I hour later). full-dose NSAIDs (such as
obtained to exclude infection. Acute gout and joint infection naproxen 500 mg twice daily). or glucocorticoids (for
occasionally coexist.
Because joint aspiration is not always feasible or success­
. TABLE 27. American College of Rheumatology Criteria for
ful, diagnosis is occasionally made on clinical grounds alone. the Classification of Acute Arthritis of Primary Gout
Criteria
TABLE 26. Differential Diagnosis of Acute Gouty Arthritis
The presence of six of the following clinical, laboratory,
Condition Comments and radiographic findings, even in the absence of crystal
identification:
Infectious arthritis Presentation may be identical to
More than one attack of acute arthritis
gout. Infectious arthritis is usually
monoarticular but can be Maximum inflammation developed within 1 day
polyarticular. Gout and infectious
Monoarthritis attack
arthritis can also coexist.
Redness observed over joints
Acute calcium Pseudogout is less likely to present in
pyrophosphate crystal the great toe (podagra), but acute First metatarsophalangeal joint painful or swollen
arthritis (also known as presentations may still be identical to
pseudogout) gout. Synovial fluid analysis can Unilateral first metatarsophalangeal joint attack
distinguish these entities. Unilateral tarsal joint attack
Basic calcium Because of their small size, basic Tophus (suspected)
phosphate deposition calcium phosphate crystals are
unlikely to be seen in synovial fluid Hyperuricemia
on light microscopy except as Asymmetric swelling within a joint visible on physical
aggregates stained with alizarin red. examination or radiography
Trauma Trauma can lead to local pain and Subcortical cysts without erosions visible on radiography
swelling, with or without a fracture,
and can also trigger a gout attack. Monosodium urate monohydrate microcrystals in joint fluid
during attack
Other forms of Clinical context, affected joints, and
inflammatory arthritis pattern of arthritis are all helpful in Joint fluid culture negative for organisms during attack
(e.g., reactive arthritis, distinguishing these entities.
Adapted with permission from John Wiley & Sons, from Wallace SL, Robinson H,
rheumatoid arthritis, Synovial fluid analysis is particularly Masi AT, et al. Preliminary criteria for the classification of the acute arthritis of pri­
psoriatic arthritis, important when there is diagnostic mary gout. Arthritis Rheum. 1977;20(3):895-900. [PMID: 856219] Copyright 1977
acute rheumatic fever) uncertainty. American College of Rheumatology.

55
Crystal Arthropathies

FIGURE 2 2. Punched-out bone lesions and erosions with overhanging edges associated with severe or long-standing gout.
Courtesy of Elaine Karis, MD.

Cl
COff
example, prednisone. 0.5 mg/kg) are all recommended.
Selection should account for patient preferences. comorbidi-
ties. and concomitant medication use. Any NSAlD, including
well as alcoholic beverages (especially beer) should be limited
because all contribute to increased semm urate levels. Dairy
intake may lower urate through uricosuric effects, and low-fat
selective cyclooxygenase-2 inhibitors. may be employed. Intra­ dairy products may thus offer benefit to patients with gout.
articular glucocorticoid injection is an excellent option for Hydration and substitution of urate-lowering for urate-raising
monoarticular attacks. with !ewer systemic toxicities than oral medications can further reduce baseline urate. For example,
glucocorticoids. but requires expertise and prior synovial nuid use of a calcium channel blocker or losartan instead of a diu­
assessment to rule out infection. Ice can reduce pain and retic for hypertension can promote decreased semm urate;
swelling. particularly in Ilares involving one or two joints. notably, the former two agents should be used preferentially to
In refractory cases. switching to a different anti­ treat hypertension in patients at risk for gout attacks.
inf"lammatory class may be attempted. For severe and/or The American College ofRheumatology (ACR) guidelines
polyarticular flares. combination therapy with two anti­ currently indicate that urate-lowering therapy should be ini­
inllammatory classes may be needed. but additive toxicities tiated in patients with gout who have had two or more attacks
should be considered. Off-label use of the interleukin-I� inhib­ within a 1-year period, one attack in the setting of chronic
itors anakinra or canakinumab can be considered in severe kidney disease of stage 2 or worse, one attack with the pres­
cases when standard therapies are ineffective. contraindicated. ence of tophi visible on examination or imaging, or one attack
or not tolerated. Patients already taking urate-lowering therapy with a history of urolithiasis. Urate-lowering therapy should
should continue at the same dose throughout a flare because usually be initiated during the intercritical phase to avoid
changes in serum urate can exacerbate attacks. Cl exacerbating active attacks. Treatment to a target serum urate
level ofless than 6.0 mg/dL (0.35 mmol/L) is mandatory; still
Urate-Lowering Therapy and Prophylaxis lower targets may be needed to improve symptoms or signs of
All patients with gout should be educated on lifestyle modifi­ gout or to resolve tophi. First-line therapy for urate lowering
cations to reduce semm urate, including weight loss if appro­ is typically the xanthine oxidase inhibitor allopurinol. An
priate (see Table 25). Intake of high-purine (for example, meat appropriate alternative is the newer xanthine oxidase inhibi­
and seafood) and high-fructose (for example, soft drinks and tor, febuxostat, which is equally or more efficacious (and
processed foods with added high-fructose corn syrup) foods as causes less hypersensitivity) but significantly more expensive.

56
Crystal Arthropathies

The uricosuric agent probenecid blocks renal uric acid resorp­


Calcium Pyrophosphate
tion and may be added to, or substituted for, a xanthine oxi­
dase inhibitor if xanthine oxidase monotherapy is inadequate Deposition
or not tolerated; however, probenecid may raise the risk of Calcium pyrophosphate (CPP) crystals are a common cause of
kidney stones and has limited efficacy in patients with calcification in connective tissues in and near joints. Calcium
impaired kidney function. In chronic refractory gout or when pyrophosphate deposition (CPPD) is associated with four
patients experience severe gout disease burden and standard distinct clinical presentations. There has been inconsistency
mate-lowering therapy has been unsuccessful or not toler­ in the literature and in practice surrounding the terminology
ated, the potent intravenous agent pegloticase is a treatment for these entities. The European League Against Rheumatism
option. Pegloticase provides uricase activity, enzymatically (EULAR) recently published guidelines to clarify the termi­
converting urate to the more soluble compound allantoin. See nology, although these have not been universally adopted.
Principles of Therapeutics for more information on urate­ EULAR advocates for using "calcium pyrophosphate deposi­
lowering therapy. tion" or "CPPD" as a broad term to refer to all instances of
Gout flare prophylaxis should be initiated along with calcium pyrophosphate deposition and redefines the termi­
urate-lowering therapy because urate reduction can provoke nology for the various presentations of CPPD. Here we use the
attacks (owing to sloughing of microcrystals within the joint). EULAR terminology but also mention the common terms
Prophylaxis should continue in the presence of any active dis­ used to describe the subtypes of CPPD.
ease. In the absence of active disease, the ACR currently rec­ The four clinical presentations are cartilage calcification
ommends that prophylaxis should be continued for the greater (also known as chondrocalcinosis), acute CPP crystal arthritis
of the following: 6 months; 3 months after achieving the target (also known as pseudogout), chronic CPP crystal inflamma­
serum urate level for a patient without tophi; or 6 months after tory arthritis, and osteoarthritis with CPPD.
achieving the target serum urate level where there has been
resolution of tophi. First-line options for gout prophylaxis Pathophysiology
include colchicine, 0.6 mg (or 0.5 mg) once or twice daily, The pathophysiology of CPPD is poorly characterized. CPPD in
adjusting for kidney function and consideration of drug-drug the joints may result from excessive body calcium or other
interactions (see Principles of Therapeutics). Low-dose metabolic derangements, overproduction of pyrophosphate by
NSA!Ds (such as naproxen, 250 mg twice daily) are an equally cartilage chondrocytes, or other local factors. Risk factors for
recommended alternative, but the gastrointestinal and cardiac CPPD include previous joint injury and familial predisposi­
toxicity of NSA!Ds must be considered. Current guidelines tion. Particularly in a young person. a diagnosis of CPPD
endorse $10 mg of daily prednisone as an acceptable second­ should prompt an evaluation for diseases associated with
line prophylactic agent, with monitoring for the side effects of CPPD, including hyperparathyroidism, hemochromatosis,
chronic glucocorticoid use. hypomagnesemia, and hypothyroidism.
KEY POINTS The mechanism of chronic low-grade joint damage seen
• Gout is characterized by intermittent painful inflamma­ in some patients with cartilage calcification is not fully eluci­
tory joint attacks resulting from crystallization of exces­ dated. In contrast, acute CPP crystal arthritis is known to
sive levels of uric acid (hyperuricemia). induce inflammation via pathways similar to those induced
by monosodium urate crystals in acute gout, triggering
• In patients with suspected gout, synovial fluid analysis is
interleukin-I� production by macrophages and leading to
used to identify monosodium urate crystals, and Gram
additional inflammation and neutrophil recruitment.
stain and cultures are ordered to exclude infection.
• Treatment of acute gouty arthritis involves control of
Clinical Manifestations
inflammation using colchicine, NSA!Ds, or glucocorti­
coids. Cartilage Calcification
Cartilage calcification (also known as chondrocalcinosis) is
• Urate-lowering therapy should be initiated in patients
a radiographic finding that is usually the result of CPPD.
with gout who have had two or more attacks within a
Cartilage calcification is typically asymptomatic but may
1-year period, one attack in the setting of chronic kid­
be associated with acute CPP crystal arthritis and/or
ney disease of stage 2 or worse, one attack with the
pyrophosphate arthropathy. In the knee, cartilage calcifica­
presence of tophi visible on examination or imaging, or
tion is seen as a linear density within and parallel to the
one attack with a history of urolithiasis.
surface of the cartilage (Figure 23); in the wrist, it is often
• Gout flare prophylaxis should be initiated along with seen in the triangular fibrocartilage distal to the ulna.
urate-lowering therapy because urate reduction can Cartilage calcification can be an isolated finding or can
transiently provoke attacks; first-line options include occur in association with osteoarthritis. Prevalence
colchicine and NSAIDs. increases with age, with a prevalence of approximately 50%
in patients aged 2':85 years.

57
Crystal Arthropathies

arthritis ("pseudo-rheumatoid artluitis"). In osteoarthritis with


CPPD, patients exhibit accelerated osteoarthritis findings in
joints not commonly involved with simple osteoarthritis (for
example, wrist, metacarpophalangeal, or shoulder joints), with
or without superin1posed acute CPP crystal arthritis flares. It is
presumed in these cases that the presence of crystals within the
cartilage promotes a degenerative process not otherwise distin­
guishable from osteoarthritis.

Management
Management of CPPD centers on controlling symptoms and
reducing inflammation (Table 28). In contrast to gout, no
therapy is currently available to eliminate the pathogenic CPP
crystals. Nonetheless, any underlying metabolic disorder that
FIGURE 2 3. Cartilage calcification (also known as chondrocalcinosis) of the may contribute to CPPD should be addressed.
knee. This radiograph shows linearly arranged calcific deposits in the articular
cartilage (ar,ow). KEY POINTS

• Calcium pyrophosphate deposition can be associated

Cl Acute Calcium Pyrophosphate Crystal Arthritis


Acute CPP crystal arthritis (also known as pscudogout) is char­
with hyperparathyroidism, hemochromatosis,
hypomagnesemia, and hypothyroidism.
acterized by painf'ul inflammatory arthritis attacks reaching
• Acute calcium pyrophosphate crystal arthritis is gener­
maximum intensity within 24 hours. The knee. wrisl. and
ally indistinguishable from an acute gout flare; synovial
shoulder are typically affected: podagrn is uncommon. Acute
fluid analysis is used to make a definitive diagnosis
C:PP crystal arthritis is generally indistinguishable f'rom a gout
based on the characteristic rhomboid shape and posi­
flare: synovial fluid analysis is needed to make a definitive
tive birefringence of calcium pyrophosphate crystals.
diagnosis (coexistence oft he two diseases is possible). In con
t rast to monosodium urate crystals. C:PP crystals are rhomboid • Management of calcium pyrophosphate deposition
shaped and positively birefringent. Cartilage calcification on centers on control of symptoms and reduction of
radiograph suggests that an acute arthritis may be clue to CPP inflammation; any underlying metabolic disorder that
crystals but is nondiagnostic. Acute CPP crystal arthritis occurs may contribute should also be addressed.
most commonly in patients over 65 years of'age. Cl

Pyrophosphate Arthropathy
Basic Calcium
Pyrophosphate arth.ropathy is a general term for joint damage Phosphate Deposition
attributed to CPPD and may present in two distinct patterns: Basic calcium phosphate (BCP) crystals (including hydroxyapa­
chronic CPP crystal inflammatory arthritis and osteoarthritis tite) are a common cause of cartilage calcification and may be
wilh CPPD (both terms advocated by EULAR). In chronic CPP radiographically indistinguishable from CPPD. Unlike CPP
crystal inflammatory arthritis, there is ongoing inflammation crystals, BCP crystals also commonly deposit in periarticular
wilh a pattern of joint involvement often resembling rheumatoid tendons, bursae, and other soft tissues. Patients are often

TABLE 28. Management of Calcium Pyrophosphate Deposition


Clinical Presentation Treatment/Comments

Cartilage calcification (also known as chondrocalcinosis) No specific treatment


Acute calcium pyrophosphate crystal arthritis (also known as Local treatment: joint aspiration; intra-articular glucocorticoid
pseudogout) injection; joint immobilization; ice packs
Systemic treatment: NSAIDs; colchicine; glucocorticoids (oral or
parenteral)
Prophylaxis if recurrent attacks (three or more annual attacks): low­
dose colchicine or daily NSAIDs (with gastrointestinal protection)
Chronic calcium pyrophosphate crystal inflammatory arthritis Low-dose colchicine or daily NSAIDs (with gastrointestinal
protection); low-dose glucocorticoids; methotrexate;
hydroxychloroquine
Osteoarthritis with calcium pyrophosphate deposition (CPPD) Same treatment as osteoarthritis without CPPD (e.g., physical
therapy; pain control; local glucocorticoids)

58
Infectious Arthritis

asymptomatic, but BCP crystals can cause destructive arthrop­ In some forms of viral-induced and other infectious
athy. Although not always visible on radiographs, BCP crystals arthritis, joint inflammation may develop as a consequence of
have been identified as a common feature of degenerative joint immune complexes elicited by the pathogen, rather than
disease in osteoarthritis (on histological examination). BCP direct infection.
crystals also can stimulate inflammation via pathways similar

Cl
to monosodium urate and CPP crystals. BCP deposition can
cause Milwaukee shoulder syndrome, a painful, swollen, and Diagnosis
often destructive process of the glenohumeral joint and rotator Infeet ion shou Id be considered in the di rterentia I diagnosis or
cuff; this condition typically occurs in women over the age of .111y acutely or chronically inllamed joint. Joint inlection
70 years. should he suspected lx1secl on clinical history. presence or risk
Diagnosis of BCP deposition is made clinically or by factors (Table 29). and initial l,iboralory studies. Although
joint aspiration. The synovial fluid is characteristically non­ elevated inflammatory markers (leukocyte count. erythrocyte
inflammatory (<2000/µL [2.0 x 109 /L]). BCP crystals are not sedimentation r.1te /1'.SR]. and C reactive protein [CRPJ) may
visible on polarized microscopy but may be visualized on suggest inlectious arthritis. th<:y ec111not con Ii rm the diagnosis
electron microscopy or under light microscopy as nonbire­ or exclude other infb111m,1tory causes.
fringent clumps that stain with alizarin red. Treatment for
symptomatic BCP deposition includes NSA!Ds, joint aspira­ Clinical Manifestations
tion and tidal lavage, and intra-articular glucocorticoid The hallmarks or.1 joint inli.:ct ion are warmth. erythema. pain.
injection. and swelling. The typical bacterial infectious arthritis develops
over clays: onset is more indolent for rungal or mycobacterial
KEY POINTS
infections. Joint infections are commonly limited to a single
• Basic calcium phosphate deposition can cause
joint: the knee is the most common site. hut the hip. wrist. and
Milwaukee shoulder syndrome, which presents with a ankle are .1lso commonly c1fkct<:d. In approximately 20'X, or
painful, swollen shoulder; is often associated with sig­
cases. bacterial or rung,11 arthritis pr<:sents with polyarthritis
nificant destruction of the glenohumeral joint and rota­
tor cuff; and typically occurs in women over the age of
70 years. TABLE 29. Risk Factors for Infectious Arthritis
• Diagnosis of basic calcium phosphate deposition is General Risk Factors
made clinically and/or by joint aspiration and staining
Age >80 years or very young children
with alizarin red.
Alcoholism
• Management of symptomatic basic calcium phosphate
deposition includes NSAIDs, joint aspiration and tidal Cutaneous ulcers or other skin infections
lavage, and intra-articular glucocorticoid injection. Diabetes mellitus
End-stage kidney disease
History of intra-articular glucocorticoid injection

Infectious Arthritis Injection drug use

CJ Introduction
Low socioeconomic status
Patients receiving immunosuppressive agents
Infectious arthritis is caused by bacteria. fungi. or viruses and Prosthetic joint or recent joint surgery
should be considered in any patient with one or more inflamed Sickle cell disease
joints. Significant morbidity and mortality can result rrorn Underlying malignancy
inadequate or delayed management. Cl
Risk Factors for Gonococcal Infection

Younger, sexually active patients


Pathophysiology
Bacterial or fungal joint infections arise from hematogenous Risk Factors for Lyme Arthritis

seeding (most cases), direct inoculation (for example, with Travel or residence in an endemic area
surgery or, rarely, glucocorticoid injection), or contiguous Documented tick bite or erythema chronicum migrans
spread from neighboring osteomyelitis, cellulitis, or septic
Risk Factors for Mycobacterial or Fungal Infection
bursitis. Some bacteria produce toxins or enzymes that directly
damage tissues. Bacteria also trigger inflammatory responses Patients with HIV infection or other immunosuppression
characterized by neutrophilic infiltration into the joint space. Patients receiving tumor necrosis factor a inhibitors
If unresolved, these processes can rapidly progress to joint
Travel or residence in an endemic area
damage or destruction.

59
Infectious Arthritis

Cl
COIIT.
and a worse prognosis. Fever and rigors may occur but are not
universally present (60% of patients): absence of fever there­
Infection with Gram-Positive Organisms
Gram-positive organisms are the most common causes of c::J
fore does not exclude an infected joint. infectious arthritis in adults. Sr-aphylococcus aureus is the
most common infecting organism. regardless of age or risk
Laboratory Studies and Imaging factors. S. aureus damages joints directly via toxic effects and
When infectious arthritis is suspected. the synovial fluid must indirectly via inflammatory responses. Symptoms develop
be aspirated and assessed for leukocyte count. Gram stain. rapidly and escalate within hours to days. T he emergence of
culture (bacterial and, depending on index of suspicion. myco­ methicillin-resistant S. aureus poses treatment challenges.
bacterial and fungal). and crystal analysis (to rule out alterna­ Streptococcus pneumoniae is another common pathogen.
tive or concomitant etiologies). Joint aspiration should be Polymicrobial infections occur uncommonly but may indicate
performed before initiating antibiotics whenever possible to a worse prognosis.
maximize the likelihood of positive cultures. In bacterial infec­
tion. the synovial fluid leukocyte count is markedly increased Infection with Gram-Negative Organisms
(usually >50.000 µL [50 x 109 /L] with neutrophil predomi­
Nongonococcal Gram-Negative Organisms
nance). Because synovial fluid cultures have a sensitivity of
Nongonococcal gram-negative bacterial arthritis is more com­
approximately 80% and may be negative due to antibiotics. a
mon in patients who are immunosuppressed. are elderly. or
fastidious organism. or other reasons. clinical assessment
have a history of injection drug use. recent trauma. or gastro­
must ultimately guide treatment.
intestinal infection. Escherichia coli and Pseudomonas aer­
Cultures of blood and other possible sources of infection
uginosa are the most common gram-negative organisms to
(such as urine or sputum) should be obtained if infectious
infect the joints. Injection drug users are at particular risk for
arthritis is under consideration. ideally before starting antibi­
Pseuc/omonas. Patients with sickle cell anemia are predis­
otics. Peripheral leukocyte count. ESR. and CRP should be
posed to Salmonella infection. Haemophilus influenzae. pre­
measured.
viously a common cause of infectious arthritis. has declined
Radiographs are nonspecific early in infectious arthritis
due to H. influenzae vaccine. Gram-negative bacterial joint
but may reveal local swelling or joint destruction if the infec­
infections cause rapid onset of symptoms and potential joint
tion has been left untreated. whether in bacterial, mycobacte­
destruction if not promptly and appropriately treated.
rial, or fungal infections. MRI. CT. and ultrasonography are not
routinely indicated but may help identify early erosions. con­
Disseminated Gonococcal Infection
current osteomyelitis. or joint effusions that are dif'Acult to
Disseminated gonococcal infection occurs in up to 3% of
assess on examination (for example. in the hip). Imaging may
patients with Neisseria gonorrhoeae and can cause t,vo dis­
also help guide arthrocentesis.
tinct clinical presentations. Risk is highest in young. sexually
Cultures or the genitourinary tract. rectum . and/or phar­
active adults.
yngeal sites should be obtained when disseminated gonococ­
Patients vvith disseminated gonococcal infection and bac­
cal infection is suspected. Diagnosis of Lyme disease and viral
teremia present with vesiculopustular or hemorrhagic macular
infections relies on serologic testing. c:J
skin lesions (Figure 24). fever. chills. and polyarthralgia. Knees.
KEY POINTS
• Infectious arthritis is caused by bacteria, fungi, or
viruses and should be considered in any patient with
one or more inflamed joints.
• The hallmarks of a joint infection are warmth, erythema,
pain, and swelling.
• Diagnosis of infectious arthritis is confirmed by joint
aspiration and aided by clinical history, physical exami­
nation, and laboratory studies.
• In bacterial infection, the synovial fluid leukocyte count
is markedly increased (usually >50,000/µL [SO x 109/L]
with neutrophil predominance).

Causes FIGURE 2 4. Disseminated gonococcal infection presents as a febrile arthritis­


dermatitis syndrome with migratory polyarthralgia evolving into frank arthritis with
Common causes of infectious arthritis are discussed in the fol­
or without tenosynovitis that involves one or more joints. Skin lesions are found in
lowing sections. For more information on the specific infec­ approximately 75% of cases, classically presenting as a small number of necrotic
tions/diseases, refer to MKSAP 17 Infectious Disease. vesicopustules on an erythematous base.

60
Infectious Arthritis

l"l"I elbows. and distal joints are typical sites of involvement. symptoms. Musculoskeletal M. tuberculosis infection most
LI.I
CONT.
Tenosynovitis of the dorsa of the hands and/or feet is a charac­ commonly presents as spondylitis (Pott disease) or vertebral
teristic feature. Synovial fluid leukocyte counts are lower than osteomyelitis. Tuberculous osteomyelitis of almost any other
in other bacterial infections (<25,000/�tL [25 x 109 /L]). Blood bone is also possible.
cultures or cultures of the genitourinary tract (for example, Arthritis caused by M. tuberculosis is usually monoarticu­
cervical or urethral specimens), rectum. or pharynx are often lar, often involving the hip or knee. Joint destruction is possible
positive, but synovial fluid cultures are usually negative. Special but occurs slowly. Synovial fluid analysis usually suggests a
media are required to culture N. gonorrhoeae. nonspecific inflammatory process; imaging may be normal or
Gonococcal infection can also present as a purnlent arthri­ show erosions that cannot be readily distinguished from other
tis without the rash or other features of bacteremia. In this infections. Acid-fast smears of synovial fluid are positive only in
case, one or two joints are typically involved, most commonly a minority of cases; synovial fluid culture or synovial biopsy
the knee followed by the wrist, ankle. and elbow. Synovial with granuloma identification are more likely to be diagnostic.
fluid cultures are more often positive for N. gonorrhoeae in Tuberculin skin tests are not uniformly positive, especially in
this setting. immunosuppressed individuals; therefore, a negative test does
not definitively exclude infection. Although signs of pulmonary
tuberculosis or tuberculosis of other sites should be sought,
Lyme Arthritis
many patients present with isolated musculoskeletal disease.
Lyme disease is caused by infection with Borre/ia species.
There are three phases of Lyme disease: early localized, early
Mycobacterium marinum
disseminated, and late. Although arthralgia is common in
Mycobacterium marinum is a freshwater and saltwater
early Lyme disease, Lyme arthritis generally refers to frank
organism that enters the skin through breaks or abrasions.
inflammation of the joints as a manifestation of late Lyme
Common scenarios include anglers and aquarium hobbyists
disease. Late Lyme arthritis frequently causes impressively
with bare hand exposure to water. M. marinum infections
large effusions, most often a knee monoarthritis with promi­
typically begin as red or violaceous plaques, nodules, or
nent stiffness but relatively little pain. Asymmetric oligoar­
abscesses in the skin. and may spread locally to involve the
ticular presentations also occur. Lyme arthritis can occur as a
hand joints. Therapy requires multidrug regimens such as
transient event, follow an intermittent pattern. or develop into
rifampin and ethambutol.
a chronic arthritis. Diagnosis is made by serologic testing
(enzyme-linked immunosorbent assay followed. if positive. by
Fungal Infections
Western blot) indicating an immune response to Borrelia
Fungal infections are a rare cause of infectious arthritis but are
burgdorferi. Joint aspiration demonstrates a moderately
important to consider. particularly in immunocompromised
inflammatory synovial fluid (leukocyte count typically
patients and/or endemic areas. Fungal aithritis is most likely
20,000-25.000/µL (20-25 x 109/L]. neutrophil predominant).
to be caused by Coccidioides (endemic in the southwestern
Synovial fluid cultures are negative, but B. burgdo1jeri DNA
United States). Sporothrix (found in farm and garden soil),
can be detected by polymerase chain reaction. Permanent
Cryptococcus. Blastomyces (endemic in the central United
joint damage is less common than in gram-positive or gram­
States), and Candida. Infections are typically monoarticular,
negative pyogenic infections.
follow an indolent course, and derive most often from hema­
togenous spread. Direct inoculation of the joint due to trauma
Mycobacterial Infections or extension from a nearby site of infection is also possible.
Several mycobacterial species can affect the musculoskeletal Both arthritis and osteomyelitis can occur. Diagnosis is often
system. the most important being Mycobacterium tuberculosis. challenging. Synovial fluid analysis typically reveals inflam­
matory fluid with negative bacterial cultures. Appropriate
Mycobacterium tuberculosis fungal cultures or synovial biopsy may yield a diagnosis. Cl
Mycobacterium tuberculosis infection exhibits extrapulmo­
nary symptoms in nearly 20% of patients. with bone or joint Viral Infections
involvement occurring in approximately 2% overall. Patients Various viral infections can lead to musculoskeletal symptoms.
at particular risk include persons who have lived in or visited Symptoms may occur via direct invasion, immune complex
an endemic area. patients with untreated HIV infection, and deposition, or less well-defined mechanisms.
patients with immunosuppression for any other reason,
including biologic therapy for autoimmune diseases. Hepatitis
Musculoskeletal M. tuberculosis can develop early after Hepatitis B virus (HBV) and hepatitis C virus (HCV) infections
infection or after an extended latency, with reactivation often frequently cause musculoskeletal symptoms. Acute HBV infec­
triggered by immunocompromise. Patients with musculo­ tion causes symmetric polyarthritis (especially in the hands
skeletal M. tuberculosis infection typically manifest a and knees) during the prodromal stage of disease, often
chronic, indolent course, often without constitutional accompanied by rash; joint pain lasts days or weeks and then

61
Infectious Arthritis

resolves. In chronic HBV, arthritis may persist, possibly due to epidermidis) are common infecting organisms. Risk factors
immune complex formation. Chronic HCV causes a wide array include superficial surgical site infection. malignancy, revision
of musculoskeletal symptoms ranging from arthralgia to arthroplasty, inflammatory arthritis. immunosuppression,
arthritis (oligoarticular or polyarticular presentations). Both obesity, and chronic illness. Infections are most likely to occur
HBV and especially HCV infection may provoke cryoglobuline­ within the first 2 years after surgery. Infections may be of early
mia, resulting in arthritis, glomerulonephritis, and other signs (<3 months after surgery), delayed (3-12 months), or late
of vasculitis such as palpable purpura. Diagnosis of HBV and (>12 months after surgery) onset. Early and delayed onset infec­
HCV is by serologic and polymerase chain reaction testing. tions are acquired during prosthesis implantation, whereas
late-onset infections arise de novo from hematogenous seeding
Parvovirus Bl9 of the joint. Prosthetic joint infection is promoted by the devel­
Parvovirus Bl 9, which causes the childhood condition known as opment of a biofilm (coalescence of bacteria) on the artificial
fifth disease (also known as erythema infectiosum), is a DNA joint surface; biofilms are resistant to treatment both because
virus with a tropism for erythrocyte precursors. Adults usually of lack of perfusion and because biofilm bacteria develop anti­
contract the vims from children; individuals at risk include microbial resistance as a consequence of their limited exposure
school workers and parents. Infection may be asymptomatic or to treatment agents.
produce a flu-like illness. As the host mounts a response, a clas­ Early-onset infections present with joint swelling, ery­
sic "slapped cheek" rash and arthritis may occur. In adults, the thema, wound drainage, and/or fever. Delayed-onset infec­
rash may be absent or atypical. The arthritis begins acutely, with tions present insidiously with prolonged joint pain , often
symmetric swelling and stiffness of the small joints of the hands without fever. Late-onset infections may present as acute pain
and feet, as well as wrists and knees, often mimicking rheuma­ and swelling. often after hematogenous seeding from a vascu­
toid arthritis. Parvovirus Bl9 arthritis typically lasts for several lar catheter or a site of infection distant from the affected joint.
weeks, but occasionally many months, before resolving. The Prompt orthopedic consultation should be sought for
presence of circulating anti-parvovims lgM antibodies provides patients with suspected prosthetic joint infection because treat­
evidence of active disease; IgG antibodies indicate prior infection ment nearly always requjres surgical intervention (in addition to
and are found in many healthy persons. Management is support­ antibiotics). ESR and CRP are typically elevated. Plain radio­
ive, using anti-inflammatories (such as NSAIDs) and analgesics. graphs are mandatory and may show erosion or looserung around
the implantation site. Aspiration of the affected joint can conJinn
Rubella the diagnosis but may be deferred if surgical intervention is
Rubella (German measles) has become uncommon in coun­ imminent. Synovial fluid cultures and blood cultures should
tries with vaccine programs; nonvaccinated populations always be obtained prior to antibiotic initiation in a stable patient.
remain at risk. Infection can cause rash, fever, and lymphade­ Surgical options include debridement. p1imary replacement of
nopathy. A polyarthritis of sudden onset may occur, affecting the affected hardware. or complete removal of the artificial joint
medium and small joints (hands, wrists, knees), usually resolv­ and replacement after the infection is fully resolved. CJ
ing within 2 weeks. Synovial fluid is inflammatory. Diagnosis is
made by documenting lgM anti-rubella antibodies or by isolat­ Infections in Previously Damaged Joints
ing the vims from synovial fluid or the nasopharynx. Arthritis Joints that have been previously damaged by trauma, inflam­
can also occur in response to the rubella vaccine. matory arthritis, degenerative arthritis, surgical procedures,
or other causes are inherently more susceptible to infection.
Chikungunya In patients with a history of joint damage or radiographic
Chikungunya virus is an alphavirus endemic to Asia and evidence of prior damage, acute pain and swelling in a joint
Africa, where it is transmitted by mosquito vectors; however, should raise suspicion for the possibility of an infected joint.
cases are increasingly seen in the United States secondary to
KEY POINTS
travel to endemic countries. Chikungunya virus causes fever,
rash, and myalgia, along with thrombocytopenia and/or leu­ • Gram-positive organisms are the most common causes
kopenia. Chikungunya arthritis is self-limited, but severe pol­ of infectious arthritis in adults, with Staphylococcus
yarthritis/tenosynovitis can persist for as long as 6 months. aureus being the most common infecting organism.
Diagnosis is by clinical context, travel history, and identifica­ • Patients with disserrunated gonococcal infection present
tion of IgM anti-Chikungunya antibodies and/or viral isolation with one of two syndromes: arthritis, tenosynovitis, and
by polymerase chain reaction. Treatment is supportive. dermatitis or a purulent monoarthritis or oligoarthritis.

C:I Prosthetic Joint Infections • Nongonococcal gram-negative bacterial arthritis is more


common in patients who are immunosuppressed, are
Prosthetic joint infection is a rare but serious complication of
elderly, or have a history of injection drug use, recent
joint replacement. Infection rates are highest in knee replace­
trauma, or gastrointestinal infection.
ments (up to 2%) and lower in hip and shoulder replacements.
(Continued)
Coagulase-negative staphylococci (for example. Staphylococcus

62
Infectious Arthritis

KEY PO I NTS (continued)

• Musculoskeletal Mycobacterium tuberculosis typically


Management
Pharmacologic Therapy
CJ
presents as a chronic, indolent process, often without
Pharmacologic therapy is the cornerstone of' treatment for
constitutional symptoms and most commonly as spon­
infectious arthritis. Bacteria I or fungal joint infections
dylitis, vertebral osteomyelitis, or hip or knee arthritis.
require antimicrobial or anlif'ungal agents (Table 30).
• Prompt orthopedic consultation should be sought for
Particularly for bacterially infected joints, antibiotic therapy
patients with suspected prosthetic joint infection because
should be initiated rapidly to prevent joint damage and other
treatment typically requires surgical intervention.
adverse outcomes. Whenever possible, synovial fluid culture

TABLE 30. Infectious Arthritis Treatment Based on the Suspected Pathogen


Likely or Identified Pathogen First-Line Therapy Second-Line Therapy Comments
Gram-Positive Cocci

If MRSA is a concern (risk Vancomycin or linezolid Clindamycin, daptomycin -


factors or known MRSA carrier)

MSSA Oxacillin/nafcillin or cefazolin - Narrow treatment to MSSA


coverage based on sensitivity
data.

Gram-Negative Bacilli

Enteric gram-negative bacilli 3rd generation cephalosporin Fluoroquinolones -


(e.g., ceftriaxone or cefotaxime)

Pseudomonas aeruginosa Ceftazidime with an Carbapenems, cefepime, -


aminoglycoside (e.g., piperacillin-tazobactam,
gentamicin) fluoroquinolones

Gram-Negative Cocci

Neisseria gonorrhoeae IV ceftriaxone x 7-14 days Fluoroquinolones (only if If suspected gonococcal


culture sensitivities support arthritis, treat for co-infection
this) with chlamydia empirically
(azithromycin or doxycycline);
in the absence of specific
culture sensitivity data,
"stepping down" to oral
therapy of any type is no
longer recommended due to
increasing resistance of N.
gonorrhoeae to commonly
used oral agents.

Gram Stain Unavailable or Inconclusive

Likely pathogen depends on E.g., vancomycin, or - Appropriate to start with broad


patient risk factors: consider vancomycin + 3rd generation antibiotic coverage and narrow
MRSA, and gram-negative cephalosporin coverage if culture data
organism if immunocompro- become available.
mised, at risk for gonococcal
infection, or with joint trauma

Borrelia burgdorferi (Lyme Oral doxycycline or amoxicillin - If concurrent neurologic


arthritis) x 28 days findings, IV ceftriaxone x 28
days.

Mycobacterium tuberculosis 3- to 4-drug treatment (e.g. - Duration may vary from 6


isoniazid, pyrazinamide, months or longer depending
rifampin, ethambutol, on drug regimen (shorter
streptomycin) treatment if rifampin is used).

Fungal infections Amphotericin B, azoles - Prolonged treatment courses


(fluconazole, itraconazole, of several months may be
voriconazole, posaconazole), needed; maintenance therapy
depending on suspected may be required in high-risk
organism or culture data patients.

IV= intravenous; MRSA = methicillin-resistant Staphylococcus aureus; MSSA = methicillin-sensitive Staphylococcus aureus.

63
Idiopathic Inflammatory Myopathies

C "'I results should guide treatment. However. if the patient is


-I unstable or culture data are unavailable. empiric therapy
CONT.
injury. DM is considered an immune complex disease with
vascular inflammation in muscle and subsequent muscle
should be initiated. Duration of treatment for bacterial damage. IBM is most likely a myodegenerative disorder with
arthritis is variable. Commonly, 2 weeks of intravenous ther­ vacuolar inclusions and a related T-cell response.
apy is followed by 2 weeks of oral therapy. However. patient
response. organism sensitivities. and severity of infection
(number of joints involved. associated bacteremia) may alter Epidemiology
the treatment course. The IIMs are rare (prevalence, 5-22/100,000). Overall female­
Management of arthritis due to viral infections is usually to-male ratio is 2:1. DM has a bimodal distribution of onset,
suppo11ive. awaiting resolution of the underlying infection. with the highest incidence in the second and fourth decades.
At1hritis due to HCV (with or without cryoglobulinemia) may PM incidence peaks in the fourth decade. IBM typically affects
improve with antiviral treatment. men older than 50 years of age.
KEY POINT
Surgical Therapy
• The idiopathic inflammatory myopathies include poly­
Drainage of purulent Ouid from a bacterially infected native
myositis, dermatomyositis, and inclusion body myositis
joint is a critical component of successful treatment. Infected
and are associated with inflammatory involvement of
native joints that can be drained fully with aspiration can be
muscle fibers and resultant weakness.
managed with either serial (usually daily) arthrocentesis or
surgical drainage. For joints that are less easily accessed (such
as the hip) or when needle drainage proves inadequate, arthro­
scopic or open surgical drainage is required. Surgical manage­
Clinical Manifestations
ment or prosthetic joint infections is discussed in Prosthetic The IIMs affect various sites and organs. PM and DM may also
Joint Infections. c:J be associated with systemic symptoms (fever, fatigue, weight
loss), Raynaud phenomenon, and arthritis. About 30% of
KEY POINTS
patients with DM and PM may present with an acute onset
• For bacterially infected joints, antibiotics should be ini­ of significant constitutional symptoms and a constellation of
tiated promptly to prevent joint damage. features called the antisynthetase syndrome. This syndrome is
• Synovial fluid culture results should guide treatment of defined by the presence of autoantibodies to aminoacyl-trans­
bacterial or fungal infections; however, if the patient is fer (t)RNA synthetase enzymes (such as anti-Jo-1) plus two of
unstable or culture data are unavailable, empiric ther­ the following clinical features: inflammatory myositis, inter­
apy should be initiated. stitial lung disease, Raynaud phenomenon, nonerosive inflam­
• Management of viral arthritis is generally supportive, matory arthritis, and mechanic's hands.
awaiting resolution of the underlying infection. IBM is rarely associated with extramuscular manifestations.
• Drainage of purulent fluid from a bacterially infected Muscular Involvement
native joint is a critical component of successful Symmetric painless proximal weakness of the arms and legs is
treatment. the classic feature of DM and PM. Onset is acute or subacute,
with disease progressing over weeks. The deltoids, arm/hip
flexors, and neck flexors are characteristically involved.
Idiopathic Inflammatory Patients report difficulty combing hair, rising from a chair, and
climbing stairs (classic triad of hair, chair, stair), and difficulty
Myopathies with routine activities such as cooking, cleaning, and shop­
ping. Muscle atrophy is absent initially but may occur in long­
Introduction standing disease.
The idiopathic inflammatory myopathies (IIMs) are character­ In contrast to DM and PM, muscle weakness in IBM char­
ized by autoimmunity and inflammatory involvement of mus­ acteristically affects both distal and proximal muscles.
cle fibers (frequently termed myositis), resulting in muscle Although typically symmetric, IBM muscle involvement may
weakness. The three IIMs are polymyositis (PM), dermatomy­ be asymmetric in up to 15% of patients. Onset is insidious and
ositis (DM), and inclusion body myositis (IBM). slowly progressive, often over years.

Cutaneous Involvement
Pathophysiology OM is associated with multiple cutaneous manifestations.
Although all three IIMs cause muscle weakness, they have dif­ Gottron sign/papules are symmetric erythematous/viola­
ferent underlying immunologic mechanisms. PM is a CD8- ceous macules, patches, or papules located on the extensor
positive T-cell-mediated immune disease with direct myocyte surfaces of the metacarpophalangeal joints (Figure 25).

64
Idiopathic Inflammatory Myopathies

FIGURE 2 5. Gottron papules. Erythematous papular and patchy eruption with


overlying scaling on the extensor surface of metacarpophalangeal joints in a
patient with dermatomyositis.

Common rashes include heliotrope rash (edematous lilac


discoloration of periorbital tissue) (Figure 26) as well as pho­
todistributed rashes such as the shawl sign (upper back)
(Figure 27) and V sign (neck/upper chest) (Figure 28).
Poikiloderma (mottled pigmentation, epidermal atrophy, and
telangiectasia) can occur in both sun-exposed and unexposed

FIGURE 2 8. V sign. Violaceous erythema and redness present on the neck and
upper chest in a patient with dermatomyositis.

areas. Gottron sign/papules and heliotrope rash are consid­


ered pathognomonic for OM. Nail changes such as cuticular
hypertrophy or nailfold capillary abnormalities can occur.
Amyopathic DM refers to classic cutaneous findings occurring
in the absence of muscle involvement. Mechanic's hands,
characterized by hyperkeratotic fissuring of the palmar and
FIGURE 2 6. Heliotrope rash. Violaceous erythematous papular eruption with lateral surfaces of the fingers , is seen in both DM and PM
hyperkeratosis on the upper eyelids in a patient with dermatomyositis. (Figure 29). See MKSAP 17 Dermatology for more information
on the cutaneous manifestations of OM.
The skin is generally uninvolved in PM and IBM.

Cardiopulmonary Involvement
Interstitial lung disease (ILD) is common in DM and PM. ILD is Cl
symptomatic in 10% to 25% of patients with DM and PM and

FIGURE 2 7. Shawl sign. Poikilodermatous (demonstrating a variety of


different shades) erythematous patchy rash on the upper back in a patient with FIGURE 2 9. Mechanic's hands. Hyperkeratotic, fissured skin on the palmar and
dermatomyositis. lateral aspects of fingers, seen in patients with polymyositis and dermatomyositis.

65
Idiopathic Inflammatory Myopathies

Cl
COl/l
can be recognized in up lo 65'7.. olpalienls screened with high­
resolution CT. I LD may precede muscle symptoms. may be
the bladder, cervix, Jung, ovaries, pancreas, and stomach.
Ovarian cancer risk may be especially increased. Age­
asymptomatic. and is associated with poor prognosis. ILD appropriate screening is recommended for all patients with
occurrence is often assoch1tecl with ,rnlisynlhelase antibodies. PM or OM, with consideration of additional testing for ovarian
including anli-Jo-1. and may cause rapidly progressive res­ cancer using imaging such as transvaginal pelvic ultrasonog­
piratory r�1ilure and death. Routine screening lor 11.D with raphy. Routine additional testing is not cost effective, but
imaging such as chest radiography or CT is appropriate in chest and abdominal CT and/or PET scanning should be con­
,1symplomatic pLllienls in the presence ofantisynthetase anti sidered if there is strong suspicion or additional risk factors
bodies. ILD is not commonly seen in IBM. are present.
All of the II Ms can cause chest wall and diaphragm mus Any relationship between malignancy and IBM remains
clc weakness. resulting in shortness or breath and. occasion poorly defined.
ally. respiratory failure. Pharyngeal muscle involvement can
KEY POINT
le,1d to aspiration pneumonia. which can worsen ILD.
Symptomatic cardiac disease is uncommon. but minor • Age-appropriate cancer screening is recommended for HVC
electroc,irdiogr,1m changes (arrhythmias and conduction patients with dermatomyositis or polymyositis, with
abnorm,ililies). presum,ibly clue to subclinical myocarditis. are consideration of additional testing for ova1ian cancer;
commonly obserYed. additional CT or PET scanning to look for underlyLng
malignancy is not cost effective unless the patient has
Gastrointestinal Involvement additional risk factors.
Weakness of the striatee! muscle of the upper esophagus and!
or oropharynx is common in IIM and can lead lo dysphagia. Diagnosis
aspiration. regurgitation. and associated pneumonitis.
IIM is a clinical diagnosis supported by tissue and laboratory
Esophageal dise,1se is more common among older patients and
studies. The unifying feature is muscle weakness; the pattern
those with 18\11. Castrointestin,11 hypomotility may occur and
of muscle and extramuscular involvement suggests the specific
promote malabsorption. C]
type. Once IIM is suspected, muscle enzyme and autoantibody
KEY POINTS studies, as well as electromyography and/or imaging studies,
• Symmetric painless proximal muscle weakness of the are performed. Muscle biopsy should be performed in most
arms and legs is the classic feature of dermatomyositis patients. Table 31 lists common and differentiating features of
and polymyositis; onset is acute or subacute. the IIMs.
• The muscle weakness associated with inclusion body
Muscle-Related Enzymes
myositis affects both distal and proximal muscle groups;
onset is insidious and slowly progressive. Muscle enzyme elevations occur in more than 90% of patients
with OM and PM, especially during active disease. Creatine
• Oermatomyositis is associated with Gottron papules/
kinase and aldolase are the most important, but others (for
sign, heliotrope rash, photodistributed rashes, and nail­
example, lactate dehydrogenase and aspartate aminotrans­
fold changes; skin is generally spared in polymyositis
ferase) also may be elevated. In OM and PM, muscle enzyme
and inclusion body myositis.
elevation is dramatic (usually 10- to SO-fold the upper limit of
• Interstitial lung disease in patients with dermatomyosi­ normal) but may or may not correlate with degree of weak­
tis or polymyositis may be asymptomatic, may precede ness. Normal muscle enzymes may be present in 5% to 10% of
muscle symptoms, is often associated with antisyn­ patients with OM or PM, particularly in chronic disease with
thetase antibodies, including anti-Jo-1, and is associ­ Joss of muscle mass. Amyopathic DM refers to OM with cuta­
ated with poor prognosis. neous involvement and normal muscle enzyme levels in the
absence of muscle manifestations.
Patients with IBM typically have mildly elevated levels of
Association with Malignancy muscle enzymes. in the range of one to threefold the upper
Increased malignancy rates are seen in both OM and PM. limit of normal.
Relative risk for malignancy is 2.4 to 7.7 for OM and 1.7 to 2.1
for PM, compared with the general population. Malignancy Autoantibodies
risk is highest during the first year after OM/PM diagnosis and A number of characteristic autoantibodies are seen in OM and
declines thereafter but remains elevated even after S years. PM. Antinuclear antibodies (ANA) are detected in approxi­
Cutaneous necrosis, vasculitis, and older age at diagnosis may mately 80% of patients with OM and PM.
be risk factors. Myositis-specific autoantibodies (MSA) are found in
Cancers associated with DM and PM are similar to those approximately 30% of patients with PM and/or OM and
seen in the general population. including adenocarcinomas of include antibodies to aminoacyl-transfer (t)RNA synthetases

66
Idiopathi c Inflam matory Myopathies

TABLE 31. Features of the Idiopathic Inflammatory Myopathies


Dermatomyositis Polymyositis Inclusion Body Myositis
Epidemiology

Sex Childhood: female= male; Female> male Male> female


Adult: female> male
Age of onset All <50years >SO years
Familial association No No Yes

Clinical Presentation

Onset of symptoms Acute or subacute Acute or subacute Insidious


Weakness Proximal Proximal Proximal and distal
Course Rapid Rapid Slowly progressive
Skin rash Yes No No

Diagnostic Findings

Muscle enzymes >10times normal >10 times normal <10times normal


Electromyography Myopathic Myopathic Myoneuropathic
Muscle biopsy Microinfarctions; myofibril Myofibril necrosis Rimmed vacuoles; inclusions
grouping

Treatment

Response Good Good Poor

(antisynthetase antibodies, including anti-Jo-1), antibodies to also be detected. Because of limited specificity, the pattern of
signal recognition particle (SRP), and antibodies to Mi-2, a muscle involvement on MRI is usually reported as either con­
nuclear helicase. sistent or likely inconsistent with IIM.
Myositis-associated autoantibodies (MAA), commonly
anti-PM-Sci, anti-Ku, anti-Ro/SSA, anti-La/SSB, and anti­ Electromyography
Ul-ribonucleoprotein (RNP) antibodies, are found in approxi­ Electromyography (EMG) is obtained in most patients with
mately 30% of patients with PM and/or OM. !IM due to almost universal availability and high sensitivity.
In clinical practice, ANA testing is usually done initially EMG can identify characteristic abnormalities and exclude
during diagnostic work-up. Positive ANA supports the diagno­ neuropathic disease. The characteristic triad of EMG findings
sis of IlM, although negative tests do not exclude it. MSA test­ includes short duration small, low-amplitude polyphasic
ing is usually performed during initial diagnostic evaluation or potentials; fibrillation potentials at rest; and bizarre, high
once diagnosis is established because MSA may be associated frequency, repetitive discharges. As with MRI, these findings
with particular clinical syndromes, patterns of organ involve­ are not adequately specific for definitive diagnosis alone
ment, certain histopathologic findings, and prognosis. MAA because they may also be seen in infectious, toxic, or meta­
testing is usually ordered if previous testing has been incon­ bolic myopathies.
clusive. Positive anti-Ul-RNP antibodies suggest an overlap
syndrome with mixed connective tissue disease. Muscle Biopsy
To date, no autoantibodies characteristic of IBM have been Biopsy of an affected muscle is the gold standard for diagnosis
identified. and should be obtained initially in most patients to distinguish
between OM, PM, and IBM as well as nonimrnune, noninflam­
Imaging Studies matory myopathic disorders. Muscle biopsy is also sometimes
Imaging studies are sensitive but nonspecific for the IIMs and needed during the course of disease to evaluate treatment
cannot definitively differentiate among inflammatory, meta­ response or rule out a new or alternative cause of weakness.
bolic, traumatic, and other myopathies. Muscle imaging can OM is typically associated with a mixed B and CD4-
identify affected sites for biopsy, aid in clinical diagnosis, and positive T-cell perivascular infiltrate, vasculitis with microin­
help follow the response to treatment and course of disease. farction and grouping of muscle fibers, and perifascicular
MRI is usually the preferred modality and can show active muscle atrophy. PM infiltrates primarily consist of C08-
inflammation of muscle and follow its course; edema, active positive T cells, involving all layers of muscle fibers with inva­
myositis, fibrosis, and calcification in advanced disease can sion and myophagocytosis. The primary target seems to be the

67
Idiopathic Inflammatory Myopathies

vascular endothelium in OM and the myofibrils themselves in drug-induced myopathies. Colchicine and hydroxychloro­
PM. IBM demonstrates a mild T-cell predominant inflamma­ quine, both used frequently in rheumatologic disease, may
tory infiltrate (similar to PM), along with the presence of cause myopathy. Glucocorticoids, used in treating IIMs, can
rimmed vacuoles and eosinophilic and basophilic inclusions themselves cause muscle weakness that may require with­
within muscle fibers. Filamentous tubules seen on electron drawal of therapy. Statins are a frequent cause of myalgia and/
microscopy are highly specific for IBM. or asymptomatic creatine kinase elevations; many cases are
mild, and not all will require discontinuation. Rarely, statin
KEY POINTS
use can result in an autoimmune myopathy associated with
• Oermatomyositis and polymyositis are associated with antibodies to the enzyme HMG-CoA reductase. See MKSAP 17
elevation of muscle enzymes such as creatine kinase Neurology for more information on myopathy.
and aldolase as well as autoantibodies such as antinu­
clear antibodies, myositis-specific autoantibodies, and KEY POINT

myositis-associated autoantibodies. • The differential diagnosis of the idiopathic inflamma­


• In the idiopathic inflammatory myopathies, muscle tory myopathies includes other myopathies that present
imaging can identify sites for biopsy, aid in clinical diag­ with muscle weakness, toxic myopathies, and disorders
nosis, and help assess the response to treatment. that present with weakness but without muscle
involvement.
• Biopsy of an affected muscle is the gold standard for
diagnosis to distinguish between dermatomyositis, pol­
ymyositis, and inclusion body myositis as well as non­ Management
immune, noninflammatory myopathic disorders. IIM management requires a comprehensive approach combin­
ing aggressive pharmacotherapy, physical therapy to maintain
muscle strength, and supportive interventions. Glucocorticoids,
Differential Diagnosis initiated at high doses and tapered once serum muscle
The differential diagnosis for myopathy is broad (Table 32) and enzymes normalize, are the mainstay treatment for PM and
includes hereditary diseases, infection, and endocrine and OM. Clinical symptoms may take months to improve.

TABLE 32. Differential Diagnosis of Myopathy


Myopathy• Common Examples Comments

Idiopathic inflammatory myopathies Dermatomyositis (DM); polymyositis (PM); DM and PM: acute/subacute, symmetric
inclusion body myositis (IBM) proximal weakness, pathognomic rash in
DM; IBM: insidious, proximal and distal
weakness

Connective tissue disease Systemic lupus erythematosus; mixed Prominent extramuscular features typical
connective tissue disease; systemic of underlying disorder
sclerosis

Endocrine disease Hypothyroidism; Addison disease; vitamin Subtle extramuscular features; routine
D deficiency testing to evaluate

Infection-induced myopathies Bacterial: Lyme disease; pyomyositis Detailed history and physical usually gives
clue; difficult to diagnose
Viral: influenza; HIV; hepatitis B; hepatitis C

Parasitic: toxoplasmosis; trichinosis

Drug- or toxin-induced myopathies Glucocorticoids; ethanol; statins; Lack of typical symptoms; muscle biopsy
colchicine; hydroxychloroquine is useful
Metabolic myopathies Acid maltase deficiency; McArdle disease Typical history; associated with exercise/
exertion; ischemic forearm testing
Mitochondrial myopathies Kearns-Sayre syndrome; Leigh syndrome Difficult to diagnose; muscle biopsy
needed to evaluate
Muscular dystrophies Duchenne dystrophy; Becker dystrophy Suggestive history; onset typically in young
adulthood
Neurologic and neuromuscular disorders Amyotrophic lateral sclerosis; myasthenia Suggestive history; some are weakness
gravis; Guillain-Barre syndrome; mimickers (lack of direct involvement of
cerebrovascular accident; multiple muscle)
sclerosis

aAsymptomatic elevation in serum muscle enzymes, especially creatine kinase, is fairly common, may occur without any underlying disease, and is more frequent in black persons.
Although usually benign, it must be distinguished from an actual myopathic disorder if weakness and other suggestive features are present.

68
Systemic Vasculitis

Immunosuppressive therapy with methotrexate or azathio­ autoimmune condition (Table 33). Vasculitis can mimic, or be
prine is used for glucocorticoid-resistant disease or glucocor­ mimicked by, other systemic conditions (Table 34). Discussion
ticoid sparing; it is also used in poor prognosis groups and here is limited to diseases in which vasculitis is the primary
patients with extramuscular disease such as ILD. Other immu­ disorder.
nosuppressive agents (such as mycophenolate mofetil and
leflunomide) have also been used. Intravenous immune globu­
lin (IVIG) therapy is recommended as an alternative treatment Large-Vessel Vasculitis
for DM. Giant cell arteritis (GCA) and Takayasu arteritis (TA) consti­
IBM is generally resistant to pharmacotherapy, although tute the large-vessel vasculitides. Although not a vasculitis,
glucocorticoids, methotrexate, and/or !VIG are often tried.
KEY POINTS
TABLE 33. Causes of Secondary Vasculitis
• Management of the idiopathic inflammatory myopathies Medications
combines pharmacotherapy, physical therapy to main­
tain muscle strength, and supportive interventions. Antimicrobial agents
Vaccines
• Glucocorticoids are the mainstay therapy for dermato­
myositis and polymyositis, with high doses used until Antithyroid agents
serum muscle enzymes normalize. Anticonvulsant agents

• In dermatomyositis and polymyositis, immunosuppres­ Antiarrhythmic agents


sive therapy is used for glucocorticoid-resistant disease Diuretics
and for glucocorticoid sparing; it is also used in poor Other cardiovascular drugs
prognosis groups or in patients with severe extramus­
Anticoagulants
cular disease such as interstitial lung disease.
Antineoplastic agents
• Inclusion body myositis is generally resistant to treat­
Hematopoietic growth factors
ment, although glucocorticoids, methotrexate, and/or
intravenous immune globulin are often tried. NSAIDs
Leukotriene inhibitors

Psychotropic drugs
Prognosis Sympathomimetic agents
DM and PM are responsive to early aggressive treatment, with Allopurinol
symptomatic and histologic improvement in most cases and a Tumor necrosis factor modulatory agents
5-year survival rate of 95%. Older age, underlying malignancy,
Interferon alfa
pulmonary fibrosis, and esophageal dysmotility are associated
with poorer prognosis; the most common causes of death are Infections
malignancies, infections (particularly aspiration pneumonia), Hepatitis A, B, and C virus
profound muscle weakness, cardiovascular disease, and res­
HIV
piratory failure.
Bacterial endocarditis
Patients with IBM tend to experience a long slow decline
of muscle function regardless of therapy, with a large impact Parvovirus B 19

on daily activities but little impact on overall survival. Neoplasms

KEY POINT Hairy cell leukemia (associated with polyarteritis nodosa)


• The most common causes of death in patients with pol­ Other hematologic and solid malignancies
ymyositis and dermatomyositis are malignancies, infec­ Autoimmune Diseases
tions, profound muscle weakness, cardiovascular dis­
ease, and respiratory failure. Systemic lupus erythematosus

Rheumatoid arthritis

Sjogren syndrome

Systemic Vasculitis Inflammatory myopathies


Systemic sclerosis
Introduction Relapsing polychondritis
Vasculitis refers to inflammation of arteries, veins, or capillaries. Inflammatory bowel disease
Involved vessels may be large, intermediate, or small. Vasculitis
Primary biliary cirrhosis
can be idiopathic, or secondary to an antigen trigger or other

69
Systemic Vasculitis

: TABLE 34. Differential Diagnosis ofVasculitis


Disease Comments

Infection (sepsis, Heart murmur, rash, and/or musculoskeletal symptoms can occur in bacterial endocarditis or viral
endocarditis, hepatitis) hepatitis.

Drug toxicity/poisoning Cocaine, amphetamines, ephedra alkaloids, and phenylpropanolamine may produce vasospasm,
resulting in symptoms of ischemia.

Coagulopathy Occlusive diseases (disseminated intravascular coagulation, the antiphospholipid antibody syndrome,
thrombotic thrombocytopenic purpura) can produce ischemic symptoms.

Malignancy Paraneoplastic vasculitis is rare. Any organ system may be affected, but the skin and nervous system are
the most common. Vasculitic symptoms may precede, occur simultaneously with, or follow diagnosis of
cancer. Lymphoma occasionally may involve the blood vessels and mimic vasculitis. Consider malignancy
in patients with incomplete or no response to therapy for idiopathic vasculitis.

Atrial myxoma Classic triad of symptoms: embolism, intracardiac obstruction leading to pulmonary congestion or heart
failure, and constitutional symptoms (fatigue, weight loss, fever). Skin lesions can be identical to those
seen in leukocytoclastic vasculitis. Atrial myxomas are rare but are the most common primary intracardiac
tumor. Myxomas also can occur in other cardiac chambers.

Multiple cholesterol Typically seen in patients with severe atherosclerosis. Embolization may occur after abdominal trauma,
emboli aortic surgery, or angiography. May also occur after heparin, warfarin, or thrombolytic therapy. Patients
may have livedo reticularis, petechiae and purpuric lesions, and localized skin necrosis.

polymyalgia rheumatica (PMR) is discussed here because of decreased pulses and hand or forearm ischemia. Aortic aneu­
its close association with GCA. rysm is a potential complication as is aortic dissection, which
may occur with or without preceding aneurysm formation. Of
Giant Cell Arteritis patients with GCA, 30% to 50% concurrently have PMR (see
Epidemiology and Pathophysiology Polymyalgia Rheumatica).
GCA is the most common primary vasculitis (10 cases/100, 000). GCA should be considered in any individual :2:50 years of
GCA is most common among white persons, especially north­ age with temporal or other atypical headache, jaw claudica­
ern Europeans. Women are affected more often than men (2:1 tion, or visual changes, with or without PMR. Nearly all
ratio). Median age of onset is 70 years; GCA in patients under patients with GCA have a markedly elevated erythrocyte
age 50 is rare. sedimentation rate (ESR), which is typically elevated to
GCA characteristically affects the second- to fifth-order greater than 50 mm/h. On examination, the temporal artery
branches of the aorta. Affected arteries include the external may be erythematous, swollen, and tender. Definitive diag­
carotids, temporal arteries (hence the alternative designation nosis should be pursued whenever possible via biopsy of a
temporal arteritis), and ciliary and ophthalmic arteries. temporal artery segment. However, the presence of skip
Subclavian and brachia! arteries can be affected. Uncommonly, lesions and the need for expert examination of the biopsied
intracranial arteritis may occur. specimen may sometimes lead to false-negative results. In
In GCA, dendritic cells in the vessel adventitia become such cases, biopsy of the contralateral artery may be war­
activated and recruit T cells and monocytes into the vessel ranted. Angiography of the arms is indicated in patients with
wall. Within the media, macrophages coalesce into multinu­ peripheral vascular insufficiency.
cleated giant cells and secrete metalloproteinases, disrupting
the internal elastic lamina. Simultaneously, intimal prolifera­ Management
tion can reduce blood flow and induce partial or complete Treatment of suspected GCA should never be deferred pending
ischemia of the affected tissue bed. biopsy because 1) treatment is rapidly effective and prevents
blindness, and 2) biopsy specimens remain interpretable for at
Clinical Manifestations and Diagnosis least 2 weeks after treatment initiation. In virtually all
Patients with GCA classically report scalp pain, headache, or instances, patients with GCA should receive oral prednisone
tenderness over the temporal artery, which is usually unilat­ (60 mg/d [or 1 mg/kg/d]). In patients with visual loss, intrave­
eral. Systemic inflammation results in malaise, weight loss, and nous (IV) pulse glucocorticoids (typically methylprednisolone,
low-grade fever. Patients may experience pain in the muscles of 1000 mg/d for 3 days) may be tried, although restoration of
mastication during chewing (jaw claudication). The most vision should not be anticipated. Prednisone should generally
feared outcome is blindness, caused by ophthalmic and/or be administered for approximately 1 month (or until resolu­
posterior ciliary artery occlusion. Blindness can be sudden or tion of signs and symptoms), with subsequent dose reduction
preceded by transient visual loss (amaurosis fugax). Diplopia is at a rate of about 10% every few weeks. The ESR usually responds
common. In some patients, brachia! artery involvement causes rapidly and can then serve as a marker of disease activity.

70
Systemic Vasculitis

Patients should be monitored for relapse, in which case aorta) and its major branches. In contrast to GCA, TA is rare
prednisone should be transiently increased; treatment dura­ (2 cases/million patient-years) and mainly affects young
tion may range from 6 to 18 months. Data from observational women (9:1 ratio) with a typical age at onset between 15 and
studies support that the coadministration of low-dose aspi­ 25 years.
rin (80 mg/d) further reduces the risk of blindness and Histologically, the pathophysiologic processes of TA and
deserves consideration. GCA are similar, with infiltration ofT cells, macrophages, and
When prednisone is contraindicated or in cases where giant cells in the vessel wall.
GCA has failed to respond to glucocorticoid therapy, metho­
trexate, the anti-interleukin-6 agent tocilizumab, and cyclo­
phosphamide have been used, although evidence of their
Clinical Manifestations and Diagnosis
Clinical manifestations of TA result from inflammation and/or [:J
effectiveness is limited. the consequences of large artery disease. Fever, fatigue,
The long-term outcomes of adequately treated GCA are malaise, and weight loss, along with arthralgia and myalgia,
good, with recurrences uncommon. often precede the onset of specific signs and symptoms.
Patients may have diminished or absent pulses in the brachia!,
Polymyalgia Rheumatica femoral, axillary, carotid. ulnar, and radial distributions, often
Epidemiology and Pathophysiology accompanied by bruits. Blood pressure measurements may
PMR is not a vasculitis but is discussed here because of its differ on the left and right sides. Aortic coarctation may cause
relationship with GCA; most experts consider that GCA and decreased kidney perfusion, compensatory renin/angiotensin
PMR exist on a spectrum of a single unifying disease. Of release, and hypertension. Wall weakness may lead to aneu­
patients with GCA, 30% to 50% have concurrent PMR; con­ rysms of the aorta and other vessels; aortic root aneurysm can
versely, 10% to 15% of patients with PMR, but no clinical result in aortic valve incompetence. Central nervous system
GCA, show occult GCA on blind temporal artery biopsy. involvement is uncommon but can lead to syncope, stroke,
PMR epidemiology is similar to GCA, but PMR is 2 to 3 times and ocular findings.
more common. TA should be suspected in patients younger than 40 years
with unexplained systemic inflammation and/or signs and
Clinical Manifestations and Diagnosis symptoms of large-vessel impairment. Angiogram demon­
Patients with PMR experience symmetric pain and stiffness strates focal arterial narrowing and/or aneurysms (Figure 30).
in the shoulder, neck, and hip regions, without actual joint Laboratory testing is nonspecific but typically reveals an
arthritis. Common symptoms include the inability to elevated ESR.
comb hair or rise from a chair unassisted. In some cases,
mild synovitis occurs in the wrists and hands. Strength
and muscle enzymes (creatine kinase, aldolase) are gener­
ally normal.
PMR is a clinical diagnosis based on the characteristic
symptoms in a patient older than SO years. Patients with PMR
demonstrate systemic inflammation (for example, elevated
ESR), along with low-grade fever, malaise, and fatigue. In the
absence of GCA signs and symptoms, temporal artery biopsies
are not routinely performed.

Management
In contrast to GCA, PMR responds well to low-dose prednisone
(10-20 mg/d). Most patients respond with dramatic improve­
ment in their symptoms within 3 days, and failure to do so
suggests an alternative diagnosis. Patients with PMR frequently
experience partial relapses during prednisone tapering,
requiring transient up-titration. PMR treatment is therefore
commonly of long duration; some patients need several years
to completely discontinue prednisone, whereas others remain
on prednisone indefinitely.
FIGURE 3 0. Takayasu arteritis. This angiogram of the aorta demonstrates high·
Takayasu Arteritis
grade stenosis of the proximal right subclavian artery (white arrow) as well as the
Epidemiology and Pathophysiology leh subclavian artery just below the origin of the leh vertebral artery (black arrow).
InTakayasu arteritis (TA), the affected arteries are primarily Incidentally noted is anatomic variation with a common origin of the right brachio·
the aorta (ascending, descending thoracic, and abdominal cephalic artery and the leh common carotid artery.

71
Systemic Vasculitis

Cl Management
(Off
Untreated TA conveys significant morbidity and mortality:
bowel infarction may ensue. Neurologic involvement com­
monly takes the form of mononeuritis multiplex. Skin find­
early recognition and management are therefore critical. ings include livido reticularis, purpura, and painful
Standard TA treatment is high-dose prednisone (1 mg/kg/ct), subcutaneous nodules.
followed by taper. Patients with structural damage to affected Elevated inflammatory markers are typically present, and
vessels may require angioplasty to improve blood flow. or, in hepatitis B antigenemia is detectable in infected patients.
severe cases. arterial bypass or reconstruction. Cl Vascular imaging studies are most useful for diagnosis.
Mesenteric and/or renal a11e1y imaging with either angiogra­
KEY POINTS
phy or CT angiography reveals medium-sized artery aneu­
• Giant cell arteritis should be considered in any individ­ rysms and stenoses.
ual older than age 50 years with temporal or other atyp­
ical headache, jaw claudication, or visual changes. Management
• Although temporal artery biopsy is indicated to PAN carries significant potential for poor outcome, including
make a definitive diagnosis of giant cell arteritis, ini­ kidney disease and mortality. Treatment is aggressive, includ­
tiation of prednisone should not be delayed; treat­ ing high-close prednisone and cyclophosphamide. Other
ment can prevent blindness, and biopsy specimens disease-modifying antirheumatic drugs (methotrexate, aza­
remain interpretable for at least 2 weeks after treat­ thioprine, mycophenolate mofetil) may be considered for
ment initiation. milder disease or as maintenance therapy. Patients with
• Patients with polymyalgia rheumatica experience sym­ hepatitis B should receive plasma exchange and antiviral
metric pain and stiffness in the shoulders, neck, and therapy whenever feasible. Cl
hips, along with low-grade fever, malaise, and fatigue;
standard treatment is low-dose prednisone. Primary Angiitis of the Central Nervous System
Epidemiology and Pathophysiology
• Takayasu arteritis should be suspected in patients
Primary angiitis of the central nervous system (PACNS) is
younger than age 40 years with unexplained systemic
exceedingly rare (2.4 cases/million person-years). Median age
inflammation and/or signs and symptoms of large­
of onset is 50 years, with men affected twice as often as
vessel impairment; standard treatment is high-dose
women. Progression may be insidious. Vascular involvement is
prednisone.
limited to intracerebral vessels; a necrotizing granulomatous
vasculitis is typical.
Medium-Vessel Vasculitis
CJ Polyarteritis Nodosa
Clinical Manifestations and Diagnosis
The most common features of PACNS are recurrent head­ Cl
Epidemiology and Pathophysiology aches and progressive encephalopathy. Strokes, transient
Polyarteritis nodosa (PAN) is rare (5-10 patients/million): men ischemic attacks, visual field defects, and seizures also occur.
and women are affected equally. In approximately 30% of Systemic vasculitis is absent, and systemic inflammatory
cases, PAN is associated with hepatitis B virus infection: hepa­ markers are not elevated. Diagnosis requires a high index of
titis B vaccination reduces PAN incidence. The remainder of suspicion and aggressive evaluation. including lumbar punc­
cases are idiopathic. PAN characteristically affects medium ture, MRI, cerebral angiography, and brain biopsy.
and small arteries; arterial inflammation results in vessel nar­ Cerebrospinal fluid analysis reveals lymphocytosis and ele­
rowing and. characteristically, aneurysms and microaneu­ vated protein. MRI reveals multiple diffuse and focal abnor­
rysms. The classic features of PAN result from ischemia and/or malities but is nonspecific. lntracranial angiography
aneurysm rupture. Organs typically affected include the kid­ typically reveals areas of ectasia and stenosis, but other enti­
neys. gastrointestinal tract (especially the mesenteric artery ties (atherosclerosis, vasospasm) may produce similar pic­
and small intestine), peripheral nervous system, and skin. tures. The definitive test is a brain biopsy revealing
Involvement of the testicles, ovaries. and breasts can occur. granulomatous vasculitis. Brain biopsy also permits identifi­
Coronary arteries can also be affected. cation and/or elimination of other important diagnoses,
including infection and malignancy. Because vascular
Clinical Manifestations and Diagnosis involvement in PACNS is patchy. a normal biopsy may some­
Patients with PAN usually present with nonspecific inflamma­ times represent a false negative. Nonetheless, brain biopsy is
tmy symptoms (fatigue, malaise, fever, myalgia. arthralgia). nearly always necessary.
Renal artery vasculitis can cause decreased kidney blood flow.
resulting in decreased glomerular filtration, renin/angiotensin Management
overproduction, and hypertension. Glomerulonephritis does PACNS can lead inexorably to cognitive decline, dementia, and
not occur. Abdominal symptoms include chronic or intermit­ death. Management is aggressive and includes high-dose glu­
tent ischemic pain. especially after eating (abdominal angina); cocorticoids and daily oral cyclophosphamide. Cl

72
Systemic Vasculitis

Kawasaki Disease granulomatosis with polyangiitis (formerly known as Churg­


Epidemiology and Pathophysiology Strauss syndrome). In addition. rapidly progressive glomeru­
Kawasaki disease (KD) is a medium-vessel vasculitis of infants lonephritis may sometimes present as an ANCA-associated
and children, most commonly boys of Asian origin. A seasonal process (see Ml<SAP 17 Nephrology, Glomerular Diseases).
predominance (late winter-spring) suggests an as-yet uniden­ ANCAs were tirst recognized when sera from patients with
tified infectious trigger. vasculitis were noted to bind to fixed neutrophils in vitro in
two distinct patterns-either perinuclear (p-ANCA) or dif­
Clinical Manifestations and Diagnosis fusely throughout the cytoplasm (c-ANCA). These patterns
Symptoms of KD include high spiking fevers, conjunctivi­ were found to reflect distinct antibody types, with p-ANCA
tis, and mucositis of the lips and oral cavity, including a mainly directed toward the neutrophil enzyme myeloperoxi­
strawberry-like tongue. A polymorphous truncal rash is dase (M PO). and c-ANCA toward the neutrophil proteinase 3
accompanied by palmar and plantar erythema, induration, (PR3). These antibodies are now specifically identified using
and desquamation. Cervical lymphadenopathy is common. enzyme-linked immunosorbent assays.
Most concerning is vasculitic involvement of the coronary Specific ANCA-associated diseases are classically associ­
arteries, resulting in coronary artery aneurysms and/or ated with one or the other, but not both, ANCA types. Although
thrombosis, and potentially myocardial infarction and the clinical pictures of c-ANCA and p-ANCA vasculitides difter.
mortality. Other cardiac manifestations include myocardi­ investigators have proposed a common process through which
tis and pericarditis. all ANCA may promote vasculitis (Figure 31). Based on the
proposed model. the extent of ANCA deposition need not be
Management large to cause disease. Thus. on immunofluorescence of histo­
Standard KD treatment consists of intravenous immune glob­ pathologic specimens, little antibody staining is seen. leading
ulin (2 g/kg) plus aspirin (30-100 mg/kg/d). This regimen
reduces inflammation and the risk of coronary aneurysms.
Patients who develop medium-large coronary artery aneu­ Fe
.
Partia!
PR3 or MPO
Neutrophil

I
receptor stimu on
rysms during the acute phase of the illness are at increased granule secretion/binding
\ �
risk of coronary artery stenosis and require close follow-up
with periodic testing that may include electrocardiography,
� �
echocardiography, and ischemic stress testing.
KEY POINTS � �

[==:=:::] [==:=:::]
• In approximately 30% of cases, polyarteritis nodosa
(PAN) is associated with hepatitis B virus infection;
hepatitis B vaccination reduces PAN incidence.
• Treatment of polyarteritis nodosa includes high-dose A B
prednisone and cyclophosphamide. ANCA binding ANCA engaging
�<./
�y
G
to MPO or PR3 � Fe receptor
• P rimary angiitis of the central nervous system presents
with recurrent headaches and progressive encephalopathy. 0
0
• Kawasaki disease occurs in infants and young children;
symptoms include high spiking fevers, conjunctivitis,
act1vat1on/
rash, and mucositis of the lips and oral cavity. endothelial
insult

CJ
Small-Vessel Vasculitis
Small-vessel vasculitis affects post-capillary venules. arteri­
oles. and capillaries. Because these vessels are ubiquitous, C
[
D
]
small-vessel vasculilides present with a wide range of mani­
FIGURE 31 . Model of presumed ANCA activity in promoting vasculitis. A,
festations. Two major categories of small-vessel vasculitis are Quiescent neutrophil and adjacent endothelial cell. B, Low-level activation of the
discussed here: ANCA-associated vasculitis and immune neutrophil ("priming") results in early partial neutrophil degranulation (release of
complex-mediated vasculitis. Cl granule contents to the extracellular environment), allowing de nova exposure of
MPO and/or PR3, which result in humoral immunity and the formation of anti-MPO
or anti-PR3 autoantibodies. C, Adherence of secreted MPO/PR3 on the neutrophil

CJ
ANCA-Associated Vasculitis
surface permits ANCA binding to the neutrophil. D, ANCA bound to neutrophil sur­
ANCA-associated vasculitis constitutes three diseases: granu­ face engages the neutrophil Fe receptor, fully stimulating the neutrophil and lead­
lomatosis with polyangiitis (formerly known as Wegener grnn­ ing to release of enzymes such as metalloproteinases, as well as toxic oxygen radi­
ulomatosis), microscopic polyangiitis, and eosinoph i I ic cals, that damage the endothelium. MPO = myeloperoxidase; PR3 = proteinase 3.

73
Systemic Vasculitis

Cl
CO�T.
to the designation of' ANCA vasculitides as pauci-immune. In
all ANCA vasculitides. fever. malaise. weight loss. myalgia. and
positivity for c-ANCA anti-PR3 is considered by many rheu­
matologists to be adequate to initiate treatment. However.
arthralgia are common. Cl because all effective treatments have toxicity. most experts
recommend that a tissue diagnosis is essential in all but the
Granulomatosis with Polyangiitis most clear-cut cases. Tissues available for biopsy include
Epidemiology and Pathophysiology skin. lung. sinuses. sural nerve. and kidneys. Lung biopsy is
Granulomatosis with polyangiitis (GPA) is the most common invasive but most reliably provides the characteristic histo­
ANCA-associated vasculitis (8-10 cases/million). GPA typically pathologic findings. including both vasculitis and necrotiz­
affects middle-aged to older patients (mean age, approxi­ ing granulomas (characteristic of GPA but not the other
mately 55 years). White persons, typically northern Europeans, ANCA diseases). Kidney biopsy is high yield when kidney
are more commonly affected. Of note, 80% to 90% of patients involvement is apparent. but because granulomas are absent.
with GPA demonstrate a c-ANCA pattern and anti-PR3 anti­ the biopsy will not distinguish GPA from the other ANCA
body positivity. conditions. Skin biopsies are less often nondiagnostic but are

Cl
readily obtained.
Clinical Manifestations and Diagnosis
Classic GPA af"fects the upper respiratory tract (including Management
sinuses and ears). lungs. and kidneys. Nasal and or sinus pain For severe or generalized disease. treatment of GPA consists of'
and stuff1ness. rhinitis. and or epistaxis are common. Nc1sal high-dose glucocorticoids plus cyclophosphamide for approx­
inflammation can cause cartilage damage and collapse (saddle im,Hely 6 months. followed by maintenance therapy with
nose deformity). Hearing loss may occur. usually due lo eus­ methotrexate. azathioprine. or mycophenolate mofeli I.
lachian tube damage. Inflammation can damage the trachea. Glucocorticoids alone are insufficient to control GPA. Recent
causing airway narrowing and obstruction. Ocular inflamma­ studies suggest that the anti-B-cell antibody rituximab is as
tion may lead to blindness. In the lungs. nodules and infiltrates ef"flcacious as cyclophosphamide, possibly with less toxicity.
occur (Figure 32): capillaritis may result in alveolar hemor­ For limited disease (no kidney disease and only mild lower
rhage and hemoptysis. In the kidneys. rapidly progressive airway disease). methotrexate and glucocorticoids alone may
crescenlic glomerulonephritis may ensue. Other organ sys­ be sul"ficient: such patients should be carefully monitored for
tems commonly involved include the skin (painful cutaneous treatment failure necessitating the more aggressive regimen.
nodules. palpable purpura. urticaria! and ulcerative lesions) Using these approaches. GPA mortality has declined from 90'Y.,
and peripheral nen·es. to around 10%. Cl
Diagnosing GPA requires the appropriate clinical and
serological picture. In the setting of a classic GPA picture. Microscopic Polyangiitis
Epidemiology and Pathophysiology
Microscopic polyangiitis (MPA) shares features with GPA but
differs with regard to organ involvement and autoimmunity.
MPA occurs about half as frequently as GPA. Men are affected
more commonly than women (2:1 ratio). Patients tend to
develop MPA in their fourth to fifth decades, but earlier and
later cases occur. Patients with MPA classically express
p-ANCA and anti-MPO antibodies.

CJ
Clinical Manifestations and Diagnosis
The typical organ involvement in MPA is vasculitis of' the kid­
neys and lungs. Kidney involvement is nearly ubiquitous: as
in GPA. the lesions are those of necrotizing glomerulonephri­
lis. Crescent formation is common. and immune deposits are
sparse or absent. Lung involvement occurs in more than half
or cases. consisting of non-granulomatous alveolar innltrates
that may be fleeting or persistent and are histologically
ref"Jected as pulmonary capillaritis with neutrophilic
inf"iltrates. Most ominous is diffuse alveolar hemorrhage.
which can be fatal if not adequately managed. Upper airv,·ays
remain uninvolved. Skin rashes (especially palpable purpura)
are common.
FIGURE 3 2. A frontal chest radiograph showing both patchy alveolar infiltrates
Diagnosis requires the appropriate conjunction of symp­
(white arrows) and nodules (black arrows) in a patient with granulomatosis with
polyangiitis (formerly known as Wegener granulomatosis). toms and serologies. Most patients are positive for p-ANCJ\ and

74
Systemic Vasculitis

Cl anti-MPO antibodies. Biopsy ofan affected organ reveals capil­ KEV PO 1 ·NTS (continued)
laritis. glomerulonephritis. and or leukocytoclastic vasculitis
CONT. • Microscopic polyangiitis typically involves the kidneys
in the absence ofgranulomas.
and lungs, and patients classically express p-ANCA and
antimyeloperoxidase (MPO) antibodies.
Management
Like GPA. MPA treatment requires high-dose glucocorticoids • Treatment of both granulomatosis with polyangiitis
plus cyclophosphamide for 6 months. followed by mainte­ and microscopic polyangiitis consists of high-dose glu­
nance therapy with melhotrexate. azathioprine, or mycophe­ cocorticoids plus cyclophosphamide for approximately
nolate mofetil. As in GPA. rituximab for MPA has recently 6 months, followed by maintenance therapy.
shown promise. CJ • Eosinophilic granulomatosis with polyangiitis is
associated with atopy, hypereosinophilia, lung dis­
Eosinophilic Granulomatosis with Polyangiitis ease, and peripheral nerve disease; glucocorticoids
Epidemiology and Pathophysiology may be sufficient for mild/limited disease, and more
Eosinophilic granulomatosis with polyangiitis (EGPA) is severe disease may be treated with glucocorticoids
the rarest ANCA-associated vasculitis. Strong associa­ plus cyclophosphamide, followed by maintenance
tion of EGPA with asthma and eosinophilia suggests an therapy.
atopic trigger.

Cl Clinical Manifestations and Diagnosis Immune Complex-Mediated Vasculitis


Several vasculitides arise from the formation of immune
Patients with EGPA report a preceding history of alopy.
including allergic rhinitis. nasal polyps. or asthma. The complexes. If not cleared, such complexes deposit in small
symptomatology of EGPA depends upon localization or the vessels, activate complement, and recruit neutrophils. Any
vasculitis. Lung disease is common. manifested as infiltrates organ may be affected, but skin and joint involvement is
and capillaritis. Peripheral nerve disease is more common in common. Dermal capillaritis characteristically allows
EGPA than in other ANCA vasculitides. presenting as mono ery throcyte extravasation (palpable purpura), worse in
or polyneuropathy or mononeuritis multiplex. Kidney dis dependent areas. Influxed neutrophils undergo cell death
ease is somewhat less common than in other ANC/\ diseases. and breakup (clasis) as well as release of pyknotic nuclei
Hypereosinophilia. in both the peripheral blood ancl involved (nuclear dust), a process designated as leukocytoclastic vas­
tissues. is characteri.slic: a diagnosis of EGPA should be culitis (Figure 33).
revisited in the absence of eosinophils. Tissue biopsy
shows necrotizing vasculitis with eosinophilic infiltrates. Cryoglobulinemic Vasculitis
Eosinophilic granulornas may be seen in the tissues as well Epidemiology and Pathophysiology
as the vessels. lgE levels are f"requently ele,·atec\. EGPA In cryoglobulinemic vasculitis, antibodies have the special
patients who are ANCA-posilive typically display the characteristic of precipitating in vitro at temperatures
p-ANCA,anti-MPO pattern. Because up to 40'X, of patients below normal body temperature (37.0 °C (98.6 °F]). The
are ANCA-negative. a negative ANCA test should not elimi­ immune complex formation that typically occurs in vivo in
nate EGPA as a diagnosis. these diseases may not relate to this cryoglobulinemic
property because body core temperature is sufficient to
Management prevent cold-induced precipitation. However, severe cold
One unique aspect or EGPA may be the sensitivity or eosino exposure of the fingertips, ear helices, and tip of the nose
phils to glucocorticoids. In patients with mild or limited dis­ may result in cold-related precipitation, vascular ischemia,
ease, glucocorlicoicl treatment alone may therefore be and infarction.
suflicient. In more se,·ere disease. glucocorticoids plus cyclo­ Three types of cryoglobulins are recognized (Table 35).
phosphamicle is preferred. followed by maintenance therapy Type I is a monoclonal antibody, often resulting from hemato­
with a less toxic immunosuppressive agent. As in all ANCA logic malignancy. Type I cryoglobulins rarely form immune
diseases, the potential lor permanent tissue damage mandates complexes in vivo but may cause hyperviscosity. Type II cryo­
an early and aggressive approach if" f"ull return lo !"unction is lo globulins consist of a polyclonal IgG antibody together with a
be anticipated. CJ monoclonal lgM with rheumatoid factor activity (that is, binds
KEV POINTS to other immunoglobulins, contributing to immune complex
formation). Type II cryoglobulins precipitate ex vivo in the
• Granulomatosis with polyangiitis (formerly known as
cold but also form immune complexes in vivo to generate vas­
Wegener granulomatosis) typically affects the upper
cular inflammation. Ninety percent of patients with type II
respiratory tract, lungs, and kidneys and is associated
cryoglobulinemia have chronic hepatitis C virus infection,
with c-ANCA and antiproteinase 3 (PR3) antibody
which furnishes the antigen(s) for immune complex forma­
positivity. (Continued) tion; the remaining 10% are idiopathic and designated as

75
Sy s te m ic Vascul itis

TABLE 35. Types of Cryoglobulins


Type Comp osition Common Underlying

[ l )[ l )!( ))
Monoclonal lgM
Conditions

Hematologic malignancy
(e.g., Waldenstrom
Antigen macroglobulinemia,
Immune complex multiple myeloma)
Antibody Polyclonal lgG + Chronic hepatitis C
Endothelial cells monoclonal lgM infection; HIV infection;
rheumatoid factor idiopathic (essential mixed)

)( )( Ill Polyclonal lgG + Autoimmune disease


polyclonal lgM (e.g., systemic lupus
rheumatoid factor erythematosus, rheumatoid
arthritis)

A Basement membrane

essential mixed cryoglobulinemia. Type Ill cryoglobulins are


also mixed but consist of polyclonal IgG together with poly­
(( )!( lI( )I( )I( ))
clonal IgM rheumatoid factor. Type Ill cryoglobulins typically
occur secondary to other autoimmune diseases and resolve

G0 0iJ with appropriate disease treatment.

(ii) Clinical Manifestations and Diagnosis


Cl
0iJ 07 As in all immune complex vasculitides. palpable purpura is
the characteristic rash in type II cryoglobulinemia. Other
organs characteristically involved include peripheral nerves
)( and the kidneys (glomerulonephritis). Less commonly. gastro­
intestinal, central nervous system, and other organ involve­
ment occurs.
Diagnosi depends upon recognition of palpable pur­
B
pura in the setting of other relevant organ involvement.
Positivity for hepatitis C suggests but does not prove a diag­
nosis of"type II cryoglobulinemia. Characteristically. patients
with cryoglobulinemic vasculitis manifest evidence of com­
plement activation. with C4 disproportionately low relative
JI( ))
to 3. Inflammatory markers are typically elevated. Absence
or rheumatoid !"actor argues against type II or type Ill cryo­
globulinemia . Cryoglobulins can be assessed directly in
serum from blood that has been allowed to clot at 37.0 °c
(98.6 °F). In patients with cryoglobulinemia. such serum.
when incubated at 4.0 °C (39.2 °F). forms cryoprecipitates.
the extent of which can be expressed as a volume-percent
(cryocrit).

CJ Management
Dennitive cryoglobulinemia treatment requires clearance of
C the driving antigen. For patients with hepatitis C-related
FIGURE 3 3. The process of classic immune complex-mediated small-vessel
type ll cryoglobulinemia. this means resolving the hepatitis
vasculitis. A, In the setting of antigen/antibody balance, immune complexes form C infection with antiviral therapy. However. if the vasculitic
that may become trapped in small vessels. B, The formation of immune complexes organ involvement is severe. reduction of cryoglobulins
lead to complement activation by the classical pathway, particularly including the themselves is imperative. Classic treatment involves gluco­
generation of C3b (an opsonin) and CSa (a chemoattractant). C, The presence of
corticoids. cyclophosphamide. and plasmapheresis. More
CSa attracts neutrophils, which engage C3b and antibody Fe tails to undergo fur­
ther activation. Neutrophils also invade the vascular wall. Some neutrophils recently. anti-8-cell therapy with rituximab has demon-
undergo cell death and breakdown (leukocytoclasis), resulting in the presence trated efficacy at reducing cryoglobulinemia and may
of cellular fragments within the vessel wall. provide a less toxic alternative. Cl

76
Systemic Sclerosis

Henoch-Schonlein Purpura
Epidemiology and Pathophysiology
Henoch-Schonlein purpura (HSP) is the most common child­
hood vasculitis (incidence, 20/100,000) and tends to appear
after upper respiratory infections. HSP is rarer among adults.
The HSP antigen is unknown, but the immune complex anti­
bodies are unique in belonging to the lgA subfamily.

c:J Clinical Manifestations and Diagnosis


The characteristic symptoms of HSP are abdominal pain and
palpable purpura. Purpuric lesions commonly occur on palms
and soles. Gastrointestinal ischemia may be severe enough to
cause intestinal bleeding. Arthritis is common. and other
organ systems may be involved; patients with I-ISP may pre­
sent with glomerulonephritis.
The diagnosis is based on the clinical picture in both chil­
dren and adults. Laboratory studies are nonspecific but con­
firm systemic inflammation. Diagnosis is established by biopsy
of the affected organ. Skin biopsy will show leukocytoclastic
vasculitis and. on immunofluorescence, lgA and C3 comple­
FIGURE 3 4. Palpable purpura is shown, the classic rash of small-vessel,
ment deposition. Kidney biopsy will show lgA nephropathy. immune complex-mediated vasculitides. The lesions are nonblanching and repre·
Serum IgA levels may be elevated. sent extravasations of blood from damaged vessels. Purpuric lesions are typically
more prominent on the lower extremities, a consequence of the superimposed
Management effect of gravity on oncotic pressure.
Pediatric I-ISP is typically self-limiting, and treatment
usually requires only supportive care. When more severe
the antigen is not readily accomplished. most clinicians con­
organ involvement occurs. glucocorticoids may be con­
sider prednisone to resolve the inflammatory component.
sidered but have limited benefit on the purpura and
Other agents that may help improve hypersensitivity vasculitis
nephritis. Adults tend to experience a more severe course,
include colchicine. dapsone. and NSA!Ds. Cl
are more likely to relapse. and are typically treated with
glucocorticoids. KEY POINTS

• Ninety percent of patients with type II cryoglobuline­


Hypersensitivity Vasculitis mia have chronic hepatitis C virus infection.
Epidemiology and Pathophysiology
• Palpable purpura is the characteristic rash in type II
Hypersensitivity vasculitis represents a hypersensitivity
cryoglobulinernia; other organs characteristically
response to a known or unknown antigen, such as a drug or
involved include peripheral nerves and the kidneys.
infection. As in the other small-vessel vasculitides, hypersen­
sitivity vasculitis typically results from immune complex for­ • Henoch-Schonlein purpura occurs mainly in chil­
mation and deposition. Complement is activated, and dren, and characteristic symptoms are abdominal
neutrophils accumulate in capillaries, arterioles. and post­ pain and palpable purpura; this disease is typically
capillary venules. often resulting in the classic pathologic self-limiting.
appearance of a leukocytoclastic vasculitis. In contrast to HSP, • Hypersensitivity vasculitis presents as palpable purpura
lgA is usually not involved. and represents a hypersensitivity response to an antigen
that typically resolves when the offending agent is
Clinical Manifestations and Diagnosis removed.
As in all immune complex vasculitides, the characteristic rash
of hypersensitivity vasculitis is palpable purpura (Figure 34).
Other rashes. including vesicles. pustules. maculopapular
lesions. and urticaria, may occur. Other organ systems are Systemic Sclerosis
usually spared.
Introduction
Management Systemic sclerosis (SSc) is an autoimmune disease character­
Definitive hypersensitivity vasculitis management consists of ized by fibrosis, with resultant thickening and hardening of
removing the offending antigen. after which the rash should the skin (scleroderma). Internal organ involvement is common
resolve within weeks. When the rash is severe, or if removal of and causes significant morbidity and mortality.

77
Systemic Sclerosis

skin induration without any other features occurs in less


Pathophysiology
than 10% of patients with SSc. Sclerodactyly (skin fibrosis
Deposition of increased amounts of structurally normal col­ of the fingers or toes) and characteristic extracutaneous
lagen is the classic finding of SSc and the cause of most clinical features are often present and support the diagnosis. Skin
manifestations. This fibrosing phenotype is accompanied by induration plus one or more of the following extracutane­
autoimmunity as well as small artery endothelial damage and ous features strongly suggests the diagnosis of SSc: Raynaud
dysfunction. Fibroblast activation and collagen deposition lead phenomenon, digital infarction, and/or pitting; heartburn,
to tissue fibrosis in skin and other organs due to the complex dysphagia, or diarrhea; hypertension and/or kidney dis­
interactions between the immune system, interstitium, and
ease; dyspnea on exertion, interstitial lung disease, or
vasculature. The impact of each of these factors is variable and pulmonary arterial hypertension; or mucocutaneous telan­
leads to heterogeneity in the clinical features.
giectasias. Skin thickening confined to a single area sug­
The triggering and initial proliferating events of SSc
gests localized scleroderma in its various forms; internal
remain undefined. No consistent environmental trigger has
organ involvement is extremely rare in these limited condi­
been identified. A role for heredity is supported by genetic loci tions (see Table 36).
associated with the disease, but these are shared with other
In patients with skin thickening and clinical features
autoimmune diseases and provide limited insight into the
suggestive of SSc, autoantibody testing is done to further sup­
generation of the SSc phenotype.
port the diagnosis. Antinuclear antibodies are often present
(70% prevalence). Anticentromere antibodies are associated
Epidemiology with LcSSc (15%-40% prevalence) and the CREST (calcinosis,
Raynaud phenomenon, esophageal dysmotility, sclerodactyly,
Annual incidence of SSc is 1 to 2/100,000 persons, and preva­
and telangiectasia) syndrome, whereas anti-Scl-70 antibodies
lence is 19 to 75/100,000. Peak age of onset occurs between 30
are associated with DcSSc (15%-40% prevalence). Patients
and so years, with a female predominance (3:1 ratio). SSc is
who have DcSSc without anti-Scl-70 antibodies may instead
more common (and perhaps more severe) in black persons.
have antibodies to an alternative antigen, RNA polymerase III.
Still other autoantibodies have been identified whose sensi­
Classification tivity for SSc is low (20%) but whose specificity is high (>95%)
(Table 37).
SSc is the most characteristic of the scleroderma spectrum
A skin biopsy is usually not needed to confirm the diag­
disorders, a group of diseases sharing the feature of skin hard­
nosis because the findings are classic and characteristic.
ening. The scleroderma spectrum disorders include SSc,
localized scleroderma (in which fibrosis is histopathologically KEY POINT
identical to SSc but limited to a single patch of the skin), and • In patients with skin thickening and clinical features
scleroderma-like conditions (which include other disorders suggestive of systemic sclerosis, autoantibody testing
associated with thickened, sclerotic skin). SSc is further char­ (antinuclear, anti-Scl-70, anticentromere, and anti­
acterized as two distinct subsets based on skin involvement: RNA polymerase III) is done to further support the
diffuse cutaneous systemic sclerosis (DcSSc) and limited diagnosis.
cutaneous systemic sclerosis (LcSSc).
Table 36 describes the common manifestations and fea­
tures of the scleroderma spectrum disorders. Clinical Manifestations
KEY POINTS
and Management
Treatments addressing the underlying mechanisms of SSc
• Diffuse cutaneous systemic sclerosis is characterized by
have yet to be developed. Instead, therapeutic options cur­
extensive distal and proximal skin thickening (chest,
rently focus on managing individual organ manifestations.
abdomen, and arms proximal to wrists) and is com­
monly accompanied by internal organ fibrosis.
Cutaneous Involvement
• Limited cutaneous systemic sclerosis is characterized by Skin thickening occurs in most patients with SSc. Skin involve­
distal (face, neck, and hands), but not proximal, skin ment varies in both the extent of body surface involved and the
thickening; it is usually unaccompanied by internal severity of induration. In LcSSc, scleroderma is restricted to
organ fibrosis but is more likely to be associated with the hands and, to a lesser extent, the face and neck. Scleroderma
pulmonary arterial hypertension. involving the chest, abdomen, forearms, upper arms, and
shoulders indicates DcSSc.
In early SSc, swelling, edema, and erythema of the hands
Diagnosis and forearms may precede skin induration. Pruritus may be a
SSc should be suspected in the presence of symmetric skin significant symptom. Sclerodactyly may develop (Figure 35, on
induration on the hands, arms, face, and/or torso. Isolated page 80), and vascular involvement (see Vascular Involvement)

78
Systemic Scleros i s

TABLE 36. Common Manifestations/Features of the Scleroderma Spectrum Disorders


Disorder Manifestation/Feature Comments
Systemic Sclerosis
Diffuse cutaneous systemic sclerosis Distal and proximal skin thickening (chest, Skin involvement is extensive and is
(DcSSc) abdomen, arms proximal to wrists); commonly accompanied by internal organ
commonly has visceral organ involvement fibrosis and ILD
Limited cutaneous systemic sclerosis Distal (face, neck, hands), but not proximal, More likely to develop PAH and Raynaud
(LcSSc) skin thickening; typically not accompanied phenomenon early in disease and display
by internal organ fibrosis features of the CREST syndrome
Systemic sclerosis sine scleroderma Fibrosing organ involvement without skin Difficult to diagnose; prognosis may be
thickening similar to LcSSc

Localized Scleroderma

Morphea Plaques generally on the trunk Systemic manifestations or Raynaud


phenomenon is extremely rare
Linear scleroderma Streaks/lines of thickened skin Same as above

Scleroderma-like Conditions•

Eosinophilic fasciitis Orange peel induration (peau d'orange) of Full-thickness skin biopsy demonstrates
proximal extremities with sparing of hands and lymphocytes, plasma cells, and eosinophils
face; peripheral eosinophilia; skin retraction infiltrating the deep fascia; glucocorticoids
over the superficial veins may be more are the mainstay of treatment
apparent with elevation of an affected limb
Nephrogenic systemic fibrosis Secondary to gadolinium in patients with Skeletal muscle fibrosis with contractures
kidney disease; brawny, wood-like and/or cardiac muscle involvement can
induration of extremities, sparing the digits occur, with cardiomyopathy and increased
mortality; changes in use and formulation
of gadolinium have reduced incidence
Scleredema lndurated plaques/patches on back, Typically seen in long-standing diabetes
shoulder girdle, and neck mellitus
Scleromyxedema Waxy, yellow-red papules over thickened Associated with paraproteinemia (lgGt..)
skin of face, upper trunk, neck, and arms; and may therefore occur in the setting of
deposition of mucin with large numbers of multiple myeloma or AL amyloidosis; more
stellate fibroblasts in the dermis frequent in men
Chronic graft versus host disease Lichen planus-like skin lesions or localized Occurs most commonly after hematopoietic
or generalized skin thickening stem cell transplantation; may occasionally
be seen after blood transfusion in an
immunocompromised host
Drug and toxin exposure Can produce scleroderma-like tissue Examples: bleomycin, docetaxel,
changes pentazocine, L-tryptophan, organic solvents

CREST= calcinosis cutis, Raynaud phenomenon, esophageal dysmotility, sclerodactyly, and telangiectasia; ILD= interstitial lung disease; PAH = pulmonary arterial hypertension.
Sclerderma-like skin changes may also occur as a manifestation of systemic endocrine, kidney, or infiltrative disorders.
11

TABLE 37. Autoantibodies and Their Associations in Systemic Sclerosis


A utoantibody Clinical Associations Comments

Antinuclear DcSSc; LcSSc Overall prevalence in SSc: 70%; not associated with specific
manifestations

Anticentromere (kinetochore LcSSc ± PAH Overall prevalence in SSc about 30%; highly associated (>90%) with
proteins) CREST variant of LcSSc

Anti-Scl-70 (DNA DcSSc; ILD Overall prevalence in SSc: about 30%; highly associated with DcSSc
topoisomerase-1)

Anti-RNA polymerase Ill DcSSc; kidney disease Useful in DcSSc with negative Scl-70
Anti-U3-RNP (fibrillarin) DcSSc; PAH; myositis Associated with poor outcome; more common in black men

Anti-PM-Sci Myositis Associated with overlap syndrome and polymyositis

Anti-Ku Myositis Rare occurrence


Anti-Th/To LcSSc; PAH Rare occurrence

CREST= calcinosis, Raynaud phenomenon. esophageal dysmotility, sclerodactyly, and telangiectasias; DcSSc = diffuse cutaneous systemic sclerosis; ILD = interstitial lung disease;
LcSSc = limited cutaneous systemic sclerosis; PAH = pulmonary arterial hypertension; RNP = ribonucleoprotein; SSc= systemic sclerosis.

79
Systemic Sclerosis

responds poorly to therapy; no treatment has been shown to


be consistently effective.

Musculoskeletal Involvement
In addition to hand swelling, patients with early SSc (espe­
cially DcSSc) often develop arthralgia, myalgia, and fatigue.
Inflammatory arthritis with palpable tenderness and joint
swelling is atypical; joint erosions should suggest an overlap
with rheumatoid arthritis. Later in the disease, fibrosis of the
periarticular structures leads to joint pain, immobility, and
contractures, especially in the fingers and extremities. Fibrosis
around tendons is sometimes associated with palpable and/or
audible deep tendon friction rubs, which occur more com­
FIGURE 3 5. Sclerodactyly in a patient with systemic sclerosis. lndurated/tight monly in DcSSc and may connote aggressive disease and inter­
skin of the fingers with resorption at the tips, leading to loss of pulp. nal organ involvement. Fibrosis also occurs around nerves,
causing cranial and peripheral entrapment neuropathies and
can result in digital ulcers and pitting of the fingertips (Figure occasionally producing autonomic neuropathy. Fibrosis within
36). Some patients develop telangiectasias and calcinosis cutis. muscle can lead to myopathy; myositis has also been reported.
Scleroderma of the face may lead to the reduction of skin Low-dose systemic glucocorticoids, methotrexate, and
wrinkles and limitation of the oral aperture. Late in the dis­ leflunomide may help treat inflammatory arthritis and myosi­
ease, induration improves and skin may become soft and thin, tis but have limited efficacy for other SSc musculoskeletal
making the diagnosis difficult. manifestations. Surgical release of entrapped nerves can
SSc pruritus is managed symptomatically with antihista­ relieve compressive symptoms. Surgical tendon release and/or
mines and skin emollients. Limited data suggest that the early reconstruction may occasionally be an option.
inflammatory phases of skin disease may respond to systemic Vascular Involvement
glucocorticoids, weekly methotrexate, or other immunosup­
Raynaud phenomenon (sequential white, blue, and red color
pressive therapies, and a therapeutic trial can be considered.
changes in the digits precipitated by cold or stress) occurs in
However, glucocorticoids may potentiate the risk of renal crisis
almost all patients with SSc. Raynaud phenomenon is initially
(see Kidney Involvement) and should be administered cau­
transient and reversible; later, structural changes develop
tiously. The indurated, thickened phase of skin involvement
within small blood vessels, resulting in permanently impaired
flow that produces acrocyanosis, digital pitting, and/or ulcera­
tions. In contrast, primary Raynaud disease is commonly seen
in young women, has a benign and usually self-limiting
course, and does not cause vascular damage and digital ulcer­
ations. It can be followed periodically in the absence of digital
pits or nailfold capillary abnormalities.
In patients with LcSSc, Raynaud phenomenon may pre­
cede other disease manifestations by years or decades. In
contrast, Raynaud phenomenon among DcSSc patients usu­
ally coincides with .the appearance of skin and/or musculo­
skeletal manifestations. Office-based examination of the
nailfold capillaries of patients with SSc using a dermatoscope
or ophthalmoscope reveals both capillary destruction and
dilated capillary loops, which can distinguish SSc from pri­
mary Raynaud as well as nonvascular causes of tissue fibrosis
(Figure 37).
Avoidance of cold exposure, keeping core temperature
warm to increase shunting of blood flow peripherally, and
avoiding smoking reduce the risk ofRaynaud episodes. Calcium
channel blockers, antiplatelet agents, and topical nitrates are
first-line pharmacotherapy and can reduce the number and
severity of episodes. Refractory patients may be treated with
FIGURE 3 6. Digital pitting in a patient with systemic sclerosis. Digital pits and phosphodiesterase-5 inhibitors (sildenafil or tadalafil).
hyperkeratosis at the tips of the fourth and fifth fingers. Endothelin-1 blockers can also be effective and may prevent

80
Systemic Sclerosis

creatinine concentration, and/or hypertension, but most do


not progress to chronic kidney disease. Kidney involvement is
more common and more severe in OcSSc. The most striking
and life-threatening manifestation of kidney disease is sclero­
derma renal crisis (SRC). SRC occurs in 10% to 15% of patients,
is more frequent in OcSSc, and tends to occur early in the
disease course; if untreated, it carries a mortality rate
approaching 90%. In SRC, involvement of afferent arterioles
leads to glomerular ischemia and hyperreninemia. The typical
presentation is acute kidney injury and severe hypertension,
mild proteinuria, urinalysis with few cells or casts, microan­
FIGURE 3 7. A 40x magnification of the nailfold using wide-field microscopy
giopathic hemolytic anemia, and thrombocytopenia. Some
revealing a capillary pattern characterized by dilatation in some areas and avascu­
larity in others during the "active phase" of diffuse cutaneous systemic sclerosis. patients develop pulmonary edema and hypertensive enceph­
alopathy. Occasionally, patients remain normotensive despite
kidney dysfunction.
recurrences of digital ulcerations. Regional sympathetic block­ ACE inhibitors significantly improve kidney survival and
ade or digital sympathectomy may be tried to reverse vasos­ decrease mortality among patients with SRC. The mechanism
pasm and save an at-risk digit; localized injection of botulinum of action of ACE inhibitors is believed to be mitigation of the
toxin A has also shown promise. Surgical debridement and effect of interstitial fibrosis and vascular dysfunction in the
amputations may be needed for severe ischemia and gangrene glomerular arterial bed. Treatment with an ACE inhibitor
that does not respond to salvage therapy. should be initiated promptly in SSc patients with even mild
hypertension or otherwise unexplained elevations in serum
Gastrointestinal Involvement creatinine levels. The ACE inhibitor should be up-titrated until
More than 70% of patients with SSc have clinical gastrointesti­ good control of blood pressure is achieved and continued even
nal involvement. Upper involvement is common, with fibrosis in the setting of kidney disease. Prophylactic use of ACE inhib­
causing pharyngeal dysfunction, esophageal hypomotility, and itors in normotensive patients is controversial, with some
lower esophageal sphincter incompetence. Consequences studies suggesting worse overall outcomes. Angiotensin recep­
include dysphagia, chronic gastroesophageal reflux, esophagi­ tor blockers (ARBs) are an alternative for patients who cannot
tis, stricture, Barrett esophagus, and pulmonary microaspira­ take ACE inhibitors, although ARBs are less effective for man­
tion. Vascular abnormalities in the stomach cause angiodys­ aging SRC.
plasias of the gastric antral mucosa! lining (so-called
watermelon stomach), resulting in recurrent bleeding and Pulmonary Involvement
chronic anemia. Involvement of the intestines causes dysmo­ Pulmonary involvement is frequent (>70%) in patients with
tility, malabsorption, and blind loop formation. Patients may SSc and can be symptomatic and disabling. The two principal
present with bloating, weight loss, alternating constipation clinical manifestations are interstitial lung disease and pul­
and diarrhea, and pseudo-obstruction. Wide-mouthed diver­ monary vascular disease. Other pulmonary manifestations
ticula may be seen on imaging studies of the colon but are include pleuritis, recurrent aspiration, organizing pneumo­
rarely of clinical significance. Primary biliary cirrhosis may nia, and hemorrhage from endobronchial telangiectasias.
occur, especially in patients with LcSSc. Anal involvement can Patients with SSc also have an increased risk for developing
cause fecal incontinence. lung cancer.
H 2 blockers and proton pump inhibitors are effective thera­
pies for esophagitis and gastritis. Oysmotility is managed with Interstitial Lung Disease
promotility agents such as metoclopramide; in refractory cases, Interstitial lung disease (!LO) usually develops subacutely with
octreotide can be tried. Malabsorption due to bacterial over­ progressive fibrosis. Patients with OcSSc and anti-Sci-70 anti­
growth is evaluated through glucose hydrogen breath testing and bodies are at higher risk for developing !LO. The most com­
may be managed with rotating courses of antibiotics. Patients mon symptom of !LO is slowly progressive dyspnea, at first on
may also need nutritional support as well as enzyme and vitamin exertion, but later at rest. Other symptoms include nonpro­
supplementation. Pseudo-obstruction is managed conserva­ ductive cough, decreased exercise tolerance, and chest pain.
tively with bowel rest and proximal decompression as needed. Auscultation reveals "Velcro" -like inspiratory crackles, most
Gastritis and vascular ectasia are evaluated with upper endos­ prominent at the lung bases. Some patients develop acute
copy; ectasias can be ablated with laser/photocoagulation. inflammatory response/alveolitis or early pulmonary fibrosis
in the absence of respiratory symptoms or physical findings.
Kidney Involvement Pulmonary function testing reveals decreased lung volumes
Kidney involvement is common in patients with SSc. Up to and decreased 0Lco. High-resolution noncontrast chest CT
50% of patients have mild proteinuria, elevation in the plasma reveals ground glass and reticular linear opacities in patients

81
Systemic Sclerosis

with alveolitis or fibrosis, and honeycombing in the later pericardia! effusion occur but are rarely symptomatic.
stages. Lung biopsy is rarely needed but may be useful in Coronary artery vasospasm and structural disease lead to
patients with atypical presentations or concern about other contraction band necrosis (pathologic finding ofischemia fol­
diseases such as infection or cancer. lowed by reperfusion due to vasospasm) and patchy myocar­
Patients with SSc who h,we evidence of alveolitis and/or dial fibrosis. promoting cardiomyopathy and heart failure.
rapidly progressive lung disease may be treated with immuno­ Conduction disturbances and arrhythmias are common and
suppressive agents. Oral or intravenous cyclophosphamide probably relate to fibrosis of the conduction system. Many
provides modest benefit in the f1rst year. but these benefits are deaths among patients with SSc are sudden and possibly a
lost by 24 months of follow-up. Azathioprine may have a role result of ventricular arrhythmia.
as mairnenance therapy following a 6-month course of cyclo­ There is no specific treatment for cardiac fibrosis.
phosphamide. High-dose glucocorticoids are frequently used Management of the complications and consequences of SSc
in SSc patients with ILD. but there is no clear e,·idence of their cardiac disease should be carried out in collaboration with an
benef1t. ancl their use may convey an increased risk of SRC. CJ experienced cardiologist. Cl
KEY POINTS
Pulmonary Arterial Hypertension
Pulmonary vascular disease occurs in up to 40% of patients • In early systemic sclerosis, swelling, edema, and ery­
with SSc. Vascular disease leading to pulmonary arterial thema of the hands and forearms may precede skin
hypertension (PAH) may occur secondary to !LO (typically in induration, and pruritus, sclerodactyly, and digital
OcSSC) or as an isolated process (typically in LcSSc). Patients ulcers and pitting may develop.
are usually asymptomatic in early disease but later develop • Upper gastrointestinal involvement is common in
dyspnea on exertion and diminished exercise tolerance. Severe systemic sclerosis, with fibrosis causing pharyngeal
disease can lead to right-sided heart failure. Pulmonary artery dysfunction, esophageal hypomotility, and lower
thrombosis can also occur and is frequently fatal. esophageal sphincter incompetence.
Pulmonary function tests and echocardiography are • Nailfold capillarioscopy can distinguish secondary
useful in the diagnosis and evaluation for PAH. Baseline and Raynaud phenomenon associated with systemic sclero­
annual monitoring of PAH is recommended in all patients sis from primary Raynaud phenomenon.
with SSc without advanced !LO. Pulmonary function tests
• ACE inhibitors significantly improve kidney survival
demonstrate a reduced 0Lco, and echocardiogram can esti­
and decrease mortality among patients with sclero­
mate pulmonary artery systolic pressure in the presence of
derma renal crisis.
tricuspid regurgitation. Right heart catheterization (RHC)
should be performed prior to initiation of treatment. RHC • Baseline and annual monitoring of pulmonary arterial
can accurately document the pressure, follow the readings, hypertension is recommended for all patients with
assess responsiveness to vasodilators, and guide choice of systemic sclerosis without advanced interstitial lung
treatment. disease.
All patients with PAH should be considered for oxygen
supplementation. Anticoagulation should be considered after
Pregnancy and Systemic Sclerosis
bleeding risk assessment. Vasodilating agents, including
phosphodiesterase-5 inhibitors (sildenafil or vardenafil) and Pregnancy in patients with SSc carries an increased risk of
spontaneous miscarriage and abortion as well as increased
prostacyclin analogues (iloprost, epoprostenol, or treprosti­
rates of premature birth and low-birth-weight full-term
nil), have demonstrated efficacy in relieving symptoms of
PAH associated with SSc. Endothelin receptor antagonists infants. Pregnant patients with SSc may develop decompen­
(bosentan and ambrisentan) have also shown efficacy in sated pulmonary arterial hypertension or scleroderma renal
crisis, each carrying a high complication and/or mortality
improving symptoms and delaying progression of PAH in SSc.
rate. It may be difficult to distinguish SSc kidney dysfunc­
Select patients may benefit from a vasodilator in combination
with an endothelin receptor antagonist. Patients with PAH tion from preeclampsia or eclampsia. Many medications
must be closely monitored using 6-minute walk tests and useful in SSc are contraindicated in pregnancy. Management
serial RHC studies. by an experienced physician is therefore imperative. See
Principles of Therapeutics for more information on medica­
Cardiac Involvement tions and pregnancy.
Cardiac in\'olvement in SSc may be due to cardiac fibrosis or KEY POINT
coronary artery disease or secondarily due to systemic or pul­
• Pregnancy in patients with systemic sclerosis carries an
monary hypertension. Intrinsic cardiac fibrosis is uncommon
increased risk of spontaneous miscarriage and abortion
,1 nd usually asymptomatic. Symptomatic cardiac involvement
as well as increased rates of premature births and low­
in SSc is rare (IO'Y.,-20'Y., prevalence) but has a poor prognosis.
birth-weight full-term infants.
with 5-year mortality rates of 75'Y.,. Pericarditis and

82
Other Rheumatologic Diseases

Other Rheumatologic TABLE 38. The International Study Group Diagnostic


Criteria for Behi;:et Syndrome
Diseases Recurrent painful oral ulceration (2:3 times in a 12-month
period) is a mandatory feature for diagnosis. Ulcers can occur
Beh�et Syndrome anywhere in the mouth or on the tongue and often resolve
spontaneously within a few weeks.
Behc;et syndrome is a form of vasculitis that affects small to
large arterial vessels and can affect veins as well. All patients At least two of the following are also required for diagnosis:

with this syndrome have recurrent painful oral ulcerations; Recurrent painful genital ulceration: These ulcers are found
on the scrotum in men or the labia in women and may lead
patients may also demonstrate additional distinctive features,
to scarring.
including hypopyon (Figure 38) and pathergy (Figure 39).
Eye involvement: Panuveitis, isolated anterior uveitis or
Behc;et syndrome is more common among individuals living posterior uveitis, or retinal vasculitis can occur and be vision
in or with ancestry from East Asia to Turkey; prevalence is threatening. Hypopyon is a distinctive feature of the anterior
6.6/100,000 in the United States, but 300/100,000 in Turkey. uveitis of Behc;:et, in which a fluid collection of leukocytes is
visible in the anterior chamber.
A genetic role in Behc;et pathogenesis is underlined by the
increased prevalence of HLA-BSl among affected individuals. Skin involvement: Erythema nodosum, pseudofolliculitis, or
acneiform lesions can occur. Acneiform lesion on trunk or
Diagnostic criteria are listed in Table 38. thighs should raise a concern for Behc;:et and may
Other major manifestations include pulmonary artery, demonstrate small-vessel vasculitis on biopsy.
aorta, and/or femoral artery inflammation (3%-12% of Pathergy test: In some patients with Behc;:et, transient
patients). Aneurysms and stenosis can occur. A rare variant of subcutaneous insertion of a sterile, large-bore needle under
the skin results in the development of a pustule at the site
Behc;et is Hughes-Stovin syndrome, characterized by pulmo­
24-48 hours later. Positive pathergy tests occur more
nary artery aneurysms and systemic thrombophlebitis. Central commonly in endemic areas and are rare in the United States.
nervous system involvement occurs in 3% to 25% and can
include headaches, stroke, pyramidal signs, behavioral
changes, or, rarely, dural sinus thrombosis. Gastrointestinal
involvement may be hard to distinguish from inflammatory bowel disease, but ulceration typically involves the ileum
rather than the rectal or perianal regions and does not lead to
fistula formation.
Treatment is commensurate with disease severity. Low­
dose prednisone or colchicine is used for oral/genital ulcers,
and high-dose prednisone and immunomodulating agents
such as azathioprine are used for more severe disease. Tumor
necrosis factor (TNF)-a inhibitors, interferon alfa, and anti­
interleukin (IL)-1� therapy have been used in recalcitrant or
severe cases.
KEY POINT

• Behc;et syndrome is characterized by recurrent painful


oral ulcers plus at least two of the following: recurrent
FIGURE 3 8. The eye of this patient with Beh�et syndrome shows hypopyon, a genital ulcers, eye involvement, skin involvement, and
layered collection of pus in the anterior chamber, representing a form of anterior
uveitis. In addition to Beh�et, hypopyon may indicate the presence of other auto·
pathergy.
immune diseases, sight-threatening infectious keratitis, or endophthalmitis.

Relapsing Polychondritis
Relapsing polychondritis (RP) is an inflammatory disease
affecting cartilaginous tissues and is thought to be driven by
an autoimmune response to type II collagen. Table 39 presents
the McAdams criteria for RP. Auricular involvement affects the
helix but spares the earlobes (Figure 40). Nasal chondritis can
result in collapse of the nasal bridge (saddle nose deformity),
which can also be seen in trauma, granulomatosis with poly­
angiitis (formerly known as Wegener granulomatosis), cocaine
use, congenital syphilis, and leprosy. Airway stenosis from
tracheal ring involvement and aortitis/large-vessel vasculitis
FIGURE 3 9. Pathergy, a pustule-like lesion or papule that appears 48 hours may occur and be life-threatening. In 30% of patients, RP
after a sterile skin prick by a 20· to 21-gauge needle in Beh�et syndrome. coexists with other inflammatory/autoimmune diseases.

83
Other Rheumat ologic Diseases

TABLE 39. McAdams Criteria for Diagnosis as tocilizumab (an IL-6 receptor antagonist) are currently
of Relapsing Polychondritis being assessed.
Criteria• Approximate KEY POINT
Frequency
• Relapsing polychondritis is characterized by chondritis
Recurrent bilateral auricular chondritis 90% of the ears, nose, and/or respiratory tract; nonerosive
Nonerosive inflammatory polyarthritis 65% inflammatory polyarthritis; ocular inflammation; and
Nasal chondritis 60% cochlear and/or vestibular dysfunction.
Ocular inflammation (conjunctivitis, 55%
keratitis, scleritis/episcleritis, uveitis)
Respiratory tract chondritis involving 50% Adult-Onset Still Disease
larynx or trachea
Adult-onset Still disease (AOSD) is a multisystem inflamma­
Cochlear and/or vestibular dysfunction 10%
(neurosensory hearing loss, tinnitus, and/ tory disease characterized by high spiking fevers, a salmon­
or vertigo) colored rash, arthritis, and high neutrophil counts (Table 40).
aRelapsing polychondritis is diagnosed when three of the six criteria are present.
The cause is uncertain, but cytokines (for example, IL-1�, IL-6,
TNF-a, and IL-18) are significantly elevated, reflecting mac­
rophage hyperactivation.
Atypical manifestations that may influence prognosis
include myocarditis, disseminated intravascular coagulation,
fulminant hepatitis, and the hemophagocytic syndrome, a life­
threatening process characterized by fever, splenomegaly,
cytopenias, elevated ferritin, and evidence of hemophagocyto­
sis on bone marrow biopsy .
AOSD diagnosis is clinically based and requires exclu­
sion of infection, malignancy, or other rheumatologic dis­
eases. Erythrocyte sedimentation rate and C-reactive
protein can be markedly elevated. Serum ferritin is elevated
in 78% of patients, often to extremely high levels, and

TABLE 40. Yamaguchi Criteria for the Diagnosis of


Adult-Onset Still Disease
Major Criteria• Approximate Frequency

Daily spiking fever to 39.0 °C 99%


FIGURE 4 0. Floppy ears in a patient with relapsing polychondritis. Chronic (102.2 °F)
inflammation of the upper section of the ear eventually results in loss of carti­
Arthralgia/arthritis >2 weeks 85%
lage and structural collapse. The ear lobe, which does not contain cartilage, is
unaffected. Nonpruritic salmon-colored 85%
macular/maculopapular rash
on trunk or extremities

Patients who develop RP in their 60s or 70s should be evalu­ Leukocyte count> 10,000/µL 90%
(10 X 109/L),>80%
ated for myelodysplastic syndrome. neutrophils
RP is diagnosed by its typical clinical manifestations.
Minor Criteria Approximate Frequency
Laboratory tests are nonspecific; acute phase reactants are ele­
vated in 80%, and mild anemia is present in 44%. Anti-type II Sore throat 66%
collagen antibodies are present in 33% but are not considered Lymphadenopathy and/or 65%/50%
diagnostic. Patients with RP should undergo imaging (CT or splenomegaly
MRI) to evaluate the large airways for inflammation/stenosis. Elevated AST, ALT, or LDH 70%
Cartilage biopsy is usually unnecessary. Negative ANA and RF N/A
Auricular chondritis typically responds to low-dose
ALT= alanine aminotransferase; ANA= antinuclear antibodies; AST= aspartate
prednisone. For more severe manifestations (for example, aminotransferase; LOH= lactate dehydrogenase; RF= rheumatoid factor.
laryngotracheal chondritis or aortitis), high-dose prednisone "Diagnosis requires five criteria with at least two major criteria.
is initiated . For persistent disease and/or glucocorticoid spar­ Adapted with permission from Yamaguchi M, Ohta A, Tsunematsu T, et al.
ing, dapsone, methotrexate, azathioprine, cyclosporine, and Preliminary criteria for classification of adult Still's diesease. J Rheumatol. 1992
Mar; 19(3):424-30. (PMID: 15 78458]
cyclophosphamide have all been used. Biologic agents such

84
Oth e r Rheuma tologic Diseases

implies macrophage activation. Lymph node biopsy shows a of inflammatory cytokines. Patients experience regular
reactive process on histology. episodes of systemic inflammation in the absence of
Therapy for AOSD includes high-dose NSA!Ds and/or infection or autoimmunity. Autoinflammatory diseases
prednisone, with methotrexate as a standard second-line may be inherited or acquired through mutation. It is antic­
agent. Recently, treatment approaches employing anti-lL-1� ipated that more syndromes will be described in the
(for example, anakinra) or anti-IL-6 (for example, tocilizumab) coming years.
therapies have shown significant promise. The classic autoinflammatory disease is familial
KEY POINTS
Mediterranean fever (FMF), associated with mutation of the
MEFV1 gene that codes for pyrin, a regulator of lL-1� pro­
• Characteristics of adult-onset Still disease include daily duction. More than 40 different mutations in MEFV1 have
spiking fever, arthritis, a salmon-colored rash, and been identified that result in FMF. Attacks last 1 to 3 days
extremely elevated serum ferritin. and are characterized by polyserositis, arthritis, erysipeloid
• Diagnosis of adult-onset Still disease is clinically based rash around the ankles, and elevation of acute phase reac­
and requires exclusion of infection, malignancy, or tants. AA amyloidosis is a potential long-term consequence
other rheumatologic diseases. of FMF and other autoinflammatory diseases due to the
production and deposition of serum amyloid A. FMF man­
agement includes chronic daily treatment with colchicine.
Autoinflammatory Diseases Increasingly, management of autoinflammatory diseases is
The autoinflammatory diseases (also known as periodic being directed at specific cytokines that are implicated
fever syndromes) are rare monogenic diseases character­ as pathogenic.
ized by autoactivation of the innate immune system, Table 41 lists the features of currently recognized autoin­
including myeloid leukocyte activation and overproduction flammatory diseases.

TABLE 41. Autoinflammatory Diseases


FMF TRAPS HIDS FCAS MWS NOMID

Inheritance Autosomal Autosomal Autosomal Autosomal Autosomal Autosomal


recessive dominant recessive dominant dominant dominant
Age at Onset 65% <10 years 50% <10 years Usually <5 years of <1 year of age Childhood Neonatal
of age; 90% <20 of age; up to 5th age period
years of age decade

Ethnicity Mediterranean All Northern European European All


European

C linical
Manif estations
Attack 12-72 h Days to weeks 3-7 days 12-24 h 1-2 days Progressive
Duration
Abdominal Pain; serositis Pain; serositis Pain; vomiting Nausea Pain

Pleuritis Common Common Rare Rare Rare

Arthritis Monoarthritis of Large joints Symmetric Arthralgia Arthralgia; Epiphyseal


lower polyarthritis of oligoarthritis overgrowth;
extremities large joints contractures

Rash Erysipeloid on Migratory with Diffuse Cold-induced; Urticaria-like Urticaria-like


lower legs underlying maculopapular urticaria-like
myalgia
Other High risk for Conjunctivitis; Cutaneous Conjunctivitis; Sensorineural Sensorineural
Manifestations amyloidosis periorbital vasculitis; headache; deafness; deafness;
edema lymphadenopathy; amyloidosis conjunctivitis; aseptic
elevated lgD amyloidosis meningitis;
mental
retardation

Treatment Colchicine Glucocorticoids; NSAIDs; IL-1� inhibition IL-1� IL-1� inhibition


TNF-a inhibitors glucocorticoids inhibition

FCAS= familial cold auto inflammatory syndrome; FMF = familial Mediterranean fever; HIDS= hyperimmunoglobulinemia D with periodic fever syndrome; IL= interleukin; MWS=
Muckle-Wells syndrome; NOMID = neonatal-onset multisystem inflammatory disease; TNF = tumor necrosis factor; TRAPS= tumor necrosis factor receptor-associated periodic
syndrome.

85
Other Rheumatologic Diseases

KEY POINT involves glucocorticoids or other immunosuppressants such


as methotrexate, azathioprine, or mycophenolate mofetil as
• Autoinflammatory diseases are characterized by epi­
dictated by the disease burden. TNF-a inhibitors may be use­
sodes of systemic inflammation in the absence of infec­
ful in more severe disease or where glucocorticoid use is
tion or autoimmunity; these rare diseases may be
problematic.
inherited or acquired through mutation.
KEY POINTS
• Sarcoidosis can manifest as Lofgren syndrome, which is HVC
Sarcoidosis characterized by acute arthritis, bilateral hilar lymphad­
Sarcoidosis is a multisystem disease characterized by noncase­ enopathy, and erythema nodosum; when all three occur
ating granulomas that form in tissues. Sarcoidosis most com­ together, there is a 95% specificity for the diagnosis, and
monly affects the lungs and is therefore discussed in detail in further diagnostic tests are unnecessary.
MKSAP 17 Pulmonary and Critical Care Medicine. Although • A chronic form of polyarticular arthritis can be found in
less common, the rheumatologic manifestations of sarcoidosis severe sarcoidosis, which may affect the shoulders,
are important and discussed here. hands, wrists, knees, and ankles; an entire digit may
Sarcoidosis can manifest as Lofgren syndrome, which is also be involved, leading to dactylitis.
seen in younger adults and is characterized by acute arthri­
tis, bilateral hilar lymphadenopathy, and erythema nodo­
sum. When all three occur together, there is a 95% specificity True Connective Tissue Diseases
for the diagnosis, and further diagnostic tests are unneces­ Ehlers-Danlos Syndrome
sary. The "arthritis" of Lofgren syndrome is actually a non­ Ehlers-Danlos syndrome (EDS) defines a group of inherited
destructive periarthritis of the soft tissue, entheses, and genetic disorders of collagen-related (COL) genes or the pro­
tenosynovium. Symmetric ankle involvement is classic, but collagen lysyl hydroxylase (PLOD) gene, leading to defect(s) in
knees, wrists, and elbows can also be involved. Lofgren is the production, structure, or function of collagen. Most EDS
more common among Europeans and is self-limited, with variants are inherited as autosomal domi.nant mutations. EDS
90% of patients remitting within 12 months. Treatment is subcategories include the benign hypermobility type, the
symptomatic and includes NSA!Ds, colchicine, or low-dose more severe kyphoscoliotic type, and the life-threatening vas­
glucocorticoids. cular type in which large- to medium-sized vessels and organs
A more chronic form of true arthritis affects 1 % to 4% such as the uterus or bowel can rupture.
of patients and occurs in more severe sarcoidosis. The EDS clinical characteristics include joint hypermobil­
arthritis is usually polyarticular. involving the shoulders, ity, hyperextensible skin, atrophic scars, and velvety skin.
hands, wrists, knees, and ankles. An entire digit may be Hypermobile joints may lead to joint dislocations or early
affected, leading to dactylitis. Black persons and those osteoarthritis. Mitra!, aortic, or tricuspid valve regurgita­
with sarcoid skin involvement are more likely to have tion may be present. Diagnosis is based on family history
chronic arthritis. and clinical examination. Genetic consultation may be use­
Granulomatous bone involvement occurs in 3% to 13% of ful if the diagnosis is uncertain. Treatment is supportive.
patients, often those with more severe disease. The hands, feet, Patients with hypermobility should avoid activities that
skull, ribs, sternum, nasal bones, pelvis, tibia, and femur may could lead to joint damage. Periodic echocardiography
be affected. The radiographic pattern is lytic (focal areas of cyst should assess cardiac status/aortic root size; invasive vas­
formation), permeative (bone has a lacey appearance), or cular procedures should be avoided for those with the
destructive. vascular type of EDS.
Myopathy is a rare manifestation of systemic sarcoidosis. EDS disorders require extensive counseling pertaining to
Acute myopathy can mimic polymyositis with proximal mus­ issues of pregnancy both in terms of maternal complications
cle weakness and muscle enzyme elevation. Chronic sarcoid of pregnancy and delivery as well as potential inherited dis­
myopathy affects older women. involves the proximal muscles, ease in the child. Patients with vascular EDS should be evalu­
may have normal muscle enzymes. and occurs in the context ated promptly for any unexplained pain and should wear a
of known disease. A nodular localized myopathy of sarcoidosis medical alert identification bracelet.
has also been described.
Granulomatous infiltration of the parotid glands may Marfan Syndrome
mimic Sjogren syndrome and produce sicca symptoms. The Marfan syndrome (MFS) is an autosomal dominant condi­
triad of sarcoid parotitis, uveitis. and fever is called uveopa­ tion characterized by a mutation in the gene for fibrillin 1
rotid fever or Heerfordt syndrome. (FBNl). Fibrillin 1 is an important structural protein in tis­
Sarcoid musculoskeletal involvement can be diagnosed sues that contain elastic fibers. such as arteries. ligaments,
via biopsy. and the extent and response to therapy can be and the structure that stabilizes the lens of the eye.
gauged via gallium or PET scanning if needed. Treatment Manifestations may relate to an overexpression of the

86
Other Rheumat ologic Diseases

cytokine transforming growth factor (TGF)-�. Clinical fea­ KEY POINTS


tures include the following:
• Clinical characteristics of Ehlers-Dantos syndrome
• Tall stature with short upper body to lower body ratio include joint hypermobility, hyperextensible skin,
• Arachnodactyly (long fingers; thumb and index finger atrophic scars, and velvety skin; diagnosis is based on
overlap around the wrist) family history and clinical examination.
• Anterior thoracic deformity: pectus excavatum or pectus • Marfan syndrome is associated with tall stature, arach­
carinatum nodactyly, anterior thoracic deformity, spinal curvature,
and skin and ocular involvement; aortopathy and mitral
• Spinal curvature: scoliosis or kyphosis
valve prolapse are also common.
• Hyperextensibility of skin
• Osteogenesis imperfecta is associated with bone fractures,
• Pes planus with a long narrow foot short stature, body deformity, hearing loss, and dental
• Myopia with dislocation of the lens (usually superiorly) deformity; blue sclerae is characteristic but nonspecific.
• Aortopathy of the ascending aorta: aortic regurgitation/dis-
section/aneurysm lgG4-Related Disease
• Mitra! valve prolapse lgG4-related disease is a recently recognized syndrome char­
• Pneumothorax from apical lung bullae acterized by abundant lgG4-producing plasma cells seen on
• Other: high arched palate, hernias, atrophic scarring over tissue biopsy; it unites an unlikely group of diseases under one
pectoral, deltoid, or lumbar areas banner (Table 42). Most of these conditions are characterized
by infiltration of the affected tissue and the clinical conse­
MFS diagnosis is made clinically, with genetic consulta­ quences that infiltration entails. Almost any organ can be
tion if necessary. Testing for FBNl mutations is available. The involved; lymph nodes are frequently affected. Most patients
2012 revised Ghent criteria emphasize aortic root dilatation are men (60%-80%) older than age 50 years.
and lens dislocation for diagnosis but also include systemic Diagnosis is made by tissue biopsy and the demonstration
features and FBNl mutations. Current guidelines recommend of a characteristic histology, which includes the following:
that echocardiography should be performed at the time of
diagnosis to determine aortic root and ascending aortic diam­ • Dense lymphoplasmacytic infiltrate
eters and 6 months later to determine their rate of enlarge­ • CD4-positive T cells and plasma cells in germinal centers
ment. Annual imaging is recommended if stability of the aortic • lgG4-staining plasma cells constituting more than 50% of
diameter is documented. If the maximal aortic diameter is 4.5 the total plasma cells
cm or greater, more frequent imaging should be considered.
• Storiform (spokes on a wheel-appearing) fibrosis
For most patients with MFS, surgical repair of the dilated
aortic root/ascending aorta is usually performed at a threshold • Obliterative phlebitis
of an external diameter of 5.0 cm. For women contemplating • Rare neutrophils and no granulomas
pregnancy, it is reasonable to prophylactically replace the aortic
root and ascending aorta if the diameter is greater than 4.0 cm. TABLE 42. Conditions Associated with lgG4-Related
�-Blockers are recommended to slow the aortopathy. Counseling Disease
regarding joint protection to prevent osteoarthritis is useful. Condition Organ Involved

Hypertrophic pachymeningitis Dura mater


Osteogenesis lmperfecta
Lymphocytic hypophysitis Pituitary gland
Osteogenesis imperfecta (01) comprises four genetic syn­
Idiopathic orbital inflammatory Periocular mass
dromes characterized by autosomal dominant or recessive disease
mutations in COL genes, leading to abnormalities in the struc­
Mikulicz disease Parotid glands
ture of type I collagen. 01 is associated with bone fractures,
Dacryoaden itis Lacrimal glands
short stature, body deformity, hearing loss, and dental deform­
ity. A characteristic feature of 01 is blue sclerae (reflecting vis­ Kuttner tumor Submandibular glands

ibility of the underlying choroid) but is not specific to 01. Riedel thyroiditis Thyroid
Patients with pseudoxanthoma elasticum, ochronosis/alkap­ Inflammatory aortitis/vascu I itis Aorta/arteries
tonuria, Ehlers-Dantos syndrome, or Marfan syndrome may Autoimmune pancreatitis Pancreas
also have blue sclerae.
Ormond disease (retroperitoneal Periaortic mass
01 diagnosis is based on clinical findings and family his­ fibrosis)
tory. Bisphosphonates may reduce fractures and improve skel­ Tubulointerstitial nephropathy Kidneys
etal growth, and orthopedic surgeons are often involved due to
Sclerosing cholangitis Biliary tract
the fractures experienced by these patients.

87
Bibliography

Because serum IgG4 levels are elevated in only 70% of Dixon WG, Watson KD, Lunt M, et al. Influence of anti-tumor necrosis factor
therapy on cancer incidence in patients with rheumatoid arthritis who
patients, a normal level does not rule out the disease. PET scan have had a prior malignancy: results from the British Society for
may identify involvement that is not clinically apparent and Rheumatology Biologics Register. Arthritis Care Res (Hoboken). 2010
Jun;62(6):755-63. [PMID: 20535785]
can document response to therapy. Initial treatment is pred­
Fleischmann R, Kremer J, Cush J. et al. Placebo-controlled trial of tofacitinib
nisone, 0.5-0.6 mg/kg/d for 2 to 4 weeks, followed by a slow monotherapy in rheumatoid arthritis. N Engl J Med. 2012 Aug 9;367(6):495-
taper over months. Azathioprine, mycophenolate mofetil, and 507. [PMID: 22873530]
methotrexate have been used as glucocorticoid-sparing agents. Furie R, Petri M, Zamani 0, et al. A phase Ill, randomized, placebo-controlled
study of belimumab, a monoclonal antibody that inhibits B lymphocyte
Recently, rituximab has shown significant benefit in patients stimulator. in patients with systemic lupus erythematosus. Arthritis
with IgG4-related disease and appears to selectively affect Rheum. 2011 Dec;63(12):3918-30. [PMID: 22127708]
Genovese MC. McKay JD. Nasonov EL, et al. lnterleukin-6 receptor inhibition
IgG4 production; it may replace prednisone as the treatment with tocilizumab reduces disease activity in rheumatoid arthritis with
of choice in severe or refractory disease. Treatment response inadequate response to disease-modifying antirheumatic drugs: the tocili­
zumab in combination with traditional disease-modifying antirheumatic
may be limited by the amount of fibrosis present prior to drug therapy study. Arthritis Rheum. 2008 Oct:58(10):2968-80. [PMID:
therapy initiation. 18821691]
See MKSAP 17 Gastroenterology and Hepatology for infor­ Hennekens CH. Dalen JE. Aspirin in the treatment and prevention of cardio­
vascular disease: past and current perspectives and future directions. Am J
mation on IgG4-related pancreatitis. Med. 2013 May:126(5):373-8. [PMID: 23499330]
KEY POINTS Jones RB, Tervaert JW. Hauser T, et al. Rituximab versus cyclophosphamide in
ANCA-associated renal vasculitis. N Engl J Med. 2010 Jul 15:363(3):211-20.
• The conditions comprising lgG4-related disease are [PMID: 20647198]
characterized by abundant IgG4-producing plasma cells Kamanamool N. McEvoy M, Attia J. lngsathit A, Ngamjanyaporn P, Thakkinstian
A. Efficacy and adverse events of mycophenolate mofetil versus cyclophos­
seen on tissue biopsy, enlargement of the affected tis­ phamide for induction therapy of lupus nephritis: systemic review and
sue, and the clinical consequences that enlargement meta-analysis. Medicine. 2010 Jul:89(4):227-35. [PMID: 20616662]
Khanna D, Fitzgerald JD, Khanna PP, et al; American College of Rheumatology.
entails. 2012 American College of Rheumatology guidelines for management of
• Serum IgG4 levels are elevated in only 70% of patients gout. Part I: systematic nonpharmacologic and pharmacologic therapeutic
approaches to hyperuricemia. Arthritis Care Res (Hoboken). 2012
with IgG4-related disease; therefore, a normal level Oct;64(10):143!-46. [PMID: 23024028]
does not rule out the disease. Mariette X. Matucci-Cerinic M. Pavelka K, et al. Malignancies associated with
tumour necrosis factor inhibitors in registries and prospective observa­
tional studies: a systematic review and meta-analysis. Ann Rheum Dis. 2011
Bibliography Nov:70(11): 1895. [PMID: 21885875]
Singh JA, Furst DE, Bharat A. et al. 2012 Update of the 2008 American College
Approach to the Patient with Rheumatologic Disease of Rheumatology recommendations for the use of disease-modifying anti­
Anderson J, Caplan L, Yazdany J, et al. Rheumatoid arthritis disease activity rheumatic drugs and biologic agents in the treatment of rheumatoid arthri­
measures: American College of Rheumatology recommendations for use in tis. Arthritis Care Res (Hoboken). 2012 May:64(5):625-39. [PMID: 22473917]
clinical practice. Arthritis Care Res (Hoboken). 2012 May;64(5):640-7. Van Assen S. Agmon-Levin N. Elkayam 0, et al. EULAR recommendations for
[PMID: 22473918] vaccination in adult patients with autoimmune inflammatory rheumatic
Brown A. How to interpret plain radiographs in clinical practice. Best Pract Res diseases. Ann Rheum Dis. 2011 Mar:70(3):414-22. [PMID: 21131643]
Clin Rheumatol. 2013 Apr;27(2):249-69. [PMID: 23731934]
Rheumatoid Arthritis
Castro C, Gourley M. Diagnostic testing and interpretation of tests for autoim­
munity. J Allergy Clin lmmunol. 2010 Feb:125(2 Suppl 2):S238-47. [PMID: Huzinga TW, Pincus T. In the clinic. Rheumatoid arthritis. Ann Intern Med.
20061009] 2010 Jul 6;153(1):ITCl-1-ITCl-15: quiz ITCl-16. [PMID: 20621898]
Colglazier C, Sutej P Laboratory testing in the rheumatic diseases: a practical Jacobs JW. Optimal use of non-biologic therapy in the treatment of rheuma­
review. South Med J. 2005 Feb:98(2):185-91. [PMID: 15759949] toid arthritis. Rheumatology (Oxford). 2012 Jun;Sl(Suppl 4):iv3-8. [PMID:
22513146]
Joshua F. Ultrasound applications for the practicing rheumatologist. Best Pract
Res Clin Rheumatol. 2012 Dec:26(6):853-67. [PMID: 23273796] Pavelka K, Kavanaugh AF. Rubbert-Roth A, Ferraccioli G. Optimizing out­
comes in rheumatoid arthritis patients with inadequate responses to dis­
Qaseem A. Alguire P Dallas P et al. Appropriate use of screening and diagnos­ ease-modifying anti-rheumatic drugs. Rheurnatology (Oxford). 2012
tic tests to foster high-value. cost-conscious care. Ann Intern Med. 2012 Jan Jul:5I(Suppl 5):vl2-21. [PMID: 22718922]
17:156(2):147-9. [PMID: 22250146]
Pisetsky DS, Ward MM. Advances in the treatment of inflammatory arthritis.
Yazdany J. Schmajuk G. Robbins M. et al: American College of Rheumatology Best Pract Res Clin Rheumatol. 2012 Apr;26(2):251-61. [PMID: 22794097]
Core Membership Group. Choosing Wisely: The American College of
Rheumatology's top 5 list of things physicians and patients should question. Radner H. Neogi T, Smolen JS, Aletaha D. Performance of the 2010 ACRI
Arthritis Care Res (Hoboken). 2013 Mar:65(3):329-39. [PMID: 23436818] EULAR classification criteria for rheumatoid arthritis: a systematic litera­
ture review. Ann Rheum Dis. 2013 Jan:73(1):114-23. [PMID: 23592710]
Principles of Therapeutics Sen D, Brasington R. Tight disease control in early RA. Rheum Dis Clin North
Barnes PM, Bloom B. Complementary and alternative medicine use among Am. 2012;38(2):327-43. [PMID: 22819087]
adults and children: United States. 2007. Natl Health Stat Report. 2008 Dec Singh JA. Furst DE, Bharat A. et al. 2012 Update of the 2008 American College
10:(12):1-23. [PMID: 19361005] of Rheumatology recommendations for the use of disease-modifying anti­
Cameron M, Gagnier JJ, Chrubasik S. Herbal therapy for treating rheumatoid rheumatic drugs and biologic agents in the treatment of rheumatoid arthri­
arthritis. Cochrane Database of Syst Rev. 2011 Feb 16:(2):CD002948. [PMID: tis. Arthritis Care Res (Hoboken). 2012 May:64(5):625-39. [PMID: 22473917]
21328257] Vermeer M, Kuper HH, Bernelot Moens HJ, et al. Adherence to a treat-to-target
Chan ES. Cronstein BN. Methotrexate-how does it really work? Nat Rev strategy in early rheumatoid arthritis: results of the DREAM remission induc­
Rheumatol. 2010 Mar:6(3):175-8. [PMID: 20197777] tion cohort. Arthritis Res Ther. 2012 Nov 23:14(6):R254. [PMID: 23176083]
Coxib and traditional NSAID Trialists· (CNT) Collaboration, Bhala N, Emberson Vermeer M. Kuper HH. Moens HJ, et al. Sustained beneficial effects of a proto­
J. et al. Vascular and upper gastrointestinal effects of non-steroidal anti­ colized treat-to-target strategy in very early rheumatoid arthritis: three year
inflammatory drugs: meta-analyses of individual participant data from results of the DREAM remission induction cohort. Arthritis Care Res
randomized trials. Lancet. 2013 Aug 31:382(9894):769-79. [PMID: 23726390] (Hoboken). 2013 Aug;65(8):1219-26. [PMID: 23436821]
Crittenden DB. Pillinger MH. New therapies for gout. Annu Rev Med. Yarilina A, Xu K. Chan C. lvashkiv LB. Regulation of inflammatory responses
2013:64:325-337. [PMID: 23327525] in tumor necrosis factor-activated and rheumatoid arthritis synovial

88
Bibliography

macrophages by JAK inhibitors. Arthritis Rheum. 2012 Dec:64(12):3856-66. Dooley MA, Jayne D, Ginzler EM. et al. Mycophenolate versus azathioprine as
[PMID: 22941906] maintenance therapy for lupus nephritis. N Engl J Med. 2011 Nov;
365(20):1886-95. [PMID: 22087680]
Osteoarthritis
Elkan KB, Wiedeman A. Type I IFN system in the development and manifesta­
Fernandes L, Hagen KB, Bijlsma J WJ, et al. EULAR Recommendations for the tions of SLE. Curr Opin Rheumatol. 2012 Sept;24(5):499-505. [PMID:
non-pharmacological core management of hip and knee osteoarthritis. 22832823]
Ann Rheum Dis. 2013 Jul;72(7):ll25-35. [PMID: 23595142]
Giannakopoulos B, Krilis SA. The pathogenesis of the antiphospholipid syn­
Hochberg MC, Altman RD. April KT, et al. American College of Rheumatology drome. N Engl J Med. 2013 Mar;368(11):1033-44. [PMID: 23484830]
2012 recommendations for the use of nonpharmacologic and pharmaco­
logic therapies in osteoarthritis of the hand. hip, and knee. Arthritis Care Hahn BH, McMahon MA, Wilkinson A, et al. American College of Rheumatology
Res. 2012 Apr:64(4):465-74. [PMID: 22563589] guidelines for screening. treatment, and management of lupus nephritis.
Arthritis Care Res. 2012 Jun:64(6):797-808. [PMID: 22556106]
Kwok WY, Kloppenburg M, Rosendaal FR, van Meurs JB. Hofman A, Bierma­
Zeinstra SM. Erosive hand osteoarthritis: its prevalence and clinical impact Hochberg MC. Updating the American College of Rheumatology revised crite­
in the general population and symptomatic hand osteoarthritis. Ann ria for the classification of systemic lupus erythematosus. Arthritis Rheum.
Rheum Dis. 2011 Jul:70(7):1238-42. [PMID: 21474485] 1997 Sep;40(9):1725. [PMID: 9324032]
Rutjes AW. Jiini P. da Costa BR. Trelle S, Niiesch E, Reichenbach S. lzmirly PM, Costedoat-Chalumeau N, Pisani CN, et al. Maternal use of hydrox­
Viscosupplementation for osteoarthritis of the knee: a systematic review and ychloroquine is associated with a reduced risk of recurrent anti-SSA/
meta-analysis. Ann Intern Med. 2012 Aug 7:157(3):180-91. [PMID: 22868835] Ro-antibody-associated cardiac manifestations of neonatal lupus.
Circulation. 2012 Jul 3;126(1):76-82. [PMID: 22626746]
Westerveld LA, Van Ufford HM, Verlaan JJ, Oner FC. The prevalence of diffuse
idiopathic skeletal hyperostosis in an outpatient population in the Lateef A. Petri M. Management of pregnancy in systemic lupus erythematosus.
Netherlands. J Rheumatol. 2008 Aug;35(8):1635-8. [PMID: 18528963] Nat Rev Rheumatol. 2012 Dec:8(12):710-8. [PMID: 22907290]
Lee SJ. Silverman E. Bargman JM. The role of antimalarial agents in the treat­
Fibromyalgia
ment of SLE and lupus nephritis. Nat Rev Nephrol. 2011 Oct 18;7(12):718-29.
Atzeni F, Cazzola M. Benucci M, Di Franco M, Salam F. Sarzi-Puttini P. Chronic [PMID: 22009248]
widespread pain in the spectrum of rheumatological diseases. Best Pract
Tsokos GC. Systemic lupus erythematosus. N Engl J Med. 2011 Dec
Res Clin Rheumatol. 2011 Apr;25(2):165-71. [PMID: 22094193]
l:365(22):2110-21. [PMID: 22129255]
Choy E. Marshall D. Gabriel ZL, Mitchell SA. Gylee E. Dakin HA. Systematic
Wu H. Birmingham DJ, Rovin B, et al. D-dimer level and the risk for thrombo­
review and mixed treatment comparison of the efficacy of pharmacological
sis in systemic lupus erythematosus. Clin J Am Soc Nephrol. 2008
treatments for fibromyalgia. Semin Arthritis Rheum. 2011 Dec:41(3):335-
Nov:3(6):1628-36. [PMID: 18945994]
45. [PMID: 21868065]
McBeth J, Mulvey MR. Fibromyalgia: mechanisms and potential impact of the Sjogren Syndrome
ACR 2010 classification criteria. Nat Rev Rheumatol. 2012 Jan 24;8(2):108- Delaleu N. Jonsson MV. Appel S. Jonsson R. New concepts in the pathogenesis
16. [PMID: 22270077] of Sjiigren's syndrome. Rheum Dis Clin North Am. 2008 Nov:34(4):833-45.
Spondyloarthritis [PMID: 18984407]
Arvikar SL, Fisher MC. Inflammatory bowel disease associated arthropathy. Helmick CG. Felson DT. Lawrence RC, et al: National Arthritis Data Workgroup.
Curr Rev Musculoskelet Med. 2011 Sep:4(3):123-31. [PMID: 21710141] Estimates of the prevalence of arthritis and other rheumatic conditions in the
United States. Part I. Arthritis Rheum. 2008 Jan;58(1):15-25. [PMID: 18163481]
Carter JD, Hudson AP. Reactive arthritis: clinical aspects and medical manage­
ment. Rheum Dis Clin North Am. 2009 Feb;35(1):21-44. [PMID: 19480995] Shiboski SC. Shiboski CH, Criswell L, et al; Sjogren's International Collaborative
Clinical Alliance (S!CCA) Research Groups. American College ofRheumatology
de Vries MK, van Eijk IC, van der Horst-Bruinsma IE. et al. Erythrocyte sedi­ classification criteria for Sjbgren's syndrome: a data-driven, expert consensus
mentation rate, C-reactive protein level, and serum amyloid a protein for approach in the Sjbgren's International Collaborative Clinical Alliance cohort.
patient selection and monitoring of anti-tumor necrosis factor treatment in Arthritis Care Res (Hoboken) 2012 Apr:64(4):475-87. [PMID: 22563590]
ankylosing spondylitis. Arthritis Rheum. 2009 Nov 15:61(11):1484-90.
[PMID: 19877087] Solans-Laque R, Lopez-Hernandez A, Bosch-Gil JA, Palacios A. Campillo M,
Vilardell-Tarres M. Risk, predictors. and clinical characteristics of lym­
Hannu T. Reactive arthritis. Best Pract Res Clin Rheumatol. 2011 Jun:25(3):347- phoma development in primary SjOgren's syndrome. Semin Arthritis
57. [PMID: 22100285] Rheum. 2011 Dec:41(3):415-23. [PMID: 21665245]
McGonagle D. Ash Z. Dickie L, McDermott M, Aydin SZ. The early phase of Theander E. Henriksson G, Ljungberg 0. Mandi T, Manthorpe R, Jacobsson LT.
psoriatic arthritis. Ann Rheum Dis. 2011 Mar:70(Suppl l):i71-6. [PMID: Lymphoma and other malignancies in primary SjOgren's syndrome: a
21339224] cohort study on cancer incidence and lymphoma predictors. Ann Rheum
Morris D, Inman RD. Reactive arthritis: developments and challenges in diagnosis Dis. 2006 Jun:65(6):796-803. [PMID: 16284097]
and treatment. Curr Rheumatol Rep. 2012 Oct:14(5):390-4. [PMID: 22821199] Theander E, Manthorpe R. Jacobsson LT. Mortality and causes of death in pri­
Ostergaard M, Lambert RG. Imaging in ankylosing spondylitis. Ther Adv mary Sjogren's syndrome: a prospective cohort study. Arthritis Rheum.
Musculoskelet Dis. 2012 Aug;4(4):301-ll. [PMID: 22859929] 2004 Apr:50(4):1262-9. [PMID: 15077310]
Poddubnyy D, van der Heijde D. Therapeutic controversies in spondyloarthri­ Mixed Connective Tissue Disease
tis: nonsteroidal anti-inflammatory drugs. Rheum Dis Clin North Am. 2012
Aug;38(3):601-ll. [PMID: 23083758] Hajas A, Szodoray P, Nakken B. et al. Clinical course, prognosis, and causes of
death in mixed connective tissue disease. J Rheumatol. 2013 Jul:40(7):1134-
Robinson PC. Brown MA. The genetics of ankylosing spondylitis and axial 42. [PMID: 23637328]
spondyloarthritis. Rheum Dis Clin North Am. 2012 Aug:38(3):539-53.
[PMID: 23083754] Swanton J, Isenberg D. Mixed connective tissue disease: still crazy after all these
years. Rheum Dis Clin North Am. 2005 Aug:31:421-36. [PMID: 16084316]
Rohekar S. Pope J. Epidemiologic approaches to infection and immunity: the
case of reactive arthritis. Curr Opin Rheumatol. 2009 Jul:21(4):386-90. Crystal Arthropathies
[PMID: 19373091]
Hamburger M. Baraf HS. Adamson TC 3rd. et al: European League Against
Rudwaleit M, Taylor WJ. Classification criteria for psoriatic arthritis and anky­ Rheumatism. 2011 recommendations for the diagnosis and management of
losing spondylitis/axial spondyloarthritis. Best Pract Res Clin Rheumatol. gout and hyperuricemia. Postgrad Med. 2011 Nov;l23(6 suppl 1):3-36.
2010 Oct:24(5):589-604. [PMID: 21035082] [PMID: 22156509]
Rudwaleit M, van der Heijde D, Landewe R, et al. The Assessment of Khanna D. Fitzgerald JD. Khanna PP. et al: American College of Rheumatology.
SpondyloArthritis International Society classification criteria for peripheral 2012 American College of Rheumatology guidelines for management of
spondyloarthritis and for spondyloarthritis in general. Ann Rheum Dis. gout. Part 1: systematic nonpharmacologic and pharmacologic therapeutic
2011 Jan:70(1):25-31. [PMID: 21109520] approaches to hyperuricemia. Arthritis Care Res (Hoboken). 2012
Stolwijk C, Boonen A, van Tubergen A. Reveille JD. Epidemiology of spondyloar­ Oct;64(l0):1431-46. [PMID: 23024028]
thritis. Rheum Dis Clin North Am. 2012 Aug;38(3):441-76. [PMID: 23083748] Khanna D, Khanna PP, Fitzgerald JD, et al: American College of Rheumatology.
2012 American College of Rheumatology guidelines for management of
Systemic Lupus Erythematosus gout. Part 2: therapy and antiinOammatory prophylaxis of acute gouty
Barber C, Gold WL. Fortin PR. Infections in the lupus patient: perspectives on arthritis. Arthritis Care Res (Hoboken). 2012 Oct:64(10):1447-61. [PMID:
prevention. Curr Opin Rheumatol. 2011 Jul:23(4):358-65. [PMID: 21532484] 23024029]

89
Bibliography

Martinon F, Petrilli V. Mayor A. Tardivel A, Tschopp J. Gout-associated uric acid Saulsbury FT. Henoch-Schonlein purpura. Curr Opin Rheumatol. 2010
crystals activate the NALP3 inflammasome. Nature. 2006 Mar 9;440(7081): Sep:22(5):598-602. [PMID: 20473173]
237-41. [PMID: 16407889] Scuccimarri R. Kawasaki disease. Pediatr Clin North Am. 2012 Apr:59(2):425-
Pascual E. Sivera F. Andres M. Synovial fluid analysis for crystals. Curr Opin 45. [PMID: 22560578]
Rheumatol. 2011 Mar:23(2):161-9. [PMID: 21285711] Specks U, Merkel PA Seo P, et al; RAV E-JTN Research Group. Efficacy of
Terkeltaub RA. Furst DE. Digiacinto JL. Kook KA. Davis MW. Novel evidence­ remission-induction regimens for ANCA-associated vasculitis. N Engl J
based colchicine dose-reduction algorithm to predict and prevent colchi­ Med. 2013 Aug l:369(5):4l7-27. [PMID: 2390248l]
cine toxicity in the presence of cytochrome P450 3A4/P-glycoprotein Unizony S, Arias-Urdaneta L, Miloslavsky E, et al. Tocilizumab for the treat­
inhibitors. Arthritis Rheum. 2011 Aug;63(8):2226-37. [PMID: 21480191] ment of large-vessel vasculitis (giant cell arteritis. Takayasu arteritis) and
Zhang W, Doherty M, Bardin T, et al. European League Against Rheumatism polymyalgia rheumatica. Arthritis Care Res. 2012 Nov:64(ll):l720-9. [PMID:
recommendations for calcium pyrophosphate deposition. Part I: terminol­ 22674883]
ogy and diagnosis. Ann Rheum Dis. 2011 Apr;70(4):563-70. [PMID: 21216817]
Systemic Sclerosis
Zhang W, Doherty M, Pascual E, et al. EULAR recommendations for calcium
pyrophosphate deposition. Part II: management. Ann Rheum Dis. 2011 Chifflot H, Fautrel B, Sordet C, Chatelus E, Sibilia J. Incidence and prevalence
Apr:70(4):571-5. [PMID: 21257614] of systemic sclerosis: a systematic literature review. Semin Arthritis Rheum.
2008 Feb:37(4):223-35. [PMID: [7692364]
Zhu Y, Pandya BJ, Choi HK: Prevalence of gout and hypernricemia in the US
general population: the National Health and Nutrition Examination Survey Gelber AC. Manno RL. Shah AA. et al. Race and association with disease
2007-2008. Arthritis Rheum. 2011 Oct;63(10):3136-41. [PMID: 21800283] manifestations and mortality in scleroderma: a 20-year experience at
the Johns Hopkins scleroderma center and review of the literature.
Infectious Arthritis Medicine (Baltimore). 2013 Jul;92(4):191-205. [PMID: 23793[08]
Kohli R. Hadley S. Fungal arthritis and osteomyelitis. Infect Dis Clin North Am. Hudson M, Baron M, Lo E, Weinfeld J, Furst DE, Khanna D. An International.
2005 Dec:19(4):831-51. [PMID: 16297735] Web-Based, Prospective Cohort Study to Determine Whether the Use of
Mathews CJ. Weston VC. Jones A. Field M. Coakley G. Bacterial septic arthritis ACE Inhibitors prior to the Onset of Scleroderma Renal Crisis is Associated
in adults. Lancet. 2010 Mar 6:375(9717):846-55. [PMID: 20206778] With Worse Outcomes-Methodology and Preliminary Results. Int J
Rheumatol. 2010 doi: 10.1155/2010/347402. [PMID: 20936135]
Osmon DR. Berbari EF. Berendt AR. et al; Infectious Diseases Society of
America. Diagnosis and management of prosthetic joint infection: clinical Johnson SR, Fransen J, Khanna D, et al. Validation of potential classification
practice guidelines by the Infectious Diseases Society of America. Clin criteria for systemic sclerosis. Arthritis Care Res (Hoboken). 2012
Infect Dis. 2013 Jan:56(1):el-e25. [PMID: 23223583] Mar;64(3):358-67. [PMID: 22052658]
Peto HM, Pratt RH, Harrington TA. LoBue PA. Armstrong LR. Epidemiology of Kumar U. Gokhle SS. Sreenivas V, Kaur S. Misra D. Prospective. open-label.
extrapulmonary tuberculosis in the United States. 1993-2006. Clin Infect uncontrolled pilot study to study safety and efficacy of sildenafil in systemic
Dis. 2009 Nov 1;49(9):1350-7. [PMID: 19793000] sclerosis-related pulmonary artery hypertension and cutaneous vascular
complications. Rheumatol Int. 2013 Apr;33(4):l047-52. [PMID: 22833239]
Rice PA. Gonococcal arthritis (disseminated gonococcal infection). Infect Dis
Clin North Am. 2005 Dec:19(4):853-61. [PMID: 16297736] Shanmugam VK. Steen V D. Renal disease in scleroderma: an update on evalu­
ation, risk stratification. pathogenesis and management. Curr Opin
Wormser GP, Dattwyler R.J, Shapiro ED, et al. The clinical assessment, treatment,
Rheumatol. 2012 Nov:24(6):669-76. [PMID: 22955019]
and prevention of lyme disease, human granulocytic anaplasmosis, and
babesiosis: clinical practice guidelines by the Infectious Diseases Society of Shenoy PD, Kumar S, Jha LK, et al. Efficacy of tadalafil in secondary Raynaud's
America. Clin Infect Dis. 2006 Nov 1:43(9):1089-34. [PMID: 17029130] phenomenon resistant to vasodilator therapy: a double-blind randomized
cross-over trial. Rheumatology (Oxford). 2010 Dec:49(12):2420-8. [PMID:
Idiopathic Inflammatory Myopathies 20837499]
Airio A, Kautiainen H, Hakala M. Prognosis and mortality of polymyositis and Spiera RF, Gordon JK. Mersten JN, et al. Imatinib mesylate (Gleevac) in the
dermatomyosilis patients. Clin Rheum. 2006 Mar:25(2):234-9. [PMID: treatment of diffuse cutaneous systemic sclerosis: results of a I-year. phase
16477398] Ila, single-arm. open-label clinical trial. Annals of rheumatic disease. 2011
Bernatsky S, Joseph L. Pineau CA, et al. Estimating the prevalence of polymy­ Jun;70(6):1003-9. [PMID: 21398330]
ositis and dermatomyositis from administrative data: age, sex and regional Tashkin DP, Elashoff R, Clements PJ, et al; Scleroderma Lung Study Research
differences. Ann Rheum Dis. 2009 Jul:68(7):1192. [PMID: 18713785] Group. Cyclophosphamide versus placebo in scleroderma lung disease. N
Buchbinder R. Forbes A, Hall S, Dennett X, Giles G. Incidence of malignant Engl J Med. 2006 Jun 22:354:2655-66. [PMID: 16790698]
disease in biopsy-proven inflammatory myopathy. A population based Walker KM, Pope J; participating members of the Scleroderma Clinical Trials
cohort study. Ann Intern Med. 2001 Jun 19;134(12):1087. [PMID: 11412048] Consortium (SCTC): Canadian Scleroderma Research Group (CSRG).
Callen JP. Cutaneous manifestations of dermatomyositis and their manage- Treatment of systemic sclerosis complications: what to use when first-line
ment. Curr Rheum Rep. 2010 Jun:12(3):192. [PMID: 20425525] treatment fails-a consensus of systemic sclerosis experts. Semin Arthritis
Rheum. 2012 Aug:42(l):42-55. [PMID: 22464314]
Dalakas MC. Hohlfeld R. Polymyositis and dermatomyositis. Lancet. 2003 Sep
20:362(9388):971-82. [PMID: 14511932] Other Rheumatologic Diseases
Marie I, Hachulla E. Hatron PY, et al. Polymyositis and dermatomyositis: short Chopra R, Chaudhary N. Kay J. Relapsing polychondritis. Rheum Dis Clin N
term and long term outcome, and predictive factors of prognosis. J Rheum. Am. 2013 May:39(2):263-76. [PMID: 23597963]
2001 Oct:28(10):2230-7. [PMID: 11669162]
Dalvi SR. Yildirim R, Yazici Y. Behcet's Syndrome. Drugs 2012 Dec 3;72(17):2223-
Oddis CV, Reed AM. Aggarwal R. et al; RIM Study Group. Rituximab in the 41. [PMID: 23153327]
treatment of refractory adult and juvenile dermatomyositis and adult poly­
myositis: a randomized, placebo-phase trial. Arthritis Rheum. 2013 Gattorno M. Martini A. Beyond the NLRP3 inflammasome: autoinOammatory
I'eb:65(2):314-24. [PMID: 23124935] diseases reach adolescence. Arthritis Rheum 2013 May:65(5):1137-47.
[PMID: 23400910]
Patwa HS, Chaudhry V. Katzberg H. Rae-Grant AD, So YT. Evidence-based
guideline: intravenous immunoglobulin in the treatment of neuromuscu­ Grahame R, Hakim AJ. Arachnodactyly-a key to diagnosing disorders of con­
lar disorders. Neurology. 2012 Mar 27;78(13),1009-15. [PMID, 22454268] nective tissue. J Nat Rev. Rheumatol 2013 Jun:9(6):358-64. [PMID:
23478494]
Targoff IN. Myositis specific antibodies. Curr Rheumatol Rep. 2006
Jun;8(3):196-203. [PMID: 16901077] McAdam LP, O'Hanlan MA, Bluestone R. Pearson CM. Relapsing polychondri­
tis: prospective study of 23 patients and a review of the literature. Medicine
Systemic Vasculitis (Baltimore). 1976 May;55(3):l93-215. [PMID: 775252]
Cacoub P. Terrier B. Saadoun D. Hepatitis C virus-induced vasculitis: thera­ O'Regan A, Berman JS. Sarcoidosis. Ann Intern Med. 2012 May I:156(9):ITC5-
peutic options. Ann Rheum Dis. 20t4 Jan:73(1):24-30.[PMID: 23921995] l-16. [PM!D: 22547486]
de Menthon M. Mahr A. Treating polyarteritis nodosa: current state of the art. Pouchot J. Ariel JB. Biologic treatment in adult-onset Still's disease. Best Pract
Clin Exp Rheuatol. 2011 Jan-Feb;29(1 Suppl 64):Sll0-6. [PMID: 21586205] Res Clin Rheumatol. 2012 Aug:26(4),477-87. [PMID: 23040362]
Mar-Lano AV. Vezzoli P. Berti E. Skin involvement in curaneous and systemic Stone JH, Zen Y, Deshpande V. lgG4-related disease. N Engl J Med. 2012 I'eb
vasculitis. Auloimmun Rev. 2013 Feb:(12)4:467-76. [PMID: 22959234] 9;366(6):539-51. [PMID: 22316447]

90
Rheumatology
Self-Assessment Test
This self-assessment test contains one-best-answer multiple-choice questions. Please read these directions carefully
before answering the questions. Answers, critiques, and bibliographies immediately follow these multiple-choice
questions. The American College of Physicians is accredited by the Accreditation Council for Continuing Medical
Education (ACCME) to provide continuing medical education for physicians.

The American College of Physicians designates MKSAP 17 Rheumatology for a maximum of 16 AMA PRA
Category 1 CreditsTM . Physicians should claim only the credit commensurate with the extent of their participation
in the activity.

Earn "Instantaneous" CME Credits Online


Print subscribers can enter their answers online to earn Continuing Medical Education (CME) credits instanta­
neously. You can submit your answers using online answer sheets that are provided at mksap.acponline.org, where
a record of your MKSAP 17 credits will be available. To earn CME credits, you need to answer all of the questions in
a test and earn a score of at least 50% correct (number of correct answers divided by the total number of questions).
Take any of the following approaches:
»- Use the printed answer sheet at the back of this book to record your answers. Go to mksap.acponline.org,
access the appropriate online answer sheet, transcribe your answers, and submit your test for instantaneous
CME credits. There is no additional fee for this service.
»- Go to mksap.acponline.org, access the appropriate online answer sheet, directly enter your answers, and
submit your test for instantaneous CME credits. There is no additional fee for this service.
»- Pay a $15 processing fee per answer sheet and submit the printed answer sheet at the back of this book by
mail or fax, as instructed on the answer sheet. Make sure you calculate your score and fax the answer sheet
to 215-351-2799 or mail the answer sheet to Member and Customer Service, American College of Physicians,
190 N. Independence Mall West, Philadelphia, PA 19106-1572, using the courtesy envelope provided in your
MKSAP 17 slipcase. You will need your 10-digit order number and 8-digit ACP ID number, which are printed
on your packing slip. Please allow 4 to 6 weeks for your score report to be emailed back to you. Be sure to
include your email address for a response.

If you do not have a 10-digit order number and 8-digit ACP ID number or if you need help creating a user name and
password to access the MKSAP 17 online answer sheets, go to mksap.acponline.org or email [email protected].

CME credit is available from the publication date of July 31, 2015, until July 31, 2018. You may submit your answer
sheets at any time during this period.

91
Directions
Each of the numbered items is followed by lettered answers. Select the ONE lettered answer that is BEST in each case.

Item 1 no other pertinent medical history and does not take any
A 59-year-old man is evaluated for a 6-month history of medications.
gout. He was doing well on colchicine and allopurinol but On physical examination, vital signs are normal. There
developed hypersensitivity to allopurinol, which resolved are tenderness and swelling of the second and fifth proximal
with cessation of the agent. He then began to have more interphalangeal joints and the second and third metacarpo­
frequent gout flares; two flares occurred in the past month phalangeal joints of the feet bilaterally, and tenderness with
and were treated with prednisone. History is also significant movement and swelling of the right wrist.
for hypertension, chronic kidney disease, and dyslipidemia. Radiographs of the hands and wrists are normal.
Current medications are colchicine, lisinopril, metoprolol, Which of the following combination of tests is most helpful
and simvastatin. in confirming the diagnosis?
On physical examination, temperature is 37.2 °C (98.9 °F),
blood pressure is 142/86 mm Hg, pulse rate is 64/min, and (A) Anti-cyclic citrullinated peptide antibodies and anti­
respiration rate is 12/min. BM! is 30. The remainder of the nuclear antibodies
examination is normal. (B) Anti-cyclic citrullinated peptide antibodies and
Laboratory studies reveal a serum creatinine level of C-reactive protein
2.3 mg/dL (203.3 µmol/L), a serum urate level of 9.2 mg/dL (C) Anti-cyclic citrullinated peptide antibodies and
(0.54 mmol/L), and normal liver chemistry studies; esti­ rheumatoid factor
mated glomerular filtration rate is 48 mL/min/1.73 m2 • (D) Antinuclear antibodies and rheumatoid factor
Which of the following is the most appropriate next step in
management?
Item 4
(A) Discontinue colchicine A 46-year-old woman is evaluated for a 3-month history of
(B) Start febuxostat a rash during the summertime. She is otherwise well and
(C) Start pegloticase takes no medications.
(D) Start probenecid On physical examination, vital signs are normal. BM! is
23. Examination of the skin reveals eyelid swelling and a peri­
orbital violaceous rash, erythema and poikiloderma of the
Item 2 anterior chest and upper back, and an erythematous papular
A 32-year-old woman undergoes a new patient evalua­ rash on the hands; there is no malar eruption, skin thicken­
tion. She was diagnosed with systemic lupus erythematosus ing, or digita! ulcers. Musele strength and reflexes are normal.
10 years ago; manifestations have included arthritis, peri­
The appearance of the hands is shown.
carditis, leukopenia, and rash. She reports increasing diffi­
culty using her hands due to joint deformities. Medications
are hydroxychloroquine and prednisone.
On physical examination, temperature is 36.8 °C
(98.2 °F), blood pressure is 130/85 mm Hg, pulse rate is 80/
min, and respiration rate is 16/min. BM! is 24. Examination
of the hands reveals subluxation and ulnar deviation of
the metacarpophalangeal joints on both hands, swan neck
deformity of fingers on both hands, flexion and subluxation
of the metacarpophalangeal joint of both thumbs, and hal­
lux valgus of the first metatarsophalangeal joints bilaterally.
Hand radiographs demonstrate no deformities or
evidence of erosions.
Which of the following is the most likely diagnosis?
(A) Hypermobility syndrome
(B) Jaccoud arthropathy
(C) Mixed connective tissue disease Laboratory studies:
Complete blood count Normal
(D) Rheumatoid arthritis Chemistry panel Normal
Aldolase 5.1 U/L (normal range, 1.0-8.0 U/L)
Creatine kinase 100 U/L
Item 3
Antinuclear antibodies Titer ofl:640
A 30-year-old woman is evaluated for a 4-month history Anti-Jo-1 antibodies Negative
of increasing foot pain. She also has a 2-month history of Urinalysis Normal
increasing pain and swelling in her fingers and right wrist
as well as morning stiffness for more than 1 hour. She has Electromyogram and chest radiograph are normal.

93
Self-Assessment Test

Which of the following is the most likely diagnosis? A radiograph of the left knee is normal.
Aspiration of the left knee is performed; synovial fluid
(A) Amyopathic dermatomyositis analysis reveals a leukocyte count of 20.000/µL (20 x 109 /L).
(B) Polymorphous light eruption extracellular and intracellular urate crystals, and a negative
(C) Rosacea Gram stain.
(D) Systemic lupus erythematosus Which of the following is the most appropriate treat­
ment?
Item 5 (A) Acetaminophen
A 62-year-old woman is evaluated for a 2-year history of (13) Colch icine
progressively frequent and severe pain in the right knee. (C) lndornethacin
She has osteoarthritis with good control of her other (D) Intra-articular glucocorticoids
joint symptoms with her current therapy that includes
medication and a daily exercise regimen. She notes about
20 minutes of morning stiffness in the right knee with Item 7
significant pain with use after rest; her activities are A 65-year-old man is evaluated for severe abdominal pain.
increasingly limited due to these symptoms. History is joint pain. and a rash. He states that he had an upper
otherwise unremarkable. Medications are acetamino­ respiratory infection about 10 days ago. Three days ago
phen and celecoxib. he noted a rash on his lower extremities. One clay later,
On physical examination, blood pressure is 135/82 he experienced pain in his knees and ankles, along with
mm Hg. BM! is 32. There are Heberden nodes of the sec­ abdominal pain that worsened over the past two days. He
ond and fifth distal interphalangeal joints bilaterally and reports no visual symptoms, numbness. weakness. or other
Bouchard nodes of the second and third proximal inter­ symptoms.
phalangeal joints bilaterally. Bony hypertrophy of the On physical examination. the patient appears uncom­
knees is present. There is a positive bulge sign for effusion fortable. ·n1e chest and cardiac examinations are unre­
of the right knee with slight warmth but no erythema. markable. Decreased bowel sounds and diffuse abdomi­
Standing radiographs of the knees show right (greater nal tenderness without rebound are noted. The knees and
than left) medial joint-space narrowing, bilateral osteo­ ankles are tender and mildly swollen. Palpable purpuric
phytes, and bilateral peaking of the tibial spines. lesions are present on the lower extremities, including the
Aspiration of the right knee is performed; synovial fluid soles or the feet. ·n,e remainder of the physical examination
analysis shows a leukocyte count of 250/µL (0.25 x 109 IL) reveals no abnormalities.
and no evidence of crystals. Laboratory studies show a normal complete blood
Which of the following is the most appropriate next step in count. an erythrocyte sedimentation rate or 88 mrn/h, a
management? serum creatinine level of 1.7 mg/dL (150.3 µmol/L), and
a urinalysis showing 3+ protein, 20-30 erythrocytes/hpf,
(A) Administer intra-articular glucocorticoids 20-30 leukocytes/hpf. and mixed granular and cellular
(B) Administer intra-articular hyaluronic acid casts. A stool test is positive for occult blood.
(C) Refer for arthroscopic lavage An abdominal ultrasound reveals thickening and
(D) Substitute indomethacin for celecoxib edema of the ileum. A biopsy of an affected skin lesion
demonstrates the presence of small-vessel, leukocytoclastic
vasculitis accompanied by deposition of lgA.
Item 6 Which of the following is the most appropriate therapy at
A 72-year-old man is evaluated in the emergency depart­ this time?
ment for acute onset of pain and swelling of the left knee. (A) Cyclophosphamide
He was diagnosed with community-acquired pneumonia
(B) Dapsone
4 days ago. and a 7-day course or clarithromycin was
started at that time. He reports marked improvement (C) Ibuprofen
of his respiratory symptoms. History is also significant (D) Prednisone
for gout. with attacks occurring approximately once a
year: hypertension: diet-controlled diabetes mellitus: and
chronic kidney disease. Other medications arc nifedipine Item 8
and hyclrochlorothiazicle. A 28-year-old woman is evaluated for a 6-month history of
On physical examination. temperature is 37.l °C joint pain and swelling. She was diagnosed with rheumatoid
(98.8 °F). blood pressure is 117 86 111111 Hg. pulse rate is arthritis 5 years ago; current medications are etanercept,
76/min. and respiration rate is 14 min. BM! is 32. Mildly sulfasalazine, and etodolac. She was initially treated with
decreased breath sounds in the right lung midfield are methotrexate, which was stopped due to gastrointestinal
noted. ll1e left knee is swollen. red, warm. tender. and fluc­ intolerance, and she refuses to retry it.
tuant with limited range of motion. On physical examination, temperature is 36.7 °C
Laboratory studies are significant for a leukocyte count (98.0 °F), blood pressure is 126/74 mm Hg, pulse rate
of 7200'µL (7.2 x 109 L) and a serum creatinine level of is 68/min, and respiration rate is 14/min. BM! is 24.
1.7 mg ell (150.3 µmol/L). Two proximal interphalangeal (PIP) joints of the left

94
Self-Assessment Test

hand and one metacarpophalangeal (MCP) joint bilater­ SLE antinuclear antibody profile:
ally are swollen and tender. Examination of the elbows, Antinuclear antibodies Positive (titer: 1:320),
wrists, knees, and feet is normal. The remainder of the speckled pattern
examination, including cardiopulmonary examination, Anti-Ro/SSA antibodies Positive
is normal. Anti-double-stranded DNA Negative
Laboratory studies, including complete blood count, antibodies
chemistry panel, and liver chemistries, are normal; erythro­ Anti-Ul-ribonucleoprotein Negative
cyte sedimentation rate is 35 mm/h, and C-reactive protein antibodies
level is 1.1 mg/dL (11 mg/L). Anti-Smith antibodies Negative
Which of the following is the most appropriate next step in The increased risk of preeclampsia and preterm deliv­
the management of this patient's disease activity? ery in SLE as well as avoidance of NSAIDs prior to concep­
(A) Add abatacept tion and in the later stages of pregnancy is discussed.
(B) Add anakinra Which of the following also needs to be discussed with this
(C) Add leflunomide patient based on her antibody profile?
(D) Add rituximab (A) Need to discontinue hydroxychloroquine
(B) Risk of congenital heart block in her child
Item 9 (C) Risk of developing lupus nephritis
A 42-year-old woman is evaluated for a 4-year history of (D) Risk of developing subacute cutaneous lupus
diffuse muscle and joint pain. most notably of her shoul­
ders, low back, hips, and knees. The pain is present in the
morning and throughout the day. She wakes unrefreshed Item 11
and reports problems with her memory. She also describes A SO-year-old man is evaluated for an 8-year history of joint
diarrhea alternating with constipation with no blood or pain, particularly in the hands, and progressively worsen­
mucus in the stool. She reports no weight loss. She quit ing fatigue. Medical history is otherwise unremarkable. He
working 2 years ago due to her symptoms, which were takes ibuprofen as needed.
made worse by her work as a baker. She has been to multi­ On physical examination, vital signs are normal. BM!
ple medical providers who have not established a diagnosis is 32. There are swelling and tenderness of the second and
despite numerous tests. third metacarpophalangeal (MCP) joints bilaterally, and
On physical examination, temperature is 37.2 °C tenderness but no swelling of the fourth and fifth MCP joints
(99.0 °F), blood pressure is 134/88 mm Hg, pulse rate is 92/ bilaterally. There is bony hypertrophy of the first MCP
min, and respiration rate is 16/min. BM! is 36. Muscles are joints and knees bilaterally. TI1e proximal interphalangeal
generally tender to light palpation but without weakness on joints and wrists are normal.
muscle strength testing. The remainder of the examination Laboratory studies are notable for an alanine amino­
is normal. transferase level of 53 U/L and an aspartate aminotransfer­
Laboratory studies, including complete blood count, ase level of 55 U/L; rheumatoid factor is negative.
chemistry panel, erythrocyte sedimentation rate, serum A radiograph of the hand is shown (see top of next page).
creatine kinase, and thyroid-stimulating hormone, are
Which of the following is the most appropriate diagnostic
normal.
test to perform next?
Which of the following is the most likely diagnosis?
(A) Anti-cyclic citrullinated peptide antibody assay
(A) Adrenal insufficiency (B) Hepatitis C antibody assay
(B) Fibromyalgia (C) Serum a-fetoprotein measurement
(C) Hypothyroidism (D) Transferrin saturation measurement
(D) Polymyositis

Item 12
Item 10 A 25-year-old man is evaluated for a 3-year history of low
A 28-year-old woman seeks preconception counseling. She back and bilateral buttock pain that has gradually increased
has a 4-year history of systemic lupus erythematosus (SLE) over the past year. TI1e pain is worse in the morning and
with manifestations of photosensitive rash, arthritis, and after inactivity; he feels better after stretching his back. He
pericarditis; she has been treated with hydroxychloroquine has 90 minutes of morning stiffness in his back. Ibupro­
and low-dose prednisone with good control of her symp­ fen provides moderate relief of symptoms. He reports no
toms for 18 months. She has never been pregnant. She also other arthritic symptoms, rash, or gastrointestinal symp­
takes vitamin D and calcium. toms. Family history is notable for his paternal uncle with
The physical examination and vital signs are normal. long-standing back problems.
Laboratory studies indicate that the patient's SLE is On physical examination, vital signs are normal. There
quiescent. A recent urinalysis is normal, and a previously is painful and diminished forward flexion and extension
checked antiphospholipid panel and lupus anticoagulant of the lumbar spine. Tenderness to palpation over both
were negative. buttocks is noted.

95
Self-Assessment Test

(A) Bronchoscopy with bronchoalveolar lavage


(B) Doppler echocardiography
(C) N-terminal proBNP (B-type natriuretic peptide)
measurement
(D) Right heart catheterization

Item 14
A 34-year-old woman is evaluated during a follow-up visit
for polymyositis. She was diagnosed 1 year ago and has
responded well to therapy. She reports no weakness, chest
pain, or shortness of breath on exertion. Current medica­
tions are prednisone and azathioprine.
On physical examination, temperature is normal,
blood pressure is 106/72 mm Hg, pulse rate is 84/min, and
respiration rate is 26/min. BM! is 23. Oxygen saturation
is 98% on ambient air. Cardiac and pulmonary examina­
tions are normal. Strength is normal in proximal and distal
muscles. Slight hyperkeratosis and cracking of the palmar
surface of the hands are present. There are no other rashes,
skin thickening, or digital ulcers.
Laboratory studies are notable for a serum creatine
kinase level of 100 U/L, an antinuclear antibody titer of
1:1280, and anti-Jo-1 antibody positivity.
Electrocardiogram is normal.
Which of the following is the most appropriate diagnostic
test to perform next?
(A) 6-Minute walk test
ITEM 11 (B) Cardiac MRI
(C) Chest radiography
(D) Exercise stress testing
Laboratory studies reveal an erythrocyte sedimenta­
tion rate of 35 mm/h; HLA-B27 testing is negative. (E) No additional testing
Plain radiographs of the lumbar spine and sacroiliac
joints are normal. Item 15
Which of the following is the most appropriate diagnostic A 52-year-old woman is evaluated during a follow-up visit
test to perform next? for a 6-year history of rheumatoid arthritis. She has not
responded to combination therapy with methotrexate
(A) CT of the sacroiliac joints
and etanercept, abatacept, or rituximab. Current medica­
(B) MRI of the sacroiliac joints tions are methotrexate and tofacitinib, which was initiated
(C) Technetium bone scan 1 month ago.
(D) Ultrasonography of the sacroiliac joints On physical examination, vital signs are normal. Exam­
ination of the joints shows mild swelling and tenderness
of the proximal interphalangeal and metacarpophalangeal
Item 13 joints and wrists bilaterally.
A 52-year-old woman is evaluated for an 8-week history of Which of the following laboratory studies should be moni­
fatigue and shortness of breath. She has gastroesophageal tored in this patient?
reflux disease, hypertension, and a 3-year history of limited
cutaneous systemic sclerosis. Medications are omeprazole, (A) Alkaline phosphatase
nifedipine, lisinopril, and aspirin. (B) Bilirubin
On physical examination, temperature is 36.4 °C (C) Glucose
(97.6 °F), blood pressure is 126/72 mm Hg, pulse rate is
(D) Lipid profile
114/min, and respiration rate is 20/min. BM! is 24. Oxygen
saturation is 88% on ambient air. A prominent single S? is
heard. The chest is clear on auscultation. Sclerodactyly a-nd Item 16
multiple healed digital pits are noted. There is no rash.
Chest radiograph is normal. A 56-year-old man is evaluated during a follow-up visit. He
was diagnosed with gout 4 months ago based on recurrent
Which of the following is the most appropriate diagnostic episodes of podagra and a serum urate level of 7.2 mg/dL
test to perform next? (0.42 mmol/L). Colchicine and allopurinol were initiated at

96
Self-Assessment Test

that time and have been maintained at their initial doses. cyte sedimentation rate is 45 mm/h, C-reactive protein level
History is also significant for chronic kidney disease and is 1.8 mg/dL (18 mg/L), and rheumatoid factor is 112 U/mL
hypertension, for which he takes losartan. (112 kU/L). Hepatitis B and C serologies are negative.
On physical examination, temperature is 37.1 °C The decision is made to start treatment with adali­
(98.8 °F), blood pressure is 130/85 mm Hg, pulse rate is mumab.
75/min, and respiration rate is 15/min. BM! is 27. There is
no swelling of the joints. The remainder of the examination Which of the following is the most appropriate screening
is unremarkable. test to perform before initiating adalimumab?
Current laboratory studies reveal a serum urate level (A) Chest radiography
of 6.4 mg/dL (0.38 mmol/L) and a serum creatinine level of (B) Immunoglobulin level measurement
2.1 mg/dL (185.6 µmol/L).
(C) Interferon-y release assay
Which of the following is the most appropriate manage­ (D) Radiography of hands and feet
ment?
(A) Discontinue colchicine Item 19
(B) Discontinue losartan A 30-year-old man is evaluated for a 1-year history of low
(C) Increase allopurinol back pain. The pain frequently spreads to the buttocks but
(D) No change in therapy does not radiate to the legs. The pain is worse in the morning
and is associated with stiffness but improves 2 hours later
after he starts working. Symptoms are worse at the end of
Item 17 the day and during the night. He takes ibuprofen with good
An 82-year-old woman is evaluated for a 2-week history relief of the pain. He is otherwise healthy and reports no
of left-sided headaches with pain on chewing, accompa­ other joint pain, rash, diarrhea, or dysuria.
nied by achiness in the shoulders and hips. She has no On physical examination, vital signs are normal. Eye
other pertinent personal or family history. She takes no examination is normal. There is mild pain with normal
medications. range of motion in all directions of the lumbar spine. Ten­
On physical examination, temperature is 38.1 °C derness over the buttocks is noted. There is no joint swelling
(100.6 °F), blood pressure is 132/86 mm Hg, pulse rate is 88/ or tenderness in the upper or lower extremities. There is no
min, and respiration rate is 18/min. BM! is 25. Eye exam­ rash or nail pitting.
ination is normal. There are tenderness and swelling over Laboratory studies are significant for an erythrocyte
the left temporal area. Moderate to severe pain on range of sedimentation rate of 40 mm/h, and HLA-B27 testing is
motion of the shoulders and hips is noted. There is no pain positive.
over the temporomandibular joints on palpation. Plain radiographs of the lumbar spine and sacroiliac
Laboratory studies, including basic metabolic panel, joints are normal.
complete blood count, and liver chemistries, are normal;
Which of the following is the most likely diagnosis?
erythrocyte sedimentation rate is 85 mm/h.
Which of the following is the most appropriate immediate (A) Ankylosing spondylitis
next step in management? (B) Lumbar degenerative disk disease
(C) Psoriatic arthritis
(A) Initiate prednisone, 15 mg/d
(D) Reactive arthritis
(B) Initiate prednisone, 60 mg/d
(C) Obtain MRI of the head
(D) Obtain temporal artery biopsy Item 20
A 52-year-old woman is evaluated for a 6-year history of
Sjogren syndrome. During the past 3 months, she has had
Item 18 low-grade fevers up to 37.5 °C (99.5 °F), weight loss of
A 25-year-old woman is evaluated for a 2-month history of 6.8 kg (15 lb), and increased fatigue and sicca symptoms.
increasing joint pain and swelling. She was diagnosed with She recently noted a rash on her legs. She reports no current
rheumatoid arthritis 1 year ago and initially treated with joint pain. Medications are hydroxychloroquine and acet­
methotrexate with good response but recently has had more aminophen as needed.
pain and swelling. With an increase of the methotrexate On physical examination, temperature is 37.2 °C
dosage and the addition of sulfasalazine and hydroxychlo­ (99.0 °F), blood pressure is 135/85 mm Hg, and pulse
roquine, there was improvement but incomplete control of rate is 82/min. BM! is 28. The oral mucosa is dry. Bilat­
the disease. She also takes naproxen daily. eral parotid fullness is present. There is bilateral cervical
On physical examination, vital signs are normal. BM! adenopathy. The tip of the spleen is palpable. There are
is 23. Two metacarpophalangeal joints of both hands are a few scattered palpable purpura on the lower legs. The
tender and swollen. There is palpable warmth and tender­ remainder of the physical examination is unremarkable.
ness of both wrists. The remainder of the examination is Laboratory studies show a normal complete blood
normal. count except for a hemoglobin level of 11 g/dL (110 g/L);
Laboratory studies, including complete blood count, serum C3 and C4 levels are low, and serum and urine pro­
chemistry panel, and liver chemistries, are normal; erythro- tein electrophoresis reveals M-component.

97
Self-Assessment Test

Chest radiograph and echocardiogram are normal. CT The cardiopulmonary and abdominal examinations are
scan of the abdomen shows numerous enlarged retroperito­ normal. There is no joint swelling.
neal lymph nodes and splenomegaly. Current laboratory studies, including complete blood
count, chemistry panel, and erythrocyte sedimentation
Which of the following is the most appropriate next step in rate, are normal.
the management of this patient?
Which of the following is the most appropriate treatment?
(A) Obtain a lymph node biopsy
(B) Obtain a skin biopsy (A) Anakinra
(C) Order heterophile antibody testing (B) Colchicine
(D) Start prednisone (C) Indomethacin
(E) Start prednisone and cyclophosphamide (D) Prednisone

Item 23
Item 21
A 75-year-old woman is evaluated for progressive left knee
A 30-year-old man is evaluated for a 6-month history of pain. She has a 20-year history of bilateral knee osteoar­
pain behind his right heel. The pain is worse after immobil­ thritis. There is no recent history of injury. She was recently
ity, and he has morning stiffness in the foot lasting 1 hour. discharged from the hospital for gastrointestinal bleeding
He has tried acetaminophen without much relief. History is related to her use of ibuprofen. She experienced only tran­
also significant for a 3-year history of intermittent left eye sient relief from previous glucocorticoid and hyaluronic
uveitis treated with a prednisolone ophthalmic solution. He acid injections. She is enrolled in physical therapy and exer­
has no other symptoms. cises to increase her quadriceps strength. History is also sig­
On physical examination, vital signs are normal. nificant for hypertension, coronary artery disease, hyper­
There is no tenderness of the lumbar spine or sacroiliac cholesterolemia, and osteoporosis. Other medications are
joints; full range of motion of the lumbar spine is noted. omeprazole, aspirin, lisinopril, propranolol, rosuvastatin,
Mild swelling and tenderness at the insertion of the Achil­ and alendronate.
les tendon are noted. The remainder of the examination On physical examination, vital signs are normal. BM! is
is normal. 24. Mild weakness on muscle group testing and atrophy are
Radiographs of the sacroiliac joints are normal. Radio­ noted in the quadriceps. There is marked bony hypertrophy
graphs of the right heel show soft-tissue swelling and an of the left knee (greater than the right) without warmth,
erosion at the insertion of the Achilles tendon. erythema, or effusion.
Which of the following is the most appropriate diagnostic Which of the following is the most appropriate manage­
test to perform next? ment for this patient?
(A) Anti-cyclic citrullinated peptide antibody assay (A) Celecoxib
(B) Antineutrophil cytoplasmic antibody assay (B) Duloxetine
(C) Antinuclear antibody assay (C) Fentanyl
(D) HLA-B27 testing (D) Prednisone

Item 22 Item 24
A 22-year-old woman is evaluated for a 2-year history of r\ "IS-year-old woman is evaluated in the emergency depart­
recurrent abdominal pain often accompanied by fever; ment l'or progressive shortness ol' breath and fatigue for
episodes occur every 3 to 4 months and last 1 to 3 days and the past 6 weeks. She also has a 5-year history of diffuse
resolve completely. She went to the emergency depart­ cutc111cous systemic sclerosis. Medicaiions are nil'edipine.
ment during an episode 6 weeks ago. She was noted to be lisinopril. omepr.:izole . .:ind clSpirin.
mildly febrile, and laboratory studies showed an eryth­ On physical examination. the patient is alen but short
rocyte sedimentation rate of 84 mm/h and a leukocyte of"breath. Temperature is 37.2 °C (99.0 °F). blood pressure is
count of 16,000/µL (16 x 109/L) with neutrophilia. She 126·92 mm Hg. pulse rate is 12-l min. and respiration rate is
was diagnosed with viral gastroenteritis and recovered 26 min. BM! is 25. Oxygen saturation is 98'Y., on 2 L of oxy­
completely with supportive treatment. She has been gen. C1 r·d iac exa Ill ina Lion is norlllal. Velcro-Ii ke crackles are
treated on several occasions for cellulitis that occurs on her heard throughout the chest. Diffuse skin thickening or the
foot or lower extremity, but the reason for repeated infec­ face. anterior chest. arms stopping at the elbows. and legs is
tion or a responsible organism has not been identified. present: sclerodactyly of the fingers is also noted. There is no
On two occasions, she had pain and swelling in the knee rash. Pedal edem.:i is present.
that lasted several weeks and was not associated with the Chest radiograph shows bilateral reticulonodul.:ir
abdominal pain. She has tried naproxen without relief. She infiltrates and ground glass opacities with normal c.:ir­
currently feels well and has no complaints. cliac silhouette. High-resolution CT scan is consistent with
On physical examination today, temperature is active nonspecific interstitial pneumonitis. An open lung
36.6 °C (97.9 °F), blood pressure is 120/74 mm Hg, pulse biopsy confirms the diagnosis of nonspecific interstitial
rate is 74/min, and respiration rate is 14/min. BM! is 23. pneu 111011itis.

98
Self-Assessment Test

[I
CONT.
Which of the following is the most appropriate treatment?
(A) Cyclophosphamide
(B) D-penicillamine
(C) lnfliximab
(D) Methotrexate

Item 25
A 72-year-old man is evaluated in the emergency depart­
ment for acute swelling. severe pain. and warmth of the
right knee that woke him from sleep. He does not recall any
inciting injury to the knee. Three months ago, he had an
acutely swollen great toe that improved within 3 days. for
which he did not seek treatment. History is also significant
for hypertension and diabetes mellitus. Medications are
hydrochlorothiazide and metformin.
On physical examination. temperature is 37.8 °c
(100.1 °F). blood pressure is 130/75 111111 Hg. pulse rate is
90/min. and respiration rate is 12/min. BMI is 33. The right
knee is warm and swollen without overlying erythema:
tenderness to palpation and decreased range of motion ITEM 26
due to pain are noted. 1l1ere is no skin breakdown or abra­
sions over the right knee. Examination of' the other joints
is unremarkable.
Which of the following is the most appropriate next step
in management?
Item 27
A 42-year-olcl man is evaluated in the hospital for a 2-week
[I
history of progressive shortness of breath, with hemoptysis
(A) Obtain a knee MRI developing in the past 48 hours. During the past week he
(B) Obtain a serum urate level has also noted weakness of the left foot, numbness in the
(C) Perform joint aspiration right hand. and the onset of a rash. He has a 7-year history
of asthma. His only medication is an as-needed albuterol
(D) Start empiric colchicine
metered-dose inhaler.
On physical examination. temperature is 38.0 °C
(J00.4 °F). blood pressure is 142/87 mm Hg. pulse rate is
Item 26 72/min. and respiration rate is 26/min. Diffuse crackles
A 40-year-old woman is evaluated for a 6-month history of are heard in the lung fields. Diminished sensation in
pain and swelling in her left thumb, left fifth finger, and left the right hand and weakness on dorsiflexion in the left
foot. She also has morning stiffness lasting 2 to 3 hours. She fool are noted. 1l1ere is palpable purpura on the arms
has a 4-year history of lumbar and thoracic back pain that ancl legs. The remainder of the physical examination is
is worse with bending and lifting and is better with rest. normal.
Naproxen is only mildly helpful for the pain. Laboratory studies:
On physical examination, vital signs are normal. Erythrocyte 98 mm/h
Patches of erythema and scaling behind the right ear and sedimentation rate
on the scalp at the occiput are noted. Fusiform swelling of Leukocyte count 16,000iµL (16 x 109/L),
the left thumb and left fifth finger is present. Tenderness 22% eosinophils
and swelling at the left third metatarsophalangeal joint are Creatinine 0.8 mg/ell (70.7 µmol/L)
noted. There is mild lumbar tenderness, and full range of lgE Elevated
motion of the lumbar spine and cervical spine is noted. No ANCA Negative
other joint swelling or tenderness is present. Antimyeloperoxidase Negative
Nail findings are shown (see top of next column). antibodies
Laboratory studies, including complete blood count Antiproteinase 3 antibodies Negative
with differential, comprehensive metabolic panel, rheu­ Urinalysis Normal
matoid factor, and urinalysis, are normal; HLA-B27 testing
is positive. Chest radiograph shows diffuse pulmonary infiltrates.
Which of the following is the most likely diagnosis? Which of the following is the most likely diagnosis?
(A) Ankylosing spondylitis (A) Cryoglobulinemia
(B) Inflammatory bowel disease--associated arthritis (B) Eosinophilic granulomatosis with polyangiitis
(C) Psoriatic arthritis (C) Granulomatosis with polyangiitis
(D) Reactive arthritis (D) Microscopic polyangiitis

99
Self-Assessment Test

Item 28 min. ancl respiration rate is 13/rnin. BMl is 22. TI1ere are
symmetric tenclerness. warmth. erytherna, ancl swelling
A 42-year-old woman is evaluated for a 3-month history of
of the wrists. proximal interphalangeal ancl metacarpo­
symmetric proximal muscle weakness. She takes no medi­
phalangeal joints. and knees bilaterally. Bilateral knee
cations.
effusions are notecl. TI1ere is milcl right upper quaclrant
On physical examination, vital signs are normal. Sym­
pain. A maculopapular rash over the trunk and legs is
metric weakness of the arm and thigh muscles is noted.
present.
There are no skin findings.
Laboratory studies are significant for a serum creatine Laboratory studies:
kinase level of 2000 U/L and a normal thyroid-stimulating Erythrocyte sedimentation rate 63 mm h
hormone level. Alanine aminotransferase 1050 U L
Electromyogram shows increased insertional activ­ Aspartate aminotransferase 800 U1L
ity, spontaneous fibrillations, and polyphasic motor unit Creatinine Norma 1
potentials in the proximal muscles. MRI of the thighs shows
inflammatory changes in the quadriceps. Which of the following is the most likely diagnosis?
A muscle biopsy is recommended, but the patient (A) Autoimmune hepatitis
refuses.
(B) Hemochromatosis
Which of the following is the most appropriate treatment (C) Hepatitis B virus-associatecl arthritis
at this time? (D) Primary biliary cirrhosis
(A) Adalimumab
(B) Cyclosporine
Item 31
(C) Leflunomide
A 65-year-old woman is seen at the request of her oph­
(D) Prednisone
thalmologist. Two days ago, she was diagnosed with scleri­
tis and began using an ophthalmic prednisolone solution.
She notes progressive fatigue and intermittent episodes of
Item 29
sinus congestion during the past 5 weeks. She has a 10-year
A 71-year-old man is evaluated for long-standing stiffness, history of joint pain in her hands and low back pain that
decreased range of motion, and pain of the neck, mid has not recently changed. History is also significant for
back, and low back. He has no history of falls or injuries. hypertension diagnosed 3 months ago, for which she takes
The stiffness and pain do not improve with activity and are hydrochlorothiazide. She reports no dyspnea, cough, rash,
not noticeably worse in bed or with inactivity. Acetamin­ diarrhea, or abdominal pain.
ophen provides minimal relief. He has no other medical On physical examination, vital signs are normal. The
problems. ears are normal. Right eye scleral injection is present.
On physical examination, vital signs and BMI are nor­ There is mild redness and crusting of the nasal mucosa.
mal. Skin examination is normal. Bony hypertrophy of the There are no oral ulcerations. There is bony enlarge­
second through fifth distal interphalangeal joints and the ment with tenderness over the distal interphalangeal
second and fifth proximal interphalangeal joints is present. joints bilaterally and squaring with tenderness over the
Marked reduction in thoracic lateral bending and reduction first carpometacarpal joints bilaterally. Mild lumbar and
of spinal flexion and extension are noted. paraspinal muscle tenderness is present; full range of
Plain radiographs of the thoracic spine show flowing motion of the lumbar spine is noted. There is no sacroiliac
osteophytes involving the anterolateral aspect of the tho­ joint tenderness. The remainder of the physical examina­
racic spine at five contiguous vertebrae; there is normal tion is normal.
disk height, the apophyseal joints are without osteophytes
or bony sclerosis, and the sacroiliac joints are without Laboratory studies:
erosions. Comprehensive Normal
metabolic panel
Which of the following is the most likely diagnosis? Erythrocyte 55 mm/h
sedimentation rate
(A) Ankylosing spondylitis
Hemoglobin 11 g/dL (110 g/L)
(B) Degenerative disk disease Leukocyte count 5000/µL (5.0 x 109/L)
(C) Diffuse idiopathic skeletal hyperostosis Platelet count 550.000/µ.L (550 X 109/L)
(D) Psoriatic arthritis Urinalysis 2+ protein; trace blood; no
leukocytes; 1 erythrocyte cast

Cl Item 30 Chest radiograph is normal.


A 46-year-old woman is evaluated for a 1-week history of Which of the following is the most likely diagnosis?
symmetric polyarthritis of the hands. wrists, ancl knees.
(A) Ankylosing spondylitis
accompanied by a rash. She is a home health aide. She has
no other pertinent history and takes no medications. (B) Behc;et syndrome
On physical examination. temperature is 37.7 °C (C) Granulomatosis with polyangiitis
(99.8 °F). bloocl pressure is 118/72 rnm Hg. pulse rate is 78 (D) Sarcoidosis

100
Self-Assessment Test

CJ Item 32 has not had any episodes of diarrhea or abdominal pain and
reports no trauma to the knee, fever, rash, or known insect
An SO-year-old man is evaluated for severe right knee pain
that began yesterday. He has a 10-year history of gout that bites. He does not have a history of sexually transmitted
has affected his great toes and knees: his last attack was infections. He has no history of injection drug use and does
2 years ago in the right great toe. Medications are allopurinol not take any medications.
and ibuprofen as needed. On physical examination, temperature is 37.1 °C
On physical examination, temperature is 37.8 °c (98.8 °F), blood pressure is 115/70 mm Hg, pulse rate is 82/
(100.0 °F). blood pressure is 150/85 mm Hg. pulse rate is 80/ min, and respiration rate is 12/min. BM! is 20. There is a large
min. and respiration rate is L6/min. BM! is 31. The right knee effusion over the left knee with warmth and mild tender­
is warm. swollen, slightly erythematous. and tender: range of ness but no overlying erythema; range of motion is limited
motion is limited to 90 degrees offlexion and associated with by swelling, but stability is intact. There is no heart murmur.
pain. There is no inflammation in the remainder of the joints. Lung and abdominal examinations are normal. TI1ere are no
Aspiration of the right knee yields 30 ml of cloudy skin lesions.
yellow fluid. Synovial fluid leukocyte count is 55.000/µL Laboratory studies reveal an erythrocyte sedimenta­
(55 x 109/L), with 95% polymorphonuclear cells. Extracellu­ tion rate of 12 mm/h and a leukocyte count of 6000/µL
lar negatively birefringent needle-shaped crystals are seen (6.0 X 10 9/L).
under polarized light. Synovial fluid Gram stain is negative. Radiograph of the left knee shows a large effusion but
Synovial fluid cultures are pending. is otherwise unremarkable.

Which ofthe following is the most appropriate treatment? Which of the following is most likely to provide the
diagnosis?
(A) Add probenecid
(A) Blood cultures
(B) Increase allopurinol
(B) Lyme serologic testing
(C) Perform an intra-articular glucocorticoid injection
(C) MRI of the knee
(D) Start antibiotics
(D) Synovial fluid cultures
(E) Start prednisone

Item 33 Item 35
A 25-year-old woman is evaluated during a follow-up visit A 21-year-old woman is evaluated for a 3-week history of
for an 18-month history of ankylosing spondylitis. She has painful nodules and a rash in the lower extremities, along
minimal lower back pain with morning stiffness lasting with pain and swelling of the wrists, knees, and ankles.
20 minutes. She is able to pursue her activities of daily living She reports a low-grade fever and a 2.7-kg (6.0-lb) weight
without any restrictions. She has been taking etanercept for loss since the onset of symptoms. She has taken naproxen
1 year with good results. with some relief. History is significant for gastroesophageal
On physical examination, vital signs are normal. Full reflux disease and acne. Medications are over-the-counter
range of motion of the thoracic and cervical spine without famotidine as needed and minocycline.
tenderness is noted. There is no lumbar or sacroiliac tender­ On physical examination, temperature is 38.2 °C
ness. The Schober test increases by 5 cm (same as at the time (100.8 °F), blood pressure is 110/60 mm Hg, pulse rate
of diagnosis). is 92/min, and respiration rate is 16/min. BM! is 24. Mild
Laboratory studies are notable for a normal erythro­ swelling of the wrists, knees, and ankles is noted. There
cyte sedimentation rate and a normal C-reactive protein are scattered 1- to 2-cm painful erythematous nodules as
level. well as livedo reticularis in the lower extremities begin­
At the time of diagnosis, radiographs showed normal ning at the thighs. The remainder of the examination is
thoracic and lumbar spine and sacroiliac joints, and an MRI normal.
showed edema of the sacroiliac joints and in the lumbar and Laboratory studies:
thoracic spine. Antinuclear antibodies Positive (titer: 1:320)
Which of the following should be performed next? Anti-double-stranded DNA Negative
antibodies
(A) Bone scan Anti-Smith antibodies Negative
(B) CT of the sacroiliac joints Anti-Ul-ribonucleoprotein Negative
(C) MRI of the sacroiliac joints antibodies
Anti-Ro/SSA antibodies Negative
(D) Plain radiography of the sacroiliac joints
Anti-La/SSB antibodies Negative
(E) No new imaging Antihistone antibodies Negative
ANCA Positive (titer: 1:320)
in a perinuclear
Item 34
pattern; negative for
A 35-year-old man is evaluated for a 2-month history of myeloperoxidase
abrupt left knee swelling. He notes prominent stiffness of Urinalysis Normal
both joints but no significant pain. He previously felt well.
He lives in Vermont and goes hiking during the summer. He Chest radiograph is normal.

101
Self-Assessment Test

Which of the following is the most appropriate next step in recurrent corneal abrasions and erosions. She has been
management? using artificial tears with minimal improvement. She saw
her ophthalmologist who inserted punctal plugs, but they
(A) Start azathioprine caused excessive tearing and were removed. She otherwise
(B) Start high-dose prednisone feels well and reports no fever, chills, weight loss, rash, joint
(C) Discontinue famotidine pain, chest pain, or dyspnea.
(0) Discontinue minocycline On physical examination, temperature is 36.7 °C
(98.0 °F), blood pressure is 135/80 mm Hg, pulse rate is 75/min,
and respiration rate is 18/min. The oral mucosa is dry. Moder­
Item 36 ately injected sclerae are noted. There is no cervical or supra­
A 61-year-old man is evaluated for a 10-month history of clavicular adenopathy and no parotid gland swelling. Lung,
generalized weakness. He reports no pain or myalgia. His­ heart, musculoskeletal, and skin examinations are normal.
tory is significant for hypercholesterolemia treated with a Which of the following is the most appropriate treatment?
stable dose of simvastatin for the past 3 years.
On physical examination, temperature is normal, blood (A) Certolizumab pegol
pressure is 138/74 mm Hg, pulse rate is 70/min, and respira­ (B) Cyclosporine drops
tion rate is 16/min. BM! is 27. There is symmetric weakness (C) Hydroxychloroquine
of the arm and thigh muscles with slightly reduced grip and
power of the finger flexors. No muscle tenderness is noted. (D) Olopatadine drops
There is no rash, skin thickening, or digital ulcers. Reflexes (E) Prednisone
and the remainder of the physical examination are normal.
Laboratory studies are notable for a normal complete
blood count, an erythrocyte sedimentation rate of 23 mm/h, Item 39
and a serum creatine kinase level of 365 U/L. A 25-year-old man undergoes a new patient evaluation. He
Chest radiograph is normal. Electromyogram and has Marfan syndrome. His clinical course has been unre­
nerve conduction studies show myopathic changes in the markable. History is significant for inguinal hernia repair;
proximal and distal muscles of the extremities as well as family history is notable for his father who has Marfan syn­
some neurogenic changes. drome. The patient takes no medications.
Which of the following is the most likely diagnosis? On physical examination, temperature is 37.4 °C
(99.3 °F), blood pressure is 134/89 mm Hg, pulse rate is 80/
(A) Amyotrophic lateral sclerosis min, and respiration rate is 14/min. BM! is 22. Tall stature
(B) Inclusion body myositis and pectus excavatum are noted. Oral examination demon­
(C) Myasthenia gravis strates a high arched palate. Arachnodactyly is noted. Scoli­
osis is present on forward bending. Examination of the feet
(D) Statin-induced myopathy
reveals bilateral pes planus without obvious osteoarthritic
mid-foot changes.
Item 37 Which of the following is the most appropriate periodic
A 72-year-old man is evaluated for a I-year history of pro­ imaging test for this patient?
gressive worsening of bilateral knee pain and stiffness. He has
had no locking, popping, or giving way in either knee. He has (A) Abdominal ultrasonography
pain in both knees at rest and at night, which awakens him (B) Chest radiography
from sleep. He has no history of injury. Acetaminophen and (C) Echocardiography
over-the-counter ibuprofen improve his pain temporarily. (D) Spine radiography
On physical examination, vital signs are normal. BM! is
35. Bilateral bony hypertrophy and valgus deformity of the
knees are noted. There is no warmth, erythema, swelling, Item 40
or effusion. Anterior drawer sign is negative, and there is no
compromise in stability. A 32-year-old woman is evaluated for a new rash on her
legs. She was diagnosed with pyelonephritis 4 days ago and
Which of the following studies of the knees is the most was started on a 7-day regimen of trimethoprim-sulfame­
appropriate diagnostic test to perform next? thoxazole based on urine culture and sensitivity data; her
urinary symptoms have improved. Medical history includes
(A) Bone scintigraphy Hashimoto thyroiditis treated with levothyroxine; her dose
(B) MRI was increased 4 weeks ago based on thyroid function stud­
(C) Standing plain radiography ies. Medical history is otherwise unremarkable.
(D) Ultrasonography On physical examination, temperature is normal,
blood pressure is 124/82 mm Hg, pulse rate is 66/min, and
respiration rate is 13/min. BMI is 21. Cardiopulmonary
Item 38 examination is unremarkable. The abdomen is soft and non­
A 60-year-old woman is evaluated during a follow-up visit tender. Musculoskeletal examination shows no evidence of
for Sjogren syndrome. She reports persistent eye discom­ joint swelling, warmth, or tenderness. The remainder of the
fort described as a sandy or gritty sensation. She has had examination is normal.

102
Self-Assessment Test

The appearance of the legs is shown. level of 7.2 mg/dL (0.42 mmol!L); symptoms resolved with
naproxen. He then presented last week with recurrent
symptoms of great toe pain, redness, and swelling that
began during sleep. Colchicine was initiated, and symp­
toms resolved. History is also significant for hypertension,
coronary artery disease, hyperlipidemia, and urolithiasis.
Current medications are colchicine, metoprolol, simvasta­
tin, and low-dose aspirin.
On physical examination, temperature is 37.l 0 c
(98.8 °F), blood pressure is 138/80 mm Hg, pulse rate is 60/
min, and respiration rate is 15/min. BM! is 30. Examination
of the joints reveals no swelling.
Laboratory studies reveal a serum urate level of 7.6 mg/
dL (0.45 mmol!L) and normal kidney and liver chemistries.
Which of the following is the most appropriate treatment
for this patient?
(A) Discontinue aspirin
(B) Discontinue colchicine
(C) Start allopurinol
(D) Start probenecid

Item 43
A SO-year-old woman is evaluated for slowly worsening
joint pain in her fingers for the past 5 years. She notes swell­
ing, morning stiffness lasting 10 minutes, and pain that is
Which of the following is the most appropriate next step in worse after housework or typing. She has no other joint
management? pain and otherwise feels well. She reports no fevers, weight
(A) Discontinue trimethoprim-sulfamethoxazole loss, rashes, alopecia, oral ulcers, dyspnea, chest pain, or
(B) Initiate prednisone abdominal pain. The patient takes no medications.
On physical examination, vital signs are normal. There
(C) Measure antihistone antibodies is squaring, crepitus, and tenderness of the first carpo­
(D) Obtain skin biopsy metacarpal joints. Bony enlargement and tenderness over
all distal interphalangeal (DIP) joints are present. Limited
range of motion of the thumbs and DIP joints is noted. There
Item 41 is no joint warmth, redness, or effusions. The remainder of
A 30-year-old woman is evaluated during a follow-up the joint examination is normal.
visit for systemic lupus erythematosus. She was diagnosed
3 months ago after presenting with pericarditis and arthri­ Which of the following is the most appropriate next step in
tis. She was initially treated with prednisone, 40 mg/d, with management?
improvement of her presenting symptoms. The prednisone (A) Anti-double-stranded DNA antibody testing
has been tapered over 3 months to her current dose of (B) Antinuclear antibody testing
10 mg/d with no recurrence. She also takes vitamin D and a
(C) Radiography of the hands
calcium supplement.
On physical examination, vital signs are normal. BM! is (D) Rheumatoid factor testing
25. Cardiac examination is normal. 1here is no evidence of (E) No further testing
arthritis. The remainder of the examination is normal.
Which of the following is the most appropriate next step in Item 44
treating this patient? A 74-year-old man is evaluated for a 2-month history of
(A) Add azathioprine progressively worsening bilateral shoulder and hip pain.
(B) Add hydroxychloroquine He currently has difficulty rising from a chair and reach­
ing overhead because of the pain. He also reports fatigue,
(C) Add mycophenolate mofetil malaise, and 4.5-kg (10-lb) weight loss during this period.
(D) Add a scheduled NSAID He reports no other symptoms. He takes acetaminophen as
needed for pain with little or no relief.
On physical examination, the patient appears
Item 42 depressed. Temperature is 37.9 °C (100.2 °F), blood pressure
A 65-year-old man is evaluated during a follow-up visit is 126/66 mm Hg, pulse rate is 72/min, and respiration rate is
for gout. He initially presented 6 months ago with acute 18/min. BM! is 26. 1here is markedly limited range of motion
pain and swelling of the right great toe and a serum urate of the shoulders and hips due to pain; strength cannot be

103
Self-Assessment Test

adequately assessed. The remainder of the physical exam­ narrowing. Plain radiographs of the knees show medial and
ination is normal. lateral joint-space narrowing.
Laboratory studies, including basic metabolic panel,
Which of the following is the most appropriate next step in
complete blood count, liver chemistries, and thyroid­
management?
stimulating hormone level, are normal. Erythrocyte sedi­
mentation rate is 68 mmlh. (A) Add etanercept
Which of the following is the most appropriate treatment? (B) Add rituximab
(C) Increase methotrexate
(A) Aspirin, 81 mg/d
(D) Increase prednisone
(B) Aspirin, 650 mg three times daily
(C) Duloxetine, 60 mg/d
(D) Prednisone, 15 mg/d Item 47
(E) Prednisone, 60 mgld A 39-year-old man is evaluated for a lower extremity rash
of 3 weeks' duration. He has no recent history of a cold, flu,
or other infection. He takes no medications.
Item 45 On physical examination, temperature is 37.3 °C
(99.2 °F), blood pressure is 136/86 mm Hg, pulse rate is 66/
A 45-year-old man is evaluated for a 2-week history of pro­
min, and respiration rate is 12/min. BM! is 24. Small vascular
gressive pain and swelling of the third and fourth toes of the
infarctions are observed on the ears and fingertips. There are
right foot. He has a rash on the soles of his feet that appeared
scattered palpable purpuric lesions on the bilateral lower
4 weeks ago. He reports no fever, back pain, chest pain,
extremities, which are less prominent on the soles. Strength
dyspnea, dysuria, diarrhea, ocular problems, oral ulcers,
is reduced in the right wrist.
Raynaud phenomenon, psoriasis, or photosensitivity. He is
sexually active. He takes no medications. Laboratory studies:
On physical examination, vital signs are normal. There Erythrocyte 66 mm/h
is no nail pitting. Dactylitis and diffuse swelling of the right sedimentation rate
third and fourth toes are noted. The soles of the feet have C3 Normal
yellow-brown vesicles and hyperkeratotic nodules with C4 Decreased
overlying keratotic crust. The remainder of the examination Creatinine 2.1 mg/dL (185.6 µmol/L)
is normal. Rheumatoid factor Positive
Laboratory studies reveal an erythrocyte sedimenta­ Hepatitis C antibodies Positive, genotype 2
tion rate of 35 mm/h; complete blood count with differential Serum protein Monoclonal spike in IgG
and urinalysis are normal. electrophoresis band
Radiographs of the toes reveal diffuse soft-tissue swell­ Urinalysis Positive for erythrocytes,
ing of the right third and fourth toes but are otherwise leukocytes, erythrocyte casts
normal.
Which of the following is most likely to establish the
Which of the following is the most appropriate diagnostic diagnosis?
test to perform next?
(A) Anti-cyclic citrullinated peptide antibody levels
(A) Anti-cyclic citrullinated peptide antibody assay (B) Anti-glomerular basement membrane antibody levels
(B) Antinuclear antibody assay (C) Antinuclear antibody levels
(C) DNA amplification urine test for Chlamydia (D) p-ANCA levels
trachoma tis (E) Serum cryoglobulin levels
(D) HLA-B27 testing

Item 48
Item 46 A 71-year-old woman is evaluated during an office visit.
A 52-year-old man is evaluated during a follow-up visit for Four months ago, she fell on an outstretched hand. During
a 2-year history of progressively symptomatic rheumatoid the next several weeks, she noted gradual pain, stiffness,
arthritis. He reports increased difficulty with his job due and swelling of her right shoulder; the pain occurs with
to persistent pain and swelling in the first proximal inter­ movement and at night. History is significant for knee
phalangeal joints, second and third metacarpophalangeal osteoarthritis, gout, and hypertension. Medications are
joints, and bilateral wrists. He also has increased difficulty acetaminophen, colchicine, allopurinol, and lisinopril.
climbing stairs due to persistent pain and swelling in the On physical examination, vital signs are normal. BM! is
right knee. Medications are methotrexate, 25 mg weekly; 25. The right shoulder has a large effusion without warmth
prednisone, 10 mg/d; naproxen; and folic acid. or overlying erythema; range of motion is limited by pain
On physical examination, vital signs are normal. There and swelling. and prominent crepitus is palpable with
isl+ tenderness to palpation andl+ swelling of the affected motion.
joints. Erythrocyte sedimentation rate, leukocyte count,
Plain radiographs of the hands and wrists show periar­ (-reactive protein level, and serum urate level are within
ticular osteopenia, multiple erosions, and carpal joint-space normal limits.

104
Self-Assessment Test

A radiograph of the shoulder is shown. (C) Rapid streptococcal antigen testing


(D) Synovial fluid analysis

Item 50
A 26-year-old woman seeks preconception counseling. She
has a 3-year history of rheumatoid arthritis. Medications
are methotrexate, hydroxychloroquine, low-dose predni­
sone, and folic acid. Currently her disease is under excellent
control.
On physical examination, vital signs are normal. There
is no warmth, erythema, swelling, or tenderness of the
joints.
Which of the following is the most appropriate next step in
management?
(A) Discontinue hydroxychloroquine
(B) Discontinue methotrexate
(C) Discontinue prednisone
(D) Discontinue prednisone, methotrexate, and hydroxy­
Aspiration of the right shoulder shows blood-tinged chloroquine
synovial fluid with a leukocyte count of 8300/µL (8.3 x 109/L);
Gram stain is negative, and there are no crystals.
Which of the following is the most likely diagnosis? Item 51
A 52-year-old man is evaluated for a 4-month history of
(A) Acute calcium pyrophosphate crystal arthritis slowly progressive unilateral proptosis. He reports enlarge­
(B) Acute gouty arthritis ment of the glands under his jaw on both sides. He generally
(C) Basic calcium phosphate deposition feels well and has no other medical problems. He takes no
(D) Infectious arthritis medications.
On physical examination, vital signs are normal.
Marked proptosis of the left eye is noted; there is no inflam­
mation of the sclerae or conjunctivae. There is bilateral
Item 49
enlargement of the lacrimal, parotid, and submandibular
A 23-year-old woman is evaluated for a 1-week history glands. There is an enlarged lymph node at the angle of
of fevers, malaise, aches of her elbows and left knee, and the jaw on the right. The remainder of the examination is
pain of her wrists, hands, and ankles that is worse with normal.
movement. She has noticed transient skin lesions resem­ Laboratory studies include a normal complete blood
bling pustules. She reports no urinary symptoms or vaginal count with differential, chemistry panel, liver chemistries,
discharge. She has no history of illicit drug use; she drinks antinuclear antibody panel, and urinalysis.
alcohol socially and is sexually active. She has not traveled MRI of the head and orbits demonstrates a homoge­
recently. Her only medication is an oral contraceptive. neous enhancing mass behind the left eye and enlargement
On physical examination, the patient appears uncom­ of the parotid and submandibular glands. Biopsy of the
fortable. Temperature is 38.3 °C (100.9 °F), blood pressure is ocular mass demonstrates a lymphoplasmacytic infiltrate
115/75 mm Hg, pulse rate is 95/min, and respiration rate is with storiform fibrosis and obliterative phlebitis, rare neu­
12/min. BM! is 23. Ulnar deviation of the wrists while hold­ trophils, and no granulomas; a monoclonal population of
ing the thumbs down elicits pain over the radial side of the cells is not identified.
wrists (positive Finkelstein test). There is generalized swell­
ing of several fingers with tenderness to palpation diffusely. Which of the following is the most likely diagnosis?
The left knee is warm, with a small effusion and pain with (A) Hodgkin lymphoma
range of motion. The ankles have tenderness to palpation
over the posterior aspects, with erythema along the Achilles (B) lgG4-related disease
tendons and pain with range of motion bilaterally. There (C) Sarcoidosis
are a few scattered vesiculopustular lesions over the palmar (D) Sjogren syndrome
aspect of the hands and upper extremities.
Laboratory studies reveal a leukocyte count ofl3,500/µL
(13.5 x 10 9/L) and normal kidney and liver chemistries. Item 52
Which of the following is most likely to provide the A 58-year-old woman is evaluated
f
for a 2-year history of
diagnosis? hand pain and increasing dif iculty using her hands. She
reports worsening grip strength as well as increasing pain
(A) Cervical culture in the distal interphalangeal (DIP) and proximal inter­
(B) Hepatitis B serologies phalangeal (PIP) joints. She has intermittent erythema

105
Self-Assessment Test

and swelling in some of these joints (such as the fifth DIP Item 54
joint and the right second PIP and left third PIP joints) A 42-year-old woman is evaluated for a 6-month history
sometimes lasting for weeks at a time. She takes naproxen of pain and swelling of several small hand joints, an elbow,
twice daily. and an ankle. She gets modest relief with naproxen. She
On physical examination, vital signs are normal. BM! has no other medical problems and takes no additional
is 29. Bony hypertrophy and malalignment of the DIP joints medications.
are noted; there is mild erythema over the right fifth DIP On physical examination, vital signs are normal. There
joint. There are bony hypertrophy of the PIP joints and are tenderness to palpation and swelling of the second and
swelling and tenderness of the right second PIP joint and third proximal interphalangeal joints bilaterally, second
the left third PIP joint. There is bony hypertrophy of the first and fifth metacarpophalangeal joints bilaterally, left wrist,
carpometacarpal (CMC) joint bilaterally. right elbow, and right ankle. The remainder of the physical
Laboratory studies, including a complete blood count, examination is normal.
erythrocyte sedimentation rate, C-reactive protein, serum Laboratory studies are significant for a rheumatoid
creatinine, rheumatoid factor, anti-cyclic citrullinated anti­ factor of 85 U/mL (85 kU/L) and positive anti-cyclic citrul­
bodies, and urinalysis, are normal. linated peptide antibodies.
Plain hand radiographs show central erosions and col­ Radiographs of the hands and wrists show periarticular
lapse of the subchondral bone in the right second PIP joint osteopenia at the metacarpophalangeal joints and a mar­
and the left third PIP joint; osteophytes at the second, third, ginal erosion at the right second metacarpal head.
and fifth DIP joints bilaterally; and joint-space narrowing
at the first CMC joint bilaterally. There is no periarticular Which of the following is the most appropriate initial
osteopenia or marginal erosions. treatment?
Which of the following is the most likely diagnosis? (A) Hydroxychloroquine
(A) Erosive osteoarthritis (B) Methotrexate
(B) Reactive arthritis (C) Rituximab
(C) Rheumatoid arthritis (D) Tofacitinib
(D) Tophaceous gout
Item 55
Item 53 A 40-year-old woman is evaluated for a 7-year history
of color changes associated with pain that occurs in her
A 44-year-old man is evaluated for a 2-month history of fingers. Her second and third fingertips turn white in
color change in the hands when exposed to the cold, finger the cold, then become blue, and eventually become dark
skin tightness with palpable nodules, fatigue, and pruritus. red and painful. These symptoms last approximately
He has no other medical problems and takes no medications 15 minutes before resolving. She also reports a 3-month
or nutritional supplements. history of pain and swelling in the second and third
On physical examination, vital signs are normal. BM! is metacarpophalangeal joints. History is also significant
23. Skin thickening of the face and fingers is noted. There are for dry eyes and dry mouth of 5 years' duration as well
a few firm, gritty nodules on the palmar aspect of the digits. as recent onset of diffusely puffy hands and increas­
There is no rash or digital pits. ing fatigue. She reports no gastrointestinal symptoms,
The appearance of the hands is shown. including gastroesophageal reflux disease. She takes no
medications.
On physical examination, vital signs are normal. No
rash or oral ulcers are noted. Slightly cool, diffusely edema­
tous fingers are noted. Scattered palmar telangiectasias are
present. There is swelling and tenderness of the second and
fourth metacarpophalangeal joints.
Laboratory studies:
C3 Normal
C4 Normal
Creatine kinase 596 U/L
Creatinine Normal
Antinuclear antibodies Positive (titer: 1:320)
Anti-double-stranded DNA Negative
antibodies
Anti-Ro/SSA antibodies Negative
Which of the following is the most likely diagnosis? Anti-La/SSB antibodies Negative
(A) Eosinophilia myalgia syndrome Anti-Scl-70 antibodies Negative
Anti-Smith antibodies Negative
(B) Limited cutaneous systemic sclerosis An ti-Ul -ribon ucleoprotein Positive
(C) Morphea antibodies
(D) Primary biliary cirrhosis Urinalysis Negative

106
Self-Assessment Test

Which of the following is the most likely diagnosis? A chest radiograph is shown.
(A) Mixed connective tissue disease
(B) Polymyositis
(C) Systemic lupus erythematosus
(D) Systemic sclerosis
(E) Undifferentiated connective tissue disease

Cl Item 56
A 28-year-old vvoman is evaluated in the emergency
department for a 1-clay history or progressive shortness of
breath. cough. and hemoptysis. She reports a fever but no
chills. She has a 2-year history of"systemic lupus erythema­
tosus. Medications are mycophenolate motetii. hyclroxy­
chloroquine. preclnisone. naproxen as needed. vitamin D.
and calcium.
On physical examination. temperature is 38.9 °C
(102.0 °F). blood pressure is 100/60 111111 Hg. pulse rate is 110
min. and respiration rate is 24 min. Oxygen saturation is
88'Y., on ambient air. BM I is 29. Diffuse hair thinning is noted.
A malar rash is present. ·n1ere is symmetric svvelling or
metacarpophalangeal and proximal interphalangeal joints
as well as both wrists and knees. ·n1ere are no cardiac rubs or
murmurs. Diffuse crackles are heard on lung auscultation.
Laboratory studies:
Hematocrit 22'1.. (30'Y., in oflicc I week ago)
Leukocyte count 3200/pL (3.2 x IO'' L)
Platelet count 90.000 pl (90 X 10'' L) Which of the following is the most appropriate treatment?
Creatinine 1.3 mg ell (115 pmol L)
Urinalysis 3+ blood: 2+ protein: erythrocyte (A) Adalimumab
casts (B) High-dose prednisone
(C) Methotrexate
Chest radiograph reveals diffuse bila tera I pulmonary
(D) Naproxen
infiltrates with sparing of" the apices.
Which of the following is most likely to establish a diag­
nosis? Item 58
A 35-year-old man is evaluated for a 4-week history of per­
(A) Bronchoalveolar lavage and biopsy
sistent pain and swelling in the left knee and right ankle.
(B) Chest CT Symptoms are worse in the morning and associated with
(C) MRl of"the chest stiffness lasting 1 hour. Ibuprofen is beneficial. Seven weeks
(D) Pulmonary angiography ago he had diarrhea that lasted one week and resolved
without treatment. One week ago he was diagnosed with
anterior uveitis, which is resolving with a prednisolone
Item 57 ophthalmic solution. He reports no current gastrointesti­
A 24-year-old woman is evaluated for a 1-week history of nal or genitourinary symptoms, rash, or cardiorespiratory
tender nodules over the legs about 2 to 3 cm in size along symptoms.
with pain and stiffness in the ankles. She also notes a non­ On physical examination, vital signs are normal.
productive cough of 3 days' duration. Her only medication Slightly injected sclera of the right eye is noted. There are
is an oral contraceptive. swelling. warmth, and tenderness of the right knee joint.
On physical examination, temperature is 38.4 °C (101.1 Tenderness and swelling at the right Achilles tendon inser­
°F), blood pressure is 110/65 mm Hg, pulse rate is 85/min, tion to the calcaneus are noted. There is no rash or nail pit­
and respiration rate is 18/min. BM! is 24. There are four3-cm ting. The remainder of the physical examination is normal.
erythematous tender nodules on the left anterior lower leg Laboratory studies done at the time of the visit:
and three on the right. Swelling of both ankles with tender­ Complete blood count with Normal
ness at the right Achilles tendon insertion into the calca­ differential
neus is noted; no other joints are swollen. Cardiopulmonary Erythrocyte sedimentation rate 30 mm/h
examination is normal. Stool cultures Negative
Laboratory studies reveal an erythrocyte sedimenta­ Urinalysis Normal
tion rate of38 mm/h and a hematocrit of35%; leukocyte and DNA amplification urine test for Negative
platelet counts are normal. Chlamydia trachomatis

107
Self-Assessment Test

Aspiration of the left knee is performed; synovial fluid (C) Rituximab


analysis reveals a leukocyte count of 15,000/µL (15 x 109/L) (D) Sulfasalazine
with 70% monocytes and no crystals. Gram stain is negative,
and cultures are pending.
Radiographs of the knee and ankle are normal. Item 61
Which of the following is the most likely diagnosis? A 28-year-old woman is evaluated for a 2-week history of
left hip pain that occurs at night and with walking. She has
(A) Infectious arthritis a 1-year history of systemic lupus erythematosus with class
(B) Psoriatic arthritis IV nephritis. Treatment has included prednisone as high as
(C) Reactive arthritis 60 mg/d and cyclophosphamide. Current medications are
(D) Rheumatoid arthritis mycophenolate mofetil, hydroxychloroquine, prednisone,
furosemide, lisinopril, calcium, and vitamin D.
On physical examination, vital signs are normal. BM! is
Item 59 26. She has a cushingoid appearance. There is pain on inter­
A 68-year-old man is evaluated during a follow-up visit for nal rotation of the left hip. There is no pain on hip adduction
long-standing tophaceous gout that caused almost monthly or with pressure over the lateral hip. The remainder of the
gout flares. Five months ago, colchicine and allopurinol examination is normal.
were initiated. He reached his serum urate goal (<6.0 mg/dL Laboratory studies reveal an erythrocyte sedimenta­
[0.35 mmol!L]) within 3 months of starting therapy, and he tion rate ofl8 mm/h, normal complement (C3 and C4) lev­
has not had a gout flare since reaching this target. He has els, and stable anti-double-stranded DNA antibodies.
not developed any new tophi; his original tophi on his hands Anterior-posterior pelvis and lateral left hip radio­
and elbows have begun to shrink in size. History is also sig­ graphs are normal.
nificant for hypertension, for which he takes losartan.
On physical examination, temperature is 37.2 °c Which of the following is the most appropriate diagnostic
(98.9 °F), blood pressure is 133/79 mm Hg, pulse rate is 79/ test to perform next?
min, and respiration rate is 12/min. BM! is 28. A moder­ (A) CT of the left hip
ate-sized tophus is visible on the right elbow as well as two (B) MRI of the left hip
small tophi on different distal interphalangeal joints. There is
(C) Repeat plain radiography in 1 month
no swelling or tenderness to palpation of any joints.
Current laboratory studies reveal a serum urate level (D) Ultrasonography of the left hip
of 5.4 mg/dL (0.32 mmol/L) and normal kidney and liver
chemistries.
Which of the following is the most appropriate next step in
management?
Item 62
\ .:;s _\L".1r llld \\·0111.111 he,·,Jlu• .1ted in Lhe hospital lcira :3-rnonth Cl
lmt1 ,n 111 1;11 igLJL' ,111tl ,1 1 ..::; kg (IO lb) weight loss. She also has
(A) Add probenecid dn111.1t1J1m()'iith that \\·as diagnosed I ye,1r ago. al which time
(B) Change allopurinol to febuxostat she urnlcr\\'L'lll del,iilcd evaluation liJr rnyopathy and age­
(C) Continue colchicine and allopurinol ,1ppr<1p1·i,IIL' 111alig11ancy screening. She has also noticed wors­
L'11i11g 111t1sLIL- \\L".1km:,s and rnsh in Lhe past 2 weeks.
(D) Discontinue colchicine 011 ph:, ,iL.il L'\,1min,1tion. temperature is normal. blood
prL''iSUl'L' is I 18 tJ I rnrn I lg. pulse rate is 90 min. and rcspira-
Item 60 1 i1111 r.1le is I(> min. BJ\ 11 b 27. C1rdi,1c ,ind pulmonary exam­
i11.1lit111s ,ll'L' 11urn1,1I. .-\hdomin,il ex,1rnination reveals ascites
A 27-year-old man is evaluated for a 2-year history of anky­ \\ it h"ul 11rg,llllllllL'g:1h. ·1 here is syrnrnet ric we.:i kness 01· the
losing spondylitis. Symptoms were initially responsive .1 rn 1 ,1nd Ihigh 111u'icle,. ,\ viol.1ccou, rash is present on the
to physical therapy and naproxen; however, for the past
L'\ten,"1· ,ur1·. 1cL' ol till' rnetacarpoph,liangeal joints. A l'ew
6 months he has experienced increasing back and buttock .1 l'L',l'i lll p.1I p:1bk- purpur:1 on the lower ex I remities are nolecl.
pain and stiffness, with difficulty bending downward. One
month ago naproxen was discontinued, and indomethacin I .ahoratory studies:
was initiated but with no improvement. ( <Jillpk-tc hl\llld C()Llllt Normal
On physical examination, vital signs are normal. BM! ('llL'lllhtry p,111el Norn1.:il
is 25. Tenderness to palpation over the sacroiliac joints \ ILi, ,l.1\l' :i1 U L (norm,11 r,rnge.
and lower lumbar spine is noted. There is limited range of I 0-8.0 U I.)
motion of the lumbar spine manifested by an increase of \,p,1 r1 .11 c ,1111 inot rans lc:r,1SL' 98 U L
1 cm of change on forward flexion. ( l'l'.I[ illL' k illcl',l' 1-100 U L
Laboratory studies reveal an erythrocyte sedimenta­ . \ 11 ti llllL'iL·,1 r ,111ti hod ic, Tiler of I :6-10
tion rate of 40 mm/h. l'ri11:1h sis Normal
In addition to continuing physical therapy, which of the Which of the follov,1 ing is the most appropriate diagnostic
following is the most appropriate treatment? te\l to perform ncx!'I
(A) Adalimumab ( \J ( hL·,t CT 11 ith contrast
(B) Methotrexate I Ill I i, L'r lli1lpsy

108
Self-Assessment Test

Cl
CONT.
(C) PET scan
(D) Thigh muscle MRI
Item 64
A 34-year-old man is evaluated for progressive left knee
(E) Transvaginal pelvic ultrasonography pain. The pain causes difficulty with his work as a mail
carrier, particularly when walking. His occupation does
not require repetitive bending. He played football in college

Cl Item 63
and experienced left knee trauma during sports participa­
tion; he underwent left meniscectomy and stopped play­
A 52-year-old man is evaluated in the hospital for fever. mal­ ing sports. His mother has osteoarthritis of the hands that
aise, arthralgia, left foot drop, abdominal pain that is worse
developed at age 65 years.
arter eating, and a 4.5-kg (10-lb) weight loss, all of which On physical examination, vital signs are normal. BM!
gradually developed over the past 2 months.
is 27. Bone hypertrophy of the left knee is noted. There is
On physical examination. temperature is 38.4 °C
crepitus but no warmth, erythema, swelling, or effusion of
(101.1 °F). blood pressure is 154/92 mm Hg, pulse rate
the knees.
is 76/min, and respiration rate is 1 8/min. BM! is 29. Plain radiographs (anteroposterior views) show medial
There is no temporal or sinus tenderness. The nasal joint-space narrowing of both knees but greater on the left
passages and oropharynx are normal. The chest is clear. as well as osteophytes and bony sclerosis of the tibial pla­
An abdominal bruit is heard on the right. inferior to the teau of the left knee; there is no periarticular osteopenia or
costal margin. The abdomen is otherwise normal. The
erosive or destructive changes.
testicles are tender to palpation. The joints are tender
to palpation without synovitis. Weakness of the left foot Which of the following is the most likely cause of this
to dorsiflexion is noted. patient's left knee osteoarthritis?
The appearance of the trunk and thighs is shown.
(A) BM!
(B) Family history
(C) Meniscectomy
(D) Occupation

Item 65
A 62-year-old woman is evaluated in the emergency depart­ Cl
ment for right knee pain. Three clays ago she developed
increasing pain, swelling, warmth, and erythema of the
right knee as well as fevers and chills. She underwent total
knee replacement of her right knee 2 months ago.
On physical examination. temperature is 38.4 °C
(101.2 °F). blood pressure is ll0/75 mm Hg. pulse rate is 98/
min. and respiration rate is 12/min. BM! is 23. The right knee
has a well-healed surgical scar. but there are significant
knee joint swelling, warmth. tenderness to palpation. mild
overlying erythema, and decreased range of motion. The
remainder of the examination is normal.
Laboratory studies: Laboratory studies reveal a leukocyte count of
Erythrocyte sedimentation rate 72 mm/h 15.500/pL (15.5 x 109/L) with 89% neutrophils, a (-reactive
Complements (C3 and C4) Normal protein level of 6.8 mg/dL (68 mg/L). and an erythrocyte
Creatinine 2.2 mg/dL (194.5 sedimentation rate of 45 rnm/h.
pmol/L) Racliograph of the right knee shows only an effusion.
lgA Normal
Hepatitis B surface antigen Positive Which of the following is the most appropriate next step
Hepatitis B surface antibodies Negative in management?
Hepatitis C antibodies Negative
(A) Begin vancomycin and cefepime
ANCA Negative
Urinalysis Negative (B) Obtain blood and synovial fluid cultures
(C) Obtain a bone scan
Chest radiograph is unremarkable. Abdominal angio­ (D) Obtain a CT of the knee
gram shows aneurysms and stenoses of the mesenteric and
renal arteries.
Which of the following is the most likely diagnosis? Item 66
A 76-year-old man seeks advice regarding dietary modifi­
(A) Goodpasture syndrome
cations to help prevent gout flares. He recently experienced
(B) Granulomatosis with polyangiitis his first episode of podagra. At his initial visit, serum urate
(C) Henoch-Schonlein purpura level was 7.2 mg/dL (0.42 mmol/L). History is also signifi­
(D) Polyarteritis nodosa cant for hypertension, for which he takes losartan.

109
Self-Assessment Test

On physical examination, temperature is 37.1 °C Item 69


(98.8 °F), blood pressure is 135/80 mm Hg, pulse rate is 80/ A 52-year-old man is evaluated for a 6-month history of
min, and respiration rate is 15/min. BM! is 27. The remainder increasingly swollen and painful joints of the fingers of
of the examination is unremarkable. both hands, both wrists, and the left ankle associated with
In addition to meat restriction, increased intake of which 90 minutes of morning stiffness. He has tried over-the­
of the following may help to decrease this patient's risk of counter ibuprofen and naproxen without sustained benefit.
gout flares? He has no other symptoms.
On physical examination, vital signs are normal.
(A) Leafy green vegetables There are swelling and tenderness of the second, third,
(B) Low-fat dairy products and fifth proximal interphalangeal joints; first, second,
(C) Red wine and third metacarpophalangeal joints; both wrists; and
(D) Shellfish left ankle. Decreased range of motion of the right wrist
is noted. The remainder of the physical examination is
normal.
Item 67 Laboratory studies reveal an erythrocyte sedimen­
tation rate of 45 mm/h and a C-reactive protein level of
A 29-year-old woman is evaluated for increasing fatigue
5.2 mg/dL (52 mg/L}; rheumatoid factor and anti-cyclic
and diffuse pain of 6 months' duration. l11e pain becomes
citrullinated peptide antibody tests are negative.
more severe for several days if she ''overdoes it." She
Hand radiographs show an erosion of the second right
reports chronically poor sleep and has difficulty concen­
metacarpal head with mild symmetric joint-space narrow­
trating at work. History is also significant for hypothy­
ing and mild periarticular osteopenia of the metacarpopha­
roidism, for which she takes levothyroxine. She takes
langeal joints; there is no bony sclerosis or osteophytes.
ibuprofen as needed for the pain, which provides minimal
benefit. Which of the following is the most likely diagnosis?
On physical examination, vital signs are normal.
BM! is 25. Tenderness to palpation of multiple muscle (A) Osteoarthritis
groups is noted. Muscle strength is normal. There is no (B) Rheumatoid arthritis
joint swelling or rash. l11e remainder of the examination (C) Sarcoidosis
is normal. (D) Systemic lupus erythematosus
Laboratory studies, including complete blood count,
chemistry panel, erythrocyte sedimentation rate, and thy­
roid-stimulating hormone, are normal. Item 70
Which of the following is the most appropriate next step in A 41-year-old man is evaluated for a 1-month history of
management? daily fever as high as 39.0 °C (102.2 °F), a 4.5-kg (10.0-lb)
weight loss, myalgia, and swollen lymph nodes. He reports
(A) Begin scheduled ibuprofen
joint pain and stiffness in the shoulders, hands, wrists, and
(B) Increase levothyroxine knees. He has also noted a pink rash over the trunk and
(C) Obtain an antinuclear antibody panel extremities associated with the fever.
(D) Start an aerobic exercise program On physical examination, temperature is 38.4 °C
(101.1 °F), blood pressure is 128/78 mm Hg, pulse rate is
100/min, and respiration rate is 18/min. BM! is 28. Multiple
Item 68 enlarged lymph nodes in the anterior cervical chain are
A 75-year-old man is evaluated for gradual progression present. Splenomegaly is noted. There is an erythematous
of right knee pain. He has a 10-year history of right knee maculopapular rash on the trunk and extremities. Swelling
osteoarthritis, which was previously controlled with acet­ of the wrists and knees is present. The remainder of the
aminophen. He recently discontinued the acetaminophen examination is normal.
because of continued pain and began over-the-counter oral Laboratory studies:
naproxen, with good results. He can walk again without Erythrocyte sedimentation rate 100 mm/h
pain or difficulty. He has no other medical problems and Hematocrit 31%
takes no other medications. Leukocyte count 30,000/µL (30 x 109/L},
On physical examination, blood pressure is 124/82 with 80% neutrophils;
mm Hg. l11e right knee demonstrates evidence of bony no blasts
enlargement and crepitus on flexion and extension; no Platelet count 350,000/µL (350 x 109/L}
warmth, tenderness, or effusion is noted. Alanine aminotransferase 68 U/L
Aspartate aminotransferase 75 U/L
Which of the following is the most appropriate next step in C-reactive protein 30 mg/dL (300 mg/L}
management? Creatinine 0.9 mg/dL (79.6 µmol/L}
(A) Continue oral naproxen Ferritin 20,000 ng/mL (20,000
(B) Discontinue oral naproxen: begin celecoxib µg/L}
Urinalysis Normal
(C) Discontinue oral naproxen; begin a topical NSAID
(D) Refer for knee joint replacement Chest radiograph is normal.

110
Self-Assessment Test

Which of the following is the most likely diagnosis? Laboratory studies reveal a serum creatinine level of
1.6 mg/dL (141.4 µmol/L).
(A) Acute myeloid leukemia
Radiographs of the right knee show mild medial joint­
(B) Adult-onset Still disease space narrowing, subchondral sclerosis of the same region,
(C) Granulomatosis with polyangiitis and small osteophytes at the medial femoral and tibial joint
(D) Systemic lupus erythematosus margins.
In addition to an exercise program, which of the following
is the most appropriate initial treatment?
Item 71
A 29-year-old man is evaluated for pain and photophobia (A) Acetaminophen
in the left eye that began 3 days ago. He reports a 6-month (B) Celecoxib
history of recurrent painful oral and genital ulcers that last (C) Colchicine
1 to 2 weeks and then resolve, as well as waxing and waning (D) lhuprofen
knee, ankle, and wrist pain during this time. Medical history
had been unremarkable until the onset of these symptoms,
and he takes no medications. Item 73
On physical examination, temperature is 38.2 °C A 24-year-old woman is evaluated for a 6-month history of
(100.7 °F), blood pressure is 134/82 mm Hg, pulse rate is 90/ color change in her hands and feet, which is worse during
min, and respiration rate is 14/min. BMI is 22. 1l1e left eye is stress. She is a nonsmoker. Family history is negative.
diffusely erythematous, and a small amount of whlte fluid On physical examination, temperature is normal,
on the bottom of the anterior chamber is noted. There is an blood pressure is 116/72 mm Hg, pulse rate is 64/min, and
ulcer on the right side of the tongue. respiration rate is 12/min. BMI is 22. There is mild reversible
The genital ulcers are shown. discoloration of the fingertips upon exposure to cold. Cool
tips of the fingers without digital pits are noted. Cardio­
pulmonary examination is normal. Muscle strength and
reflexes are normal. 1l1ere is no rash. The remainder of the
examination is normal.
Laboratory studies, including complete blood count
and chemistry panel, are normal. Nailfold capillary exam­
ination is normal.
Which of the following is the most appropriate next step in
management?
(A) Measure antinuclear and anti-Ul-ribonucleoprotein
antibodies
(B) Measure antiphospholipid antihody panel and cryo­
globulins
Laboratory studies show an erythrocyte sedimentation
rate of 76 mm/h as well as a normal complete blood count (C) Obtain digital arteriography
and metabolic profile. (D) Clinical observation
The patient is urgently referred to an ophthalmologist.
Which of the following is the most likely diagnosis? Item 74
(A) Behc;et syndrome A 64-year-old man is evaluated in the emergency depart­
ment for progressively deteriorating mental status. His
(B) Cytomegalovirus infection
wife states that he has been experiencing episodic head­
(C) Herpes simplex virus type 1 infection aches during the past several months, and his mental sta­
(D) Reactive arthritis tus has changed progressively over the past several clays.
History is significant for atherosclerosis. hypertension,
and coronary artery disease. He has a 40-pack-year history
Item 72 of smoking. Medications are atorvastatin, lisinopril, and
A 66-year-old man is evaluated for a 2-month history of low-close aspirin.
right knee pain. The pain is worse with walking and is On physical examination. the patient is alert and ori­
accompanied by approximately 5 minutes of morning ented to self but not to place or year. Vital signs are normal.
stiffness. Medical history is significant for chronic kidney BMl is 26. On the Mini-Mental State Examination, he is
disease, hypertension, and mild gastroesophageal reflux unable to do serial sevens. is able to recall only one object
disease. Medications are lisinopril, hydrochlorothiazide, out of three. and cannot draw a geometric figure that is
and ranitidine as needed for heartburn. shown to him. The remainder or the examination, including
On physical examination, vital signs are normal. BMI is the neurologic assessment. is within normal limits.
29. Medial joint line tenderness and mild crepitus are noted Laboratory studies. including complete blood count.
in the right knee. There is no redness, warmth, or instability basic metabolic panel. liver chemistries, and urinalysis, are
of the affected joint; minimal swelling is noted. normal; erythrocyte sedimentation rate is 22 mm/h.

111
Self-Assessment Test

Cl
CONT.
Lumbar puncture is performed: cerebrospinal fluid
analysis reveals a leukocyte count of 15/µL (15 x 106/L). 90%
lymphocytes. and a protein level of 45 mg/dL (450 mg/L).
disease, hypertension, chronic kidney disease (estimated
glomerular filtration rate of 55 mL/min/1.73 m2), and non­
alcoholic fatty liver disease. Current medications are dilti­
Chest radiograph is unremarkable. A brain MRI shows azem and azathioprine, which he has been taking for the
scattered lesions. mainly in the white matter. and an MR past 9 months.
angiogram shows possible narrowing of the intracerebral On physical examination, temperature is 37.1 °C
arteries. (98.8 °F), blood pressure is 125/70 mm Hg, pulse rate is 80/
min, and respiration rate is 12/min. BM! is 28. The exam­
Which of the following is the most appropriate next step
ination is unremarkable, including no joint abnormalities.
in management?
Which of the following is a contraindication to the use of
(A) Initiate azathioprine febuxostat in this patient?
(B) Initiate cyclophosphamide and glucocorticoids
(C) Obtain functional MRI (A) Azathioprine
(D) Obtain intracerebral angiography and brain biopsy (B) Diltiazem
(C) Mild to moderate chronic kidney disease
(D) Nonalcoholic fatty liver disease
Item 75
A 45-year-old woman is evaluated for a 2-week history of
Item 77
C]
nausea, right upper quadrant abdominal pain and fullness,
and malaise. She has rheumatoid arthritis that was diag­ A 28-year-old woman is evaluated in the emergency depart­
nosed 2 years ago. Initial treatment with methotrexate lost ment for a 3-week history of progressively worsening pain
its efficacy after 6 months, and she was switched to leflun­ in the left arm. ll1e pain worsens with use of the arm. She
omide. She had partial response to leflunomide and was also notes fatigue. malaise. and the inability to walk long
started on etanercept in combination. Other medications distances due to discomfort in her legs. She reports no
are sulfasalazine and naproxen. In the past 6 months, she cough. nausea. vomiting. or burning on urination. She takes
has had no active swollen or tender joints. no medications.
On physical examination, vital signs are normal. BM! On physical examination, temperature is 38.l °C
is 28. Icterus is noted. The liver is palpable with slight ten­ (100.5 °F), blood pressure is 166/95 111111 Hg in the right arm
derness. Murphy sign is negative. The remainder of the and ll5/56 111111 Hg in the left arm, pulse rate is 72/min. and
examination is normal. respiration rate is 14/min. BMI is 27. Pallor of the fingertips
Laboratory studies: and delayed capillary refill of the nail beds are noted in the
Hemoglobin 11.1 g/dL (111 g/L) left hand. A diminished radial pulse of the left arm and
Leukocyte count 12,500/µL (12.5 x 109/L) decrea ed dorsalis pedis pulses bilaterally are noted. A bruit
Alkaline phosphatase 162 U/L is heard over the mid abdomen. ll1ere is no rash.
Alanine aminotransferase 73 U/L Laboratory studies:
Aspartate aminotransferase 81 U/L Erythrocyte sedimentation rate 115 mm/h
Total bilirubin 2.6 mg/dL (44.5 µmol/L) Creatinine 1.3 mg 'dL (115 µmol/L)
Hepatitis B serologies Negative Partial thromboplastin time Normal
Hepatitis C serologies Negative Prothrombin time Normal
D-dimer Negative
Abdominal ultrasound shows multiple gallstones, no
Urinalysis Normal
thickening of the gallbladder, and normal extrahepatic bile
ducts. Which of the following is the most appropriate diagnostic
Which of the following is the most appropriate next step in test to perform next?
management? (A) Antimyeloperoxidase antibody assay
(A) Discontinue etanercept (B) Antiphospholipid antibody assay
(B) Discontinue leflunomide (C) Aortic arteriography
(C) Schedule a cholecystectomy (D) Temporal artery biopsy
(D) Schedule a liver biopsy

Item 78
Item 76 A 28-year-old woman seeks preconception counseling.
A 55-year-old man is evaluated during a fo llow-up visit for She has a 5-year history of systemic lupus erythematosus,
gout. Two years ago, he had been treated with allopurinol which initially presented with nephritis, rash, and arthri­
and developed a hypersensitivity reaction. Over the past tis. Her disease has been well controlled for 1 year with
several months, he has had recurrent attacks of acute, hydroxychloroquine, mycophenolate mofetil, and predni­
episodic swelling of the first metatarsophalangeal joints sone, 5 mg/d.
with increasing involvement of other joints, including On physical examination, vital signs are normal. BM! is
the ankles and knees. Laboratory studies showed signifi­ 28. There is a discoid rash on the ear pinna, unchanged since
cant hyperuricemia. History is also significant for Crohn the last examination. No other rashes or ulcers are noted.

112
Self-Assessment Test

The remainder of the examination, including cardiopulmo­ drop. Reflexes are normal. The remainder of the examina­
nary examination, is normal. tion is normal.
Laboratory studies, including complete blood count, Laboratory studies indicate that her SLE appears to be
chemistry panel, liver chemistries, complement levels, and active with an elevation of erythrocyte sedimentation rate
urinalysis, are normal. compared with baseline, leukopenia, and anemia typical of
her previous SLE flares.
Which of the following is the most appropriate next step in
management? Which of the following is the most appropriate next step in
management?
(A) Discontinue hydroxychloroquine
(B) Discontinue mycophenolate mofetil (A) Discontinue hydroxychloroquine
(C) Discontinue prednisone (B) Obtain electromyography/nerve conduction studies
(D) Continue current regimen (C) Obtain MRI of the cervical spine
(E) Stop all medications (D) Obtain skin biopsy for small-fiber neuropathy

Item 79 Item 81
A 42-year-old woman is evaluated for a 4-month history A 40-year-old man has a 15-year history of well-controlled
of progressive right hip pain. Plain radiographs obtained chronic plaque psoriasis and psoriatic arthritis and is now
2 months ago showed early osteoarthritis. Her symptoms evaluated for a severe flare of both the skin and joint disease.
have steadily worsened and are now limiting mobility. She One month ago, he developed severe pain and swelling of
reports no recent trauma or injuries, fevers, or pain of other the hands, elbows, knees, ankles, and toes; symptoms have
joints. She is originally from India. She has lived in the been unresponsive to ibuprofen. He also developed sudden
United States for 15 years but visits her family abroad occa­ worsening of psoriasis over the trunk and extremities. He
sionally. She reports no injection drug use and is not sexu­ notes increased fatigue and intermittent lymphadenopathy
ally active. She was found to be positive for latent tubercu­ in the neck for the past 3 months. He has no other symp­
losis infection several years ago and underwent standard toms. His only medication is sulfasalazine.
treatment at that time. History is significant for rheumatoid On physical examination, temperature is 37.8 °C (100.0
arthritis, for which she takes methotrexate and folic acid; °F), blood pressure is 130/85 mm Hg, and pulse rate is 80/min.
she also started etanercept 3 months ago. Swelling and tenderness of the bilateral elbows, wrists, proxi­
On physical examination, temperature is 37.1 °C mal interphalangeal joints, knees, ankles, and metatarsopha­
(98.8 °F), blood pressure is 135/80 mm Hg, pulse rate is langeal joints are noted. Oropharyngeal candidiasis is present.
75/min, and respiration rate is 12/min. BM! is 23. Range of Cervical lymphadenopathy is noted bilaterally. Except for the
motion of the right hip is limited by pain without overlying skin, the remainder of the physical examination is normal.
erythema or warmth. There are no visible joint or skin abnor­ The appearance of the skin is shown.
malities. There are no heart murmurs, and the lungs are clear.
Laboratory studies reveal an erythrocyte sedimenta­
tion rate of 40 mm/h; complete blood count and kidney and
liver chemistries are normal.
f
Radiograph of the right hip reveals an ef usion and new
erosive changes.
Arthrocentesis of the right hip is performed.
Which of the following is the most likely cause of this
patient's hip pain?
(A) Gout
(B) Mycobacterium tuberculosis infection
(C) Neisseria gonorrhea infection
(D) Rheumatoid arthritis

Item 80
A 35-year-old woman is evaluated for weakness in the
right foot and left wrist with paresthesia in the right leg,
right foot, left forearm, and left hand. She also reports facial
erythema and joint stiffness. She has a 6-year history of sys­
temic lupus erythematosus (SLE). Medications are hydroxy­
chloroquine, prednisone, vitamin D, and calcium.
On physical examination, vital signs are normal. There
is a new malar rash. Swelling of the second through fourth
metacarpophalangeal joints of the hands is present. There
is dorsiflexion weakness of the right ankle and a left wrist

113
Self-Assessment
--- ------- Test

Which of the following is the most appropriate diagnostic Sinus radiogrnph shows bony erosion of the sep
test to perform next? tum and turbinates. Chest radiograph shows diffuse infil­
trates.
(A) Heterophile antibody testing
(B) HIV antibody testing Which of the following is most likely to establish the
(C) HLA-B27 testing diagnosis?
(D) Lyme antibody testing (,\) Anti-double stranded DNA antibody levels
(E) Rapid streptococcal testing (13) Antimyeloperoxiclase antibody levels
(C) /\ntiproteinase 3 antibody levels
Item 82 (I)) Serum cryoglobulin levels

A 28-year-old woman is evaluated for a 1-week history of


pain and morning stiffness in her hands. Three weeks ago, Item 84
she had muscle aches, malaise, fevers, and coryza, all of
A 31-year-old woman is evaluated during a follow-up visit
which have resolved. She is an elementary school teacher;
for systemic lupus erythematosus. She was diagnosed
prior to her initial illness. several children in her class had
6 months ago after presenting with a malar rash, pericardi­
similar symptoms accompanied by an erythematous rash
tis, and arthritis. She was initially treated with prednisone,
on the cheeks. She does not have other pertinent personal
40 mg/ct, and hydroxychloroquine with good control of
or family history, and she takes no medications.
symptoms. The prednisone was subsequently tapered to the
On physical examination, temperature is 37.3 °c
current dose of 5 mg/cl.
(99.2 °F), blood pressure is 120/78 mm Hg, pulse rate is 66/
On physical examination, temperature is normal,
min, and respiration rate is 13/min.BM! is 22. Symmetric wrist,
blood pressure is 130/92 mm Hg, pulse rate is 90/min, and
metacarpophalangeal, and proximal interphalangeal joint ten­
respiration rate is 16/min. BM! is 27. There is edema of the
derness and pain with motion are noted without significant
lower extremities to just above the ankles. There are no car­
joint swelling. The remainder of the examination is normal.
diac or pleural rubs. No rash is present.
Laboratory studies are significant for an erythrocyte
sedimentation rate of 38 mm/h. Laboratory studies:
One month ago Today
Which of the following is the most appropriate initial treat­ C3 Normal Decreased
ment? Normal
C4 Decreased
(A) Azithromycin Creatinine 0.7 mg/dL 1.3 mg/dL
(B) Ibuprofen (61.9 µmol/L) (115 µmol!L)
Anti-double- 225 U/mL 721 U/mL
(C) Interferon alfa
stranded DNA
(D) Prednisone antibodies
Urinalysis Trace eryth­ l+ erythrocytes;

Cl
rocytes; trace 2+ protein;
Item 83
protein 1 erythrocyte cast;
A 52-ye<ir old man is e\·aluated in the hospital for several no bacteria
episodes of' hemoptysis that developed over the past day. Spot urine 300 mglg 1200 mg/g
I le reports reeling well until about 3 weeks ago when he protein­
developed myalgia. arthralgia. occasional epistaxis. and creatinine ratio
diminished hearing. One week ago he developed a rnsh ancl
weakness in his right hand. History is otherwise unremark­ In addition to an increase in prednisone, which of the fol­
able. and he takes no medications. lowing is the most appropriate next step in management?
On physical examination. temperature is 38.0 °c
(A) Add methotrexate
(l00.-1 °F). blood pressure is 152 100 mm Hg. pulse rateis72 min.
anc! respiration rate is 2-1 min. Conjunctivitis is present in both (B) Repeat laboratory testing in 1 month
eyes. Decreased hearing in both ears is noted. Bilateral maxillary (C) Schedule kidney biopsy
sinus tenderness is present. Chest examination reveals diffuse (D) Schedule renal artery Doppler examination
rhonchi. ll1e 1ight hand has decreased grip strength. Palpable
purpurn ofthe bilateral lm,·cr extremities is present.
Laboratory studies: Item 85
Erythrocyte 8-1 mm h A 55-year-old man is diagnosed with hypertension. Other
seclimcntation rate than a single episode of podagra 6 months ago, his medical
Leukocyte count 12.300 pL (12.3 x JO" u. history is unremarkable. He takes no medications. Family
eosinophils <2",, history is notable for his father and brother who have gout
Complements (C3. C-1) Normal and hypertension.
Creatinine 2.1 mg dL (185.6 pmol I.) On physical examination, temperature is 36.6 °c
Urinalysis 3+ protein: SO erythrocytes (97.9 °F), blood pressure is 152/100 mm Hg, pulse rate is 82/
hpf': 20 leukocytes hpl': sev min. and respiration rate is 14/min. BM! is 24. The remainder
era! mixed cellular casts of the physical examination is unremarkable.

114
Self-Assessment Test

Laboratory studies are significant for normal blood Laboratory studies:


urea nitrogen, serum creatinine, and electrolyte levels; the Leukocyte count 3000/µL (3.0 x 109/L}, with
serum urate level is 7.9 mg/dL (0.47 mmol/L}.
900 lymphocytes
Which of the following antihypertensive drugs is the most Creatinine Normal
appropriate for this patient? Electrolytes Normal
Urinalysis 2+ protein; trace blood
(A) Hydrochlorothiazide
(B) Lisinopril Which of the following tests should be obtained next?
(C) Losartan (A) Anti-double-stranded DNA antibodies
(D) Metoprolol (B) Antinuclear antibodies
(C) Anti-Ro/SSA and anti-La/SSB antibodies
(D) Anti-Smith antibodies
Item 86
(E) Anti-Ul-ribonucleoprotein antibodies
A 71-year-old man is evaluated for severe tophaceous
gout. Colchicine has been effective in reducing flares

Cl
to approximately two in the past year. On initial eval­ Item 88
uation 1 year ago, serum urate level was 10.2 mg/dL
(0.60 mmol/L}. Allopurinol was initiated but subse­ A 31-year-old woman is evaluated in the hospital for head­
quently discontinued because of gastrointestinal intoler­ ache. blurred vision, and nausea occurring for the past
ance. He was switched to febuxostat, which was increased 12 hours. She has a 2-year history of diffuse cutaneous sys­
to maximum dose without success in reaching the serum temic sclerosis with recent worsening of Raynaud phenom­
urate goal of less than 6.0 mg/dL (0.35 mmol/L}. He enon that is treated with nifedipine.
recently had a gout flare of his right great toe, which has On physical examination, the patient is alert but is
nearly resolved. somnolent and has altered sensorium. Temperature is nor­
On physical examination, temperature is 36.6 °c mal. blood pressure is 150/92 mm Hg. pulse rate is 104/min.
(97.9 °F), blood pressure is 140/80 mm Hg, pulse rate is and respiration rate is 16/min. BM! is 22. Oxygen saturation
89/min, and respiration rate is 15/min. BMI is 32. Bulky is 95% on ambient air. Cardiopulmonary examination is
tophi are present over bilateral elbows, hands, and feet normal. Examination of the skin reveals diffuse skin thick­
with drainage of pasty material from a large tophus over the ening of the face. anterior chest. and distal extremities;
second metacarpophalangeal joint of the left hand. There sclerodactyly; and multiple healed digital pits. Neurologic
are mild swelling and tenderness to palpation over the right examination is nonfocal.
first metatarsophalangeal joint. Laboratory studies:
Laboratory studies reveal a serum urate level of Complete blood count Normal
10.9 mg/dL (0.64 mmol/L}, an estimated glomerular Albumin 3.0 g/dL (30 g/L)
filtration rate of 42 mL/min/1.73 m2, and a normal glu­ Bicarbonate 32 mEq/L (32 mmol/L}
cose-6-phosphate dehydrogenase level. Creatinine 4.2 rng/dL (371.3 µmol/L);
Which of the following is the most appropriate next step in baseline, 0.8 mg/dL (70.7
management? µmol/L)
Urinalysis 2+ protein: 3 erythrocytes/
(A) Add pegloticase hpf: 5 leukocytes/hpf;
(B) Start prednisone few granular casts
(C) Switch colchicine to anakinra Urine protein-creatinine 1200 rng/g
ratio
(D) Switch febuxostat to pegloticase
Chest radiograph is normal. Noncontrast CT of the head
is normal. MRI of the brain shows bilateral parietal lobe
white matter prominence.
Item 87
A 25-year-old woman is evaluated during a follow-up visit Which of the following is the most appropriate treatment?
for systemic lupus erythematosus. She was feeling well (A) Captopril
until 2 weeks ago when she developed increased fatigue and
(B) Cyclophosphamide
diffuse arthralgia. Medications are hydroxychloroquine and
ibuprofen as needed. (C) Methylprednisolone
On physical examination, temperature is 37.2 °C (D) Sildenafll
(99.0 °F), blood pressure is 140/80 mm Hg, pulse rate is
80/min, and respiration rate is 16/min. There is diffuse
alopecia of the scalp. Malar erythema is noted. Heart Item 89
sounds are normal, and the chest is clear. Examination A 55-year-old woman is evaluated for a 3-year history
of the abdomen is normal. Tenderness with minimal of gradual left knee pain. She reports increased difficulty
swelling of the proximal interphalangeal joints is present with stair cUmbing and an increase in pain over the past
bilaterally. Small effusions on both knees with pain on 6 months. She has no history of injury. She was prescribed
range of motion are noted. acetaminophen, 1000 mg three times daily, and an exercise

115
Self-Assessment Test

program 3 months ago but continues to have activity-limiting


symptoms. Family history is notable for her mother who
Item 91
A 35-year-old woman is evaluated in the hospital for a [:J
had a total knee replacement at the age of 65 years. 6-month history of worsening fatigue and a 3-week history
On physical examination, vital signs are normal. BMI is of progressive shortness of breath. Over the past 2 weeks
31. There is bony hypertrophy of the left knee and the first she has developed orthopnea and leg edema. Medical his-
metacarpophalangeal joints without warmth, erythema, tory is significant for diffuse cutaneous systemic sclerosis
swelling, or effusion. and gastroesophageal reflux disease. Her only medication
Laboratory studies, including an erythrocyte sedimen­ is omeprazole.
tation rate and serum creatinine, are normal. On physical examination. the patient is alert but in
Knee radiographs (including standing views) show respiratory distress. Temperature is 37.2 °C (99.0 °F), blood
medial joint-space narrowing and small osteophytes of the pressure is 106/74 mm Hg. pulse rate is 108/min. and res­
left knee; there is no periarticular osteopenia or marginal piration rate is 24/min. BM! is 31. Oxygen saturation is 92%
erosions. on ambient air. An S 1 and elevated jugular venous pressure
Which of the following is the most appropriate next are noted. Crackles are noted at the lung bases. Diffuse skin
treatment? thickening of the face. anterior chest. arms stopping at the
elbows. and legs is noted: there is sclerodactyly ofthe hands.
(A) Capsaicin ll1ere is lower extremity edema to the knees.
(B) Diclofenac Laboratory studies are normal except for a serum cre­
(C) Duloxetine atinine level of2.2 mg/dL (194.5 µrnol/L).
(D) Hyaluronic acid Chest radiograph shows bilateral pleural effusions and
diffuse alveolar infiltrates. Echocardiogram shows gener­
(E) Hydrocodone
alized myocardial hypokinesis and a left ventricular ejec­
tion fraction of 20%. Electrocardiogram shows nonspecific
T-wave changes.
Item 90
Which of the following is the most likely cause of this
A 56-year-old man is evaluated for painless intermittent patient's clinical presentation?
bloody urine of 6 weeks' duration. History is significant
for granulomatosis with polyangiitis (formerly known as (A) Carcliomyopathy
Wegener granulomatosis) diagnosed 10 years ago, which (B) Constrictive pericarditis
is now in remission; he was treated with prednisone for (C) Pulmonary arterial hypertension
3 years and oral cyclophosphamide for 1 year. He also has
(D) Scleroderma renal crisis
hypertension and hyperlipidemia. Current medications are
metoprolol and atorvastatin.
On physical examination, temperature is 36.7 °C (98.0 °F), Item 92
blood pressure is 146/94 mm Hg, pulse rate is 68/min, and A 32-year-old woman is evaluated for a 2-month history
respiration rate is 14/min. BM! is 28. There are no rashes or of weight loss, abdominal cramping, and loose stools. Her
ulcers. Genitalia are normal. The remainder of the examina­ stools are malodorous, but she has not noted any blood
tion, including cardiopulmonary examination, is normal. associated with her bowel movements. Although her appe­
Laboratory studies: tite is good, she has lost 3.2 kg (7.0 lb). She has an 8-year
Chemistry panel and kidney Normal history of diffuse cutaneous systemic sclerosis.
function tests On physical examination, temperature is normal,
Hemoglobin 12.1 g/dL (121 g/L) blood pressure is 146/92 mm Hg, pulse rate is 94/min, and
Erythrocyte sedimentation rate 35 mm/h respiration rate is 16/min. BMI is 19. Cardiopulmonary
p-ANCA Negative examination is normal. The abdomen is soft and nontender
Antimyeloperoxidase antibodies Negative with normal bowel sounds. Diffuse skin thickening of the
Antiproteinase 3 antibodies Negative face, anterior chest, and distal extremities is noted as well
Urinalysis Trace protein; as sclerodactyly and multiple healed digital pits. There is no
10-20 erythrocytes/hpf; rash. Muscle strength and reflexes are normal.
0-2 leukocytes/hpf; Laboratory studies:
no casts Hematocrit 30%
Urine cultures Negative Albumin 2.6 g/dL (26 g/L)
Alanine aminotransferase Normal
A chest radiograph is normal. Aspartate aminotransferase Normal
Which of the following is the most appropriate diagnostic Total bilirubin Normal
test to perform next? Lipase Normal
Urinalysis Normal
(A) CT of the abdomen and pelvis with contrast
(B) Cystoscopy Which of the following is the most appropriate diagnostic
(C) Kidney and bladder ultrasonography test to perform next?
(D) Urine eosinophil measurement (A) Colonoscopy
(E) Urine protein-creatinine ratio (B) CT of the abdomen and pelvis with contrast

116
Self-Assessment Test

(C) Endoscopic retrograde cholangiopancreatography ago after developing shoulder and hip girdle pain and
(D) Glucose hydrogen breath test morning stiffness. Symptoms resolved on prednisone,
15 mg/d. She feels well and reports no headache, jaw
claudication, visual changes, or recurrence of myalgia
Item 93 or stiffness. History is significant for type 2 diabetes
A 65-year-old woman is evaluated for bilateral hand and mellitus and hypertension. Medications are metformin,
wrist pain that worsens with activity. She reports no swell­ lisinopril, and prednisone, which has been tapered to
ing or redness but has morning stiffness lasting less than 10 mg/d.
30 minutes. History is also significant for hypertension On physical examination, temperature is normal,
and diabetes mellitus. There is no personal or family his­ blood pressure is 140/80 mm Hg, pulse rate is 70/min,
tory of psoriasis. Medications are hydrochlorothiazide and and respiration rate is 14/min. BM! is 31. There is no tem­
metformin. poral tenderness or induration. No carotid or subclavian
On physical examination, vital signs are normal. BM! bruits are present. Good range of motion without pain
is 29. The right wrist has a mild effusion and slightly in the shoulders and hips is noted. Proximal strength is
reduced range of motion. There is mild pain with range of normal.
motion of both wrists. The hands have bony hypertrophy Laboratory studies:
of the proximal and distal interphalangeal joints, with Initial Current
mild tenderness to palpation but no swelling. Bilateral Erythrocyte sed­ 90 mm/h 42 mm/h
crepitus of the knees is noted. There are no rashes or nail imentation rate
changes. Hemoglobin 11.S g/dL (llS g/L) 12 g/dL (120 g/L)
Laboratory studies reveal a negative rheumatoid factor,
and erythrocyte sedimentation rate, C-reactive protein, and Which of the following is the most appropriate manage­
serum urate levels are within normal limits. ment at this time?
A radiograph of the wrist is shown. (A) Increase prednisone
(B) Increase prednisone and add methotrexate
(C) Schedule temporal artery biopsy
(D) Continue current treatment

Item 95
A 47-year-old woman is evaluated in the emergency depart­ Cl
ment for sharp mid-chest pain that developed abruptly.
Tl1e pain is exacerbated by lying clown, deep inspiration. or
coughing but improves when she sits up.
On physical examination. temperature is 37.8 °C
(100.0 °r). blood pressure is 1-10/88 111111 Hg. pulse rate is
100 min. and respir,ition rate is 22/min. A friction rub al
the lef't sternal border is heard. ll1e lungs are clear. Tl1ere are
swelling and tenderness of" the second and third proximal
inlerphalangeal and metacarpophalangeal joints.
Electrocardiogram shows diffuse ST-segment eleva­
tions in all leads except aVR and \/ 1 and PR-segment depre -
sion in leads V� lo v,,.
Which of the following is the most likely cause of this
patient's pericarditis?
Aspiration of the wrist is performed, and results are (A) Ankylosing sponclylitis
pending. (B) Polymyalgia rheumalica
Which of the following is the most likely diagnosis? (C) Psoriatic arthritis
(D) Rheumatoid arthritis
(A) Chronic gouty arthropathy
(B) Osteoarthritis with calcium pyrophosphate deposi­
tion
(C) Psoriatic arthritis
Item 96
An SO-year-old woman was hospitalized 2 days ago for CJ
(D) Rheumatoid arthritis upper gastrointestinal bleeding due lo peptic ulcer disease.
She vvas placed on omeprazole and given intravenous nor-
mal saline For hydration. Today she has developed right
Item 94 knee pain and swelling. History is also significant For osteo­
A 74-year-old woman is evaluated during a follow-up arthritis of the lrnnds and knees. Her only medication prior
visit for polymyalgia rheumatica diagnosed 8 weeks to admission was ibuprofen as needed.

117
Self-Assessment Test

CJ
CONT.
On physical examination. temperature is 37.8 °C
(100.0 °F); the remainder of the vital signs is normal. Hand
findings are consistent with osteoarthritis. The right knee is
Which of the following synovial fluid tests will be most
helpful in establishing a diagnosis?
f
warm with a large efusion. is tender lo palpation, and has (A) Antinuclear antibody measurement
limited flexion to 90 degrees. (B) Glucose measurement
Aspiration of the right knee is performed: the syno­ (C) Gram stain, culture. and crystal analysis
vial fluid is yellow and cloudy. and the leukocyte count is (D) Protein measurement
15.000/µL (15 x 109/L).

118
Answers and Critiques

Item 1 Answer: B Item 2 Answer: B


Educational Objective: Treat hyperuricemia with Educational Objective: Diagnose Jaccoud arthropathy
febuxostat in a patient with an adverse reaction to in a patient with systemic lupus erythematosus.
allopurinol.
The most likely diagnosis is Jaccoud arthropathy, which is most
Febuxostat is indicated for this patient with frequent gout commonly caused by systemic lupus erythematosus (SLE). SLE
attacks. He had been taking allopurinol, a first-line agent for arthritis is nonerosive, but persistent periarticular inflamma­
serum urate reduction in patients with gout. Urate-lowering tion that affects the structural integrity of the joint capsule/
therapy is indicated for patients with gout who experience supporting joint ligaments can result in Jaccoud arthropa­
repeated attacks (:2:2 per year), have one attack in the setting thy, or reversible hand deformities, which is characterized by
of chronic kidney disease (CKD) of stage 2 or worse, have reducible subluxation of the digits, swan neck deformities,
tophaceous deposits found on examination or imaging, or and ulnar deviation of the fingers due to attenuation of the
have a history of urolithiasis. This patient developed an joint-supporting structures. It is reported to occur in 5% of
adverse reaction to allopurinol but still needs urate-lowing patients with SLE and can be confused with rheumatoid arthri­
therapy. Febuxostat is a newer non-purine, non-competitive tis. Jaccoud arthropathy can be seen in other inflammatory
xanthine oxidase inhibitor, which is a viable alternative to illnesses, including scleroderma, mixed connective tissue dis­
allopurinol. It can be used in patients with mild to moderate ease, and Sjogren syndrome, and was first described in patients
CKD and is safe to try after an adverse reaction or failure with recurrent episodes of rheumatic fever. This patient with
of allopurinol. SLE demonstrates the classic features of Jaccoud arthropathy,
Anti-inflammatory prophylaxis to prevent gout attacks including subluxation, ulnar deviation, and swan neck defor­
is recommended when urate-lowering therapy is initiated mities, with radiographs that do not show evidence of erosions.
because of the paradoxical increased risk of acute gout Joint hypermobility refers to the ability to painlessly move
attacks when serum urate levels are rapidly decreased by a joint beyond normal range of movement. Hypermobility syn­
medication. Prophylaxis should be continued in the pres­ drome describes a disorder characterized by musculoskeletal
ence of any active disease (tophi or flares). Colchicine is pain and generalized joint hypermobility occurring in other­
a first-line option for gout prophylaxis and should not be wise healthy individuals. Patients with joint hypermobility can
discontinued in this patient who requires flare prophylaxis rarely have swan neck deformities but do not generally have
during urate-lowering therapy. deformities of the severity seen in this patient. In addition, the
Pegloticase is an intravenous synthetic uricase replace­ presence of another disease that can cause joint hypermobility
ment approved for treatment-failure gout. Pegloticase is (SLE in this patient) excludes hypermobility syndrome.
immunogenic, and the development of antibodies eventu­ Mixed connective tissue disease (MCTD) is character­
ally occurs in most patients taking the drug, which leads to ized by features of systemic sclerosis, polymyositis, and
reduced effectiveness and increases the risk of hypersensi­ SLE and is by definition associated with high titers of
tivity reactions. anti-Ul-ribonucleoprotein antibodies. This patient does
The uricosuric drugs probenecid and sulfinpyra­ not have any of the characteristic symptoms of MCTD,
zone promote kidney clearance of uric acid by inhibiting including Raynaud phenomenon, hand edema, puffy fin­
urate-anion exchangers in the proximal tubule responsible gers, and/or prominent synovitis.
for urate reabsorption. These agents are relatively contrain­ Rheumatoid arthritis generally causes nonreducible
dicated in patients with impaired kidney function or those hand deformities; furthermore, severe hand changes asso­
at risk for kidney stones. ciated with rheumatoid arthritis typically show erosions
on radiograph and periarticular osteopenia, which are not
KEY POINT
present in this patient.
• In patients with gout who require urate-lowering
KEY POINT
therapy, febuxostat is a viable alternative for those
who have an adverse reaction to allopurinol. • Jaccoud arthropathy is a nonerosive arthritis most
commonly caused by systemic lupus erythematosus
Bibliography and is characterized by reducible subluxation of the
Khanna D, Fitzgerald JD, Khanna PP, et al; American College or digits, swan neck deformities, and ulnar deviation of
Rheumatology. 2012 American College of Rheumatology guidelines for
management ofgoul. Part l: systematic nonpharmacologic and pharma­ the fingers due to attenuation of the joint-supporting
cologic therapeutic approaches to hyperuricemia. Arthritis Care Res structures.
(Hoboken). 2012 Oct,64(10):1431-46. [PMlD: 23024028]

119
Answers and Critiques

Bibliography ltem4 Answer: A


Skare TL, Godoi AL, Ferreira V. Jaccoud arthropathy in systemic lupus ery­ Educational Objective: Diagnose amyopathic
thematosus: clinical and serologic findings. Rev Assoc Med Bras. 2012
Jul-Aug:58(4):489-92. [PMID: 229300301 dermatomyositis.
The most likely diagnosis is amyopathic dermatomyositis.
This patient has a clinical presentation of heliotrope eruption
ltem3 Answer: C
in the form of a violaceous periorbital rash, Gottron pap­
Educational Objective: Diagnose rheumatoid arthritis ules over the extensor surface of small joints of the hands,
with appropriate laboratory testing. and photodistributed violaceous poikiloderma (V sign and
This patient most likely has rheumatoid arthritis (RA), Shawl sign) without muscle weakness. These are findings
and testing for both anti-cyclic citrullinated peptide of skin involvement seen in dermatomyositis without any
(CCP) antibodies and rheumatoid factor will be most clinical, serum, or electromyogram (EMG) findings of muscle
helpful in confirming the diagnosis. RA is an autoim­ involvement or myositis, suggesting the diagnosis of amyop­
mune disorder that typically presents as a symmetric athic dermatomyositis. Amyopathic dermatomyositis is seen
inflammatory polyarthritis affecting the proximal inter­ in about 20% to 25% of patients with dermatomyositis, but
phalangeal and metacarpophalangeal joints of the fin­ some of these patients have evidence of myositis on one of the
gers, the wrists, and the analogous joints of the feet. Pro­ evaluation studies (muscle enzymes, EMG, or muscle biopsy)
longed morning stiffness is common. Anti-CCP antibody in the absence of muscle weakness. To qualify for the diagno­
testing has the greatest specificity (95%) for the diagnosis sis of amyopathic dermatomyositis, the patient should have
of RA. Although no single laboratory test will diagnose the characteristic rash but no clinical, laboratory, or muscle
RA, the combination of a compatible clinical presentation evaluation findings of myositis. Amyopathic dermatomyositis
and a positive rheumatoid factor and positive anti-CCP may be triggered by sunlight exposure and also is associated
antibodies is more specific for the diagnosis than any with an underlying malignancy. Treatment is usually with
other combination of tests. Approximately 75% of patients glucocorticoids and immunosuppressive agents.
with RA are rheumatoid factor positive, but specific­ Polymorphous light eruption (PMLE) is another der­
ity is only around 80%. Rheumatoid factor positivity matologic condition in which patients develop skin lesions
frequently occurs in other autoimmune disorders and after exposure to sunlight; these lesions last several days
chronic infections, most notably chronic active hepatitis and resolve spontaneously in the absence of reexposure. A
C virus infection. variety of skin lesions may be seen in PMLE, including urti­
Testing for antinuclear antibodies (ANA) is usually per­ caria! wheals, papules, plaques, and vesicles. PMLE usually
formed in patients with suspected systemic lupus erythe­ develops early in the spring, with the first few exposures to
matosus (SLE). A new onset of polyarticular inflammatory sunlight, and can be triggered by intense exposures. These
arthritis as seen in this patient can be indicative of SLE; lesions occur in photodistributed areas but lack character­
however, she has no other signs or symptoms suggestive of istic heliotrope or Gottron eruptions. A diagnosis of PMLE is
SLE such as alopecia, aphthous ulcers, malar rash, pericar­ unlikely in this patient.
dia! and pleural serositis, or cytopenias. Furthermore, an Rosacea is a chronic, inflammatory condition that
ANA test may be positive in 40% of patients with RA and causes an acneiform eruption and flushing on the mid-face.
would not distinguish between RA and SLE with as much There are two types, vascular and papular pustular (inflam­
specificity as the combination of anti-CCP antibodies and matory) rosacea. Vascular rosacea presents as persistent
rheumatoid factor. flushing, especially of the central face, with prominent tel­
Although an elevated C-reactive protein may provide angiectasias. Pustules and papules are seen in the inflam­
laboratory evidence of inflammation that can complement matory variant, but in contrast to acne, rosacea pustules are
the physical examination findings of inflammatory synovi­ not follicular based. The patient's findings are not consistent
tis, this inflammatory marker lacks diagnostic specificity with rosacea.
and does not distinguish RA from other forms of inflam­ Although this patient has positive antinuclear anti­
matory arthritis. bodies, she lacks the associated findings of systemic lupus
erythematosus (SLE), including malar/discoid rash, arthri­
KEY POINT
tis, organ involvement, and kidney disease. SLE can cause
• The combination of anti-cyclic citrullinated pep­ a rash on the hands similar to Gottron papules, but it more
tide antibodies and rheumatoid factor has the typically involves skin located between the joints.
greatest specificity for the diagnosis of rheumatoid
KEY POINT
arthritis.
• Amyopathic dermatomyositis refers to dermatomyosi­
Bibliography tis with cutaneous involvement in the absence of clin­
Pincus T, Sokka T. Laboratory tests to assess patients with rheumatoid ical, laboratory, electromyogram, or biopsy evidence
arthritis: advantages and limitations. Rheum Dis Clin N Am. 2009 of myositis.
Nov:35(4):731-4. [PMID: 199626171

120
Answers and Critiques

Bibliography
Gerami P. Schope JM, McDonald L. Walling HW, Sontheimer RD. A system­
_
atic review of adult-onset clinically amyopathic der111atomyosn1s (der­
ltem6 Answer: D
Educational Objective: Treat an acute monoarticular
Cl
matomyositis sinemyositis): a missing link within the spectrum of the gouty attack with intra-articular glucocorticoids.
idiopathic inflammatory myopathies. J Am Acad Dermatol. 2006
Apr:54(4):597-61 3. [PMID: 16546580] Intra-articular glucoco11icoid therapy is appropriate tor this
patient. Intra-a11icular glucocorticoid injections are a good
treatment strategy when only one or tvvo joints are aflected, the
Item 5 Answer: A presence of joint infection has been ruled out, and oral therapies
Educational Objective: Treat inadequately controlled have potential adverse events. This patient has a history of gout
osteoarthritis with intra-articular glucocorticoids. and currently presents with evidence of'an acute gouty attack in
tJ1e knee. Given the synovial fluid leukocyte count (<50.000/µL
Intra-articular glucocorticoids are appropriate for this
(50 x 109/L]). the documented presence of intracellular crystals.
patient. She has osteoarthritis in multiple joints based on
and the negative synovial fluid Gram stain. the probability of a
her history, physical examination, and plain radiographs.
joint infection is very low. The onset of gout Ln this patient was
Although her overall osteoarthritis symptoms appear to be
preceded by community-acquired pneumonia. which may have
well controlled, the right knee is clearly more affected than
promoted the gout attack by causLng fever and dehydration.
any other joint. Therefore, therapy targeted toward this
Treatment for acute gout should focus on anti-inflammatory
joint is indicated, and intra-articular glucocorticoids are the
tl1erapy. typically using colchicine. an NSAID. or a glucocor­
most appropriate choice for this patient to address her local­
ticoid. Although all three would eflectively treat this patient's
ized symptoms. Meta-analyses of clinical trials evaluating
oout the best choice would be an intra-articular glucocorticoid
the use of glucocorticoid injection in osteoarthritis suggest b

because of potential adverse effects of the otl1er anti-in.flamma-


that the technique may be particularly helpful. Individual
tories. Although both systemic and local glucocorticoid therapy
studies have shown that the presence of an effusion, with­
are effective in treatLng acute gout. Lntra-articular glucocorti­
drawal of fluid from the knee, severity of disease, absence
coids are preferred in tl1is patient to avoid systemic inm1unosup­
of synovitis, injection delivery under ultrasound guidance,
pressive eflects in the setting of resolving pneumonia and tl1e
and greater symptoms at baseline may all improve the like­
possible adverse impact of systemic glucocorlicoids in a patient
lihood of response.
witl1 diabetes mellitus.
Hyaluronic acid injections have shown only a min­
Acetaminophen is analgesic but not anti-inflammatory
imal degree of benefit in the treatment of knee osteoar­
and does not promote the resolution of a gouty attack.
thritis. They generally require a series of three weekly
Colchicine effectively treats acute gout, especially in the
injections and are more invasive, considerably more
early phases of an attack. However, colchicine is metabo­
expensive, and less predictably efficacious than gluco­
lized by the hepatic CYP3A4 enzyme. which clarithromycin
corticoid injections.
strongly inhibits; coadministration raises the risk of col­
Arthroscopic lavage for knee osteoarthritis is a tech­
chicine toxicity and even death. and colchicine should be
nique in which fluid is instilled and aspirated from the
avoided while this patient is taking clarithromycin.
joint through an arthroscope with the intention of remov­
The presence of significant kidney disease makes the
ing the debris often present in these joints. Although
use of an NSAID such as inclomethacin undesirable because
observational studies originally suggested that the tech­
cyclooxygenase inhibition adversely affects the kidneys.
nique might be of benefit, more recent studies and
meta-analyses have suggested otherwise. No differences KEY POINT
in pain measured by visual analogue scale or function • Intra-articular glucocorticoid injections are a good
measured by the Western Ontario McMaster University treatment strategy when only one or two joints are
Index have been identified in controlled trials using sham affected, the presence of joint infection has been ruled
procedures.
out, and oral therapies have potential adverse events.
The patient's symptoms are reasonably well controlled
on her current NSAID. Therefore, there is no clear benefit to
Bibliography
switching to another medication within this class.
Khanna D. Khanna PP. Fitzgerald JD. et al: American College of Rheumatology.
2012 American College ofRheumatology guidelines for management of gout.
KEY POINT
Part 2: therapy and antiinflammatory prophylaxis of acute gouty arthritis.
• Targeted therapy with an intra-articular glucocorti­ Arthritis Care Res (Hoboken). 2012 Oct:64(10):1447-61. [PMID: 23024029]
coid injection is appropriate in patients with osteoar­

Cl
thritis who have one symptomatic joint. Item 7 Answer: D
Educational Objective: Treat an adult with severe
Bibliography
Henoch-Schonlein purpura using prednisone.
Maricar N. Callaghan MJ. Felson D'I'. O'Neill TW. Predictors or response to
intra-articular steroid injections in knee osteoarthritis--a systematic
review. Rheumatology (Oxford). 2013 Jun;52(6):1022-32. [PMID:
Prednisone is appropriate for this patient. His findings
23264554] demonstrate the presence of a small-vessel vasculitis affecting

121
Answers and Critiques

Cl
CONT.
the skin, joints. kidneys, and gastrointestinal tract. Depo­
sition of lgA in the skin confirms the diagnosis of adult­
improve the outcomes in patients such as in this case, and
additional therapy is needed to achieve a low disease activity
onset Henoch-Schbnlein purpura (HSP). In children, HSP is or remission. A combination of a biologic agent (preferably a
generally a benign, self�limited condition, and treatment is tumor factor necrosis a inhibitor) and a nonbiologic DMARD
most commonly supportive pending spontaneous remission. is thought to be the best option to achieve this goal. Because
HSP in adults is less common and typically is more severe. of her side-effect history with methotrexate, the addition of
Although adult HSP also tends to run a self-limited course. the alternative nonbiologic DMARD leflunomide at this point
adults with HSP are more likely to experience severe disease would be the next best strategy.
and to accumulate irreversible organ damage before the acute Biologic agents are frequently used in combination with
disease resolves. In this case, the involvement of multiple a nonbiologic DMARD. However, concurrent use of two or
organ systems, including the gastrointestinal tract, probably more biologic agents is not recommended because infection
warrants prednisone treatment based upon expert consensus rates are significantly increased with minimal, if any, added
recommendation. efficacy. Therefore, the addition of abatacept, anakinra, or
Cyclophosphamide is an alkylating agent and potent rituximab is inappropriate.
immunosuppressant. It is commonly used for treatment or
KEY POINT
severe autoimmune disease such as systemic lupus erythe­
matosus and ANCA-associated vasculitis. Its use in adult­ • Concurrent use of two or more biologic agents is not
onset HSP is less well established: although it is sometimes recommended because infection rates are signifi­
used in conjunction with prednisone for severe HSP nephri­ cantly increased with minimal, if any, added efficacy.
tis. it would not be a first-choice therapy in the absence of
prednisone use. Bibliography
Dapsone is an antibiotic that has antileukocyte activity Singh JA, Furst DE, Bharat A. et al. 2012 update of the 2008 American College
of Rheumatology recommendations for the use of disease-modifying
and is occasionally used to treat the leukocytoclastic vas­ antirheumatic drugs and biologic agents in the treatment of rheumatoid
culitides, including HSP. However. given the severity of this arthritis. Arthritis Care Res (Hoboken). 2012 May:64(5):625-39. [PMID:
patient's condition, dapsone is less likely to be effective. and 22473917]
prednisone use is warranted.
Like all NS.A.IDs. ibuprofen may help alleviate joint pain Item 9 Answer: B
and swelling and may have some modest effect on reduc­
ing the inflammation of small-vessel vasculitis. However, Educational Objective: Diagnose fibromyalgia.
ibuprofen is unlikely to be adequately effective in a serious The most likely diagnosis is fibromyalgia, which is charac­
case such as this. Moreover. the nephrotoxic and antiplatelet terized by chronic widespread pain, tenderness of skin and
efTecls of an NSAID would be undesirable in this patient who muscles to pressure (allodynia), fatigue, sleep disturbance,
already has acute kidney injury and intestinal bleeding. and exercise intolerance. Previous lack of response to multi­
KEY POINT ple medications, including NSAIDs, also provides a diagnos­
tic clue. Examination is generally unremarkable except for
• Treatment with prednisone should be considered for allodynia. An association with other pain syndromes, includ­
patients who have severe Henoch-Schonlein purpura ing irritable bowel syndrome, irritable bladder, pelvic pain,
with involvement of multiple organ systems. vulvodynia, headache, and temporomandibular jaw pain,
is not uncommon. This patient fulfills the 2010 American
Bibliography College of Rheumatology diagnostic criteria for fibromyalgia
Reamy BV, Williams PM, Lindsay TJ. Henoch-Schonlein purpura. Am Fam (widespread pain, wakes unrefreshed, significant fatigue, and
Physician. 2009 Oct 1:80(7):697-704. [PMID: 198173401
cognitive difficulties), with symptoms present for more than
3 months.
Item 8 Answer: C The clinical manifestations of adrenal insufficiency are
often insidious, with fatigue and malaise being the domi­
Educational Objective: Avoid combining biologic agents
nant symptoms; a high degree of clinical suspicion may be
when treating rheumatologic disease.
needed to pursue the diagnosis in the presence of subtle sys­
Addition of the nonbiologic disease-modifying antirheumatic temic symptoms. The patient's 4-year history of symptoms
drug (DMARD) leflunomide is indicated for this patient. She and a normal chemistry panel makes adrenal insufficiency
has chronic moderate to severe rheumatoid arthritis with one unlikely.
or more poor prognostic markers, which may include young The numerous and largely nonspecific clinical manifes­
age, involvement of more than three joints, seropositivity, ele­ tations of hypothyroidism include fatigue, reduced endur­
vated inflammatory markers, and radiographic changes. She ance, weight gain, cold intolerance, constipation, impaired
has continued to have active disease despite treatment with concentration and short-term memory, dry skin, edema,
sulfasalazine and the biologic agent etanercept as suggested mood changes, depression, psychomotor retardation, mus­
by continued synovitis of four joints and elevated inflamma­ cle cramps, myalgia, menorrhagia, and reduced fertility.
tory markers. A "treat to target" approach has been found to Some patients with mild hypothyroidism will exhibit few

122
Answers and Critiques

or none of these symptoms. A normal thyroid-stimulating This patient is not at an increased risk of lupus nephritis
hormone level makes hypothyroidism very unlikely. because it has not been a feature of her disease to date, and
The classic findings of polymyositis are symmetric prox­ she is negative for anti-double-stranded DNA antibodies.
imal muscle weakness with little or no pain and elevation in Although anti-Ro/SSA antibodies increase the risk of
muscle-associated enzymes. This patient has pain, a normal developing subacute cutaneous lupus, pregnancy does not
serum creatine kinase level, and no true weakness on exam­ increase this risk further and does not need to be part of
ination, making polymyositis an unlikely diagnosis. preconception counseling. Because a photosensitive rash has
been one of the features of this patient's illness, she is aware
KEY POINT
of the hazard of sun exposure.
• Current criteria for the diagnosis of fibromyalgia
KEY POINT
include chronic widespread pain, fatigue, waking
unrefreshed, and cognitive symptoms, with symp­ • Neonatal congenital heart block affects approximately
toms present for more than 3 months. 2% of pregnancies in which the mother is positive for
anti-Ro/SSA or anti-La/SSB antibodies.
Bibliography
Wolfe F. Clauw DJ, Fitzcharles MA. et al. The American College of Bibliography
Rheumatology preliminary diagnostic criteria for fibromyalgia and Lateef A, Petri M. Management of pregnancy in systemic lupus erythemato­
measurement of symptom severity. Arthritis Care Res (Hoboken). 2010 sus. Nat Rev Rheumatol. 2012 Dec;8(12):7l0-8. [PMID: 22907290]
May:62(5):600-10. [PMID: 20461783]

Item 11 Answer: D
Item 10 Answer: B Educational Objective: Diagnose hemochromatosis as a
Educational Objective: Provide preconception counsel­ cause of secondary osteoarthritis.
ing to a patient with systemic lupus erythematosus who
Measurement of transferrin saturation is the most appropriate
has positive anti-Ro/SSA antibodies.
diagnostic test to perform next in this patient. He has signs
Preconception counseling regarding congenital heart block and symptoms suggestive of hemochromatosis, an autosomal
in her child is appropriate for this patient with systemic lupus recessive disorder characterized by increased absorption of iron
erythematosus (SLE) who is positive for anti-Ro/SSA anti­ from the gut. Approximately 40% to 60% of patients with hemo­
bodies. Patients with SLE experience miscarriage, stillbirth, chromatosis develop arthropathy that is osteoarthritis-like, but
preeclampsia and premature delivery two to five times more characteristically involves the second and third metacarpo­
often than patients without the disease. This patient has mild phalangeal (MCP) or wrist joints. Transferrin saturation and
SLE, and her disease has been quiescent for 18 months; there­ serum ferritin levels are usually elevated in patients with
fore, this is an appropriate time to attempt pregnancy. Expert arthropathy due to hemochromatosis. Although a variety
opinion recommends conception when SLE has been qui­ of more specific diagnostic maneuvers may be undertaken,
escent for at least 6 months. A major risk to her child would including liver biopsy and genetic testing for homozygosity
be congenital heart block, which affects approximately 2% of for the C282Y mutation of the HFE gene, the most appropri­
pregnancies in which the mother is positive for anti-Ro/SSA ate and cost-effective next step is measurement of transferrin
or anti-La/SSB antibodies. Some of these newborns require saturation. A consensus does not exist for transferrin satu­
pacing from birth if there is complete heart block at delivery. ration cut-off levels for diagnosis of hemochromatosis, with
Pregnancies in mothers who are positive for anti-Ro/SSA some guidelines recommending a value of greater than 60%
or anti-La/SSB antibodies should be monitored closely and in men or greater than SO% in women, and others suggesting
should include input from high-risk obstetrics and neonatol­ a level of greater than 55% for all patients. Measurement of
ogy because these antibodies can pass the placenta and affect ferritin levels is indicated in patients with an elevated trans­
the developing cardiac conduction system. If the mother has ferrin saturation; a markedly elevated level further supports
had a previously affected child, subsequent pregnancies carry the diagnosis and predicts the development of symptoms.
a 12% risk of congenital heart block. Positivity for anti-Ro/ The presence of clinical MCP involvement with radiographic
SSA or anti-La/SSB antibodies also confers a risk for neonatal evidence of hook-shaped osteophytes is most characteristic of
lupus erythematosus, which is characterized by rash as well as hemochromatosis.
hematologic and hepatic abnormalities that generally resolve Anti-cyclic citrullinated peptide antibodies are never
when the antibody dissipates. The use of phototherapy for associated with hemochromatosis, and rheumatoid factor
neonatal hyperbilimbinemia may cause the rash to develop is generally negative in patients with hemochromatosis as
because the antibody is associated with photosensitivity. seen in this patient. These autoantibodies have specificity for
Hydroxychloroquine is thought to be safe in pregnancy rheumatoid arthritis.
and has been shown to reduce the risk of congenital heart Arthritis may occur in up to 20% of patients with hep­
block in newborns whose mothers are positive for anti-Ro/ atitis C virus infection and may mimic rheumatoid arthritis
SSA or anti-La/SSB antibodies. It should therefore not be clinically and radiographically. However, the characteristic
discontinued in this patient. findings of hemochromatosis on this patient's radiographs

123
Answers and Critiques

and the lack of more typical findings of erosions or bony first-line imaging modalities are negative but the suspicion
decalcification adjacent to the involved joints make a diag­ of disease remains high.
nosis of hepatitis C-associated arthritis less likely. Ultrasonography is relatively inexpensive, poses no
Serum a-fetoprotein is elevated in liver disease such as radiation hazard, can scan across three-dimensional struc­
acute or chronic viral hepatitis infection as well as in hepa­ tures, and may be used concurrently with physical exam­
tocellular and numerous other cancers. It would have little ination to evaluate moving structures (for example, tendon
diagnostic specificity in this clinical setting. evaluation). Musculoskeletal ultrasonography can be helpful
in detecting evidence of peripheral enthesitis and arthri­
KEY POINT
tis but has not demonstrated usefulness in detecting axial
• Secondary osteoarthritis may occur in the setting of involvement such as sacroiliitis.
hemochromatosis, which is associated with an
KEY POINT
arthropathy that is osteoarthritis-like, but characteris­
tically involves the metacarpophalangeal and wrist • MRI is the most sensitive imaging technique for
joints. detecting early inflammation in the spine and sacroil­
iac joints in patients with suspected spondyloarthritis.
Bibliography
Husar-Memmer E. Stadlmayr A. Datz C. Zwerina J. HFE-Related hemochro­ Bibliography
matosis: an update for the rheumatologisl. Curr Rheumatol Rep. 2014 Ostergaard M. Lambert RG. Imaging in ankylosing spondylitis. Ther Adv
Jan:16(1):393. [PMID: 24264720] Musculoskelet Dis. 2012 Aug,4(4):301-11. lPMID: 22859929]

Item 12 Answer: B Item 13 Answer: B


Educational Objective: Diagnose spondyloarthritis Educational Objective: Diagnose puhnonary arterial
using MRI. hypertension in a patient with limited cutaneous systemic
sclerosis.
MRI of the sacroiliac joints and/or spine is the most appro­
priate diagnostic test to perform next in this patient with Doppler echocardiography is the most appropriate test to
suspected spondyloarthritis, considered in patients with perform next in this patient with a 3-year history of limited
chronic inflammatory back pain beginning before the age of cutaneous systemic sclerosis (LcSSc) who now presents with
45 years. lt is important to establish the diagnosis of spondy­ shortness of breath and fatigue, a prominent single S2, and a
loarthritis even if it will not change immediate management normal pulmonary examination and chest radiograph. LcSSc
because it requires life-long monitoring for the development is characterized by isolated distal skin thickening (face, neck,
of cardiovascular and other major organ damage. A positive and hands distal to wrists), is typically not accompanied by
HLA-B27 can be supportive of this diagnosis, but a negative internal organ fibrosis, and is more likely to be associated with
result does not rule it out. Conventional radiographs can pulmonary arterial hypertension (PAH). The initial screening
demonstrate sacroiliitis (erosive changes and sclerosis) but test for those with systemic sclerosis who have suspected PAH
may be normal in early disease. If there is high suspicion is echocardiography, which can rapidly and noninvasively
for axial inflammation and conventional radiographs are estimate elevated pulmonary pressure as well as rule out some
normal, MRI of the sacroiliac joints and/or spine should be etiologies in the differential diagnosis such as intracardiac
considered to further evaluate for inflammation. MRI is the shunts, valvular heart disease, or heart failure. A moderate to
most sensitive imaging technique for detecting early inflam­ high tricuspid gradient correlates well with PAH confirmed
mation in the spine and sacroiliac joints. Although his radio­ with gold standard right heart catheterization, which is 97%
graphs and HLA-B27 testing were negative, this 25-year-old specific but may not be sensitive.
patient has probable inflammatory back (morning stiffness Bronchoscopy with lavage is often used in immuno­
lasting 90 minutes) and sacroiliac pain, making spondyloar­ compromised patients with rapidly deteriorating lung func­
thritis, specifically ankylosing spondylitis, a likely diagnosis. tion to assess for infection and/or pulmonary hemorrhage.
Advanced imaging is often needed to show sacroiliac joint This test is not indicated in a patient with findings suggestive
abnormalities. of PAH.
CT of the sacroiliac joints can provide evidence of ero­ 8-type natriuretic peptide (BNP) or N-terminal proBNP
sive changes in the bone but has limited ability to detect levels should be assessed in patients suspected of hav­
soft-tissue inflammation of the spine and may be normal ing heart failure. P reliminary data suggest that N-termi­
until bony changes are present. nal proBNP may be helpful in the assessment of PAH and
When injected intravenously. technetium-99m binds to may provide prognostic information. BNP and N-terminal
hydroxyapatite crystals. Increased uptake reflects increased proBNP measurement cannot be recommended at this time
bone turnover related to infection, cancer, trauma, and until further studies validate their usefulness in patients
arthritis. Because of these characteristics. a positive scan with PAH.
is a sensitive but nonspecific indicator of bone, joint, and In patients with echocardiographic findings suggesting
periarticular disorders and may be most useful when other PAH, an array of studies (such as imaging of the chest to

124
Answers and Critiques

assess parenchymal lung disease; V/Q scanning to assess MRI is not necessary in a patient with a low suspicion for
potential chronic thromboembolic disease; pulmonary this condition.
function testing with OLCO; serologic studies for connective Coronary artery disease most classically presents with
tissue disease, liver disease, and HIV; and sleep studies) are exertional substernal chest pain relieved with rest or nitro­
helpful in selected patients. All patients suspected of having glycerin. Variant presentation may include dyspnea on exer­
PAH should be considered for right heart/pulmonary artery tion and exertional fatigue. Exercise stress testing would be
catheterization to confirm the diagnosis suggested by clinical indicated if there was a high level of suspicion for coronary
presentation, echocardiography, and pulmonary function artery disease, which is not the case here.
tests and to accurately measure the arterial pressure. It is No additional testing is incorrect because !LO may be
also very useful in evaluating responsiveness to therapeutic asymptomatic in some patients and can be missed without
medications and helps guide therapy. However, right heart additional testing.
catheterization follows these preliminary diagnostic tests
KEY POINT
and would not be done next.
• Interstitial lung disease is strongly associated with
KEY POINT
polymyositis and the presence of positive autoanti­
• Echocardiography can rapidly and noninvasively esti­ bodies to transfer RNA synthetases, including anti­
mate elevated pulmonary pressure as well as rule out Jo-1 antibodies.
some etiologies in the differential diagnosis of pulmo­
nary arterial hypertension. Bibliography
Connors GR. Christopher-Stine L, Oddis CV, Danoff SK. Interstitial lung
disease associated with the idiopathic innammatory myopathies: what
Bibliography progress has been made in the past 35 years? Chest. 2010 Dec:138(6):1464-
Steen V. Advancements in diagnosis of pulmonary arterial hypertension in 74. [PMID: 21138882]
scleroderma. Arthritis Rheum. 2005 Dec:52(12):3698-700. [PMID:
16320319]

Item 15 Answer: D
Item 14 Answer: C Educational Objective: Identify the association oftofac­
Educational Objective: Recognize the risk of interstitial itinib with a risk of causing an abnormal lipid profile.
lung disease in patients with polymyositis. The lipid profile should be monitored in this patient with
Chest radiography is the most appropriate diagnostic test to rheumatoid arthritis who began taking tofacitinib 1 month
perform next in this patient with polymyositis who has pos­ ago. Elevation of all components of the lipid panel, including
itive anti-Jo-1 antibodies and features of the antisynthetase cholesterol, triglycerides, HOL cholesterol, and LOL choles­
syndrome. Pulmonary manifestations of dermatomyositis and terol, has been found to occur as rapidly as 1 month after
polymyositis are common and may result from interstitial initiation of therapy with the biologic agent tofacitinib. Gen­
lung disease (!LO), hypoventilation (weakness of respiratory erally, these elevations remain stable over time. In the first
muscles), aspiration pneumonia, and, rarely, pulmonary arte­ 3 months of clinical trials evaluating the efficacy of tofac­
rial hypertension (PAH). Clinical manifestations of !LO range itinib, mean LOL cholesterol increased by 15%, and mean
from being asymptomatic to severe progressive cough and HOL cholesterol increased by 10%. In a subsequent clinical
dyspnea. !LO is strongly associated with the presence of pos­ trial, statin therapy resulted in a return to pretreatment
itive autoantibodies to transfer RNA synthetases, including levels of LOL cholesterol. It is unknown to what extent
anti-Jo-1 antibodies. In clinical practice, chest radiography, these lipid abnormalities may impact the long-term risk of
high-resolution chest CT, and pulmonary function testing are cardiovascular disease in patients treated with tofacitinib.
used to evaluate for the presence of this manifestation, and Tofacitinib may initially raise then lower leukocyte counts.
periodic follow-up in an asymptomatic patient is appropri­ Furthermore, lymphopenia, neutropenia, and anemia may
ate. Various patterns of !LO occur, ranging from nonspecific be seen with long-term use.
interstitial pneumonitis (most common) to usual interstitial Elevated aminotransaminase levels may be seen with
pneumonia or bronchiolitis obliterans organizing pneumo­ exposure to tofacitinib. However, abnormalities of bilirubin,
nia. The pattern of involvement determines glucocorticoid glucose, and alkaline phosphatase would not be expected to
responsiveness and, ultimately, prognosis. result from exposure to 1 month of therapy with tofacitinib.
6-Minute walk testing is an important test used in the KEY POINT
evaluation and follow-up of patients with an established
• The biologic agent tofacitinib is associated with a risk
diagnosis of PAH, but there is no evidence that this patient
has PAH. of causing an abnormal lipid profile.
Myocarditis has a highly variable presentation, includ­
ing fatigue, chest pain, heart failure, cardiogenic shock, Bibliography
Fleischmann R, Kremer J. Cush J. et al. Placebo-controlled trial of tofacitinib
arrhythmias, and sudden death. This patient has no cardio­ monotherapy in rheumatoid arthritis. N Engl J Med. 2012 Aug 9;367(6):
vascular symptoms suggesting myocarditis, and a cardiac 495-507. [PMID: 22873530]

125
Answers and Critiques

Item 16 Answer: C hip symptoms is consistent with polymyalgia rheumatica


Educational Objective: Prevent gout by titrating allopu­ (PMR), which commonly co-occurs in patients with GCA
rinol to achieve a target serum urate level. (approximately 50% of cases). Despite a lack of visual symp­
toms to date, the patient is at risk of acute and potentially
The allopurinol dose should be increased for this patient with catastrophic visual loss. Immediate treatment is therefore
gout who has not yet reached the serum urate target goal of warranted, the standard regimen being prednisone at a dose
less than 6.0 mg/dL (0.35 mmol/L). Allopurinol is consid­ of 60 mg/d. (In the setting of severe visual loss, high-dose
ered a first-line agent for serum urate reduction in patients pulse glucocorticoids might be considered.) The addition of
with gout. Historically, concern has been expressed regarding low-dose aspirin has been shown in limited studies to further
allopurinol dosing and the risk of hypersensitivity reaction; reduce the risk of visual loss in patients with GCA already
however, it appears that gradual dose escalation with moni­ receiving prednisone and is favored by some experts.
toring for side effects is a safe approach, even in patients with Low-dose prednisone in the range of 10 to 20 mg/d is
chronic kidney disease. Recent American College of Rheu­ generally adequate treatment for isolated PMR but has not
matology recommendations advocate for a starting dose of been shown to adequately treat GCA or to prevent visual
100 mg/d (or 50 mg/d in those with stage 4 or 5 chronic kid­ complications.
ney disease) with titration upward every 2 to 5 weeks, aiming MRI of the head permits the visualization of structures
for a target serum urate level of less than 6.0 mg/dL (0.35 that could potentially be associated with headache and/or
mmol!L). Despite the patient's kidney dysfunction, it is safe visual symptoms, including tumors, hydrocephalus, and/or
to up titrate the allopurinol gradually over several months to large aneurysms. However, the presence of jaw claudication,
reach the target goal. If intolerance to allopurinol emerges, as well as the presence of PMR symptoms, is not consistent
alternative mate-lowering therapy (such as the xanthine oxi­ with an intracranial lesion.
dase inhibitor febuxostat) should be pursued. A temporal artery biopsy should be obtained as rapidly
Gout flare prophylaxis such as colchicine should as possible to confirm the GCA diagnosis and to help direct
be maintained during mate-lowering therapy because long-term management; however, the histopathology of the
patients are paradoxically at increased risk of gout flares disease will still be readable up to 1 to 2 weeks after initi­
during this time. ation of treatment, and treatment should not be deferred
This patient is taking losartan to treat hypertension. This pending biopsy.
agent should not be discontinued because it has uricosuric
KEY POINT
effects and thus may be helping with efforts to lower his
serum urate level. • Immediate treatment with prednisone, 60 mg/d (or
This patient is not yet at the target serum urate 1 mg/kg/d), is indicated for patients with suspected
goal; therefore, continuation of the same therapy is not giant cell arteritis to prevent visual complications.
appropriate.
KEY POINT
Bibliography
Frazer JA, Weyand CM, Newman JN, Biousse V. The treatment of giant cell
• Gradual dose escalation of allopurinol, with monitor­ arteritis. Rev Neurol Dis. 2008 Summer:5(3):140-52. [PMID: 18838954]
ing for side effects, is a safe approach for patients
(even those with chronic kidney disease) with gout
who have not reached a target serum urate level of Item 18 Answer: C
less than 6.0 mg/dL (0.35 mmol/L). Educational Objective: Screen for tuberculosis prior to
starting biologic therapy.
Bibliography Screening for tuberculosis using an interferon-y assay is
Khanna D. Khanna PP. Fitzgerald JD. et al; American College of indicated for this patient before initiation of a tumor necro­
Rheumatology. 2012 American College of Rheumatology guidelines for
management of gout. Part 2: therapy and antiinflammatory prophylaxis sis factor (TNF)-a inhibitor. This patient with rheumatoid
of acute gouty arthritis. Arthritis Care Res (Hoboken). 2012 Oct:64(10): arthritis has active disease despite treatment with triple
1447-61. lPMID: 23024029]
therapy using nonbiologic disease-modifying antirheumatic
drugs. In addition, she has poor prognostic markers, includ­
Item 17 Answer: B ing positive rheumatoid serology and high inflammatory
markers. Appropriate therapy with a biologic agent is being
Educational Objective: Treat giant cell arteritis with
planned. TNF-a inhibitors are the mainstay of initial biologic
high-dose prednisone.
therapy for rheumatoid arthritis. Reactivation of tuberculo­
Treatment with prednisone, 60 mg/d (or 1 mg/kg/d), is indi­ sis is a significant risk for most biologic agents, and particu­
cated immediately for this patient. She has temporal artery larly with TNF-a inhibitors because they inhibit formation
pain and tenderness, along with jaw claudication in the set­ of granuloma. Prior to starting any biologic agent, appro­
ting of low-grade fever and a very high erythrocyte sedimen­ priate testing for latent tuberculosis is needed by obtaining
tation rate. Given her age, these findings are most consistent either a tuberculosis skin test or an interferon-y release assay
with giant cell arteritis (GCA). The presence of shoulder and (IGRA). Either of these two tests can be used to screen for

126
Answers and Critiques

latent tuberculosis. IGRA is more costly but may be more prototype disease in the spectrum of axial spondyloarthritis.
sensitive in patients on immunosuppressive therapy. These criteria allow patients who have not yet developed
Chest radiography will be needed if the patient is radiographic sacroiliitis to be classified as having "non­
symptomatic with pulmonary symptoms or has positive radiographic" axial spondyloarthritis.
testing for latent tuberculosis infection but is currently not Distinguishing between inflammatory and non.inflam­
necessary. matory joint pain is c1;tical in evaluating patients with muscu­
Common variable immunodeficiency (CVID) occurs in loskeletal conditions. Inflammation may be the only symptom
both adults and children. Serum IgG levels are markedly that distinguishes ankylosing spondylitis from lumbar degen­
reduced, and serum lgA and/or lgM levels are frequently erative disk disease. Subjective manifestations of joint inflam­
low. Patients with CVID frequently develop chronic lung mation include morning stiffness for more than 1 hour. Lumbar
diseases, autoimmune disorders such as rheumatoid arthri­ degenerative disk disease is not likely in this patient because his
tis, malabsorption, recurrent infections, and lymphoma. radiographs are normal and he has inflammatory back pain.
Recurrent sinopulmonary infections, ear infections, and Characteristic features of psoriatic arthritis include
conjunctivitis are common. Measuring immunoglobulin lev­ enthesitis, dactylitis. tenosynovitis, arthritis of the distal
els in this patient without evidence of recurrent infection is interphalangeal joints, asymmetric oligoarthritis, and spon­
not indicated. dylitis. The HLA-B27 antigen may be positive in patients with
Radiographs of the hands and feet can confirm the axial involvement. Psoriatic arthritis involving only the axial
presence of erosions in active rheumatoid disease but are skeleton is possible in this patient but less likely because he
not needed in this patient prior to starting a biologic because has no evidence of psoriasis.
disease activity has already been established by physical Reactive arthritis (formerly known as Reiter syndrome)
examination and laboratory testing. is a postinfectious arthritis that occurs in both men and
women. Infections may include urethritis or diarrhea,
KEY POINT
although patients may be asymptomatic. Arthritis, usually
• Screening for tuberculosis is indicated before initia­ oligoarticular, develops several days to weeks after the infec­
tion of any biologic agent. tion. ll1e HLA-B27 antigen may be positive in these patients.
Reactive arthritis is also less likely as this patient has no
Bibliography history of a gastrointestinal or genitourinary infection pre­
Singh JA. Furst DE. Bharat A. et al. 2012 update of the 2008 American College ceding the onset of arthritis.
of Rheumatology recommendations for the use of disease-modifying
antirheumatic drugs and biologic agents in the treatment of rheumatoid KEY POINT
arthritis. Arthritis Care Res (Hoboken). 2012 May;64(5):625-39. [PMID:
22473917] • Ankylosing spondylitis is characterized by inflamma­
tory back pain that manifests as pain and stiffness in
the spine that is worse after immobility and better
Item 19 Answer: A
with use.
Educational Objective: Diagnose ankylosing spondylitis.
The most likely diagnosis is ankylosing spondylitis, which is Bibliography
characterized by inflammatory back pain that manifests as Rudwaleit M, van der Heijde D. Landewe R. et al. The Assessment of
SpondyloArthritis International Society classification criteria for periph­
pain and stiffness in the spine that is worse after immobility eral spondyloarthritis and for spondyloarthritis in general. Ann Rheum
and better with use. Symptoms are prominent in the morning Dis. 2011 Jan:70(1):25-31. [PMID: 21109520)
(>l hour), and patients can be symptomatic during the night.
Buttock pain is common and correlates with sacroiliitis, Item 20 Answer: A
which is typically bilateral. This patient has symptoms/signs Educational Objective: Recognize lymphoma in a
consistent with ankylosing spondylitis, including more than patient with Sjogren syndrome.
3 months of inflammatory back pain of primarily axial
involvement, age of onset younger than 45 years, a positive The most appropriate next step in the management of this
HLA-B27, and a good response to an NSAID. The lack of sac­ patient is to obtain a lymph node biopsy. This patient has
roiliitis or other inflammatory changes on his radiographs Sjogren syndrome, an immune-mediated disease manifest­
does not rule out this diagnosis; these changes may not be ing primarily as inflammation of exocrine glands, including
evident early in the disease course and may not be seen on the major and minor salivary glands, lacrimal glands, and,
plain radiographs if there are no bone erosions. He fulfills less commonly, other exocrine glands such as the pancreas.
the Assessment of SpondyloArthritis international Soci­ Patients with Sjogren syndrome are at significant risk of
ety (ASAS) classification criteria for axial spondyloarthritis developing lymphoma, the most common being diffuse B-cell
because he has a positive HLA-B27 plus at least two other and mucosa-associated lymphoid tissue (MALT) lymphomas;
features of spondyloarthritis, including inflammatory back this risk is up to 44-fold higher than in the general population.
pain and a good response to NSAIDs. The ASAS classification It is thought that chronic B-cell activation may lead to the
criteria use a nomenclature that defines spondyloarthritis as development of a clone of malignant B cells. Hypocomple­
axial or peripheral, and ankylosing spondylitis would be the mentemia, splenomegaly, lymphadenopathy, gammopathy,

127
Answers and Critiques

skin vasculitis, and cryoglobulinemia predict the develop­ of anterior uveitis, chronic inflammatory back pain, and
ment of, and accompany, lymphoma. With this patient's clini­ radiographic evidence of sacroiliitis).
cal history and findings, lymphoma must be in the differential Testing for anti-cyclic citrullinated peptide antibod­
and should be evaluated with a lymph node biopsy. ies can be useful in patients with suspected rheumatoid
Biopsy of the rash will demonstrate leukocytoclastic arthritis, which is characterized by an inflammatory poly­
vasculitis, which frequently accompanies lymphoma, but arthritis of small joints. Rheumatoid arthritis is unlikely in
will not be able to diagnose the underlying lymphoma. this patient because he only has enthesitis, an uncommon
Heterophile antibody testing is not indicated because presentation of rheumatoid arthritis.
the complete blood count results do not demonstrate lym­ The ANCA-associated vasculitides include granuloma­
phocytosis, and the patient's symptoms and findings are tosis with polyangiitis (formerly known as Wegener gran­
much more concerning for lymphoma. ulomatosis), microscopic polyangiitis, and eosinophilic
Prednisone and cyclophosphamide therapy may be granulomatosis with polyangiitis (formerly known as Churg­
helpful for some of this patient's symptoms, including vas­ Strauss syndrome). Granulomatosis with polyangiitis is a
culitis, but are premature at this time until lymphoma has systemic necrotizing vasculitis that predominantly affects
been either identified or excluded. If there is no lymphoma the upper and lower respiratory tract and kidneys. More
and Sjogren syndrome is believed to be active and causing than 70% of patients have upper airway manifestations such
vasculitis, then these therapies could be considered. as sinusitis or nasal, inner ear, or laryngotracheal inflamma­
tion. Microscopic polyangiitis is a necrotizing vasculitis that
KEY POINT
predominantly affects the lungs and kidneys. Eosinophilic
• Patients with Sjogren syndrome have up to a 44-fold granulomatosis with polyangiitis is an eosinophil-rich nec­
higher risk of developing lymphoma, the most com­ rotizing vasculitis predominantly affecting the respiratory
mon being diffuse B-cell and mucosa-associated lym­ tract and other major organs. This patient does not have
phoid tissue (MALT) lymphomas. clinical evidence of ANCA-associated vasculitis; therefore,
testing for ANCA is not indicated.
Bibliography Antinuclear antibody testing is useful for patients with
Jonsson MV. Theander E. Jonsson R. Predictors for the development of non­ suspected systemic lupus erythematosus (SLE); however,
Hodgkin lymphoma in primary Sjogren·s syndrome. Presse Med. 2012
Sep:41(9 Pt 2):eSll-6. [PMID: 22867948]
SLE does not usually cause enthesitis or uveitis as seen in
this patient.
KEY POINT
Item 21 Answer: D
• HLA-827 testing can be helpful in supporting the
Educational Objective: Utilize HLA-827 testing to aid in diagnosis of spondyloarthritis in the absence of other
the diagnosis of spondyloarthritis. sufficient manifestations.
The most appropriate diagnostic test to perform next is
HLA-827 testing. The patient has uveitis and enthesitis at Bibliography
the Achilles tendon, which are suggestive of peripheral Rudwaleil M, van der Heijde D. Landewe R. et al. The Assessment of
SpondyloArthritis International Society classification criteria for periph­
spondyloarthritis. The Assessment of SpondyloArthritis eral spondyloarthritis and for spondyloarthritis in general. Ann Rheum
international Society classification criteria for peripheral Dis. 2011 Jan;70(1):25-31. [PMID: 21109520]
and axial spondyloarthritis are primarily used for research
purposes, although they include many of the common
symptoms, signs, and tests that are useful in diagnosing Item 22 Answer: B
these disorders. Classification criteria include several other
Educational Objective: Treat a patient who has familial
manifestations, including psoriasis, inflammatory bowel
Mediterranean fever with colchicine.
disease, preceding infection, and sacroiliitis, which are
absent in this patient. HLA-827 is included in the criteria Colchicine is appropriate for this patient who has familial
and, in the absence of other sufficient manifestations, can Mediterranean fever (FMF), an autosomal recessive disease
be helpful in supporting the diagnosis of peripheral or characterized by episodes of fever, polyserositis, arthritis, ery­
axial spondyloarthritis. HLA-827 testing has limitations sipeloid rash around the ankles, and elevated acute phase
because of its approximately 5% prevalence in the general reactants. Attacks last 1 to 3 days and are self-limited but can
population, which can lead to false positives in diagnosis. be dramatic. FMF is associated with mutation of the MEFV1
Therefore, HLA-827 is not a useful test in a patient in whom gene; testing for MEFV mutations is available and should be
clinical suspicion for spondyloarthritis is low (for example, considered as an aid to diagnosis, although not all mutations
a 65-year-old patient with noninflammatory or mechan­ have been identified. AA amyloidosis is a potential long-term
ical back pain). Furthermore, it does not add anything in consequence of FMF due to the production and accumulation
the setting of a high suspicion for spondyloarthritis when of serum amyloid A. Colchicine affects the function of various
there are sufficient other findings to establish a diagnosis inflammatory cells that are thought to play a role in the cyto­
(for example, a 35-year-old man with a 12-month history kine overproduction seen in FMF. A response to colchicine is

128
Answers and Critiques

useful in the clinical diagnosis of FMF because it can prevent genase-2 inhibitor celecoxib, is inadvisable because the risk
attacks as well as AA amyloidosis. Treatment with colchicine of recurrence with repeated exposure is high.
is therefore indicated for this patient who most likely has FMF, Narcotic use in the elderly, although not associated
as manifested by her history of febrile attacks of abdominal with ulcer risk or gastrointestinal bleeding, should also
serositis, erysipelas-like skin lesions that mimic cellulitis, be approached cautiously. Short-acting narcotics should be
and inflammatory arthritis occurring independently of the tried first, and long-acting agents such as fentanyl may be
febrile episodes. used when other agents have failed. However, in this elderly
FMF genetic mutations affect the function of pyrin that patient at risk for falling due to her recent hospitalization,
results in overactivation of the inflammasome. The inflam­ debilitation, quadriceps weakness, and a history of osteopo­
masome is an important constituent of the innate immune rosis, fentanyl is not the optimal choice.
system that is responsible for production of interleukin (IL}- Although intra-articular glucocorticoids can be of ben­
1; dysregulation of the inflammasome, as in FMF, results in efit in the management of individual joints in osteoarthritis,
overproduction of lL-1. The lL-1 inhibitor anakinra can be there is no evidence that oral glucocorticoids would be of
used in patients with FMF who are unresponsive to colchi­ benefit. Furthermore, the likelihood of adverse effects with
cine; however, this agent is not first-line therapy. extended use is high in this patient.
NSA!Ds and prednisone have not been shown to have a
KEY POINT
significant impact on the disease progression or symptoms
associated with FMF. • Compared with placebo, duloxetine significantly
reduces pain and improves physical functioning in
KEY POINT
patients with knee osteoarthritis.
• Familial Mediterranean fever is characterized by epi­
sodes of fever, polyserositis, arthritis, erysipeloid rash Bibliography
around the ankles, and elevated acute phase reac­ Hochberg MC. Altman RD. Toupin AK. et al. American College of
tants; a response to colchicine is useful in the clinical Rheumatology 2012 recommendations for the use of nonpharmacologic
and pharmacologic therapies in osteoarthritis of the hand. hip. and knee.
diagnosis. Arthritis Care Res (Hoboken). 2012 Apr:64(4):465-74. [PMlD: 225635891

Cl
Bibliography Item 24 Answer: A
Ben-Chetrit E. Touitou I. Familial Mediterranean fever in the world. Arthritis
Rheum. 2009 Oct t5;61(10):t447-53. [PMlD: 197901331 Educational Objective: Treat a patient who has inter­
stitial lung disease associated with diffuse cutaneous sys­
temic sclerosis.
Item 23 Answer: B
Cyclophospharnide is appropriate for this patient who has
Educational Objective: Treat osteoarthritis with
interslit ial lung disease (I LD) associated with diffuse cuta­
duloxetine.
neous systemic sclerosis (DcSSc). This patient with DcSSc
The serotonin-norepinephrine reuptake inhibitor duloxetine presents with clyspnea, decreased exercise tolerance, and
is appropriate for this patient with osteoarthritis. The phar­ characteristic Velcro-like crackles. These clinical findings are
macologic management of osteoarthritis pain can be difficult strongly suggestive or ILD and are supported by the imaging
because therapy for symptom relief does not halt or reverse studies and confirmed by open lung biopsy. Patients with
the disease process. Furthermore, all potential agents have systemic sclerosis who have active inflammatory lung disease
side effects, some severe, and elderly patients can be at par­ may be treated vvilh immunosuppressive agents. Cyclophos­
ticularly high risk for developing them. The optimal choice phc1mide is the only treatment shovvn to have some benefit in
of therapy relies on the risks and benefits of each agent in patients with ILD associated with OcSSc. Cyclophosphamide
the context of the patient's comorbidities. This patient has given orally or int rnvenously for I. year provides modest ben­
already tried an NSAID (ibuprofen) as well as glucocorticoid efit. ,.\!though it has shown limited clinical improvement. it
and hyaluronic injections without symptomatic relief and has is the only evidence-based therapy and is therefore appropri­
recently experienced an episode of gastrointestinal bleeding. ate. High-close glucocorticoicls are frequently used in these
Duloxetine is a reasonable choice given the patient's comor­ patients but are of unclear benefit and may precipitate sclero­
bidities and recent history of gastrointestinal bleeding. Com­ clerma renal crisis: therefore. if used. low closes are typically
pared with placebo, duloxetine significantly reduces pain and recommended by experts. Azathioprine may have a role as
improves physical functioning in patients with knee osteoar­ maintenance therapy.
thritis. In short-term studies. duloxetine was not associated /\gents such as D-penicillamine or methotrexate have
with an increase in the adverse event rate compared with pla­ not been shown to be beneficial in these patients and should
cebo. Unlike NSA!Ds, duloxetine does not increase the risk for not be used to treat I LO.
a recurrence of peptic ulcer disease. There are no medication Biologic agents. including tumor necrosis factor a
interactions that contraindicate its use in this patient. inhibitors such as inniximab. have not been shown to have
In an elderly patient who has had a recent bleeding therapeutic benefit in I LO or systemic sclerosis and should
peptic ulcer, the use of any NSAID, including the cyclooxy- not be used in this patient.

129
Answers and Critiques

KEY POINT Bibliography


Neogi T. Clinical practice. Gout. N Engl J Med. 2011 Feb 3;364(5):443-52.
• Cyclophosphamide has been shown to have some [PMID: 21288096]
benefit in patients who have interstitial lung disease
associated with diffuse cutaneous systemic sclerosis.
Item 26 Answer: C
Bibliography Educational Objective: Diagnose psoriatic arthritis.
Tashkin DP. Elashoff R. Clements PJ. et al; Scleroderma Lung Study
Research Group. Cyclophosphamide versus placebo in scleroderma The most likely diagnosis is psoriatic arthritis. Although esti­
lung disease. New Engl J Med. 2006 Jun 22;354(25):2655-66. [PMID:
16790698] mates of the prevalence of psoriatic arthritis in patients with
psoriasis vary, more recent studies using standardized diag­
nostic criteria indicate that psoriatic arthritis is present in

Cl Item 25 Answer: C
Educational Objective: Perform joint aspiration to
approximately 15% to 20% of those with psoriasis. Patients
who have features consistent with psoriatic arthritis should be
diagnose acute monoarticular arthritis. examined closely for psoriasiform skin lesions on the umbili­
cus, gluteal cleft, extensor surfaces, posterior auricular region,
Aspiration of the right knee is the most appropriate next step and scalp. Nails should be examined for pitting or onycholysis.
in management. This patient is likely to have gout based on his Characteristic features of psoriatic arthritis include enthesitis,
risk factors (older man. hype11ension. diabetes mellitus. obe­ dactylitis, tenosynovitis, arthritis of the distal interphalangeal
sity). the description of the symptoms (sudden onset al night joints, asymmetric oligoarthritis, and spondylitis. The recently
with e,·ere pain). and the recent episode of great toe swelling developed Classification Criteria for Psoriatic Arthritis
consistent with podagra. The gold standard for diagnosing (CASPAR) have a sensitivity and specificity of more than 90%,
gout is identiAcation of monosodium needle-shaped urate especially for the diagnosis of early psoriatic arthritis. This
crystals within leukoc:'.>1es via synovial fluid analysis. Further­ patient fulfills the CASPAR criteria because she has inflamma­
more. infectious arthritis must be excluded in a patient with tory articular disease with psoriasis, psoriatic nail dystrophy,
monoarticular arthritis. ll1is patient is at increased risk for dactylitis, and a negative rheumatoid factor.
joint infection given his age and presence of diabetes. Thus. This patient does not have symptoms or findings of
joint aspiration should be performed and synovial fluid sent inflammatory back pain associated with ankylosing spon­
for Gram stain. cultures. leukocyte count. and c1ystal analysis. dylitis; her back pain is related to use and improves with
Although uncommon. il is important to note that gout and an rest, which is noninflammatory. HLA-B27 positivity alone is
infected joint can coexist. insufficient to diagnose this disease, and peripheral articular
MRI may be useful for a patient with a history of trauma disease is not typical for ankylosing spondylitis.
or other reason to suspect a mechanical cause for knee pain. Nearly SO% of patients with inflammatory bowel disease
Although MRI may demonstrate inflammation. it does not (IBD) develop musculoskeletal symptoms. Peripheral arthritis
typically distinguish between infectious and noninfectious may be acute and remitting with a pauciarticular distribution
causes. ll1erefore. MRI is not currently indicated in this commonly involving the knee. Peripheral arthritis can also
patient with warmth over the joint as well as fever. which be chronic or relapsing, with prominent involvement of the
suggests an inflammatory process. metacarpophalangeal joints and less correlation with intes­
Obtaining a serum urate level may assist in the diag­ tinal inflammation. IBD-associated arthritis is also unlikely
nosis of this patient because an elevated level (>6.8 mg,dL because this patient has no symptoms of bowel disease.
(0.40 11111101 L]) would help support a diagnosis of gout. Reactive arthritis (formerly known as Reiter syndrome)
However. this test is not deAnitive for the diagnosis. An ele­ is a postinfectious arthritis triggered by infections causing
vated level does not prove that the patient has gout because urethritis or diarrhea, although patients may be asymptom­
asymptomatic hyperuricemia is common in the general pop­ atic. Arthritis, usually oligoarticular, develops several days
ulation. A relatively low serum urate level also does not to weeks after the infection. Reactive arthritis can cause
exclude gout because serum urate levels can be p�1radoxi­ dactylitis; however, this patient has no history of a preceding
cally low during acute gout attacks. infection, making this an unlikely diagnosis.
Starting colchicine would be an option for the treat­
ment of acute gout in this patient. and resolution of'inflam­ KEY POINT

mation with colchicine may in fact support a diagnosis or • Psoriatic arthritis is associated with psoriasis, enthesi­
crystal-induced arthritis. However. the diagnosis must Arst tis, dactylitis, tenosynovitis, arthritis of the distal
be established and infectious arthritis excluded. interphalangeal joints, asymmetric oligoarthritis, and
KEY POINT
spondylitis.
• Analysis of synovial fluid from joint aspiration is Bibliography
the gold standard to diagnose gout and exclude Coates LC. Conaghan PG. Emery P. et al. Sensitivity and specificity of the
infection. classification of psoriatic arthritis criteria in early psoriatic arthritis.
Arthritis Rheum. 2012 Oct,64 (10):3150-5. [PMID: 22576997]

130
Answers and Critiques

Cl Item 27 Answer: B
Educational Objective: Diagnose eosinophilic granulo­
Nonetheless, treatment should be started as soon as possible
to prevent complications of active disease, improve symptoms,
matosis with polyangiitis. and have a better long-term prognosis. Treatment should not
be withheld if the biopsy is delayed or cannot be obtained.
This patient most likely has eosinophilic granulomatosis with TI1e initial treatment of polymyositis or dermatomyositis with
polyangiitis (EGPA: formerly known as Churg-Strauss syn­ muscle involvement is systemic glucocorticoids, most com­
drome). EGPA is characterized by eosinophilia. migratory monly prednisone given at 1 mg/kg/d. Some patients with
pulmonary infiltrates, purpuric skin rash, and mononeuritis severe disease require treatment with intravenous methyl­
multiplex in the setting of antecedent atopy. Although EGPA prednisolone, and many physicians use methotrexate or aza­
is considered an ANCA-associated vasculitis (specifically, thioprine at onset for their glucocorticoid-sparing benefits. If
antimyeloperoxidase/p-ANCA), 40% of patients with EGPA refractory or recurrent disease is noted, additional agents such
are negative for ANCA. This patient has involvement of the as mycophenolate mofetil, intravenous immune globulin, rit­
lungs (likely capillaritis with hemoptysis), nerves (mononeu­ uximab, cyclophosphamide, or tumor necrosis factor (TNF)-a
ritis multiplex), and skin. In the presence of systemic eosin­ inhibitors can also be considered.
ophilia, elevated serum lgE levels, and a history of asthma. The TNF-a inhibitor adalimumab is not recommended
this patient's most likely diagnosis is EGPA. Other systemic as initial therapy prior to a trial of prednisone with or with­
multiorgan system diseases are less Likely. For example, gran­ out a glucocorticoid-sparing agent in patients with poly­
ulomatosis with polyangiitis (GPA: formerly known as Wege­ myositis or dermatomyositis. TNF-a inhibitors have been
ner granulomatosis) and microscopic polyangiitis would be reported to be effective in some patients with refractory
unusual in the absence of ANCA and the presence of eosin­ disease.
ophilia (and. in the case of GPA, lack of sinus involvement). Cyclosporine is an immunosuppressant agent that
Diagnosis of EGPA can be made most definitively by tissue preferentially targets T cells and demonstrates efficacy in
biopsy. Sura! nerve biopsy is most likely to yield pathology several rheumatologic and autoimmune diseases, including
specific for EGPA, specifically, the presence of necrotizing rheumatoid arthritis, systemic lupus erythematosus, inflam­
vasculitis with eosinophilic granulomas. In addition. confir­ matory myositis, psoriasis, pyoderma gangrenosum, and
mation that the patient's neurologic findings are due to EGPA inflammatory bowel disease. Toxicity is relatively common
is impo11ant for characterizing the extent of disease and the (hypertension, nephrotoxicity, tremor, hirsutism); therefore,
level of immunosuppression needed. Such a biopsy would cyclosporine is mainly used as a third-line agent in rheuma­
also allow the exclusion of other eosinophilic diseases. such tologic diseases.
as hypereosinophilic syndrome. Leflunomide is approximately as effective as methotrex­
Although cryoglobulinemia commonly affects the ate for rheumatoid arthritis; its use in other diseases is less
nerves and skin (along with the kidneys, which are not explored. Toxicities include liver and hematopoietic abnor­
involved in this patient). it uncommonly affects the lungs. malities, infection, and interstitial lung disease. This agent
Furthermore. the high level of eosinophilia is inconsistent has an extremely long half-life (months) and undergoes
with cryoglobulinemia. enterohepatic circulation.
KEY POINT KEY POINT
• Eosinophilic granulomatosis with polyangiitis is char­ • The initial treatment of polymyositis or dermatomy­
acterized by eosinophilia, migratory pulmonary infil­ ositis with muscle involvement is glucocorticoids,
trates, purpuric skin rash, and mononeuritis multi­ most commonly prednisone.
plex in the setting of antecedent atopy.
Bibliography
Bibliography Amato AA, Griggs RC. Treatment of idiopathic inflammatory myopathies.
Mahr A, Moosig F. Neumann T. et al. Eosinophilic granulomatosis with Curr Opin Neurol. 2003 Oct:16(5):569-75. [PMID: 14501840)
polyangiitis (Churg-Strauss): evolutions in classification, etiopathogen­
esis. assessment and management. Curr Opin Rheumatol. 2014
Jan:26(1):16-23. [PMID: 24257370] Item 29 Answer: C
Educational Objective: Diagnose diffuse idiopathic
skeletal hyperostosis.
Item 28 Answer: D
The most likely diagnosis is diffuse idiopathic skeletal hyper­
Educational Objective: Treat polymyositis using
ostosis (DISH), which is defined by the presence of flowing
prednisone.
osteophytes involving the anterolateral aspect of the thoracic
Treatment with prednisone is indicated. This patient's find­ spine at four or more contiguous vertebrae with preservation
ings of wealkness, elevated muscle enzymes, electromyogram of the intervertebral disk space and the absence of apophyseal
and MRI abnormalities, and no skin involvement suggest joint or sacroiliac inflammatory changes such as erosions.
polymyositis. A definitive diagnosis can only be made by mus­ DISH may occur with or without osteoarthritis or inflamma­
cle biopsy, and every effort should be done to obtain it quickly. tory arthritis and represents a separate finding of calcification

131
Answers and Critiques

and ossification of spinal ligaments and the regions where vated aminotransferase levels. a history of' possible exposure
tendons and ligaments attach to bone (entheses). DISH is a risk (works as a home health aide), and the presence of a rash
noninflammatory condition of unknown cause that is com­ point toward the acute prodromal arthritis of HBV infection.
mon in the elderly population. Patients may be asymptomatic which often presents before frank jaundice. Testing to docu­
or may describe stiffness and reduced range of motion, par­ ment acute HBV infection, specifically assessment of hepati­
ticularly at the thoracic spine. tis B core lgM and hepatitis B surface antigen. is indicated in
Ankylosing spondylitis is an inflammatory disorder this patient.
that usually becomes symptomatic in adolescence and early Hemochromatosis is characterized by excessive body
adulthood. It is characterized by progressive morning stiff­ stores of iron and can cause joint symptoms and liver dys­
ness and low back pain and typically becomes symptomatic function: unlike HBV-associated arthritis. it tends to occur
in the lumbar spine rather than the thoracic spine early in gradually rather than abruptly. and the arthritis tends to
the course. Radiographs of DISH and ankylosing spondylitis resemble osteoa11hritis without frank synovitis. Hemochro­
have similarities; however, ankylosing spondylitis demon­ matosis is also uncommon in women. who are protected
strates vertical bridging syndesmophytes rather than the from iron accumulation due to menstruation.
flowing osteophytes that occur in DISH. Plain radiographs of Primary biliaty cirrhosis and autoimmune hepatitis can
ankylosing spondylitis also characteristically show changes cause acute liver enzyme elevation but do not cause acute
of the sacroiliac joints that can include erosions, evidence arthritis or rash.
of sclerosis, and widening, narrowing, or partial ankylosis.
KEY POINT
Degenerative disk disease is thought to arise from
age-related changes in proteoglycan content in the nucleus • The prodromal stage of hepatitis B virus infection is
pulposus of the disk. Disks shrink as they become desic­ characterized by rapid-onset symmetric polyarthritis,
cated and more friable. Age-related changes also occur in which is often present before frank jaundice.
the annulus fibrosus, which becomes more fibrotic, less
elastic, and can shift its position. Vertebral body endplates Bibliography
adjacent to the disk develop sclerosis, and osteophyte for­ Vassilopoulos D, Calabrese LH. Viral hepatitis: review of arthritis complica­
tions and therapy for arthritis in the presence of active HBV/HCV. Curr
mation occurs at the vertebral margins. Rheumatol Rep. 2013 Apr:15(4):319. [PMID: 23436024]
Psoriatic arthritis is an inflammatory disorder that
can affect the spine as well as peripheral joints. When
axial involvement is prominent, sacroiliitis and spondy­ Item 31 Answer: C
litis can both be present; however, axial disease rarely
presents in the absence of frank inflammatory arthritis of Educational Objective: Diagnose vasculitis as a cause
peripheral joints. of scleritis.

KEY POINT The most likely diagnosis associated with this patient's scleri­
tis is granulomatosis with polyangiitis (formerly known as
• Diffuse idiopathic skeletal hyperostosis is a nonin­
Wegener granulomatosis). Scleritis is inflammation of the
flammatory condition defined by the presence of fibrous layer of the eye underlying the conjunctiva and epis­
flowing osteophytes involving the anterolateral aspect clera. Scleritis can be caused by autoimmune diseases such as
of the thoracic spine at four or more contiguous verte­ rheumatoid arthritis, relapsing polychondritis, inflammatory
brae with preservation of the intervertebral disk space bowel disease, or vasculitis. This patient has evidence of sys­
and the absence of apophyseal joint or sacroiliac temic inflammation on laboratory testing, hypertension, and
inflammatory changes such as erosions. an abnormal urinalysis that could represent glomerulone­
phritis but no clinical evidence of rheumatoid arthritis, chon­
Bibliography dritis, or inflammatory bowel disease. Vasculitis, especially
Mader R. Verlaan JJ, Buskila D. Diffuse idiopathic skeletal hyperostosis: ANCA-associated vasculitis, should always be considered in
clinical features and pathogenic mechanisms. Nat Rev Rheumatol. 2013
Dec:9(12):741-50. [PMID: 241898401 the differential dfagnosis of scleritis because delay in diagnosis
can result in permanent loss of vision. In this case, the sinus
symptoms, abnormal urinalysis suggestive of glomerulone­

CJ Item 30 Answer: C
Educational Objective: Diagnose the acute prodromaJ
phritis, and scleritis are suggestive of granulomatosis with
polyangiitis.
arthritis of hepatitis B virus infection. Ankylosing spondylitis is characterized by inflamma­
tory back pain that usually presents during the second to
The most likely diagnosis is hepatitis B virus (HBV )-associ­ third decade of life. The type of ocular inflammation most
ated arthritis. The patient has rapid-onset symmetric poly­ commonly associated with this entity is anterior uveitis, not
arthritis. rash. and elevated aminotransferase levels. which scleritis. Uveitis (inflammation of the uvea) commonly pres­
are consistent with the prodromal phase of HBV infection. ents as a red eye with pain, photophobia, and blurred vision.
The differential diagnosis of this symmetric pattern encom­ Anterior uveitis is characterized by circumferential redness
passes numerous arthritides: however, the presence of ele- (ciliary flush) at the corneal limbus Uunction of the cornea

132
Answers and Critiques

and sciera). Furthermore, the duration of this patient's back gouty attack. and it will not treat an iniected joint. It may be
pain (which began after age 45 years) and evidence of osteo­ indicated long term if the patient has persistently elevated
arthritis in her fingers suggest degenerative, rather than serum urate with recurrent attacks of gout.
inflammatory, disease of the spine. An intra-articular glucocorticoid injection can be used
Behc;et syndrome is characterized by recurrent oral and/ to treat an acute crystalline arrack; however. if an infected
or genital ulcers, eye and skin involvement, and pathergy. joint were suspected. it would not be appropriate because
This patient has no clinical symptoms to suggest this diagno­ it may worsen infection. It is therefore inappropriate at this
sis, and it is more commonly associated with uveitis rather time because the cause of this patient's acute inflammatory
than scleritis. monoarthritis has not been determined.
Sarcoidosis is a multisystem disease characterized by Similarly. prednisone should not be started because it
granulomas that form in tissues and most commonly affects may cause worsening of infection. Prednisone is sometimes
the lungs. Sarcoidosis is more likely to cause uveitis than used short term to treat acute attacks of gout. but if a single
scleritis, and this patient has no other clinical symptoms large joint is involved. a local injection would avoid the sys­
such as respiratory complaints or an abnormal chest radio­ temic side effects of' prednisone therapy.
graph to suggest this diagnosis.
KEY POINT
KEY POINT
• Empiric therapy with antibiotics should be started
• Scleritis can be caused by autoimmune diseases such immediately in the setting of an acute inflammatory
as rheumatoid arthritis, relapsing polychondritis, monoarthritis if infection is suspected, even if not yet
inflammatory bowel disease, or vasculitis. confirmed on cultures.

Bibliography Bibliography
Sims J. Scleritis: presentations, disease associations and management. Yu KH. Luo SF. Liou LB. et al. Concomitant septic and gouty arthritis-an
Postgrad Med J. 2012 Dec;88(1046):713-18. [PMID: 22977282] analysis of 30 cases. Rheumatology (Oxford). 2003 Sep:42(9):1062-6.
[PMID: 12730521]

C] Item 32 Answer: D
Educational Objective: Treat a suspected infected joint Item 33 Answer: E
in the setting of gout. Educational Objective: Monitor ankylosing spondylitis
disease activity with physical examination.
Antibiotics are appropriate for this patient who has an acute
inflammatory monoarthritis. possibly infectious arthritis. No new imaging is required for this patient with ankylos­
Acute crystalline attack can also cause fever and an inflam­ ing spondylitis. She is currently feeling well, continues to
matory monoarthritis. but a negative Gram stain and,or the respond well to treatment, and has normal inflammatory
presence of crystals do not rule out infection. If' infection is markers, making new imaging unnecessary. As with any test,
suspected. even if not yet confim1ed on cultures. empiric imaging to follow disease activity should be performed only
therapy with antibiotics should be started without delay. The if clearly indicated by the clinical situation (for example, if
crystals seen in this patient's synovial fluid are extracellular. the result is likely to change management). According to the
which is consistent with a diagnosis or history of gout but not 2010 Assessment of SpondyloArthritis international Soci­
diagnostic of an acute gouty attack; only intracellular crystals ety/European League Against Rheumatism (ASAS/EULAR)
are diagnostic of an acute crystalline attack. In the absence of guidelines, serial imaging of patients with ankylosing spon­
positive findings on Gram stain. initial empiric therapy usu­ dylitis can be part of a comprehensive monitoring plan that
ally includes coverage for gram-positive organisms (including also includes patient history (such as questionnaires like
methicillin-resistant Staphylococcus aureus [M RSA], which the Bath Ankylosing Spondylitis Functional Index [BASF!]
is increasing in prevalence in many communities) as well as or Bath Ankylosing Spondylitis Disease Activity Index
coverage for gram-negative organisms if immunocompro­ [BASDAl]), clinical parameters (such as physical examina­
mised. at risk for gonococcal infection. or with trauma to the tion findings like the Schober test), and laboratory tests
joint. Therapy can be adjusted once results from stains and (such as erythrocyte sedimentation rate and C-reactive pro­
cultures are available. tein). The ASAS/EULAR recommendations state that spinal
Probenecid is sometimes added lo allopurinol to control radiography should not be repeated more frequently than
gout when allopurinol alone is insufficient. If the patient every 2 years unless absolutely necessary in specific cases.
were to experience persistent gout. increasing allopurinol The Schober test measures range of motion of the lumbar
to lower the serum urate below 6.0 mg/dL (0.35 mmol/L) spine and is an inexpensive and noninvasive physical exam­
would be more appropriate than combination therapy with ination tool for assessing spine involvement and progression;
two agents. greater than 4 cm is normal.
Allopurinol is considered a first-line agent for serum Bone scan, CT, MRI, and plain radiography may demon­
urate reduction in patients with gout. Increasing this strate evidence of inflammation and/or progression of dis­
patient's allopurinol will not be helpful in treating an acute ease, but without a clear indication may unnecessarily

133
Answers and Critiques

expose the patient to radiation and to expense. This patient Bibliography


is feeling well, has intact activities of daily living, and had Hu LT. In the clinic. Lyme disease. Ann Intern Med. 2012 Aug 7:157(3):ITC2-
2-16. [PMID: 22868858]
radiographs less than 2 years ago; therefore, no additional
imaging is necessary at this time.
KEY POINT
Item 35 Answer: D
• Patients with ankylosing spondylitis who are
Educational Objective: Treat a patient with drug­
responding well to treatment should be monitored
induced lupus erythematosus caused by minocycline.
clinically and do not require periodic imaging
studies less than every 2 years unless absolutely Discontinuation of minocycline is indicated for this patient.
necessary. Minocycline is one of the known causes of drug-induced
lupus erythematosus (DILE). Criteria for DILE include a
Bibliography positive antinuclear antibody (ANA) test, exposure to a
Braun J. van den Berg R. Baraliakos X. et al. 2010 update ofrhe ASAS/EULAR drug associated with DILE, and at least one clinical feature
recommendations for the management of ank-ylosing spondylitis. Ann of lupus in a patient without a known history of lupus.
Rheum Dis. 2011 Jan;70(6):896-904. [PMID: 21540199]
Common symptoms include malaise, fever, arthritis, and
rash. Diagnosis is typically confirmed when symptoms
Item 34 Answer: B resolve several weeks to months after discontinuation of
the offending agent. The agents classically associated with
Educational Objective: Diagnose Lyme arthritis.
DILE, such as procainamide and methyldopa, are not com­
Serologic testing for Lyme disease is appropriate for this monly used at present. However, the spectrum of DILE
patient with arthritis characterized by prominent swelling includes other medications that are more commonly used,
with stiffness without significant joint pain. He has a risk including hydralazine, diltiazem, isoniazid, minocycline,
factor for Lyme arthritis, given his frequent hiking in an and certain tumor necrosis factor a inhibitors (such as
endemic area. Patients may not recall a tick bite; therefore, infliximab and etanercept). Other agents that possibly
Lyme disease should be suspected even without this history. cause DILE include specific anticonvulsants, antithyroid
The knee is most commonly affected, although other large agents, and certain antibiotics.
joints can also be involved, usually in a monoarticular or The diagnostic laboratory evaluation for DILE is simi­
oligoarticular pattern. Serologic testing for Borrelia burg­ lar to that for patients with suspected idiopathic systemic
dorferi is the diagnostic test of choice for this disease and lupus erythematosus. Antinuclear antibodies are typically
is typically done with an enzyme-linked immunosorbent positive, whereas anti-double-stranded DNA antibodies
assay (ELISA) screening test, followed by confirmation by are usually negative in DILE, as are most other lupus­
Western blot. associated extractable nuclear autoantibodies. Antihis­
Blood cultures should always be obtained when there tone antibodies have traditionally been associated with
is a suspicion for bacterial infectious arthritis. This patient DILE caused by older medications, but their presence may
does not have risk factors for infectious arthritis. with be more variable with DILE induced by newer agents. In
Lyme being the most compelling diagnosis. Arthrocentesis addition to causing DILE, certain medications, such as
is recommended for routine synovial fluid analysis and minocycline and hydralazine, may be associated with a
symptomatic relief, but blood cultures will not provide the p-ANCA syndrome that may also cause a small- to medium­
correct diagnosis. vessel vasculitis with organ involvement. Treatment of
An MRI of the knee is unlikely to have diagnostic DILE, regardless of offending agent, requires discontinu­
benefit in new-onset Lyme arthritis (and even in recurrent ation of the drug. The patient can also be treated symp­
cases, it tends to be a nonerosive arthritis). MRI can be tomatically until manifestations resolve. An NSAID such
helpful in cases in which mechanical damage needs to be as naproxen may be helpful, as can low-dose prednisone.
excluded or if there is concern for osteomyelitis or other Rarely, more substantial therapy may be needed if there is
bone pathology. internal organ involvement.
Synovial fluid cultures tend to be negative in Lyme Neither azathioprine nor high-dose prednisone is indi­
arthritis. Synovial fluid will show inflammatory fluid with a cated in this patient because she does not have evidence of
neutrophil predominance. B. burgdorferi can be detected by internal organ involvement.
polymerase chain reaction in synovial fluid. Famotidine has not been associated with DILE.
KEY POINT KEY POINT

• In patients with risk factors for Lyme arthritis (even • The diagnosis of drug-induced lupus erythematosus is
without a history of a tick bite), serologic tests show­ typically confirmed when symptoms resolve several
ing an immunologic response to Borrelia burgdorferi weeks to months after discontinuation of the offend­
are indicated to establish the diagnosis. ing agent.

134
Answers and Critiques

Bibliography Bibliography
Chang C. Gershwin ME. Drug-induced lupus erythematosus: incidence, Engel WK, Askanas V. Inclusion body myositis: clinical, diagnostic, and
management and prevention. Drug Saf 2011 May 1;34(5):357-74. lPMID: pathologic aspects. Neurology. 2006 Jan 24;66(2 Suppl l):S20. [PMlO:
21513360] 164321411

Item 36 Answer: B Item 37 Answer: C


Educational Objective: Diagnose inclusion body myositis. Educational Objective: Diagnose knee osteoarthritis
The most likely diagnosis is inclusion body myositis (IBM), an with standing plain radiography.
insidious and slowly progressive inflammatory myopathy that Standing plain radiography is appropriate for this patient
occurs more commonly in men and in those over the age of 50 who most likely has osteoarthritis. Osteoarthritis is often
years. Muscle weakness may be diffuse and involve both the clinically identifiable by the presence of bony hypertrophy in
distal and proximal muscles. Although typically sy mmetric, a characteristic pattern of joint involvement and the absence
IBM muscle involvement may be asymmetric in up to 15% of overt inflammatory synovitis. The history generally con­
of patients. Skin is generally spared. IBM is rarely associated sists of long-standing and gradually worsening symptoms
with extramuscular manifestations such as rash, fever, or pul­ in middle-aged or older patients. Confirmatory plain radio­
monary involvement. Patients with IBM typically have only graphs are appropriate to solidify the diagnosis and rule out
mildly elevated (typically <1000 U/L), or even normal, levels less common findings such as osteonecrosis, fractures, or
of muscle enzymes. The characteristic triad of electromyo­ malignancies. They are also noninvasive, widely available,
graphic findings for myopathy includes short-duration, small, and the least expensive radiographic modality. Standing views
low-amplitude polyphasic potentials; fibrillation potentials at of the knees can demonstrate a more accurate picture of the
rest; and bizarre, high-frequency, repetitive discharges. 'This joint-space narrowing that is present during functioning,
older male patient has developed slowly progressive weak­ including standing and walking, than radiographs that are
ness affecting both the proximal and distal muscles without obtained supine.
any significant pain or stiffness. This presentation suggests a Bone scintigraphy can visualize areas of bone turnover
myopathy with weakness based on his history and physical change due to osteophyte formation, subchondral sclerosis,
examination, mild elevation of muscle enzymes, and abnor­ subchondral cyst formation, and bone marrow lesions. How­
mal electromyogram (EMG) results, all of which are most ever, its limited anatomic resolution and the use of ionizing
consistent with IBM. radiation make it less useful for the diagnosis of osteoarthritis.
Amyotrophic lateral sclerosis is characterized by pro­ MRI is capable of demonstrating numerous findings
gressive dysfunction of both upper motoneuron and lower in soft tissue and cartilage that provide information about
motoneuron pathways in one or more areas of the body. the joint as a whole organ. As such, MRI is a critical tool in
Common upper motoneuron features are spasticity, hyper­ osteoarthritis research, but its high cost argues against its
reflexia, and pathologic reflexes, including extensor plantar routine use, particularly because there are no end points
responses. Typical lower motoneuron features are muscle apart from joint-space narrowing (easily assessed on plain
weakness, atrophy, fasciculations, and cramps. These find­ radiographs as well) visualized on MRI that confer prognos­
ings are not present in the patient. tic information.
Myasthenia gravis is characterized by fluctuating, fat­ Although ultrasonography is noninvasive and is
igable muscle weakness that worsens with activity and appealing because it provides real-time information, its
improves with rest. Neurologic examination may reveal primary use in the management of osteoarthritis is for
bilateral asymmetric ptosis worsened by prolonged upward needle placement in difficult arthrocenteses. Limitations
gaze, an expressionless or sagging appearance of facial mus­ of ultrasonography include that it is an operator-depen­
cles, a "snarling" smile, nasal speech worsened by prolonged dent technique and that the physical properties of sound
speaking, and limb weakness that increases with exercise. limit its ability to assess deep articular structures and the
None of these findings are present in this patient. subchondral bone.
Statin-induced myopathy most commonly presents
KEY POINT
with muscle pain, tenderness, and cramping typically
within the first 6 months of therapy, and EMG results are • In patients with suspected osteoarthritis, confirma­
normal. This patient has no muscle pain, has an abnormal tory plain radiographs with standing views are appro­
EMG, and has been taking a stable dose of a stain for years. priate to solidify the diagnosis and rule out less com­
making statin-induced myopathy unlikely. mon findings such as avascular necrosis, fractures,
and malignancies.
KEY POINT

• Inclusion body myositis has an insidious onset, with Bibliography


muscle weakness that may be diffuse and involve both Guermazi A. Hayashi D. Roemer FW. OT Felson. Osteoarthritis: a review of
the distal and proximal muscles. strengths and weaknesses of different imaging options. Rheum Dis Clin
N Am. 2013 Aug;39(3):567-91. [PMID: 23719076]

135
Answers and Critiques

Item 38 Answer: B with Marfan syndrome. This autosomal dominant condition


is characterized by a mutation in the gene FBNl responsible
Educational Objective: Treat dry eyes in a patient with
for producing fibrillin 1, a structural protein in tissues that
Sjogren syndrome.
contain elastic fibers, such as the arterial wall. Clinical fea­
Cyclosporine drops are appropriate for this patient. She has tures include tall stature, arachnodactyly, anterior thoracic
primary Sjogren syndrome with dry eyes (keratoconjunctivitis deformity, spinal curvature, and skin hyperextensibility.
sicca), which has not responded to topical lubrication or punc­ The most common cause of morbidity and mortality is
tal plugs. Sjogren syndrome is an immune-mediated disease of aortic root dilatation with possible dissection, rupture, or
unknown cause manifesting primarily as inflammation of exo­ aortic valve insufficiency. Current guidelines recommend
crine glands, including the major and minor salivary glands, that echocardiography should be performed at the time
lacrimal glands, and, less commonly, other exocrine glands of diagnosis to determine aortic root and ascending aortic
such as the pancreas. The most prominent clinical feature is diameters and 6 months later to determine their rate of
dryness (sicca), particularly of the eyes and mouth. Dry eyes enlargement. Annual imaging is recommended if stability
can lead to corneal damage and visual impairment. Sicca symp­ of the aortic diameter is documented. If the maximal aortic
toms are primarily treated with hydration and lubrication, diameter is 4.5 cm or greater, more frequent imaging should
although other local measures and medications may be helpful. be considered.
Topical cyclosporine has been demonstrated in trials to improve Patients with Marfan syndrome may develop abdom­
the symptoms of dry eyes in patients with primary Sjogren syn­ inal aortic aneurysm, but thoracic involvement or thoracic
drome. An alternative therapy is punctal occlusion (placement and abdominal aneurysms are much more common. There
of plugs in the tear drainage duct openings of the lower eyelids currently are no recommendations from any organization
to increase eye moisture). There is controversy as to the timing to routinely screen these patients with abdominal ultraso­
and type of plug to use when performing this procedure. nography.
Certolizumab pegol is a tumor necrosis factor (TNF)-a Apical lung bullae can form and lead to pneumothorax
inhibitor used to treat diseases such as rheumatoid arthritis, in patients with Marfan syndrome; however, there are no
psoriatic arthritis, and ankylosing spondylitis. This agent is data to suggest that monitoring with annual chest radiogra­
not appropriate for this patient because trials of other TNF-a phy has any impact on morbidity or mortality.
inhibitors (etanercept and infliximab) did not demonstrate Likewise, routine monitoring for progressive skeletal
benefit for sicca symptoms. abnormalities, including annual spine radiography to mon­
Hydroxychloroquine is used to treat diseases such as itor this patient's scoliosis, is not indicated because it has
systemic lupus erythematosus and rheumatoid arthritis. This no impact on outcome and should be evaluated only when
agent has not been demonstrated to improve sicca symp­ symptomatic. However, counseling regarding joint protec­
toms, although it could be useful for treating arthritic and tion to prevent osteoarthritis is useful.
other systemic symptoms associated with Sjogren syndrome.
KEY POINT
Olopatadine drops reduce histamine release from mast
cells and are used to treat allergic conjunctivitis; they would • In patients with Marfan syndrome, echocardiography
therefore not be helpful for Sjogren-related sicca symptoms. should be performed at the time of diagnosis to deter­
Prednisone is a potent anti-inflammatory agent that mine aortic root and ascending aortic diameters and
is very effective in many rheumatologic diseases. It has not 6 months later to determine their rate of enlargement.
been demonstrated to improve sicca symptoms, although
it could be useful for treating systemic manifestations of Bibliography
Sjogren syndrome. Dean JCS. Marran·s syndrome: clinical diagnosis and management 2007
Jul:15(7):727-33. [PMID: 17487218]
KEY POINT

• Topical cyclosporine improves the symptoms of dry


Item 40 Answer: A
eyes in patients with primary Sjogren syndrome.
Educational Objective: Treat a patient who has hyper­
Bibliography sensitivity vasculitis.
Ramos-Casals M. Tzioufas AG. Stone JH. Sis6 A. Bosch X. Treatment of' pri­
mary Sjogren syndrome: a systematic review. JAMA. 2010 Jul 28:304(4):
Discontinuation of trimethoprim-sulfamethoxazole is
452-60. [PMID: 20664046] indicated for this patient who developed palpable purpura
within days of starting trimethoprim-sulfamethoxazole for
a urinary tract infection. The most likely diagnosis is hyper­
Item 39 Answer: C sensitivity vasculitis, which is caused by a hypersensitivity
reaction to antigens such as medication or infection. Hyper­
Educational Objective: Monitor a patient with Marfan
sensitivity vasculitis typically resolves when the offending
syndrome using echocardiography.
agent is removed. Similar to other forms of small-vessel
Echocardiography to evaluate for aortic root dilatation is the vasculitis, hypersensitivity vasculitis results from anti­
most appropriate routine monitoring test for this patient bodies directed toward the antigens that result in immune

136
Answers and Critiques

complex formation. Complement is activated, and neutro­ toxicity. Azathioprine has generally been supplanted by the
phils are attracted to accumulate in capillaries, arterioles, use of mycophenolate mofetil in SLE.
and post-capillary venules. Although a similar reaction asso­ Mycophenolate mofetil may be appropriate for this
ciated with the infection itself may occur, this is less likely patient if she had more serious disease activity such as
given the time course of onset and the high degree of associ­ nephritis or if her arthritis or pericarditis recurred while
ation of trimethoprim-sulfamethoxazole with this reaction. taking hydroxychloroquine.
Because her reaction is mild and limited to the skin, the most NSA!Ds, often with colchicine, are first-line therapy for
important aspect of treatment is to remove the likely offend­ most patients with pericarditis, although glucocorticoids
ing agent, after which the condition will resolve. may be indicated in patients with pericarditis associated
Prednisone has shown efficacy in hypersensitivity vas­ with a systemic inflammatory disease such as in this patient.
culitis and can be used if the vasculitis is severe, is causing However, there is no indication to start an NSAID now given
discomfort of the skin or other organ damage, or if it is resolution of her symptoms, and doing so would increase
imperative that the agent be continued despite the reactions. her risk of gastrointestinal complications if used along with
Because this is not the case, discontinuing the offending her daily glucocorticoid.
agent remains the highest priority.
KEY POINT
Antihistone antibodies occur frequently in patients
with either systemic or drug-induced lupus erythematosus. • Antimalarial therapy such as hydroxychloroquine in
This patient's disease is more consistent with hypersensitiv­ systemic lupus erythematosus (SLE) has documented
ity vasculitis without specific features of drug-induced lupus benefit for reducing disease activity, improving sur­
(for example, malar rash); however, even if drug-induced vival, and reducing the risk of SLE-related thrombosis
lupus were the likely diagnosis, discontinuing the offending and myocardial infarction.
agent would still be the highest priority.
Palpable purpura is a clinical diagnosis for which a Bibliography
skin biopsy is not generally needed. A skin biopsy could be Lee SJ, Silverman E, Bargman JM. The role of antimalarial agents in the
useful in cases where the mechanism of the rash needs to treatment of SLE and lupus nephritis. Nat Rev Nephrol. 2011 Oct
18;7(12):718-29. [PMID: 22009248]
be understood to facilitate diagnosis or treatment, or when
the rash fails to resolve despite management. In this case, the
cause of the rash is known, and management should consist Item 42 Answer: C
of removal of the offending agent before any further work-up
is considered. Educational Objective: Treat gout with urate-lowering
therapy.
KEY POINT
Initiation of allopurinol is appropriate for this patient with
• Hypersensitivity vasculitis is caused by a hypersensi­ gout who has had two attacks of podagra within the past year.
tivity reaction to antigens such as medication or The American College of Rheumatology (ACR) guidelines cur­
infection and typically resolves when the offending rently recommend that mate-lowering therapy should be ini­
agent is removed. tiated in patients with gout who have had two or more attacks
within a 1-year period, one attack in the setting of chronic
Bibliography kidney disease of stage 2 or worse, one attack with the pres­
Chen KR. Carlson JA. Clinical approach 10 cutaneous vasculitis. Am J Clin ence of tophi visible on examination or imaging, or one attack
Dermatol. 2008;9(2):71-92. [PMID: 18284262)
with a history of urolithiasis. Allopurinol, a xanthine oxidase
inhibitor, is an appropriate first-line agent for urate reduction.
Item 41 Answer: B Flare prophylaxis should be maintained when mate-lowering
therapy is undertaken.
Educational Objective: Treat mild systemic lupus
Low-dose aspirin can increase serum urate due
erythematosus.
to effects on renal uric acid transport; however, the ACR
Hydroxychloroquine is an appropriate agent to address does not currently recommend aspirin discontinuation in
milder systemic manifestations of systemic lupus erythe­ patients for whom it is indicated, such as this patient with
matosus (SLE) such as arthritis and pericarditis, and it can coronary artery disease.
act as a glucocorticoid-sparing agent. All patients with SLE This patient needs to begin urate-lowering therapy, and
who can tolerate it should be taking hydroxychloroquine. his flare prophylaxis (colchicine) should be maintained during
Antimalarial therapy such as hydroxychloroquine in SLE has this period given the paradoxical increased risk of flare during
documented benefit for reducing disease activity, improving acute serum urate reduction. In the absence of active disease,
survival, and reducing the risk of SLE-related thrombosis the ACR currently recommends that prophylaxis should be
and myocardial infarction. continued for the greater of the following: 6 months; 3 months
Azathioprine is generally reserved for more severe man­ after achieving the target serum urate level for a patient with­
ifestations of SLE not responsive to low-dose prednisone out tophi; or 6 months after achieving the target serum urate
and hydroxychloroquine but can be associated with serious level where there has been resolution of tophi.

137
Answers and Critiques

Probenecid is a uricosuric agent (promotes kidney uric acterizes early RA. Rheumatoid factor positivity is character­
acid excretion) and is a viable alternative first-line urate-low­ istic of RA, although rheumatoid factor has a low specificity
ering agent in patients who cannot tolerate or have a contra­ for diagnosis of RA. Rheumatoid factor may be present in
indication to xanthine oxidase inhibitor therapy. However, healthy persons, especially at older ages. Because this patient
allopurinol is generally more appropriate in patients such as has no clinical evidence of RA, testing for rheumatoid factor
in this case who have no contraindication to urate-lowering is unnecessary.
therapy. Moreover, the patient's history of urolithiasis makes KEY POINT
probenecid relatively contraindicated because this medica­
tion increases the risk of kidney stones. • Additional testing such as autoantibody measure­
ments or radiography is unnecessary in patients with
KEY POINT clinically diagnosed hand osteoarthritis.
• Urate-lowering therapy should be initiated in patients
with gout who have had two or more attacks within a Bibliography
I-year period, one attack in the setting of chronic kid­ Hunter DJ. ln the clinic. Osteoarthritis. Ann Intern Med. 2007 Aug
7;147(3):lTCS-1-16. [PMlD: 17679702]
ney disease of stage 2 or worse, one attack with the
presence of tophi visible on examination or imaging,
or one attack with a history of urolithiasis. Item 44 Answer: D
Educational Objective: Treat polymyalgia rheumatica
Bibliography
with low-dose prednisone.
Khanna D. Fitzgerald JD, Khanna PP, et al; American College of
Rheumatology. 2012 American College of Rheumatology guidelines for
management of gout. Part l: systematic nonpharmacologic and pharma­
Treatment with prednisone, 15 mg/d, is appropriate. This
cologic therapeutic approaches to hyperuricemia. Arthritis Care Res 74-year-old man presents with shoulder and hip girdle pain
(Hoboken). 2012 Oct:64(10):1431-46. [PMID: 23024028) and limitation accompanied by signs of systemic inflamma­
tion, including low-grade fever, weight loss, malaise, and a
Item 43 Answer: E markedly elevated erythrocyte sedimentation rate (ESR). This
constellation of findings is classic for polymyalgia rheumat­
Educational Objective: Clinically diagnose osteoarthri­
ica (PMR), especially in this age group. Treatment of PMR,
tis of the hands.
typically prednisone initiated at a dose of 10 to 20 mg/d, is
No further testing is necessary for this patient who clinically warranted and should result in rapid resolution of symp­
appears to have hand osteoarthritis. Osteoarthritis is a clinical toms. Prednisone can be tapered over a 6-month period in
diagnosis, and the cardinal symptom is pain with activity that some patients, but others experience flares with tapering and
is relieved with rest. Affected patients also typically expe­ require more prolonged therapy, for as long as 1 to 3 years.
rience morning stiffness that lasts for less than 30 minutes Methotrexate can be tried as a glucocorticoid-sparing agent,
daily. Bony hypertrophy is commonly detected in the fingers, but studies suggest limited efficacy.
and Heberden and Bouchard nodes may be easily palpated. Low-dose aspirin (81 mg/d) may be useful to reduce
Osteoarthritis also may cause squaring or boxing of the car­ ocular complications in patients with giant cell arteritis
pometacarpal joint at the base of the thumb. (GCA), which can co-occur with PMR; however, this patient
This patient has no clinical signs or symptoms suggestive has no signs or symptoms consistent with GCA such as jaw
of a systemic inflammatory disease and therefore does not claudication, temporal headache, or visual loss.
require diagnostic testing with antinuclear antibodies (ANA) Aspirin, 650 mg three times daily, functions in a manner
or anti-double-stranded DNA antibodies. A positive ANA test similar to other traditional NSA1Ds, with analgesic, anti-inflam­
result has low predictive value when the pretest probability matory, and antipyretic effects. However, like other NSAIDs, it is
of systemic lupus erythematosus or a related disease is low. not a treatment for PMR and is not effective for this condition.
Therefore, this test should not be used to screen indiscrimi­ Ouloxetine is a dual serotonin-norepinephrine reuptake
nately for the presence of rheumatologic disease. The Amer­ inhibitor that is used as an antidepressant, to modulate
ican College of Rheumatology recommends not testing ANA pain due to fibromyalgia and other chronic central pain
subserologies such as anti-double-stranded DNA without the syndromes, and for chronic musculoskeletal pain. Although
combination of a positive ANA and elevated clinical suspicion the patient has pain and a depressed affect-a common con­
of autoimmune disease, which is not present in this patient. stellation in fibromyalgia-his pain is in a classic distribution
Radiography is not needed to confirm the diagnosis of for PMR and his depressed affect is common in patients with
osteoarthritis in patients with a history and physical exam­ PMR pain. Since low-dose prednisone will likely be curative,
ination compatible with this condition. Clinical examination duloxetine therapy should not be needed.
is more sensitive and specific for the diagnosis of hand osteo­ High-dose prednisone is indicated for GCA and severe
arthritis compared with radiography. or life-threatening forms of autoimmunity but carries a high
The key features of rheumatoid arthritis (RA) are swell­ rate of toxicity. This patient has no signs or symptoms of GCA
ing and tenderness in and around the joints. Prominent or any other disease except PMR; therefore, high-dose pred­
morning stiffness that usually lasts more than 1 hour char- nisone is not warranted.

138
Answers and Critiques

KEY POINT Bibliography


• Treatment of polymyalgia rheumatica typically con­ Caner JD. Inman RD. Chlamydia-induced reactive arthritis: hidden in plain
sight? Best Pract Res Clin Rheumatol. 2011 Jun:25(3):359-74. [PMID:
sists of low-dose prednisone, initially at 10 to 20 mg/d, 22100286]
which should result in rapid resolution of symptoms.
Item 46 Answer: A
Bibliography
Educational Objective: Treat inadequately controlled
Kermani TA, Warrington KJ. Polymyalgia rheumatica. Lancet. 2013 Jan
5:381(9860):63-72. [PMID: 23051717] rheumatoid arthritis.
Addition of a tumor necrosis factor (TNF)-a inhibitor such
Item 45 Answer: C as etanercept is indicated for this patient with inadequately
controlled rheumatoid arthritis (RA). He has been appropri­
Educational Objective: Diagnose chlamydia infection ately started on the recommended initial agent, methotrex­
associated with reactive arthritis. ate. with the dose appropriately titrated up because of con­
The most appropriate diagnostic test to perform next is a tinued disease activity. Symptomatic relief has been sought
DNA amplification urine test for Chlamydia trachomatis. with the use of prednisone and naproxen, but he continues to
This patient has a presentation consistent with reactive have active synovitis. Because he has been given an appropri­
arthritis (oligoarticular lower extremity dactylitis, kera­ ate dose of methotrexate for an adequate period of time, the
toderma blennorrhagicum). Up to 30% of patients with most appropriate next step is to add a TNF-a inhibitor such
reactive arthritis develop keratoderma blenorrhagicum, as etanercept. TNF-a inhibitors remain the most widely used
a hyperkeratotic rash found on the soles and palms that biologics for RA and are highly effective in the treatment of
may be indistinguishable from pustular psoriasis. Chla­ RA, leading to a 20% improvement in signs and symptoms of
mydia infection is a common cause of reactive arthritis disease within weeks for over half of patients.
and is often asymptomatic. Patients with reactive arthri­ Rituximab is indicated for use in patients with moderate
tis, even if asymptomatic, should be tested for chlamydia to severe RA who are also taking methotrexate but have not
because they may have persistent infection or carriage of responded to TNF-a inhibitors. Having never been treated with
this organism. If chlamydia is identified, these patients, as a TNF-a inhibitor, it is most appropriate to add a TNF-a inhib­
well as their partners, should receive treatment to prevent itor to this patient's regimen rather than rituximab. Other bio­
recurrence or transmission of infection. More studies are logics are available, and a number have different mechanisms
needed to determine whether antibiotics are helpful in of action and can be used in combination with methotrexate.
treating arthritic symptoms. Antibiotics are not indicated The patient has been on methotrexate since diagnosis
for non-chlamydia-related reactive arthritis unless there and is taking a dose that would be expected to improve his
is documentation of persistent infection because they have symptoms; however, he continues to have significant disease
not been shown to alter the course of arthritis. activity. It is unlikely that continuing to increase the dose
Anti-cyclic citrullinated peptide antibodies are associ­ will adequately control his disease; this will also increase
ated with rheumatoid arthritis, which is characterized by a the risk of toxicity.
symmetric small joint polyarthritis typically without dac­ Increasing prednisone may offer short-term relief of
tylitis. This patient does not have this typical presentation; flares in patients with RA. However, this patient has been
therefore, testing for these antibodies is not indicated. on chronic glucocorticoids and high-dose methotrexate, yet
Antinuclear antibodies are associated with systemic continues to have a considerable amount of synovitis. Given
lupus erythematosus (SLE); with the exception of arthritis, the chronic nature of RA and need for long-term treatment,
this patient has no clinical evidence of SLE, including alope­ exposing patients to the numerous side effects associated
cia, aphthous ulcers, pericardia! and pleural serositis, kidney with higher doses of glucocorticoids is not optimal. Fur­
disease, rash, and cytopenias, and does not need to be tested thermore, in this patient with known seropositive erosive
for these antibodies. disease, therapy with disease-modifying agents is required,
Although many patients with reactive arthritis are and prednisone does not halt bony destruction.
positive for HLA-B27, it will not add any further value KEY POINT
to the diagnosis or management. This patient has several
• In patients with inadequately controlled rheumatoid
typical features to establish a diagnosis of reactive arthri­
arthritis who are taking methotrexate, the addition of
tis, and management with anti-inflammatory medication
a tumor necrosis factor a inhibitor is appropriate to
for arthritis will be effective in the presence or absence of
HLA-B27. improve signs and symptoms of disease.

KEY POINT Bibliography


• Patients with reactive arthritis, even if asymptomatic, Singh JA, Furst DE, Bharat A, et al. 2012 Update of the 2008 American
College Of Rheumatology recommendations for the use of disease­
should be tested for chlamydia infection using a DNA modifying antirheumatic drugs and biologic agents in the treatment of
amplification urine test. rheumatoid arthritis. Arthritis Care Res (Hoboken). 2012 May:64(5):625-
39. [PMID: 22473917]

139
Answers and Critiques

Item 47 Answer: E and swelling that tend to occur gradually over time, often
with a preceding trauma or history of overuse on the affected
Educational Objective: Diagnose cryoglobulinemic
side; this entity is classically described in women over the age
vasculitis.
of 70 years. This patient has typical features of the syndrome,
Measurement of serum cryoglobulin levels is most likely to including a large effusion on examination and synovial fluid
establish the diagnosis of cryoglobulinemic vasculitis. This that is blood tinged with a low leukocyte count. The crystals
patient presents with palpable purpura, glomerulonephritis are often not visible by routine light microscopy due to their
(elevated serum creatinine, active urine sediment with cel­ small size, and they are not birefringent (hence, not appre­
lular casts), mononeuritis, and skin infarctions of the fingers ciated with a polarizing microscope); however, they may be
and ears. Although several vasculitic diseases may present revealed with alizarin red staining (with crystals visualized
with this picture, the presence of ear infarctions is most as red, globular clumps). This patient's shoulder radiograph
consistent with a diagnosis of cryoglobulinemia. Moreover, reveals narrowing of the glenohumeral joint, calcification of
low C4 with a normal (or relatively unaffected) C3, in the the periarticular cartilage, and erosive changes of the humeral
presence of rheumatoid factor and a monoclonal paraprotein head, all typical findings in this syndrome. Upward sublux­
(the rheumatoid factor itself). is the classic pattern for cryo­ ation of the humeral head due to rotator cuff destruction and
globulinemic vasculitis. Thus, the test most likely to establish bony cysts are also common.
the diagnosis is a serum cryoglobulin level. Because more Acute calcium pyrophosphate crystal arthritis (also
than 90% of patients with essential mixed cryoglobulinemia known as pseudogout) can lead to significant joint swelling;
are infected with hepatitis C virus, this patient should be however, the absence of inflammatory synovial fluid makes
screened and treated for this infection. this diagnosis unlikely.
Anti-cyclic citrullinated peptide antibodies are found This patient has a history of gout; however, her cur­
in patients with rheumatoid arthritis (RA). Because this rent symptoms are atypical for a gout attack, particularly
patient lacks arthritis, this diagnosis can be excluded, with the gradual onset of these symptoms. The synovial fluid is
the rheumatoid factor (also associated with RA) explained by also very uncharacteristic for gout, which typically causes
the patient's cryoglobulinemia. an inflammatory effusion. The absence of crystals in the
Anti-glomerular basement membrane antibodies can synovial fluid further speaks against gout, although mono­
produce glomerulonephritis but would be less likely to cause sodium urate crystals are sometimes missed on synovial
the skin findings that are seen in this patient and would be fluid analysis.
inconsistent with the other serologies presented. Monoarticular joint swelling must always raise suspi­
Antinuclear antibodies are present in nearly all patients cion for an infected joint; however, in this case, the slow
with systemic lupus erythematosus, a disease of protean onset of symptoms, lack of fever, and normal serum leuko­
manifestations that could indeed present in this manner. cyte count go against this diagnosis. Synovial fluid with low
However, the presence of a normal C3 with a low C4 is atyp­ leukocyte count and negative Gram stain also point away
ical in lupus. from a diagnosis of an infected joint.
p-ANCA is found in several forms of vasculitis, all of
KEY POINT
which may involve the kidneys and the skin. However, exter­
nal auricular involvement is not classic, and the C4 is not • Milwaukee shoulder syndrome, caused by basic cal­
reduced. cium phosphate deposition, is characterized by pain,
stiffness, and swelling that tend to occur gradually
KEY POINT
over time, often with a preceding trauma or history of
• Patients presenting with a multisystem vasculitic dis­ overuse on the affected side, with a predilection for
ease should be considered for cryoglobulinemia, par­ women older than the age of 70 years.
ticularly if the C4 is low, the C3 is relatively preserved,
and rheumatoid factor is present. Bibliography
Halverson PB. Crystal deposition disease of the shoulder (including ca lei tic
Bibliography tendonitis and Milwaukee shoulder syndrome). Curr Rheumatol Rep.
2003 Jun:5(3):244-77. [PMID: 12744818]
Saadoun D. Landau DA. Calabrese U. Cacoub PP. Hepatitis C-associated
mixed cryoglobulinaemia: a crossroad bet,,veen autoimmunity and lym­
phoproliferation. Rheumatology. 2007 Aug:46(8):1234-42. [PM ID:
17566058]
Item 49 Answer: A
Educational Objective: Diagnose disseminated
Item 48 Answer: C gonococcal infection.
Educational Objective: Diagnose basic calcium
Cervical culture is the most appropriate diagnostic test to per­
phosphate deposition.
form in this patient who most Likely has disseminated gono­
This patient has Milwaukee shoulder syndrome, caused by coccal infection (DGI). DGI occurs in up to 3% of patients with
basic calcium phosphate deposition. Milwaukee shoulder Neisseria gonon-hoeae and can cause tvvo distinct clinical
syndrome is characterized by symptoms of pain, stiffness, presentations. In the setting of bacteremia from DGI, patients

140
Answers and Critiques

present with vesiculopustular or hemorrhagic macular skin adverse maternal or fetal outcomes have been observed in a
lesions, fever, polyarthralgia, and tenosynovitis. Arthritis typ­ number of studies in which hydroxychloroquine has been
ically is nonpurulent, and synovial fluid cultures tend to be continued throughout pregnancy.
negative. Blood cultures or cultures of the genitourinary tract Low-dose glucocorticoids are frequently used but
or pharynx are often positive. Therefore, a cervical culture is should be avoided if possible before 14 weeks of gestation
appropriate for this patient; cultures of the throat, skin, and because of the risk of cleft palate. Glucocorticoid use can
rectum should also be ordered to maximize the chance of contribute to gestational diabetes and hypertension. How­
organism identification. ever, they can be useful in the management of RA in preg­
The second presentation of DGI is purulent arthritis nancy if the benefit of treatment is thought to exceed risk.
without the rash or other features of gonococcal bacteremia.
KEY POINT
Synovial fluid cultures are often positive for N. gonorrhoeae
in these patients. This patient does not exhibit these symp­ • Women taking methotrexate must discontinue this
toms; therefore, synovial fluid analysis is unnecessary. medication 3 months prior to attempting to conceive.
Hepatitis B virus infection can also present with arthral­
gia, fever, and rash; however, this patient's rash is not typical Bibliography
for hepatitis 8, and her liver chemistry studies are normal. Barbhaiya M. Bermas BL. Evaluation and management of systemic lupus
erythematosus and rheumatoid arthritis during pregnancy. Clin
Hepatitis B serologies therefore will not be diagnostic. lmmunol. 2013 Nov:149(2):225-35. [PMID: 23773975]
A positive rapid streptococcal antigen test is helpful to
diagnose acute rheumatic fever; however, this patient lacks
the sore throat, characteristic rash, and other findings such
Item 51 Answer: B
as cardiac or neurologic involvement that are associated with
acute rheumatic fever. Educational Objective: Diagnose IgG4-related disease.

KEY POINT The most likely diagnosis is JgG4-related disease, a recently


described condition characterized by lymphoplasmacytic
• Blood cultures or cultures of the genitourinary tract
infiltration and enlargement of various structures, includ­
or pharynx are useful diagnostic tests in the setting of ing the pancreas, lymph nodes, salivary glands, periaortic
bacteremia from disseminated gonococcal infection. region leading to retroperitoneal fibrosis, kidneys, and skin.
Most patients are men (60%-80%) over the age of SO years.
Bibliography Typical presentation is the subacute development of a mass
Rice PA. Gonococcal arthritis (disseminated gonococcal infection ). Infect in the affected organ (for example, an orbital pseudotumor)
Dis Clin North Am . 2005 Dec;l9(4):853-61. [PMID: 16297736]
or diffuse organ enlargement. Up to 90% of patients have
multiple organ involvement. Lymphadenopathy is common,
Item 50 Answer: B as is lacrimal and salivary gland involvement. Most patients
lack constitutional symptoms at the time of diagnosis and
Educational Objective: Manage rheumatoid arthritis
generally feel well. Staining for JgG4-producing plasma cells
medications in a patient of childbearing age.
reveals large numbers in the tissue.
Discontinuation of methotrexate is indicated for this patient Hodgkin lymphoma usually presents with palpable
with rheumatoid arthritis (RA) who is interested in becoming lymphadenopathy or a mediastinal mass that requires tissue
pregnant. This nonbiologic disease-modifying antirheumatic biopsy for diagnosis. Biopsy is likely to show mainly clonal
drug is both highly teratogenic and abortifacient and must malignant Hodgkin/Reed-Sternberg cells in a background
be discontinued 3 months prior to attempting to conceive. of granulocytes, plasma cells, and lymphocytes. Reed­
Although this patient is taking folic acid to help reduce her Sternberg cells appear as large cells with large, pale nuclei
incidence of methotrexate side effects, taking folic acid sup­ containing large purple nucleoli, and their appearance is
plements during pregnancy can reduce the risk of certain indicative of Hodgkin disease. The patient's biopsy findings
neural tube birth defects. Therefore, she should not discon­ are not consistent with Hodgkin lymphoma.
tinue folic acid even if she discontinues methotrexate. Sarcoidosis is a multisystem disease characterized by
Considerable epidemiologic evidence in patients with noncaseating granulomas that form in tissues and most
systemic lupus erythematosus as well as RA supports the commonly affects the lungs. The absence of granulomas on
use of hydroxychloroquine during pregnancy. The risks to biopsy is helpful to distinguish IgG4-related disease from
mothers and their fetuses appear low, particularly when sarcoidosis.
balanced against the consequences of discontinuing treat­ Sjogren syndrome can present with enlargement of
ment in anticipation of pregnancy. Greater disease activity the salivary glands. However, it is associated with focal,
during pregnancy is associated with small gestational age centrilobular collections of lymphocytes on biopsy without
and preterm delivery. In addition, patients in whom all med­ the histology characteristic for IgG4-related disease present
ications are stopped, including hydroxychloroquine, run the in this patient. Patients with Sjogren syndrome also usu­
risk offlare, which can impair their physical functioning and ally have positive anti-Ro/SSA and anti-La/SSB antibodies,
make coping with pregnancy more difficult. No increases in which are not present in this patient.

141
Answers and Critiques

KEY POINT Bibliography


Kwok Y./Y. Kloppenburg M. Rosendaal FR. et al. Erosive hand osteoarthritis:
• IgG4-related disease is characterized by the lympho­ its prevalence and clinical impact in the general population and sympto­
plasmacytic infiltration and enlargement of various matic hand osteoarthritis. Ann Rheum Dis. 2011 Jul:70(7):1238-42.
[PMID: 21474485]
structures, including the pancreas, lymph nodes, sali­
vary glands, periaortic region leading to retroperito­
neal fibrosis, kidneys, and skin. Item 53 Answer: B
Educational Objective: Diagnose limited cutaneous
Bibliography systemic sclerosis.
Mahajan VS, Matto H. Deshpande V. Pillai SS, Stone JS. lgG4-Related disease.
Ann Rev Pathol. 2014:24(9):315-47. [PMID: 24111912] The most likely diagnosis is limited cutaneous systemic scle­
rosis (LcSSc), a form of systemic sclerosis that is charac­
terized by distal (face, neck, and hands) but not proximal
Item 52 Answer: A skin thickening and is typically not accompanied by internal
organ fibrosis. Patients with LcSSc may display features of the
Educational Objective: Diagnose erosive osteoarthritis.
CREST (calcinosis cutis, Raynaud phenomenon, esophageal
The most likely diagnosis is erosive osteoarthritis (OA). Dis­ dysmotility, sclerodactyly, and telangiectasia) syndrome and
tinguishing erosive OA from rheumatoid arthritis (RA) can are more likely to develop Raynaud phenomenon early in the
be difficult because both diseases disproportionately affect disease course. Pulmonary arterial hypertension is more com­
women, are polyarticular, and preferentially affect the hand mon in patients with LcSSc compared with diffuse cutaneous
joints. Signs of inflammation (warmth, erythema, swelling, systemic sclerosis. This patient presents with skin thickening
tenderness, reduced function) are present in both entities, involving the distal extremities and face as well as Raynaud
although generally involving more joints for longer periods phenomenon; he also has nodules on the extremities, which
of time in RA. In contrast to RA, erosive OA is common in are suggestive of calcinosis cutis. A number of patients may
the distal interphalangeal and carpometacarpal joints, does present with only one or two clinical features, and negative
not typically affect the wrists or elbows, and is not associ­ serologies (including antinuclear antibodies) are seen in up
ated with rheumatoid factor, anti-cyclic citrullinated peptide to 10% to 25% of patients with LcSSc. He does not have CREST
antibodies, or an elevated erythrocyte sedimentation rate or syndrome, which is a variant of LcSSc characterized by the
C-reactive protein level. Both types of arthritis can lead to presence of telangiectasias and esophageal dysmotility as well
erosions seen on plain radiographs. In erosive OA, these ero­ as positive antinuclear and anticentromere antibodies.
sions are centrally located in the joint and are accompanied by Eosinophilia myalgia syndrome is characterized by fasci­
proliferative changes. In RA, erosions are located at the joint itis and dermal induration linked to consuming contaminated
margins, and periarticular osteopenia is likely to be present. L-tryptophan, a nutritional supplement. Patients also develop
Reactive arthritis is a noninfectious inflammatory neuropathy and myopathy but not Raynaud phenomenon or
arthritis that can occur after a gastrointestinal or genitouri­ scleroderma-specific autoantibodies. New cases are rare since
nary infection. It is not commonly associated with erosive the identification of the toxin several years ago. This patient
changes on radiographs. consumes no nutritional supplements and has Raynaud phe­
Tophaceous gout can occur in either gender and is more nomenon, making eosinophilia myalgia syndrome unlikely.
frequent in women after menopause. It can occasionally Morphea is characterized by a localized area of skin
have a polyarticular presentation in the hands, but extensive thickening, usually on the torso; systemic manifestations
disease is typically accompanied by the obvious presence of or Raynaud phenomenon is extremely rare in patients with
multiple tophi in the skin. When inflammation is evident, an this condition.
acute gout flare is occurring in the presence of tophaceous Primary biliary cirrhosis (PBC) is a chronic choles­
gout. Tophi can occur in many locations and can create the tatic liver disease of unknown cause affecting middle-aged
appearance of erosions with overhanging edges in affected women. Fatigue, dry eyes, dry mouth, and pruritus are the
bones, which are sometimes found adjacent to joints. Highly most common symptoms. Jaundice, cutaneous hyperpig­
destructive changes can occur that can lead to loss of joint mentation, hepatosplenomegaly, and xanthelasmas are
space with long-standing tophaceous gout. rarely observed at diagnosis. Raynaud phenomenon and
skin thickening are not consistent with PBC.
KEY POINT
• In contrast to rheumatoid arthritis, erosive osteoar­ KEV POINT

thritis is common in the distal interphalangeal and • Limited cutaneous systemic sclerosis is characterized
carpometacarpal joints, is associated with erosions by distal skin thickening (face, neck, and hands) and is
that are centrally located in the joint and accompa­ typically not accompanied by internal organ fibrosis;
nied by proliferative changes, does not typically affect patients may display features of the CREST (calcinosis
the wrists or elbows, and is not associated with cutis, Raynaud phenomenon, esophageal dysmotility,
inflammatory markers. sclerodactyly, and telangiectasia) syndrome.

142
Answers and Critiques

Bibliography Item 55 Answer: A


Van den Hoogen F, Khanna D. Fransen J. et al. 2013 classification criteria for
systemic sclerosis: an American College of Rheumatology/European Educational Objective: Diagnose mixed connective
League against Rheumatism collaborative initiative. Arthritis Rheum. tissue disease.
2013 Nov,65(11):2737-47. [PMID: 24122180]
The most likely diagnosis is mixed connective tissue disease
(MCTD), an overlap syndrome that includes features of systemic
Item 54 Answer: B lupus erythematosus (SLE), systemic sclerosis, and/or polymy­
Educational Objective: Treat rheumatoid arthritis with ositis in the setting of positive anti-Ul-ribonucleoprotein (RNP)
methotrexate. antibodies. Patients with MCTD must have positive anti-Ul-RNP
Treatment with methotrexate is indicated for this patient with antibodies and at least three of the following five features: Ray­
naud phenomenon, edema of the hands, sclerodactyly, syno­
rheumatoid arthritis (RA). She has a polyarticular inflamma­
vitis, and myositis. Patients with MCTD usually have positive
tory arthritis involving the small joints of the hands as well
antinuclear antibodies (ANA), negative anti-double-stranded
as a wrist and an ankle. with radiographically demonstrated
DNA antibodies, nom1al complement levels, and a low inci­
marginal erosions and periarticular osteopenia and positive
dence of kidney disease. These criteria have a sensitivity of 63%
anti-cyclic citrullinated peptide antibodies and rheumatoid
and specificity of 86% in the diagnosis of MCTD. These patients
factor, all of which support a diagnosis of RA. Methotrexate
are more likely to have myositis and pulmonary hypertension,
with or without the addition of another disease-modifying
an important cause of mortality, than patients with SLE.
antirheumatic drug (DMARD) should be instituted immedi­
Polymyositis is a form of inflammatory myopathy charac­
ately in patients with erosive disease documented at disease
terized by symmetric proxin1al muscle weakness with little or
onset. Methotrexate is the gold standard therapy because it
no pain, and muscle enzymes such as creatine kinase are usu­
is usually better tolerated than other DMARDs and has good
ally elevated 10- to SO-fold the upper limit of normal. However,
efficacy, long-term compliance rates, and relatively low cost.
polymyositis is not associated with anti-Ul-RNP antibodies
Hydroxychloroquine is indicated to treat early, mild, and
and does not result in puffy hands or sicca symptoms.
nonerosive disease. Hyd.roxychloroquine therapy alone has not
Although the patient has a positive ANA, the diagnosis
been shown to retard radiographic progression of RA and there­
of SLE is less likely in the context of the negative anti-Smith
fore should be used only in patients whose disease has remained
and anti-double-stranded DNA antibodies as well as the
nonerosive for several years. This patient has erosive disease, and
normal complement levels, serum creatinine, and urinalysis.
hydroxychloroquine as a single agent is not appropriate.
TI1e absence of sclerodactyly, sclerodermatous skin
Rituximab, the anti-CD20 B-cell depleting mono­
changes, and esophageal involvement makes systemic scle­
clonal antibody, is FDA approved for the treatment of
rosis an unlikely diagnosis.
moderately to severely active RA in combination with
Undifferentiated connective tissue disease (UCTD) is
methotrexate in patients who have had an inadequate
not an overlap syndrome but instead refers to nonspecific
response to tumor necrosis factor a inhibitor therapy.
clinical features (such as Raynaud phenomenon and arthral­
Rituximab may also be considered for patients with high
gia). no disease-specific findings, and nonspecific positive
disease activity and poor prognostic features despite
autoantibodies (such as ANA). UCTD may exist for some
sequential nonbiologic DMARDs or methotrexate in
time prior to a clear emergence of symptoms characteristic
combination with other DMARDs. It is not appropriate
enough to define a single rheumatologic disease.
initial treatment for RA in a patient who has not been
given a trial of methotrexate. KEY POINT
Tofacitinib is also indicated for use in the management • Mixed connective tissue disease is an overlap syn­
of RA but only in patients who have already not responded drome that includes features of systemic lupus ery­
to methotrexate alone. This relatively recent addition to the thematosus, systemic sclerosis, and/or polymyositis in
treatment armamentarium for RA is the first oral agent to the setting of positive anti-Ul-ribonucleoprotein anti­
be introduced in decades but is indicated for use in patients
bodies.
who are intolerant to or have had an inadequate response to
methotrexate.
Bibliography
KEY POINT Hajas A. Szodoray P. Nakken B. et al. CHnical course. prognosis, and causes
of death in mixed connective tissue disease. J Rheumatol. 2013
• Methotrexate is the initial treatment of choice for Jul:40(7):1134-42. [PMID: 236373281
patients with new-onset, rapidly progressive, or ero­
sive rheumatoid arthritis.

Bibliography
Item 56 Answer: A
Educational Objective: Diagnose diffuse alveolar hem­
Cl
Singh JA. Furst DE. Bharat A. et al. 2012 Update of the 2008 American orrhage in a patient with systemic lupus erythematosus.
College Of Rheumatology recommendations for the use of disease­
modifying antirheumatic drugs and biologic agents in the treatment of
rheumatoid arthritis. Arthritis Care Res (Hoboken). 2012 May:64(5):625- Bronchoscopy with bronchoalveolar lavage (BAL) and biopsy
39. [PMID: 224739171 is the most appropriate diagnostic test to perfom1 nell.1 in this

143
Answers and Critiques

Cl rare
patient with suspected diffuse alveolar hemorrhage (DAH). a and inflammatory bowel disease are the most frequently
but severe manffestation of systemic lupus erythematosus associated systemic diseases. The lesions of erythema nodo­
CONT. sum are tender, subcutaneous nodules presenting as barely
(SLE). The t1iad of hypoxemia. new infiltrates found on chest
radiograph. and decreasing hematocrit is highly predictive of appreciable convexities on the skin surface, with a reddish
underlying DAH; only about SO% of patients have hemopty­ hue in the acute phase. As the lesions resolve, a dull brown
sis. DAH occurs in the setting of active SLE. and up to 90% of circular patch is often left behind. Erythema nodosum is fre­
patients have evidence of nephritis. Chest radiograph demon­ quently bilateral and symmetric, and it usually occurs on the
strates bilateral infiltrates often sparing the lung apices. Diag­ distal lower extremities. Lofgren is more common among
nosis is by BAL du1ing bronchoscopy to demonstrate bleeding Europeans. Initial treatment is generally with NSAJDs (such
and rule out infection. Lung biopsy is the definitive diagnostic as naproxen), colchicine, or low-dose prednisone.
test. which typically shows a capillaritis with immune com­ Methotrexate, high-dose prednisone, or a tumor necro­
plex deposition. Mechanical ventilation and aggressive immu­ sis factor a inhibitor such as adalimumab are reserved for
nosuppression are generally required. but mortality rates still patients with chronic, organ-damaging forms of sarcoidosis.
are as high as 70%.
KEY POINT
Although DAH can be suggested by the presence of
ground-glass opacities on chest CT. the diagnosis is estab­ • Lofgren syndrome is a self-limiting form of sarcoido­
lished only with bronchoalveolar lavage and biopsy. sis characterized by acute arthritis, bilateral hilar lym­
MRI of the chest can be used to determine if the infil­ phadenopathy, and erythema nodosum.
trates are blood and could be used in rare cases in which
bronchoalveolar lavage and biopsy are unable to be done. Bibliography
Because DAH is typically a pulmonary capillaritis. angi­ O'Regan A, Berman JS. Sarcoidosis. Ann Intern Med. 2012 May 1;156(9):
ITC5:!-16. [PMID: 22547486]
ography will not image vessels of this size and vvill not be
useful in establishing the diagnosis.
KEY POINT Item 58 Answer: C
• The triad of hypoxemia, new pulmonary infiltrates on Educational Objective: Diagnose reactive arthritis.
chest radiograph, and decreasing hematocrit is highly The most likely diagnosis is reactive arthritis (formerly known
predictive of underlying diffuse alveolar hemorrhage as Reiter syndrome), a noninfectious inflammato1y arthritis
associated with systemic lupus erythematosus. that can occur approximately 3 to 6 weeks after a gastrointes­
tinal or genitourinary infection. It is common for infection to
Bibliography have resolved and stool culture to become negative by the time
Martinez-Martinez MU, Abud-Mendoza C. Predictors of mortality in diffuse arthritis begins. Asymmetric monoarthritis or oligoarthritis
alveolar haemorrhage associated with systemic lupus erythematosus.
Lupus. 2011 May:20(6):568-74. [PMID: 21558137] in the lower extremities is the most common presentation,
but up to 20% of patients have polyarthritis. Enthesopathy
(inflammation at the site where ligaments, tendons, joint
Item 57 Answer: D capsule or fascia attaches to bone), dactylitis, and sacroiliitis
may occur. Erosive disease is uncommon. Ophthalmologic
Educational Objective: Treat a patient who has Lofgren
inflammation is a feature of this disease in up to 30% of
syndrome.
patients and can manifest as uveitis or conjunctivitis. This
An NSAJD such as naproxen is appropriate for this patient patient's manifestations, including a diarrheal illness that
who has Lofgren syndrome, a self-limiting form of sarcoid­ preceded the onset of arthritis by 2 to 3 weeks, inflammatory
osis characterized by a triad of acute arthritis in combina­ oligoarticular involvement of lower extremities, inflammation
tion with bilateral hilar lymphadenopathy and erythema of the insertion of the Achilles tendon, and anterior uveitis,
nodosum. The "arthritis" associated with Lofgren syndrome are typical of reactive arthritis.
is actually a nondestructive periarthritis of the soft tissue, In bacterial infections, synovial fluid analysis typically
entheses, and tenosynovium around the joints. Symmet­ reveals a neutrophilic leukocytosis (typically >S0,000/µL
ric involvement of the ankles is classic, but knees, wrists, [SO x 109/L]), but synovial fluid leukocyte counts <e:;S0,000/µL
and elbows can also be involved. Ninety percent of patients (SO x 109 IL) do not definitively rule out infection. In this
remit within 12 months. When the triad of features occurs, patient, infectious arthritis is less likely because this
it has a 9S% specificity for diagnosis, and further diag­ patient has no fever or leukocytosis on complete blood
nostic tests (such as radiography or serologic testing) are count.
unnecessary. This patient presents with the classic triad Although psoriatic arthritis can present as an oligoar­
of hilar lymphadenopathy, erythema nodosum, and acute thritis, it is also less likely because the patient has no evi­
arthritis involving the ankles and therefore has Lofgren syn­ dence of psoriasis on physical examination.
drome. Erythema nodosum may occur in association with Rheumatoid arthritis is unlikely because it usually is a
a wide array of causes, including infections, medications, symmetric small joint polyarthritis rather than large joint
and systemic disease, or it may be idiopathic. Sarcoidosis oligoarthritis, and enthesopathy is not common.

144
Answers and Critiques

KEY POINT Item 60 Answer: A


• Reactive arthritis can occur after a gastrointestinal or Educational Objective: Treat ankylosing spondylitis
genitourinary infection and is characterized by an with a tumor necrosis factor a inhibitor.
asymmetric monoarthritis or oligoarthritis in the
Treatment with adalirnumab is appropriate for this patient
lower extremities as well as enthesopathy, dactylitis,
with ankylosing spondylitis, a form of spondyloarthritis that
and sacroiliitis. predominantly affects the axial skeleton. Inflammatory back
pain is a hallmark feature, manifesting as pain and stiffness
Bibliography in the spine that is worse after immobility and better with
Morris D. Inman RD. Reactive arthritis: developments and challenges in use. Symptoms are prominent in the morning and can be
diagnosis and treatment. Curr Rheumatol Rep. 2012 0cl;l4(5):390-4.
[PMID: 22821199] symptomatic during the night. Buttock pain is common and
correlates with sacroiliHis. Fusion of the spine may occur over
time, leading to rigidity and kyphosis. Exercise to preserve
Item 59 Answer: C range of motion and strengthen the spine extensor muscles to
Educational Objective: Treat tophaceous gout. prevent kyphosis is essential. Physical therapy may be indicated
to assist patients in developing a home exercise routine. NSAIDs
Continuation of colchicine for flare prophylaxis and allo­
are considered first-line therapy for symptomatic patients. If
purinol for urate lowering is indicated for this patient with
the patient does not adequately respond to a minimum of
tophaceous gout. He is having a good response to colchicine
two different trials of NSAIDs used at least 4 weeks total, the
and allopurinol, with a serum urate level currently at the
Assessment of SpondyloArthritis international Society/Euro­
target goal of less than 6.0 mg/dL (0.35 mmol/L), resolu­
pean League Against Rheumatism (ASAS/EULAR) guidelines
tion of flares, and reduction in size of tophi (reservoirs of
recommend treatment with a tumor necrosis factor (TNF)-a
monosodium urate). The American College of Rheumatology
inhibitor. This patient has not adequately responded to either
currently recommends continuation of flare prophylaxis if
naproxen or indomethacin; therefore, a TNF-a inhibitor such
there is any evidence of active disease, including flares or
as adalimumab is appropriate. 1l1e currently available TNF-a
tophi as seen in this patient. Hence, both agents should be
inhibitors appear to be equally effective when compared with
continued at this time. The guidelines further suggest that in
placebo. Response is rapid, often within the first 6 weeks of
the absence of active disease and once target serum urate is
therapy Patients who do not respond to one agent may respond
reached, colchicine should be continued for the longer of the
to an alternative. Because long-term safety is unknown, TNF-a
following: 6 months; 3 months after reaching target serum
inhibitors remain second-line therapy to NSAIDs.
urate in a patient without baseline tophi; or 6 months after
Nonbiologic disease-modifying antirheumatic drugs
reaching target serum urate in a patient with baseline tophi
(DMARDs) such as methotrexate and sulfasalazine have not
that have resolved.
been demonstrated to be efficacious for axial disease but
Changing this patient's urate-lowering regimen by
may be considered for peripheral arthritis.
adding probenecid or changing allopurinol to febuxostat
Rituximab is a biologic DMARD used to treat moderate
is unnecessary in this patient who has reached the target
to severe rheumatoid arthritis in combination with metho­
serum urate level by taking allopurinol. His gout symptoms
trexate in patients who have had an inadequate response to
have improved, and the tophi have begun to shrink in size.
TNF-a inhibitor therapy and is also used in ANCA-associated
If he was not at goal or was having ongoing flares or resis­
vasculitis.1l1ere is no evidence showing benefit of this agent
tance of tophi to dissolve on his current regimen, options
in ankylosing spondylitis.
would be to further increase allopurinol, switch to febux­
ostat (a newer, more expensive xanthine oxidase inhibitor), KEY POINT
or add probenecid. • In patients with ankylosing spondylitis, treatment with
Colchicine should be maintained at daily dosing a tumor necrosis factor a inhibitor is currently recom­
because the patient still needs ongoing protection against mended if fust-line therapy with NSAIDs is inadequate.
flares, given the presence of tophi.
KEY POINT Bibliography
van der Heijde D. Sieper J. Maksymowych WP, et al; Assessment of
• In patients with gout, continuation of flare prophy­ SpondyloArthritis international Society. 2010 Update of the 111ter rnt1onal
laxis and urate-lowering therapy is currently indi­ ASAS recommendations for the use of anti-TNF agents .m patients with axial
spondyloarthritis. Ann Rheum Dis. 20U Jun:70(6):905-8. [PMID: 21540200]
cated if there is any evidence of active disease, includ­
ing flares or tophi.
Item 61 Answer: B
Bibliography Educational Objective: Diagnose acute osteonecrosis in
Khanna D, Khanna PP. Fitzgerald JD, et al; American College of a patient with systemic lupus erythematosus.
Rheumatology. 2012 American College of Rheumatology guidelines for
management of gout. Part 2: therapy and antiinflammatory prophylaxis MRI of the left hip is the most appropriate diagnostic test
of acute gouty arthritis. Arthritis Care Res (Hoboken). 2012 Oct:64(10):
1447-61. [PMID: 23024029] to perform next in this patient with systemic lupus

145
Answers and Critiques

erythematosus (SLE) who has symptoms associated with an underlying malignancy, especially in the first 2 years after
osteonecrosis. Patients with SLE who have pain or limitation of diagnosis. Malignancy risk may also be higher in the presence
motion of the large joints, especially the hips, should be eval­ of vasculitis (as seen in this patient) or cutaneous necrosis as
uated for osteonecrosis. MRI is the best method for detecting well as in patients who are of older age at onset. Patients with
early bone edema caused by osteonecrosis when plain radio­ dermatomyositis have an increased standardized incidence
graphs are normal. MRI can also be prognostic: If more than ratio or solid malignancies such as adenocarcinomas of the
20% of the femoral volume demonstrates necrosis and edema Jung. cervLx, ovaries, pancreas. colorectal. stomach. and blad­
on MRI, progressive disease (subchondral fracture and femoral der as well as non-Hodgkin lymphoma. Risk of ovarian cancer
head collapse) is the rule, whereas smaller infarcts rarely prog­ may be especially increased. In a population-based study.
ress. Up to 37% of patients with SLE may have osteonecrosis by the standardized incidence ratio was 10.S for ovarian can­
serial MRI monitoring, although less than 10% become symp­ cer. highest among all cancers diagnosed. TI1erefore, obtain­
tomatic. Cushingoid features indicate a risk for osteonecrosis ing a transvaginal pelvic ultrasonography to look for ovarian
because enlargement of fat cells in the face is a marker for pathology, especially ovarian cancer. is recommended for this
enlargement of fat cells in the ends of long bones. Increased patient who has worsening dermatomyositis and new-onset
adipose volume causes compression of small sinusoidal vessels ascites on examination.
that leads to interosseous hypertension and impairment of Chest CT is helpful in diagnosing interstitial lung dis­
arterial inflow. Use of daily oral prednisone more than 20 mg/d ease in patients with myositis and in evaluating malignancy:
for 4 to 6 weeks is also a risk factor, whereas use of intravenous however. this patient has no evidence of respiratory symp­
glucocorticoids may not have the same risk. This patient has toms but does have new-onset ascites. making ovarian can­
hip pain, cushingoid features, and recent use of high-dose cer a more likely diagnosis.
prednisone, all indicators that she should be evaluated for Although this patient has elevated aspartate amino­
osteonecrosis despite the normal hip radiograph. transferase, it is likely of muscle origin and abnormal due to
MRI remains the gold standard to diagnose osteonecro­ active myositis. Liver biopsy is not needed prior to obtaining
sis. CT is less sensitive than MRI and exposes the patient to other noninvasive studies for her abnormal liver chemistry
unnecessary radiation. tests and evaluation for ovarian cancer.
Plain radiography may not detect changes of osteonecro­ TI1e diagnosis of advanced ovarian cancer is usually
sis for several months following the onset of symptoms. Early made by CT- or ultrasound-guided biopsy of a suspicious
radiographic findings include bone density changes, sclerosis, mass or cytologic examination of ascitic fluid. PET scan is
and, eventually, cyst formation. Subchondral radiolucency likely to show abnormalities in her abdomen but will not
producing the "crescent sign" indicates subchondral collapse. lead to the correct diagnosis and is not cost-effective.
End-stage disease is characterized by collapse of the femoral Thigh muscle MRI with specialized (STIR protocol)
head, joint-space narrowing, and degenerative changes. images may show evidence of actively involved muscle with
Ultrasonography can be used to evaluate for trochan­ myositis and support the diagnosis of dermatomyositis.
teric bursitis as the cause of lateral hip pain but would not be However. this study is not needed in this patient who already
useful to check for osteonecrosis. Trochanteric bursitis can has been diagnosed with dermatomyositis and has evidence
be confirmed in patients in whom hip adduction intensifies of the classic rash. weakness. and elevated muscle enzymes.
the pain or in those in whom the examination reveals pain
KEY POI NT
and tenderness over the bursa. Pain at night is present when
the patient sleeps on the affected side. • The risk of ovarian cancer may be especially increased
in women with dermatomyositis.
KEY POINT
• Patients with systemic lupus erythematosus who have Bibliography
pain or limitation of motion of the large joints, espe­ Hill CL. Zhang Y, Felson DT. et al. Frequency of specific cancer types in
cially the hips, should be evaluated for osteonecrosis dermatomyositis and polymyositis: a population-based study. Lancet.
2001 Jan 13:357(9250):96. [PMID: lll97446]
using MRI when plain radiographs are normal.

Cl
Bibliography Item 63 D
Answer:
Shigemura T. Nakamura J. Kishida S, et al. Incidence of osteonecrosis associ­
ated with corticosteroid therapy among different underlying diseases: Educational Objective: Diagnose polyarteritis nodosa.
prospective MRI study. Rheumatology (Oxford). 2011 Nov;SO(ll):2023-8.
[PMID: 21865285] TI1e most likely diagnosis is polyarteritis nodosa (PAN), the
most common medium-sized vasculitis that affects the

Cl Item 62 Answer:E
Educational Objective: Diagnose ovarian cancer in a
mesenteric and renal arteries. Patients usually present with
nonspecific inflammatory symptoms as well as abdominal
patient with dermatomyositis. symptoms (chronic or intermittent ischemic pain). neuro­
Jogic involvement (mononeuritis multiplex). and skin find­
Transvaginal pelvic ultrasonography is indicated for this ings (livedo reticularis. purpura. and painful subcutaneous
patient with dermatomyositis. which may be associated with nodules). Kidney disease is based on decreased renal artery

146
Answers and Critiques

Cl
CONT.
blood flow rather than glomerulonephritis. The presence of
hepalitis B virus (HBV) infection is a strong risk factor for the
40-year follow-up concluded that meniscectomy leads to
osteoarthritis of the knee with a resultant 132-fold increase
development of PAN. although in areas where HBV vaccina­ in the rate of total knee replacement in comparison to their
tion is common. the incidence of HBV-associated PAN has matched controls. The risk of osteoarthritis of the knee fol­
diminished greatly. This patient with HBV inlection (positive lowing meniscus injury and removal is also well documented
hepatitis B surface antigen) presents with involvement of the for adolescent athletes and, as recognition of this link has
abdomen. kidneys. nerves. joints. and skin. accompanied by become more widespread, the incidence of meniscus repair
hypertension but sparing the upper and lower respiratory rather than meniscectomy has risen.
tracts. He also has mononeuropathy. arthralgia. fever. and Other factors for osteoarthritis are advancing age, obe­
livedo reticularis (shown in the ngure) but lacks the active sity, female gender, and genetic factors. For example, obesity
urine sediment characteristic of glomerulonephritis. Together is the most important modifiable risk factor for osteoarthri­
these findings suggest a systemic vasculitis of which PAN tis of the knee, but this patient is not obese. The incidence
is the most likely. Testicular involvement is also common. as in of knee osteoarthritis is also increased by occupations with
this case. Diagnosis of PAN is best established by demonstrat­ repetitive bending, which this patient does not experience.
ing necrotizing arteritis in biopsy specimens, or characteristic The prevalence of osteoarthritis of the hip and knee is nearly
medium sized artery aneurysms and stenoses on imaging two times higher in women than in men. Osteoarthritis of
studies or the mesenteric or renal a11eries using either angi­ the hand has strong female and genetic predilections; it is
ography or CT angiography. also associated with obesity. His mother's hand osteoarthri­
Goodpasture syndrome affects the kidneys and often tis is probably not relevant for this patient who developed
the lungs. The lack of aclive urine sediment ,rnd the presence knee osteoarthritis at an early age following meniscectomy.
or medium arterial involvement are inconsistent with Good­
KEY POINT
pasture syndrome. as are the abdominal symptoms.
Granulomatosis with polyangiitis (GPA: formerly • There is an increased risk of early-onset knee osteoar­
known as Wegener granulomatosis) can involve the kidneys. thritis in patients with a history of prior injury fol­
and neuropathy and purpura can occur: however. this dis­ lowed by meniscectomy.
ease is associated with glomerulonephritis. which is absent
in this patient. Furthermore. medium-sized artery involve­ Bibliography
ment does not occur in GPA. and the absence of ANCA makes Pengas JP, Assiotis A, Nash W. et al. Total meniscectomy in adolescents: a
40-year follow-up. J Bone Joint Surg Br. 2012 Dec:94(12):1649-54. [PMID:
GPA much less likely. 23188906]
Henoch-Schonlein purpura (HSP ) can affect the kid­
neys. nerves. and skin as well as cause abdominal pain.

Cl
However. HSP is an immune complex disease afTecting small Answer: B
Item 65
vessels: active urine sedimenl is common. complements
(particularly C4) are typically low. and lgA levels may be Educational Objective: Manage a prosthetic joint
elevated. none of which is the case in this patient. infection.

KEY POINT
Obtaining blood and synovial nuid cultures is the most appro­
priate next step in management for this patient with suspected
• Patients with polyarteritis nodosa ty pically present prosthetic joint infection. Infections may occur early (within
with fever, arthralgia, myalgia, skin findings, abdomi­ 3 months of surgery). have delayed onset (3-12 months), or
nal pain, weight loss, and peripheral nerve manifesta­ have late onset (>12 months after surgery). Early-onset infec­
tions, most commonly mononeuropathy or mono­ tions typically present with joint swelling. erythema. wound
neuritis multiplex. drainage. and/or fever. Delayed-onset infections present more
insidiously with prolonged joint pain, often without fever.
Bibliography Late-onset infections present as acute pain and swelling, often
Ebert EC, Hagspiet KD, Nagar M, Schlesinger N. Gastrointestinal involve­ in the setting of a nidus for hematogenous seeding such as
ment in polyarteritis nodosa. Clin Gastroenterol Hepatol. 2008
Sep:6(9):960-6. [PMlD: 18585977] a vascular catheter or other site of infection remote from
the affected joint. l11is patient has acute onset of pain and
swelling of the right knee. along with fevers. an elevated
Item 64 Answer: C leukocyte count with a left shift. and elevated inflammatory
markers. occurring 2 months after knee replacement. Urgent
Educational Objective: Recognize the risk of early­
surgical consultation is warranted. and blood and synovial
onset knee osteoarthritis following meniscectomy.
fluid cultures should be obtained before administration of
The most likely cause of this patient's left knee osteoarthri­ antibiotics. whenever possible. Lo allow for more accurate
tis is meniscectomy. The history of prior injury followed by culture data. Blood cultures are essential (even when fever is
meniscectomy puts this patient at substantial risk for the absent); although the infection likely arose locally at the sur­
development of osteoarthritis at an earlier age than would gical site in this patient. most cases of infectious a11hritis arise
otherwise be predicted. A recent prospective study with a from hematogenous spread. Blood cultures are also imp011ant

147
Answers and Critiques

Cl because. even in cases in which lhe infection arises directly at


the joinl. lhe organism may o casionally be identified in the
CONT.
KEV POINT
• In patients with gout, lifestyle and dietary modifica­
blood cullures. Before initiating antibiotics. synovial nuid cul­ tions, including weight loss if appropriate, reduction
lures should be obtained via arthrocentesis or in the operating
of high-fructose and high-purine foods, alcohol
room if surgical intervention is imminent and the patient is
restriction, and increased low-fat dairy intake, may
stable enough to withhold antibiotics until surgery.
help decrease the risk of gout flares.
TI1is patient will need antibiotics shortly. but she is
stable enough to awail surgical evaluation and collection of
blood and synovial fluid cultures. Bibliography
Choi HK. Atkinson K. Karlson EW. Willett W. Curhan G. Purine-rich foods,
More advanced imaging such as a CT. bone scan, or MRI dairy and protein intake. and the risk of gout in men. N Engl J Med. 2004
is generally not indicated in the preliminary evaluation and Mar ll;3SO(ll):1093-ll03. [PMID: 150141821
treatment of' a suspected prosthetic joint infection because
these studies delay the more urgent management of the
patient and do nol change the initial management. Item 67 Answer: D
KEV POINT Educational Objective: Treat a patient who has
fibromyaigia.
• In patients with suspected prosthetic joint infection,
blood and synovial fluid cultures should be obtained An aerobic exercise program is appropriate for this patient
before initiation of antibiotics to allow for more accu­ with fibromyalgia, which is characterized by chronic wide­
rate culture data. spread pain, tenderness of skin and muscles to pressure,
fatigue, sleep disturbance, and exercise intolerance. Non­
Bibliography pharmacologic therapy is the cornerstone of treatment and
Osmon DR. Berbari EF. Berendt AR. et al: Infectious Diseases Society of should be initiated in all affected patients. Regular aerobic
America. Diagnosis and management of prosthetic joint infection: clini­ exercise has been shown to be effective in this setting.
cal practice guidelines by the Infectious Diseases Society of America. Clin
Infect Dis. 2013 Jan;S6(1):el-e25. [PMID: 23223583] Exercise regimens should be individualized and titrated
up to 30 minutes most days of the week. Physical therapy
may also be helpful initially to develop a stretching and
progressive aerobic program. Cognitive behavioral therapy
Item 66 Answer: B
has been shown to be beneficial but is not always covered
Educational Objective: Manage gout with dietary by insurance plans.
modifications. NSAIDs such as ibuprofen have not been shown to be
The addition of low-fat dairy products is appropriate for particularly useful in fibromyalgia, and most patients have
this patient with gout. Low-fat dairy products have been tried them before seeking medical care for their symptoms.
shown to decrease the risk of gout flares both through Although possibly helpful when taken on an as-needed basis
uricosuric and anti-inflammatory properties. He should for other musculoskeletal pain, NSAIDs as a primary therapy
also be advised to reduce intake of high-fructose beverages for fibromyalgia would not be appropriate.
such as soft drinks because they are associated with gout This patient also has hypothyroidism but has a normal
flares due to metabolic pathways utilized in the metabolism thyroid-stimulating hormone level, indicating that she is
of fructose, which lead to increased uric acid generation. being treated properly and does not need an increase of her
Obesity is also a risk factor fo'r gout and should be addressed levothyroxine dose.
as needed. Fibromyalgia may co-occur in patients with inflamma­
Some leafy green vegetables are high in purines, the tory diseases such as rheumatoid arthritis, systemic lupus
nucleic acid component that is metabolized to uric acid. erythematosus, and Sjogren syndrome, but this patient does
Thus, a recommendation to increase leafy greens as a not have any clinical signs or laboratory features that would
dietary approach to gout treatment would be incorrect. suggest an inflammatory disease. Tirns, an antinuclear anti­
However, intake of leafy green vegetables has not been body (ANA) panel would not be helpful, and a clinically
shown to increase the risk of flares in population-based insignificant low-level positive ANA may actually lead to
studies. further unnecessary testing and specialist referral.
Alcohol is a well-established trigger for gout, proba­ KEV POINT
bly due to several mechanisms, including uric acid pro­
• Nonpharmacologic therapy, including regular aerobic
duction and kidney urate handling. Although wine has
exercise, is the cornerstone of fibromyalgia treatment
been found less likely to trigger gout flares than beer,
alcohol consumption of any sort will increase the risk of and should be initiated in all affected patients.
flares overall.
Shellfish have long been established as a food that is Bibliography
Fitzcharles MA. Ste-Marie PA, Pereira JX: Canadian Fibromyalgia Guidelines
likely to trigger a gout flare due to the high purine load and Committee. Fibromyalgia: evolving concepts over the past 2 decades.
should therefore be restricted in this patient's diet. CMAJ. 2013 Sep 17:185(13):E645-51. [PMID: 23649418]

148
Answers and Critiques

Item 68 Answer: C diagnostic specificity for RA. Some patients with RA also lack
Educational Objective: Manage NSAID gastrointestinal rheumatoid factor. Seronegative RA has an identical clinical
risk in an older patient by using topical NSAID therapy. appearance as seropositive RA but is more likely to occur
in men. Despite a negative rheumatoid factor and anti-CCP
Discontinuation of oral naproxen and initiation of a topical antibodies, this patient's clinical presentation of polyartic­
NSAID is the most appropriate therapy for this 75-year-old ular inflammatory arthritis involving multiple and bilateral
patient with knee osteoarthritis. The major risks of NSA!Ds interphalangeal joints of the fingers, metacarpophalangeal
include gastrointestinal toxicity, cardiovascular disease, joints, a wrist, and an ankle as well as prolonged morning
hypertension, and kidney disease. Among these, NSAID gas­ f
stifness and radiographic findings of marginal erosion and
trointestinal risk is higher among older patients and needs periarticular osteopenia, is characteristic of RA. Over time,
to be managed. The American College of Rheumatology cur­ some patients who are initially seronegative develop a posi­
rently recommends topical NSA!Ds rather than oral NSA!Ds tive rheumatoid factor.
for patients aged 75 years or older. A 2012 Cochrane review of This patient does not have monoarticular or oligoar­
topical NSA!Ds for chronic musculoskeletal pain included 34 ticular disease or radiographs showing bony sclerosis or
studies with 7688 participants. Topical NSA!Ds were superior osteophyte formation, all of which are typical of osteoar­
to placebo for pain relief, with the most data available for thritis. This patient's symmetric polyarticular inflammatory
topical diclofenac. Topical and oral NSA!Ds did not differ with arthritis associated with prolonged morning stiffness is not
regard to pain relief. Topical NSA!Ds led to more skin reactions consistent with osteoarthritis, in which joint swelling is not
than placebo or oral NSAIDs and fewer gastrointestinal events found and morning stiffness lasts less than 30 minutes.
than oral NSA!Ds. Although sarcoidosis can occasionally cause joint
Discontinuing naproxen and adding celecoxib would involvement, it is unlikely to present with joint symptoms
reduce the risk of gastrointestinal toxicity to an extent alone. Chronic sarcoid arthropathy most commonly involves
similar to that of adding a proton pump inhibitor (PP!) to the ankles, knees, hands, wrists, and metacarpophalangeal
naproxen but would be unwarranted in the setting of a good and proximal interphalangeal joints and is usually accom­
response to topical naproxen. In patients with particularly panied by parenchymal pulmonary disease. It is unlikely to
high risk who require therapy with oral NSA!Ds, simulta­ be the cause of inflammatory polyarthritis in a previously
neously switching to celecoxib and adding a PP! could be healthy middle-aged man.
considered. Although systemic lupus erythematosus (SLE) can
Joint replacement should be considered in patients cause seronegative polyarticular inflammatory arthritis, the
who have knee osteoarthritis with function and/or pain initial presentation in a middle-aged man as an explanation
that cannot be managed using nonsurgical interventions. for polyarticular inflammatory arthritis would be exceed­
Because this patient has neither pain nor limitation on his ingly unlikely, and erosions are not seen as a result of arthri­
current therapy, consideration of joint replacement would tis in SLE. The patient has no signs or symptoms otherwise
be premature. suggestive of SLE such as brain, kidney, lung, heart, or skin
KEY POINT
manifestations.

• Older patients with osteoarthritis who require NSAID KEY POINT

therapy to control pain should be considered for topi­ • Seronegative rheumatoid arthritis has an identical
cal NSAID therapy to manage gastrointestinal toxicity. clinical appearance as seropositive rheumatoid arthri­
tis but is more likely to occur in men.
Bibliography
Hochberg MC, AltmanRD. April KT. et al; American College ofRheumatology. Bibliography
American College of Rheumatology 2012 recommendations for the use of
nonpharmacologic and pharmacologic therapies in osteoarthritis of the Aletaha D. Neogi T. Silman AJ. et al. 2010 rheumatoid arthritis classification
hand, hip, and knee. Arthritis Care Res (Hoboken). 2012 Apr:64(4):465- criteria: an American College ofRheumatology/European League Against
74. [PMID: 225635891 Rheumatism collaborative initiative. ArthritisRheum. 2010 Sep:62(9)2569-
81. [PMID: 20872595]

Item 69 Answer: B Item 70 Answer: B


Educational Objective: Diagnose seronegative Educational Objective: Diagnose adult-onset Still
rheumatoid arthritis. disease.
The most likely diagnosis is rheumatoid arthritis (RA), which The most likely diagnosis is adult-onset Still disease (AOSD),
is characterized by a symmetric inflammatory polyarthritis a multisystem inflammatory disease characterized by high
of the small joints. Autoantibodies such as rheumatoid factor spiking fevers, arthritis, rash, high neutrophil counts , and
or anti-cyclic citrullinated peptide (CCP) antibodies may be markedly elevated serum ferritin. The rash is a nonpruritic
present, although autoantibodies are neither necessary nor salmon-colored macular/maculopapular rash on the trunk
sufficient for diagnosis. Anti-CCP antibodies occur less fre­ or extremities. Serum territin is elevated in many patients,
quently than rheumatoid factor, but their presence has more often to extremely high levels, and is a marker of macrophage

149
Answers and Critiques

activation. Erythrocyte sedimentation rate and C-reactive and pathergy (development of a pustule following a needle
protein can also be impressively elevated. Diagnosis is clin­ stick). Oral ulcers typically resolve spontaneously after 1 to
ical, based on exclusion of infection, malignancy, or other 3 weeks. Eye involvement can be severe, especially when there
rheumatologic diseases. Clinical criteria have been devel­ is a posterior uveitis/retinal vasculitis, and can lead to blind­
oped to assist in the diagnosis of AOSD. lhe most sensitive ness. Hypopyon (suppurative fluid seen in the anterior cham­
of these are the Yamaguchi classification criteria. Diagnosis ber) is also a distinctive feature. The combination of recurrent
requires fulfilling at least five criteria, two of which must be painful oral ulcerations, genital ulcerations, and uveitis make
major. This patient fulfills three major criteria (fever, rash, Behc;et syndrome the most likely diagnosis in this patient.
joint involvement) and three minor criteria (splenomegaly, Other clinical findings include gastrointestinal ulceration that
lymphadenopathy, elevated liver chemistries). Very few other may make it challenging to differentiate Behc;et syndrome
diseases elevate ferritin to this level, although this is not cur­ from Crohn disease with extraintestinal manifestations.
rently one of the criteria. Arthritis affecting medium and large joints is common, and
Acute myeloid leukemia ( AML) is a malignancy of the vasculitic process may cause neurologic, cardiopulmonary,
myeloid progenitor cells. Clinical manifestations of bone kidney, and vascular complications.
marrow failure develop over days to months and include Clinical manifestations of cytomegalovirus infection
fatigue. dyspnea. and easy bleeding. Fever is commonly (CMV) include a mononucleosis-like illness with findings
caused by infection. 1l1e leukocyte count can be low, nor­ ranging from fever and lymphadenopathy to colitis, hep­
mal, or high, but circulating myeloblasts are present in most atitis, and even retinitis. Oral ulcers, genital ulcers, and
cases. 1l1is patient has a significant leukocytosis but no cir­ hypopyon are not features of acute CMV infection.
culating blasts, thus excluding the diagnosis of AML. Herpes simplex virus type 1 (HSV-1) infection has been
Granulomatosis with polyangiitis (formerly known classically associated with causing recurrent painful oral
as Wegener granulomatosis) is also a multisystem disorder ulcerations. Although HSV-1 is being increasingly recog­
characterized by upper respiratory, lower respiratory, and nized as a cause of genital ulceration, it is much less com­
kidney involvement. More than 70% of patients have upper mon than with HSY type 2 infection. Systemic symptoms,
airway manifestations such as sinusitis or nasal, inner ear, such as fever, may be associated with primary herpes virus
or laryngotracheal inflammation. This patient has none of infection but are less common with recurrent episodes of
these manifestations. viraJ activation. Additionally, HSV-1 may also be associated
Systemic lupus erythematosus (SLE) is also a multi­ with ocular disease, although keratitis is the most common
system disease with early nonspecific constitutional symp­ manifestation seen with HSV-1 infection and not hypopyon,
toms, including fever, fatigue, and weight loss. Common as seen in this patient.
presentations include a photosensitive rash and symmetric Reactive arthritis (formerly known as Reiter syndrome)
polyarthritis. Cytopenia is common, whereas leukocytosis is a postinfectious, noninfectious arthritis that occurs in
and extremely elevated serum ferritin levels are not charac­ both men and women. Arthritis, usually oligoarticular,
teristic of SLE. develops several days to weeks after a genitourinary or gas­
trointestinal infection and can be associated with oral and
KEY POINT
genital lesions as well as uveitis. Unlike Behc;et, the oral
• Adult-onset Still disease is characterized by high spik­ ulcers are painless, the genital lesions are hyperkeratotic
ing fevers, arthritis, rash, high neutrophil counts, and rather than ulcerative, and hypopyon is not typical.
markedly elevated serum ferritin.
KEY POINT

Bibliography • Behc;et syndrome is characterized by recurrent painful


lliou C, Papagoras C, Tsifetaki N, Voulgari PV. Drosos AA. Adult-onset Slill's oral ulcers plus at least two of the following: recurrent
disease: clinical. serological and therapeutic considerations. Clin Exp painful genital ulcers, eye involvement, skin involve­
Rheumatol. 2013 Jan-Feb:31(1):47-52. [PMID: 230100971
ment, and pathergy.

Item 71 Answer: A Bibliography


Dalvi SR, Yildirim R. Yazici Y. Behcet's Syndrome. Drugs. 2012 Dec
Educational Objective: Diagnose Behc;:et syndrome. 3:72(t7):2223-41. [PMID: 23153327]

The most likely diagnosis is Behc;et syndrome, a form of vas­


culitis that can affect small to large arterial vessels and is one Item 72 Answer: A
of the few forms of vasculitis that also can affect veins. Behc;et
Educational Objective: Select acetaminophen as a
syndrome has an increased prevalence in a belt from East
first-line phannacologic agent in the treatment of knee
Asia to Turkey and therefore conveys an ethnic/genetic risk in
osteoarthritis.
those with a Mediterranean/Asian background. Behc;et syn­
drome is characterized by recurrent painful oral ulcers plus at Treatment with acetaminophen is appropriate for this patient
least two of the following: recurrent painful genital ulcers, eye with knee osteoarthritis (OA). The presence of progressive
involvement, skin involvement (typically acneiform lesions), knee pain in this older individual that is worse with walking

150
Answers and Critiques

T
and is accompanied by unicompartmental joint-space nar­ low risk for progressing to a C D, should be reassured, and
rowing and osteophytosis in the absence of extensive inflam­ can be followed periodically. In these patients, most episodes
mation is pathognomonic for OA. Although OA includes a ofRaynaud phenomenon are self-limiting and do not require
component of low-level inflammation, the goal of treatment treatment. Persistently symptomatic patients can be treated
is relief of pain and restoration of function. Most treatment with a peripherally acting calcium channel blocker such as
guidelines suggest the initial use of acetaminophen for pain nifedipine or amlodipine. Sildenafil and endothelin-1 block­
control in patients with knee OA. Acetaminophen is usually at ers can be used in refractory cases.
least moderately effective in OA management; at doses of up This patient's likelihood of developing a CTD is low,
to 3 to 4 g/d, it is considered safe and well tolerated. Addition­ and measuring antinuclear and anti-Ul-ribonucleoprotein
ally, it causes little or no gastrointestinal intolerance in most antibodies would be of extremely low yield and would not
patients, does not affect blood pressure, and has significantly be cost-effective.
less nephrotoxicity than NSAIDs . Similarly, obtaining an antiphospholipid antibody panel
Both selective cyclooxygenase (COX)-2 inhibitors and and cryoglobulins in a patient who has no history or evi­
traditional nonselective NSA!Ds (such as ibuprofen) are of dence of thrombosis, pregnancy loss, or vasculitis is unnec­
proven benefit in patients with OA and may be incremen­ essary and not cost-effective.
tally more effective than acetaminophen; selective COX-2 Digital arteriography is an invasive test and is usually
inhibitors have improved gastrointestinal tolerance and normal in patients with Raynaud phenomenon. Obtaining
might be a better choice than a traditional NSAID in this this study would be appropriate if thromboangiitis oblit­
patient given his gastric symptoms. However, both selective erans ("Buerger disease") were suspected, which is a non­
COX-2 inhibitors and traditional NSAIDs promote hyper­ atherosclerotic vascular inflammatmy disease affecting the
tension and can cause or exacerbate kidney disease; this medium and small vessels of the extremities and digits that
patient's chronic kidney disease therefore makes them even has a strong association with smoking and male gender.
less desirable as first-line therapy compared with acetamin­
KEY POINT
ophen. Other options for pharmacotherapy include local
and topical therapy, intra-articular management, tramadol, • Raynaud phenomenon may be the initial symptom of
and, if absolutely necessary, opiates. an w1derlying connective tissue disease; tl1e predictors/
Colchicine is an anti-inflammatory agent commonly features include severe and prolonged vasospastic epi­
used in gout and is not recommended for OA therapy. sodes, asymmetric involvement of the digits, and
abnormal nailfold capillary examination and/or digi­
KEY POINT
tal pitting.
• In patients with osteoarthritis, initial treatment with
acetaminophen for pain control is generally recom­ Bibliography
mended. Lambova SN. Mliller-Ladner U. The role of capillaroscopy in differentiation
of primaiy and secondary Raynaud's phenomenon in rheumatic dis­
eases: a review of the literature and two case reports. Rheumalology Int.
Bibliography 2009 Sep;29(11):1263-71. [PMID: 19547979]
Hochberg MC. Allman RD. April KT, et al; American College of Rheumatology.
American College of Rheumatology 2012 recommendations for the use of
nonpharmacologic and pharmacologic therapies in osteoarthritis of the

Cl
hand, hip and knee. Arthritis Care Res (Hoboken). 2012 Apr;64(4):465-
74. [PMID: 22563589]
Item 74 Answer: D
Educational Objective: Evaluate a patient for primary
angiitis of the central nervous system.
Item 73 Answer: D
The most appropriate next step in management is to obtain
Educational Objective: Evaluate a patient with primary
an intracerebral angiography and brain biopsy. This patient,
Raynaud phenomenon.
presenting with recurrent headaches and rapidly progressive
Clinical observation is the most appropriate next step in encephalopathy but no clear evidence of stroke or infection,
management. This patient's presentation is suggestive of requires consideration for prima1y angiitis of the central ner­
primary Raynaud phenomenon, a common occurrence in vous system (PACNS). PACNS is a rare and challenging. but
young women that may be seen in up to 30% of women treatable. diagnosis because it is isolated lo the CNS with no
who are white. Raynaud phenomenon may also be the ini­ evidence or systemic involvement. His change i_n mental status
tial symptom of an underlying fibrosing connective tissue could alternatively represent Alzheimer disease. or dementia
T
disease (C D) such as mixed connective tissue disease or on a vascular basis given his history of hypertension and
systemic sclerosis; the predictors/features include severe and smoking; however, the time course would be unusual, head­
prolonged episodes of vasospasm, asymmetric involvement aches are somewhat atypical for dementia, and the presence
of the digits, and abnormal nailfold capillary examination of an abnormal cerebrospinal fluid suggests an inflammatory
and/or digital pitting. This patient has none of these features, process. Other diagnoses to consider include reversible vaso­
has a negative family history for CTD, and has undergone constriction syndrome, infection, and intravascular malig­
nailfold capillary examination, which is normal. She is at a nancy. MR angiography (MRA) is insufficiently sensitive for a

151
Answers and Critiques

Cl negative result. as in this patient's case, to suggest an absence treatment guidelines recommend periodic monitoring of liver
of vasculitis. and insufficiently specific for a positive finding chemistry tests (every 8-12 weeks) in patients being treated
CONT.
to abrogate further work-up. Because treatment or PACNS is with leflunomide.
aggressive and not without hazard, it is important to defini­ Etanercept is a receptor fusion protein, which blocks
tively establish a diagnosis whenever possible. Angiography tumor factor necrosis a. Etanercept-induced liver disease
is generally used to define the extent of vasculitic disease. is extremely rare and should not be considered the likely
Although it is more sensitive than MRA, it is still insufficiently diagnosis.
sensitive and specific to establish a PAC S diagnosis alone. The usual presentation of biliary colic is episodic, with
Brain biopsy is the gold standard, although it also has sensitiv­ severe abdominal pain typically in the epigastrium and/
ity limitations. TI1e combination of intracerebral angiography or right upper quadrant. The pain rapidly intensifies over a
and brain biopsy is therefore the preferred approach. 15-minute interval to a steady plateau that lasts as long as
If PACNS were confirmed. treatment would generally 3 hours and resolves slowly. 1he pain is often associated with
require initiation of cyclophosphamide and high-dose glu­ nausea or vomiting, and there is no jaundice. This patient's
cocorticoids: however. such treatment should only be ini­ symptoms are not suggestive of biliary colic; therefore, cho­
tiated after the brain biopsy or empirically if a brain biopsy lecystectomy is not indicated.
cannot be carried out. Liver biopsy may be needed to evaluate the cause of
Azathioprine is a somewhat safer immunosuppressant liver disease, but it is an invasive process and is only recom­
than cyclophosphamide but has not been shown to be a mended if repeat testing and discontinuation of a known
treatment of first choice in PACI S: however. azathioprine offending agent do not lead to resolution, if the cause of liver
may be utilized to reduce relapse rates among patients disease is uncertain, or if there is evidence of chronic liver
already treated and in remission. disease.
Functional MRI (fMRI) could provide information about
KEY POINT
this patient's overall brain functioning; however. to elate
there are no specific signatures on fMRI that would permit • Leflunomide can induce elevation of liver chemistries,
specific diagnosis of PACNS. which is usually reversible with dose reduction or
drug discontinuation.
KEY POINT

• Intracerebral angiography with brain biopsy can pro­ Bibliography


vide a definitive diagnosis in patients with suspected Singh JA. Furst DE, Bharat A. et al. 2012 update ofthe 2008 American College
primary angiitis of the central nervous system. of Rheumatology recommendations for the use of disease-modifying
antirheumatic drugs and biologic agents in the treatment of rheumatoid
arthritis. Arthritis Care Res (Hoboken). 2012 May:64(5):625-39. [PMID:
Bibliography 22473917]
Birnbaum J. Hellmann DB. Primary angiitis of the central nervous system.
Arch Neural. 2009 Jun:66(6):704-9. [PMID: 195061301
Item 76 Answer: A
Educational Objective: Identify azathioprine as a con­
Item 75 Answer: B traindication to the use of febuxostat.
Educational Objective: Manage liver toxicity in a
Azathioprine is a contraindication to the use of febuxostat in
patient taking leflunomide.
this patient with gout. Febuxostat is a purine analogue that
Discontinuation of the nonbiologic disease-modifying anti­ blocks urate synthesis by inhibiting xanthine oxidase, the
rheumatic drug leflunomide is appropriate for this patient final enzyme in the pathway of urate synthesis from purine
with rheumatoid arthritis who has developed abdominal precursors. It can be utilized when a patient has intolerance to
pain and tenderness with acute hepatitis. The most likely or failure of allopurinol. Azathioprine, a purine analogue used
diagnosis is leflunomide-induced hepatitis. Leflunomide­ in the treatment of inflammatory bowel disease, undergoes
induced elevation of liver chemistries can occur in up to 20% metabolism via xanthine oxidase. Thus, concomitant use of
of patients taking the medication, and a threefold elevation of febuxostat(a xanthine oxidase inhibitor) can lead to danger­
serum aminotransferase levels has been noted in up to 13% of ously high levels of azathioprine. Of note, use of allopurinol
patients treated with leflunomide. She is at a slight increased (also a xanthine oxidase inhibitor) concomitantly with aza­
risk for it due to concomitant NSAID use. Mild elevations less thioprine also poses a risk and is relatively contraindicated;
than three times the upper limit of normal are usually revers­ however, some practitioners have used allopurinol in this
ible with dose reduction or drug discontinuation. Hence, setting with dose reduction and careful monitoring.
temporary or permanent discontinuation of leflunomide is Use of diltiazem is a relative contraindication to col­
indicated in this patient. In patients with more severe eleva­ chicine, not febuxostat. Diltiazem is a moderate CYP 34A
tions(> three times), additional therapy with cholestyramine inhibitor, and coadministration of this agent with colchicine
to quickly decrease the drug levels would also be indicated can cause elevated colchicine levels.
because leflunomide undergoes significant enterohepatic Moderate chronic kidney disease (estimated glomerular
circulation. The current American College of Rheumatology filtration rate, 30-59 mL/min/1. 73 m2) is not a contraindication

152
Answers and Critiques

to the use of febuxostat; no dose adjustment to this medi­ rarely. if ever. involves temporal arteries. Thus. a temporal
cation is needed in the setting of mild to moderate kidney artery biopsy is nol indicated.
impairment.
KEY POINT
Nonalcoholic fatty liver disease is not a contraindication
to febuxostat use, although monitoring of hepatic function • Arteriography of the aorta and its branches can be
with administration of the drug is indicated. used to confirm the diagnosis of Takayasu arteritis.
KEY POINT
Bibliography
• T he xanthine oxidase inhibitor febuxostat is contrain­ \,\/en D. Ma CS. Takayasu arleritis: diagnosis, treatment and prognosis. lnl
dicated in patients taking azathioprine, which under­ Rev lrnmunol. 2012 Dec:31(6):462-73. [PMID: 23215768].
goes metabolism via xanthine oxidase; concomitant
use of these agents can lead to dangerously high levels
Item 78 Answer: B
of azathioprine.
Educational Objective: Manage pregnancy planning in
Bibliography a patient with systemic lupus erythematosus who is taking
Khanna D, Fitzgerald JD, Khanna PP. et al; American College of mycophenolate mofetil.
Rheumatology. 2012 American College of Rheumatology guidelines for
management of gout. Part I: systematic nonpharmacologic and pharma­ Discontinuation ofmycophenolate mofetil is indicated for this
cologic therapeutic approaches to hyperuricemia. Arthritis Care Res patient with systemic lupus erythematosus (SLE) who plans
(Hoboken). 2012 Oct;64(10):1431-46. [PMID: 23024028]
to become pregnant. Pregnancy outcomes in patients with

CJ
SLE are better if their disease has been well controlled for
Item 77 Answer: C 6 months prior to becoming pregnant. SLE can worsen during
pregnancy in up to one third of patients, and hydroxychloro­
Educational Objective: Diagnose Takayasu arteritis.
quine can reduce this risk. In addition, most rheumatologists
The most appropriate diagnostic test lo perform next is aortic continue stable low-dose prednisone during the pregnancy.
arleriography in this patient who most likely has Takayasu Many medications used in SLE are contraindicated in preg­
arteritis. She presents with arterial compromise in the setting nancy; permitted medications include prednisone, hydroxy­
of a systemic febrile illness. Asymmetric blood pressure in the chloroquine, and azathioprine. Mycophenolate mofetil was
arms suggests arm involvement. and a midabdominal bruit. developed to prevent transplant rejection but in recent years
leg symptoms. and hypertension suggest aortic ancl renal has been used as a treatment for SLE. Mycophenolate works
artery obstruction. ll1e cliflerential diagnosis includes other by inhibiting the purine pathway in nucleotide synthesis and
forms ofvasculilis ancl/or thrombosis. Given her age. sex. ancl may be at least as effective as cyclophosphamide for SLE
high erythrocyte sedimentation rate. a diagnosis of'Takayasu (including lupus nephritis) but with fewer and milder side
arteritis is likely. Because there are no specific laboratory tests effects. ll1is agent is teratogenic and must be stopped for 3
used to diagnose or define Takayasu arteritis. arteriography of' months prior to becoming pregnant. Mycophenolate mofetil
the aorta and its branches is used to confirm the diagnosis and use may also be associated with difficulty in conception in
define the extent ofthe problem. Alternative im,1ging modali­ some cases. This patient with SLE plans to become pregnant
ties such as er angiography or MR angi graphy might also be and has stable disease, and her laboratory parameters show
used for the same purpose. no significant activity. Stopping mycophenolate is the only
An antimyeloperoxidase an1ibocly assay would be use­ necessary intervention, and both hydroxychloroquine and
ful if' microscopic polyangiitis (M PA) were a diagnostic con prednisone should be continued unchanged.
sideration: however. the kidney disease seen in lhis patient is Stopping all three medications would put this patient at
clue 10 renal artery obstruction and occurs in the absence of unnecessary increased risk of a flare-up during pregnancy.
active urine sec! iment such as wou lei be expected in MPA glo­
KEY POINT
meru lonephritis. Moreover. peripheral artery involvement
would not be expected in MPA. a small-vessel disease. • Mycophenolate mofetil is teratogenic and must be
ll1e presence of' antiphospholipid antibodies would be stopped for 3 months prior to becoming pregnant.
consistent wilh the anliphospholipid antibody syndrome and
wilh the presence of thrombotic disease potentially occluding Bibliography
the arm. aorta. and renal arteries. However. her normal pro­ 0stensen M. Forger F. How safe are anti-rheumatic drugs during preg­
nancy? Curr Opin Pharmacol. 2013 Jun:13(3):470-5. [PMID: 23522967]
lhrombin and partial thromboplasti.n limes indicate the absence
or a lupus anticoagulant (one criterion lor anliphospholipid
antibody syndrome). ancl the lack of' elevation in fibrin degra Item 79 Answer: B
elation proclucls (D-climer) argues against a thrombolic disease.
Educational Objective: Diagnose Mycobacterium
Although Takayasu arteril is and giant cell arteritis
tuberculosis infection.
(GCA) share remarkably similar pathology. GCA occurs in
older patients and is characterized by temporal arteritis. The most likely diagnosis is Mycobacterium tuberculosis
whereas 'fakayasu arteritis is a disease of' the young that infection of the hip. Joint i.nfections with M. tuberculosis

153
Answers and Critiques

present as an indolent process, often in the hip, knee, or The peroneal nerve is the most commonly aflected nerve.
spine (Pott disease). Constitutional symptoms are frequently Approximately 14% of patients with SLE have a peripheral
absent, and imaging may reveal nonspecific erosions that neuropathy with the majority (60%) due to SLE. Risk factors
may be interpreted as osteoarthritis. Moderate elevation of for the development of SLE-associated peripheral neuropa­
the etythrocyte sedimentation rate is common. The diagnosis thy include moderate to severe disease and the presence of
is made by joint aspiration with fluid sent for mycobacterial other neuropsychiatric SLE manifestations. Approximately
cultures. This patient is at increased risk due to origination two thirds of patients improve with more aggressive immu­
from and travel to an endemic area (India) and the recent ini­ nosuppression. EMG/NCS can identify a nerve (usually the
tiation of the tumor necrosis factor (TNF)-a inhibitor etaner­ sural nerve) that might be amenable to biopsy to document
cept for treatment of rheumatoid arthritis. TNF-a inhibitors the vasculitis prior to aggressive immunosuppression.
increase the risk of tuberculosis reactivation. Patients should Hydroxychloroquine can cause a neuromyopathy
be screened for latent infection ptior to start of therapy and manifested by proximal muscle weakness and areflexia.
monitored for signs of infection dming therapy. In this case, Biopsy demonstrates vacuoles in the muscle cells. How­
the patient had appropriate treatment for latent tubercu­ ever, hydroxychloroquine has not been associated with
losis infection in the past but may have had an incomplete mononeuritis multiplex.
response or contracted another latent infection. SLE may rarely cause transverse myelitis, which is
Gout rarely occurs in premenopausal women and characterized by a rapidly progressing paraparesis associ­
typically presents in petipheral joints, with the great toe ated with a sensory level. Autonomic symptoms, including
(podagra) the classic site of the first attack. increased urinary urgency, bladder and bowel incontinence,
This patient is not sexually active and thus is not at risk and sexual dysfunction, may be present. The patient has no
for Neisseria gonorrhoeae infection. Furthermore, gonococ­ symptoms suggesting transverse myelitis, and a spine MR! is
cal arthritis typically spares the axial skeleton. not indicated.
Rheumatoid arthritis (RA) is always a consideration A small-fiber neuropathy causes a burning pain in the
in a patient with RA and joint pain. However, it is unusual extremities and has been associated with autoimmune dis­
for a single joint to be involved in an RA flare, and her eases such as SLE but does not cause motor symptoms. Diag­
arthritis appears well controlled overall. Isolated inflam­ nosis is made by skin biopsy, which demonstrates a reduced
mation of a single joint out of proportion to other joints is density of small sensory nerve fibers in the skin.
a clue to infection.
KEY POINT
KEY POINT
• Mononeuritis multiplex is characterized by abnormal
• Patients should be screened for latent tuberculosis findings in the territory of two or more netves in
prior to start of tumor necrosis factor a inhibitor separate parts of the body and is highly specific for
therapy and monitored for signs of infection during vasculitis but can occur in systemic inflammatory
therapy. disorders such as systemic lupus erythematosus.

Bibliography Bibliography
Mariconda M. Cozzolino A. Attingenti P. Cozzolino F. Milano C. Osteoarticular Fiorica B. Aghdassi E. Su J, et al. Peripheral neuropathy in patients with
tuberculosis in a developed country. J Infect. 2007 Apr:54(4):375-80. systemic lupus erythematosus. Semin Arthritis Rheum. 2011
[PMID: 16860392] Oct:41(2):203-tl. [PMID: 2164!018]

Item 80 Answer: B Item 81 Answer: B


Educational Objective: Evaluate a patient with systemic Educational Objective: Diagnose HIV infection as the
lupus erythematosus who has developed mononeuritis cause ofa severe flare ofpsoriatic arthritis.
multiplex.
HIV antibody testing is indicated for this patient with a severe
Electromyography (EMG) and nerve conduction studies flare of psoriatic arthritis. HIV infection can trigger the onset
(NCS) are appropriate for this patient with systemic lupus of or exacerbate preexisting psoriatic arthritis and psoriasis.
erythematosus (SLE) who most likely has rnononeuritis mul­ Skin and joint symptoms tend to be severe. Explosive onset
tiplex. Mononeuritis multiplex is characterized by abnormal or severe flare-up of psoriatic arthritis should therefore raise
findings in the territory of two or more nerves in separate suspicion for concomitant HIV infection. This patient had
parts of the body. She has a foot drop with normal reflexes previously well-controlled ps01iatic arthtitis w1til a recent
that suggests an injury to the peroneal ne1ve and wrist drop severe flare, as manifested by severe pain and swelling of
that suggests injury to the radial ne1ve. EMG/NCS would multiple joints and psoriasis, and should therefore be tested
most likely document a petipheral neuropathy. Mononeu­ for HIV infection.
rilis multiplex is highly specific for vasculitic disorders that Although viral infections other than HIV can trigger
affect the vasa vasorum or netve vascular supply but can flares of psoriasis, infectious mononucleosis is characterized
also occur in systemic inflammatory disorders such as SLE. by sore throat, lymphadenopathy, and splenomegaly, none of

154
Answers and Critiques

which (except adenopathy) is seen in this patient. Therefore, form of spondyloarthritis associated with a specific group
heterophile antibody testing is not indicated. of urogenital and gastrointestinal pathogens. However, anti­
HLA-B27 is associated with psoriatic arthritis but not biotic therapy is not indicated for parvovirus Bl9 infection.
with psoriasis; furthermore, it neither confirms the diagno­ Interferon alfa is of value for treating some viruses, par­
sis nor explains the flare-up of psoriasis. ticularly hepatitis. However, this agent is not needed to treat
The initial clinical manifestation of early localized Lyme parvovirus 819 infection, which is seu·-limited.
disease is erythema migrans, an erythematous skin lesion Glucocorticoid therapy is not indicated in uncompli­
that is noted in 70% to 80% of patients with confirmed cated parvovirus Bl9 infection due to the self-limited nature
infection. Early disseminated Lyme disease develops several of the infection and associated arthritis.
weeks after the initial infection. Patients frequently present
KEY POINT
with a febrile illness associated with myalgia, headache,
fatigue, and lymphadenopathy. Although Lyme disease may • Parvovi.rus B19 infection and its associated arthritis
trigger a flare of psoriasis, as most acute infections can, it are generally self-limited; therefore, management is
cannot account for the patient's thrush. symptomatic, and an NSAID such as ibuprofen should
Streptococcal pharyngitis is a common trigger of guttate alleviate symptoms until the episode resolves.
psoriasis, especially in children. Guttate psoriasis can also be
the first sign of a flare in previously stable chronic plaque pso­ Bibliography
riasis. Guttate psoriasis consists of many small raindrop-like Young NS, Brown KE. Parvovirus Bl9. N Engl J Med. 2004 Feb 5;350(6):586-
papules and plaques on the trunk. 111is patient's psoriatic 97. [PMID: 14762186]
pattern does not suggest guttate psoriasis, and testing for

Cl
streptococcal infection is not indicated.
Item 83 Answer: C
KEY POINT Educational Objective: Diagnose granulomatosis with
• The development of explosive onset or severe flare of polyangiitis.
psoriatic arthritis should raise suspicion for concomi­ ·1 he presl'nce of'anliprolein:1se :i (IJ Jfl) antibodies will be cliag
tant HIV infection. nostic in lhis palient 11·ho most Iikcly has granulom,llosis with
polyangiii is (i(irmerly known as \\·egener gr,rnulom:llosis).
Bibliography :1 systemic necrotizing 1-a-;culilb that preclomin:rntly aflects
Fernandes S, Pinto GM, Cardoso J. Particular clinical presentations of pso­ the upper :11Kl lower respir:1tory tr.1ct and kidneys. More th,1n
riasis in HIV patients. Int J STD AIDS. 2011 Nov;22(ll):653 4. [PMID:
22096050] 70",. of' p,1lients have upper .1irway manifest,llions such as
�inu�itis or n,1sal. inner car. or laryngotracheal inflamma
lion. Purpura and ulcers .ire common skin m,rnilcstations.
Item 82 Answer: B Mononeuritis multiplex may :1lso occur. Pulmonary maniles
t.11ions rnn present as cough. hemoptysis. and pleurisy. Char
Educational Objective: Treat a patient who has par­
,lclcristic radiographic findings include 111ultilc1cal infiltrates
vovirus Bl 9 infection.
or nodules. some of' which may ca1·italc: dilf'use opacities are
111is patient has parvovirus Bl9 infection, and an NSAID such ,cen in palicnts with pulmonary hemorrhage. Pauci immune
as ibuprofen is the appropriate initial treatment. This infection glomeruloncphritis occurs in up to 80",. or p,1lients. Diagnosis
most commonly occurs in children and is characterized by is best established by lung or kidney biopsy. I lowevcr. the
acute polyarthritis with synunetric swelling and stiffness, the presence or anti PR:l :1mibodies is sullicient to establish a
classic "slapped cheek" rash, and flu-like symptoms. Adults diagnosis in p.llients 11·ith cfossic upper :1irway manilesla
tend to contract the virus from children, but the rash may be lions. pulmon,11) infiltrates nodules. :11Kl urinary abnorm,1li
absent or atypical in adults. Therefore, diagnosis should be ties consistenl with glomeruloncphrilb.
suspected in adults with other characteristic findings as well ,\nli double stranded l)N,\ antibodies nre specilic
as exposme to sick children. This patient shows evidence of a but rl'latilely insensitive i<Jr systemic lupus erythematosus
synunetric, small joint arthritis of the hands, a pattern consis­ (SI.El. SU: is an unlikely diagnosis in this patient because the
tent with rheumatoid arthritis as well as several forms of viral irn·ol\'emenl or the upper :1irw,1ys. particularly with erosion
artlu·itis, most characteristically the artlu·itis that accompanies or the sinuses. is uncommon in SI.I-:. the disease is much less
parvovirus 819 infection. This patient is at risk for parvovirus common in men than women. and complement levels are
819 infection given her occupation as an elementary school typically reduced during act i1·c disease.
teacher, and the presence of lgM antibodies for parvovirus is A111i111yelopcroxidc1sc ,1111ibmlies reflect the presence
definitive and establishes the diagnosis. Parvovirns Bl9 infec­ or ANCA in a pcrinuclc:ir rnther than cytosolic pattern
tion and its associated arthritis are generally seli�limited; there­ and are associated with microscopic polyangiilis. eosino
fore, management is symptomatic, and an NSA!D such as ibu­ philic granulomalosis with polyangiitis (l<>rmerly known as
profen should alleviate symptoms w1til the episode resolves. Churg Strauss syndrome). and rapidly progressive glomeru
Azithromycin therapy may be appropriate for treatment lonephritis. Howe1·er. l he presence of' upper airway disease
of Chlamydia infection associated with reactive arthritis, a csscntially rules uul these conditions in this patient.

155
Answers and Critiques

Cl emia.Serum
CONT
cryoglobulins can be elevated in cryoglobulin­
which can characteristically aflect the kidneys, skin.
KEY POINT

• The diagnosis of lupus nephritis, suggested by pro­


and nerves ami can less commonly affect the lungs. How­ teinuria (>500 mg/24 h) or cellular casts (erythrocytes
ever. upper airway involvement is uncommon. and the nor­
or leukocytes) in the urine sediment, must be con­
m,11 complement levels. especially a normal C4. also argue
firmed and classified with a kidney biopsy.
against cryoglobulinemia as a diagnosis.
KEY POINT Bibliography
• The presence of antiproteinase 3 antibodies is suffi­ Hahn BH. McMahon MA, Wilkinson A, et al. American College of
Rheumatology guidelines for screening, treatment, and management of
cient to establish a diagnosis of granulomatosis lupus nephritis. Arthritis Care Res. 2012 Jun:64(6):797-808. [PMID:
with polyangiitis in patients with classic upper air­ 22556106]
way manifestations, pulmonary infiltrates/nodules,
and urinary abnormalities consistent with glomer­ Item 85 Answer: C
ulonephritis.
Educational Objective:Treat hypertension in a
patient with hyperuricemia who is at increased risk for
Bibliography
acute gout.
Lutalo PMK. D'Cruz DP. Diagnosis and classilication of'granulomatosis with
polyangiitis (aka Wegener's granulomatosis). J Autoimmun. 2014 Feb­ The angiotensin receptor blocker (ARB) losartan is the
Mar:48-49:94-8. [PMID: 24485158]
most appropriate antihypertensive drug for this patient
with hyperuricemia who is at increased risk for acute gout.
Item 84 Answer: C Hypertension is a common comorbidity of gout and is
found in approximately 74% of patients with gout. Antihy­
Educational Objective: Evaluate a patient with
pertensive drugs have variable effects on serum urate levels
suspected lupus nephritis using a kidney biopsy.
and risk of acute gout. A population-based, nested-case
A kidney biopsy is appropriate for this patient with sys­ control study compared nearly 25,000 patients with a
temic lupus erythematosus (SLE). Kidney disease occurs in new diagnosis of gout with 50,000 control patients. The
some form in up to 70% of patients with SLE, especially in risk of gout was assessed according to antihypertensive
those who express anti-double-stranded DNA antibodies, drug class. Losartan, but not other ARBs, and calcium
which typically rise and fall with disease activity. Diagnosis channel blockers were associated with a reduced risk of
of lupus nephritis is suggested by proteinuria (>500 mg/24 gout (relative risk for losartan: 0.81 [95% CI, 0.7-0.84];
h) or cellular casts (erythrocytes or leukocytes) in the urine relative risk for calcium channel blockers: 0.87 [95% CI,
sediment of patients fulfilling the formal criteria for the 0.82-0.93)). Both losartan and calcium channel blockers
diagnosis of SLE. Most patients with active lupus nephritis lower serum urate. Losartan, like probenecid, interferes
have low serum complement levels. According to the 2012 with the urate-reabsorbing transporter, thereby promot­
American College of Rheumatology guidelines for evalu­ ing kidney urate excretion. The mechanism by which
ating and treating lupus nephritis, this patient meets the calcium channel blockers lower urate levels is unclear
criteria for kidney biopsy. Unless contraindicated, biopsy but may be mediated through increased glomerular fil­
should be done before initiating therapy. This patient will tration rate and increased urate clearance. Based upon
most likely have a proliferative form of lupus nephritis these data, losartan and calcium channel blockers are the
(class Ill/IV) and will require aggressive immunosuppres­ preferred antihypertensive agents if reducing the risk of
sive therapy. gout is clinically relevant.
The usual therapeutic agents for proliferative forms In this same study, ACE inhibitors, non-losartan
of lupus nephritis are either cyclophosphamide or myco­ ARBs, P-blockers, and diuretics were all associated with an
phenolate mofetil. There are no data to suggest that meth­ increased risk of gout. The absolute risk of gout was greatest
otrexate is useful in this setting. Methotrexate is renally with diuretics, with an estimated risk of six events per 1000
excreted, and toxicity increases in the setting of kidney person-years.
disease.
KEY POINT
Without adequate management, class Ill, class IV, or
class V combined with class 111 or class IV generally are • The angiotensin receptor blocker losartan and cal­
progressive, with a probability of end-stage kidney dis­ cium channel blockers lower serum urate and may be
ease as high as 50% to 70% after 5 to 10 years of diagnosis. useful to treat patients in whom hypertension and
TI1erefore. there is no reason to wait another month to gout are both clinical concerns.
repeat laboratory testing in this patient given the present
abnormalities. Bibliography
Because the vascular lesion in lupus nephritis is at the Choi HK, Soriano LC, Zhang Y, Rodriguez LA. Antihypertensive drugs
level of the arteriole, a renal artery Doppler examination will and risk of incident gout among patients with hypertension: popula­
tion based case-control study. BMJ. 2012 Jan l2:344:d8J90. [PMID:
not be useful for treatment decisions. 22240117]

156
Answers and Critiques

Item 86 Answer: D Measurement of anti-double-stranded DNA antibodies is


Educational Objective: Treat tophaceous gout with appropriate for this patient who is having a flare of system.ic
pegloticase in a patient who has not responded to oral lupus erythematosus (SLE). She has symptoms of fatigue, joint
urate-lowering therapy. pain, rash, leukopenia, and lymphopenia. Urinalysis shows
proteinuria and hematuria, indicating that she may have glo­
Switching febuxostat to pegloticase is appropriate for this mernlonephritis as well. Levels of anti-double-stranded DNA
patient. He has severe tophaceous gout and persistent hyper­ antibodies correlate with SLE disease activity; in particular,
uricemia and has not responded to oral urate-lowering ther­ they correlate with active k.idney disease or glomerulone­
apy, including the xanthine oxidase inhibitors allopurinol phritis and might prompt further evaluation such as k.idney
and febuxostat. He therefore warrants a trial of the synthetic biopsy. Thus, measuring anti-double-stranded DNA antibody
uricase replacement pegloticase, an intravenous medica­ titers may be useful in assessing this patient's recent symp­
tion FDA approved for treatment-failure gout. Patients with toms. Following anti-double-stranded DNA antibody titers
bothersome persistent tophi or recurrent gout flares despite over time can be useful because it is a marker for risk of devel­
oral urate-lowering therapy (or with contraindications to oping lupus nephritis.
available oral therapy) should be considered for pegloticase Antinuclear antibody (ANA) testing is a useful screen­
treatment, in consultation with a rheumatologist. ing tool for SLE because more than 95% of patients with SLE
Xanthine oxidase inhibitors such as febuxostat should be are positive for ANA; however, ANA does not correlate with
discontinued when initiating pegloticase. Immunogen.icity to disease activity.
pegloticase can result in infusion reactions; loss of response to Anti-Ro/SSA and anti-La/SSB antibodies can be pres­
the drug suggests the presence of neutralizing antibod.ies to the ent in patients with Sjogren syndrome as well as SLE. These
drug and indicates increased risk of infusion reactions. There­ antibodies correlate with SLE rashes and photosensitivity
fore, drugs that m.ight otherwise lower the serum urate level and are a risk factor for the development of neonatal lupus
should be d.iscontinued because they may mask a rising sernm erythematosus; however, they do not correlate with disease
urate level that would signal the presence of pegloticase anti­ activity.
bodies and its attendant loss of effectiveness and increased risk Anti-Smith antibodies are highly specific for the diag­
of a serious infusion reaction. Pegloticase is contraindicated in nosis of SLE; however, these antibodies also do not correlate
patients with glucose-6-phosphate dehydrogenase deficiency. with disease activity.
This patient's current gout flare is nearly resolved and Anti-Ul-ribonucleoprotein antibodies are found in
therefore does not require prednisone. Moreover, prednisone patients with SLE and with mixed connective tissue disease
is not effective in reducing serum urate levels, and it is not an but do not correlate with disease activity.
appropriate drug to prevent recurrent attacks of acute gout
KEV POINT
due to its many long-term side effects.
Colchicine is generally working well in reducing gout • Anti-double-stranded DNA antibodies correlate with
flares in this patient. If this were ineffective or not tolerated, systemic lupus erythematosus disease activity, par­
another agent would need to be considered for prophylaxis ticularly active kidney disease or glomerulonephritis.
(or treatment of acute flares). Anakinra is an interleukin-IP
inhibitor that may be considered for off-label use for gout Bibliography
prophylaxis and flare treatment in patients in whom other Andrejevic S, Jeremie 1, Sefik-Bukilica M, Nikolic M, Stojimirovic B. Bonaci­
more conventional agents are ineffective or contraindicated, Nikolic 8. lmmunoserological parameters in SLE: high-avidity anti­
dsDNA detected by ELISA are the most closely associated with the disease
which is not the case for this patient. activity. Clin Rheumatol. 2013 Nov;32(11):16l9-26. [PMID: 23857662]

KEV POINT

• Pegloticase may be considered for patients with resist­


ant gout who have not responded to oral urate-lowering
therapy.
Item 88 Answer: A
Educational Objective: Treat a patient who has sclero­
Cl
derma renal crisis.
Bibliography 'Jhe ACE inhibitor captopril is the most appropriate treat­
Khanna D. Fitzgerald JD, Khanna PP. el al; American College of
Rheumalology. 2012 American College of Rheumalology guidelines for ment for this patient who most likely has scleroclerma renal
management of gout. Part I: systematic nonpharmacologic and pharma­ crisis (SRC) in the sening or cliff'use cutaneous systemic
cologic therapeutic approaches to hyperuricemia. Arthritis Care Res sclerosis (DcSSc). SRC occurs in 10% to 15% of patients
(Hoboken). 2012 Oct:64(10):1431-46. [PMID: 23024028]
with systemic sclerosis and is more frequent in DcSSc com­
pared with limited cutaneous systemic sclerosis. Vascular
involvement of aflerent arterioles leads to glomerular isch­
Item 87 Answer: A emia and hyperreninemia. The typical presentation is acute
Educational Objective: Confirm a flare of systemic onset of oliguric kidney disease and severe hypertension.
lupus erythematosus with an anti-double-stranded DNA mild proteinuria. urinalysis with few cells or casts. micro­
antibody measurement. angiopathic hemolytic anemia, and thrombocytopenia.

157
363

Answers and Critiques

Cl encephalopathy.
Some patients develop pulmonary edema and hypertensive
Normal blood pressure may be present in
with concomitant use of proton pump inhibitors. Cardiovas­
cular risks can be mitigated by appropriate patient selection
CONT.
up to 10%. l11is patient presents acutely with a rapid rise in for chronic NSAID use.
serum creatinine consistent with acute kidney injury, with Topical capsaicin can be used at any time to treat osteo­
a bland urinalysis and non-nephrotic range proteinuria as arthritis as well; however, in the absence of an effect from
well as neurologic symptoms suggestive of encephalopa­ acetaminophen, an NSAID is likely to give this patient more
thy. Although her blood pressure is almost normal. these substantial relief of symptoms.
[indings are highly suggestive of SRC. Treatment with an Duloxetine is a serotonin-norepinephrine reuptake
ACE inhibitor is essential to restore kidney function and inhibitor approved to treat osteoarthritis pain but is slower
manage hypertension associated with SRC. Captopril is acting than NSA!Ds and requires ongoing, rather than inter­
the preferred ACE inhibitor because it has been the most mittent and as-needed, administration.
extensively studied agent in this clinical setting. and its Hyaluronic acid injections have shown only a minimal
short half-life allows rapid titration. degree of benefit in the treatment of knee osteoarthritis;
Cyclophosphamide is a potent immunosuppressant they also require an invasive procedure for administration
used to treat severe or life threatening manifestations of and are expensive. Therefore, they would not be preferred to
certain diseases such as systemic lupus erythematosus or treatment with an NSAID.
systemic vasculitis. It is ineftective in treating SRC. which is Narcotics such as hydrocodone should be reserved for
vascular and noninflammatory. patients who have not responded to nonpharmacologic mea­
l11is patient does not have inflammatory end-organ sures in addition to NSA!Ds. An alternative to hydrocodone
involvement: therefore. methylprednisolone is not needed. is tramadol, a centrally acting synthetic opioid analgesic
Glucocorticoids are not useful in SRC. and intravenous glu­ that binds to µ-opioid receptors and inhibits reuptake of
cocorticoids may cause worsening symptoms. norepinephrine and serotonin. It can be used for analgesia
Sildenafil can be used to treat pulmonary hypertension when NSA!Ds are not tolerated or are contraindicated. Side
or finger ulcerations but is not appropriate for SRC. which effects include headaches and dizziness. Tolerance can occur
is primarily mediated through the renin-angiotensin axis. with long-term use; withdrawal symptoms can occur with
discontinuation.
KEY POINT
KEY POINT
• In patients with scleroderma renal crisis, treatment
with an ACE inhibitor is essential to restore kidney • An NSAID should be initiated in patients with osteo­
function and manage hypertension. arthritis if first-line therapy with acetaminophen does
not provide adequate relief
Bibliography
Penn H. Howie AJ. Kingdon EJ, et al. Scleroderma renal crisis: patient char­ Bibliography
acteristics and long-term outcomes. QJM. 2007 Aug;IOO(S):485-94. Hochberg MC. Altman RD. Toupin AK, et al. American College of
[PMID: 17601770] Rhewnatology 2012 recommendations for the use of nonpharmacologic
and phannacologic therapies in osteoarthritis of the hand, hip. and knee.
Arthritis Care Res (Hoboken). 2012 Apr;64(4):465-74. [PMID: 22563589]

Item 89 Answer: B
Educational Objective: Treat a patient who has
inadequately controlled knee osteoarthritis. Item 90 Answer: B
Educational Objective: Diagnose bladder cancer
An NSAID such as diclofenac is indicated for this patient
in a patient who has received cyclophosphamide.
with knee osteoarthritis. In addition to the implementation
of nonpharmacologic measures such as an exercise regimen Cystoscopy is the most appropriate diagnostic test to perform
and/or assistive devices, the initial pharmacologic manage­ next in this patient. He has painless hematuria with a his­
ment of osteoarthritis recommended in guidelines issued tory of granulomatosis with polyangiitis (formerly known as
by various societies is acetaminophen in doses ::;3 g/d. If Wegener granulomatosis), which was treated with the non­
this offers inadequate relief; NSAIDs can be used. NSAIDs biologic disease-modifying antirheumatic alkylating agent
are more efficacious than acetaminophen in the relief of cyclophosphamide. Both the underlying rheurnatologic con­
osteoarthritis pain. Treatment guidelines suggest using the dition and the medication used for its treatment are associ­
lowest possible effective dose for the shortest time period ated with increased risk of malignancy, especially bladder
because side effects are common and occasionally severe. cancer. Bladder cancer usually presents with painless frank
However, many patients require years of NSAID use given (usually not microscopic) hematuria, and cystoscopy with
the prolonged timeframe over which the disease is symp­ biopsy is most likely to lead to the correct diagnosis. In con­
tomatic and the small number of alternative pharmacologic trast, kidney involvement due to the disease is associated with
treatments. NSAIDs are associated with important toxicities, glomeruloneph1itis, and urinalysis shows e1ythrocyte casts
particularly with prolonged exposure. TI1e risk of peptic or dysmorphic e1ythrocytes, which is not the case here. The
ulcer disease and gastrointestinal bleeding can be reduced risk of bladder cancer is higher if the patient has received oral

158
Answers and Critiques

cyclophosphamide because there is prolonged daily exposure malities may also contribute to the cardiac dysfunction but
to the metabolites associated with causing mucosal irritation are less common causes of systemic sclerosis heart disease.
and metaplasia. The incidence of cystitis and bladder cancer is Constrictive pericarditis is a chronic disorder resulting
lower with intermittent intravenous cyclophosphamide, espe­ from inflammation and fibrosis of the pericardium with loss
cially when given with mesna, an adjuvant therapy given with of elasticity and resulting noncompliance of the pericar­
cyclophosphamide to detoxify urotoxic metabolites. Impor­ dium. Although it may be associated with connective tissue
tantly, bladder cancers associated with cyclophosphamide disease. this patient does not have specific clinical findings
exposure may be more aggressive and should be urgently typically associated with constrictive pericarditis such as
evaluated even when suspicion is low. a pericardia! knock or pulsus paradoxus: furthermore. her
CT and ultrasonography may show large lesions affect­ echocardiogram shows evidence of diffuse myocardial dys­
ing the kidneys and gastrointestinal tract but do not detect function and no evidence of impaired cardiac filling with
small and superficial lesions, which can only be detected on normal ventricular function as would be expected with
cystoscopy. constrictive pericarditis.
The patient had no new drng exposure, and urinalysis Pulmonary arterial hypertension is a common compli­
does not show significant findings of nephritis; therefore, cation of systemic sclerosis and may lead to cor pulmonale
there is no reason to suspect a drng reaction or interstitial and primarily right-sided heart failure. This patient's clinical
nephritis and obtain urine eosinophils. presentation is more consistent with generalized myocardial
Urine protein-creatinine ratio to look for glomerular dysfunction due to cardiomyopathy.
disease is not helpful in evaluating a patient with hematuria TI1e kidneys are frequently involved in systemic scle­
when suspicion for the underlying vasculitis is low, as seen rosis. with scleroderma renal crisis (SRC) occurring in 10%
in this patient with a negative p-ANCA. to LS'Yn of patients. SRC causes acute-onset oliguric kidney
disease. severe hypertension. and often microangiopathic
KEY POINT
hemolysis and thrombocytopenia. Except for evidence of
• The use of cyclophosphamide is associated with impaired kidney function likely due to heart failure. this
increased risk of malignancy, especially bladder can­ patient's clinical presentation is not consistent with SRC.
cer, and patients should be evaluated accordingly.
KEY POINT

Bibliography • Microvascular cardiomyopathy is the most common


Talar-Williams C, Hijazi YM, Walther MM, et al. Cyclophosphamide-induced symptomatic manifestation of heart involvement in
cystitis and bladder cancer in patients with Wegener granulomatosis. systemic sclerosis and presents with heart failure.
Ann Intern Med. 1996 Mar 1;124(5):477. [PMID: 86027051

Bibliography
C] Item 91 Answer: A
Tyndall AJ, Banner! B, Vonk M, et al. Causes and risk factors for death in
systemic sclerosis: a study from the EULAR Scleroderma Trials and
Educational Objective: Diagnose cardiomyopathy in a Research (EUSTAR) database. Ann Rheum Dis. 2010 Oct;69(10):1809-15.
[PMID: 205511SS]
patient with systemic sclerosis.
The most likely diagnosis is cardiomyopathy in this patient
with systemic sclerosis. Cardiomyopathy due to systemic scle­ Item 92 Answer: D
rosis-induced coronary vasospasm and microvascular disease
Educational Objective: Diagnose bacterial overgrowth
leading to patchy myocardial fibrosis is the most common
syndrome due to systemic sclerosis-associated intestinal
symptomatic manifestation of heart involvement in systemic
disease.
sclerosis. Although accelerated macroscopic coronary athero­
sclerosis has been associated with other autoimmune inflam­ A glucose hydrogen breath test is indicated. This patient has an
matory diseases. coronary vascular involvement in systemic 8-year history of diffuse cutaneous systemic sclerosis (DcSSc)
sclerosis is most often microvascular, with ischemia due to and now presents with weight loss, abdominal cramping, and
structural changes and recurrent spasm of small vessels. This loose stools. She is at high risk for developing malabsorption
results in contraction band necrosis, a pathologic finding due from bacterial overgrm,vth (also known as blind loop syn­
to episodes of myocardial ischemia followed by reperfusion. drome) due to altered peristalsis caused by fibrosis associated
This process may ultimately lead to patchy fibrosis resulting with her underlying disease. She has unexplained weight
in cardiomyopathy and heart failure. Patients who have sys­ loss as well as loose stools without any increase in symp­
temic sclerosis with symptomatic cardiac involvement have toms of dysphagia, nausea, or vomiting. The most appropriate
a poor prognosis and mortality rate of 75% at 5 years. This study for her evaluation at this time is the glucose hydrogen
patient presents with evidence of fluid overload due to heart breath test. The gold standard for the detection of bacterial
failure as seen on her physical examination and confirmed by overgrowth is small bowel aspiration, but this study is not
an abnormal chest radiograph and an echocardiogram sug­ frequently performed because it is invasive. By comparison,
gesting diffuse myocardial dysfunction, consistent with this the glucose hydrogen breath test is noninvasive and has a high
cause. Infiltrative myocardial fibrosis and conduction abnor- sensitivity and specificity. Barium study may also be done to

159
Answers and Critiques

confirm these findings, but obtaining a CT scan at this point is involvement of the DIP joints, which tend to be spared in
unnecessary and costly. MRI may also be useful in the future rheumatoid arthritis.
for assessment of disease and exclusion of other pathologies.
KEY POINT
She does not have bloody bowel movements or colitis,
and performing a colonoscopy is unlikely to lead to the cor­ • Osteoarthritis with calcium pyrophosphate deposition
rect diagnosis because the primary pathology is in the small is a form of pyrophosphate arthropathy in which
bowel and not the colon. patients often have osteoarthritis in joints not typi­
Endoscopic retrograde cholangiopancreatography is the cally involved with traditional osteoarthritis, includ­
diagnostic test of choice for suspected pancreatic or extrahe­ ing non-weight-bearing joints such as the shoulders
patic biliary tract pathology. She has no evidence of biliary and wrists.
tract blockage or pancreatic disease.
Bibliography
KEY POINT
Zhang W, Doherty M, Bardin T, et al. European League Against Rheumatism
• In patients with systemic sclerosis, malabsorption due recommendations for calcium pyrophosphate deposition. Part I: termi­
nology and diagnosis. Ann Rheum Dis. 2011 Apr;70(4):563-70. lPMID:
to bacterial overgrowth is evaluated by obtaining a 21216817]
glucose hydrogen breath test.

Bibliography D
Item 94 Answer:
Forbes A, Marie I. Gastrointestinal complications: the most frequent internal
complications of systemic sclerosis. Rheumatology. 2008 Jun;48(Suppl Educational Objective: Identify the cause ofan elevated
3):iii36-39. [PMID: 19487222] erythrocyte sedimentation rate.
Continuing this patient's current treatment is appropriate at
Item 93 Answer: B this time. This patient has polymyalgia rheumatica (PMR); she
feels well, and her laboratory studies have improved over time.
Educational Objective: Diagnose osteoarthritis with
Although her erythrocyte sedimentation rate (ESR) remains
calcium pyrophosphate deposition.
elevated, it likely does not represent ongoing disease activity.
This patient most likely has pyrophosphate arthropathy, spe­ The most likely cause of this patient's persistently elevated
cifically osteoarthritis with calcium pyrophosphate deposi­ ESR is her age; it may also be elevated because of uncontrolled
tion (CPPD). She has symptoms consistent with degenerative diabetes mellitus (possibly exacerbated by prednisone). The
arthritis (pain worse with activity, brief morning stiffness) degree of elevation is related to the serum globulin con­
and signs of osteoarthritis of her hands. Her radiograph shows centration, the albumin-globulin ratio, the serum fibrinogen
calcification (also known as chondrocalcinosis) of the trian­ concentration, and the percent of hemoglobin A,c but not the
gular fibrocartilage, seen as calcific densities in the region of fasting serum glucose concentrations. ESR is dictated by char­
the distal ulna and ulnar styloid, consistent with CPPD; there acteristics of the erythrocytes themselves and by the presence
is also some narrowing of the carpal metacarpal joints consis­ of specific plasma proteins that alter the normal repulsive
tent with osteoarthritis. In osteoarthritis with CPPD, patients forces betv,reen erythrocytes and influence their ability to
often have osteoarthritis in joints not typically involved with aggregate, form rouleaux, and sediment more quickly. These
traditional osteoarthritis, including non-weight-bearing plasma proteins include acute phase reactants (such as fibrin­
joints such as the shoulders and wrists. ogen) produced by the liver in response to proinflammatory
This patient has risk factors for gout (postmenopausal cytokines occurring in rheumatologic disease, infection, and
woman, hypertension and taking a diuretic, diabetes mel­ malignancy that neutralize these negative surface charges and
litus, overweight). However, she lacks a history of episodic increase ESR. Noninflammatory conditions causing elevated
joint inflammation that typically precedes chronic gouty fibrinogen, including kidney disease, diabetes, pregnancy,
arthropathy. and obesity, can also result in an elevated ESR. Normal aging
The distribution of involved joints, including the dis­ can also cause an elevated ESR; for this female patient, an
tal interphalangeal (DIP) joints, is consistent with psoriatic equation to find the estimate of the maximal expected ESR is
arthritis; however, there is no evidence or symptoms of (age in years+ 10)/2, resulting in 42 mm/h.
inflammatory arthritis. This patient also has no skin or nail It is important to recognize underlying factors that
findings to support the diagnosis of psoriasis. Although influence laboratory studies such as ESR; misinterpreting
some patients develop skin involvement after the onset of an elevated ESR as indicative of persistent inflammation or
arthritis, psoriatic arthritis cannot account for the finding of other disease can lead to inappropriate treatment such as
chondrocalcinosis seen in this patient. prolongation or increase of glucocorticoid therapy. Thus,
The absence of synovial thickening and limited morn­ increasing prednisone or adding methotrexate is not indi­
ing stiffness are not consistent with inflammatory arthritis cated at this time.
such as rheumatoid arthritis. Rheumatoid factor is also This patient reports no headache, jaw claudication, or
negative, and inflammatory markers are within normal lim­ visual changes, all of which are clinical signs of giant cell
its. Finally, examination and radiographic findings indicate arteritis; therefore, temporal artery biopsy is not indicated.

160
Answers and Critiques

KEY POINT KEY POINT

• Noninflammatory conditions (kidney disease, diabe­ • Pericarditis is the most common cardiac manifestation
tes mellitus, pregnancy, obesity) as well as normal of rheumatoid arthritis and is often asymptomatic.
aging can cause an elevated erythrocyte sedimenta­
tion rate. Bibliography
Kitas G, Banks MJ, Bacon PA. Cardiac involvement in rheumatoid disease.
Clin Med. 2001 Jan-Feb:l(l):18-21. [PMID: 11358070]
Bibliography
Colglazier C, Sutej P. Laboratory testing in the rheumatic disease: a practical
review. South Med J. 2005 Feb;98(2):185-91. [PMID: 15759949]
Item 96 Answer: C
Educational Objective: Evaluate synovial fluid for infec­
C] Item 95 Answer: D tion and crystal-related disease.
Educational Objective: Identify rheumatoid arthritis as Gram stain. culture. and crystal analysis are the most help­
the cause of pericarditis. ful and appropriate diagnostic tests to perform next on this
Rheumatoid arthritis (RA) is the most likely cause of this patient's synovial fluid. SynoviaJ fluid aspiration is essential
patient's pericarditis. RA is an independent risk factor for when evaluating for infection and crystal-related disease and
both coronary artery disease and heart failure; patients with is uset"ul in distinguishing betvveen inflammatory and non­
severe extra-articular disease are at particularly increased inflammato1y disease. On physical examination, this febrile
risk of cardiovascular death. Pericarditis is the most com­ patient has evidence of a monoarthritis with inflammation
mon cardiac manifestation of RA and is often asymptomatic. in her knee. which is conArmed by the cloudy appearance
Approximately one third of patients with RA can be found of the synovial fluid at the bedside and her elevated synovial
to have an asymptomatic pericardia! effusion. and 10% of fluid leukocyte count (15,000/µL [15 x 109 /L]). Synovial fluid
patients with RA will have symptomatic pericarditis at some leukocyte counts greater than 2000/µL (2.0 x lOq/L) are con­
point during the course of their disease. Most of those with sistent with inflammatory fluid: the higher the count is. the
symptomatic disease have a positive rheumatoid factor and more inflammat01y the fluid and the greater the suspicion for
active synovitis; however. when symptomatic, the manifes­ c1ystal-related or infectious disease. An acute hospitalization
tations are likely to be similar to those of any other cause of or illness can precipitate an attack of crystal-related arthritis
pericarditis. Diagnosis is most often made by confirming rwo (either gout or pseudogout) and/or infection, and these enti­
of three classic findings: chest pain, often with a pleuritic ties can be evaluated for by synovial fluid analysis. It is import­
component: friction rub; and diffuse ST-segment elevation ant to note that the presence of c1ystals does not rule out
on electrocardiogram. concomitant infection. TI1e most useful tests to obtain when
Ankylosing spondyl itis is a form of spondyloarthritis analyzing synovial fluid are leukocyte count. stains. cultures,
and c1ystal analysis. Sometimes the amount of synovial fluid
that manifests primarily by axial inflammation and bony
available for analysis may be small: therefore. it is important
ankylosis (fusion across joints). Inflammatory arthritis
involvement of the hands tends to present as "sausage to order only the most useful tests.
digits" rather than the symmetric polyarthritis seen in Antinuclear antibodies. glucose levels. and protein lev­
els do not add any useful information and do not distinguish
RA. Although conduction defects and aorlitis with dil­
between infectious and noninfectious synovial fluid.
atation of the aortic valve ring and aortic regurgitation
occur, pericarditis is not seen in patients with ankylosing KEY POINT
spondylitis. • The most useful tests to obtain from synovial fluid are
Polymyalgia rheurnatica occurs in patients over the age
leukocyte count, Gram stains, cultures, and crystal
of SO years and causes diffuse achiness at the neck. shoulder
analysis to evaluate for infection and crystal-related
girdle. and pelvic girdle. It is rarely associated with synovitis
disease and to distinguish between inflammatory and
and is not associated ·with pericarditis.
noninflammatory disease.
Psoriatic arthritis is associated with an increased risk
of coronary artery disease. as is RA. It can also cause a sym­
metric polyarticular inflammatory arthritis involving the Bibliography
Courtney P, Doherty M. Joint aspiration and injection and synovial fluid
small joints of the hands. However. psoriatic arthritis is not analysis. Best Pract Res Clin Rheumatol. 2013 Apr;27(2):137-69. [PMID:
a common cause of pericarditis. 23731929]

161
Index

Note, Page numbers followed by f and t denote figures and tables, respectively. Test Antiproteinase 3 (PR3) antibodies, in granulomatosis with
questions are indicated by Q. polyangiitis, Q83
Anti-Ro/SSA antibodies
A in rheumatic disease, St
Abatacept, 14, lSt in systemic lupus erythematosus, 43
in pregnancy, l9t Anti-Scl-70 antibodies, in rheumatic disease, St
in rheumatoid arthritis, 27 Anti-Smith antibodies, in rheumatic disease, St
ACE inhibitors Antisynthetase syndrome, 64
and drug-induced lupus erythematosus, 48t Antithyroid drugs, and drug-induced lupus
in scleroderma renal crisis, 81 erythematosus, 48t
Acetaminophen, 11 Anti-Ul-RNP antibodies
in osteoarthritis, 32, Q72 in mixed connective tissue disease, 52
in pregnancy, 19t in rheumatic disease, St
Acute calcium pyrophosphate crystal arthritis, 58 Aortic arteriography, in Takayasu arteritis, Q77
Acute cutaneous lupus erythematosus (ACLE), 43, 43f APLA/LAC antibodies, in systemic lupus erythematosus, 46
Acute lupus pneumonitis, in systemic lupus erythematosus, 45 Apremilast, 12t, 13
Adalimumab, 13-14, !St Arthritis, 2
in ankylosing spondylitis, Q60 infectious (see Infectious arthritis)
Adult-onset Still disease (AOSD), 84-85, 84t, Q70 monoarthritis, 2
dermatologic manifestations of, 4t oligoarthritis, 2, 3t
diagnosis of, 84-85 polyarthritis, 2
therapy for, 85 psoriatic (see Psoriatic arthritis)
Yamaguchi criteria for, 841 rheumatoid (see Rheumatoid arthritis [RA])
Allopurinol, 17 Arthroscopic synovectomy, 18
in gout, 56, QI6, Q42, QS9 Arthroscopy, 18
in pregnancy, 19t in knee osteoarthritis, 33
Amaurosis fugax, 70 Aspirin (acetylsalicylate), 10
Ambrisentan, for pulmonary arterial hypertension in systemic in Kawasaki disease, 73
sclerosis, 82 Autoantibodies
Amyopathic dermatomyositis, Q4 in idiopathic inflammatory myopathies, 66-67
dermatologic manifestations of, 4t in systemic lupus erythematosus, 47-48, 47t
Anakinra, !St, 16 in systemic sclerosis, 78, 79t
in acute gouty arthritis, 56 Autoantibody tests, 6-7, St
in pregnancy,l 9t Autoinflammatory diseases, 85-86, 8St
Analgesics Azathioprine, l2t, 13, Q76
in pregnancy, l 9t in Beh,et syndrome, 83
in rheumatologic diseases, 11 in dermatomyositis/polymyositis, 69
topical, 11 in granulomatosis with polyangiitis, 74
ANCA-associated vascLLlitis,73-75, 73f in lgG4-related disease, 88
eosinophilic granulomatosis with polyangiitis, 75 in microscopic polyangiitis, 75
granulomatosis with polyangiitis, 74, 74f in mixed connective tissue disease, 52
microscopic polyangiitis, 74-75 in polyarteritis nodosa, 72
Anemia, in systemic lupus erythematosus, 46 in pregnancy, l9t
Angiotensin receptor blockers (ARBs), in scleroderma renal in relapsing polychonclritis, 84
crisis, 81 in sarcoidosis, 86
Ankylosing spondylitis, 3t, 35-37, 361, 41-42, Ql9, Q33. See also in systemic lupus erythematosus, 49, 49t
Spondyloarthritis in systemic sclerosis with interstitial lung disease, 82
radiographic findings in, 9t
Antibiotics, in infectious arthritis, 63t, Q32 B
Anticentromere antibodies, in rheumatic disease, St Basic calcium phosphate deposition, 58-59, Q48
Anti-cyclic citrullinated peptide antibodies, 6, St Beh,et syndrome, 83, 83f, Q71
in rheumatic disease, St dermatologic manifestations of, 4t
in rheumatoid arthritis, 20, Q3 hypopyon,83f
Anti-double-stranded DNA antibodies International Study Group diagnostic criteria for, 83t
in rheumatic disease, St pathergy, 83f
in systemic lupus erythematosus, Q87 Belimumab, 151, 16
Anti-endothelin therapies, for pulmonary arterial in pregnancy, 19t
hypertension, 52 in systemic lupus erythematosus, 49-50, 491
Antifungal agents, for infectious arthritis, 63t Biofilm, on prosthetic joint, 62
Antihistone antibodies, in rheumatic disease, St Bone marrow biopsy, in systemic lupus erythematosus, 46
Anti-inflammatory agents, 10-ll Bosentan, for pulmonary arterial hypertension in systemic
colchicine, 10 sclerosis, 82
glucocorticoids, 10 Bouchard node, 30
NSAIDs, 10, lOt Brain biopsy, in primary angiitis of central nervous system, 72
in pregnancy, 19t Bronchoalveolar lavage and biopsy, in systemic lupus
Anti-interleukin (IL)-1� therapy, in Beh,et syndrome, 83 erythematosus, Q56
Anti-Jo-I antibodies, in rheumatic disease, St
Anti-La/SSB antibodies, in rheumatic disease, St C
Antinuclear antibodies (ANA), 6, St Calcium channel blockers
in dermatomyositis/polymyositis, 66, 67 and drug-induced lupus erythematosus, 48t
in systemic lupus erythematosus, 47-48, 47t in mixed connective tissue disease, 52

163
Index

Calcium pyrophosphate deposition (CPPD), 57-58 D


clinical manifestations of, 57 Dactylitis. in psoriatic arthritis. 37, 37f
acute calcium pyrophosphate crystal arthritis. 58 Dapsone
cartilage calcification, 57. 58f in hypersensitivity vasculitis, 77
pyrophosphate arthropathy, 58 in relapsing polychondritis, 84
European League Against Rheumatism (EULAR) on, 57 Dem1atomyositis (OM), 64. See also Idiopathic inflammatory myopathies (!!Ms)
management of, 58, 58t dermatologic manifestations of, 4t
pathophysiology of, 57 Diclofenac, 10
radiographic findings in, 9t in knee osteoarthritis, Q89
Canakinumab, 15t, 16 Dietary supplements, 18
in acute gouty arthritis, 56 Diffuse alveolar hemorrhage, Q56
in pregnancy, I 9t in systemic lupus erythematosus, 45
c-ANCA. in rheumatic disease, St Diffuse cutaneous systemic sclerosis (DcSSc), 78. 79t. See also Systemic
Capsaicin, II sclerosis (SSc)
Captopril. in scleroderma renal crisis, Q88 Diffuse idiopathic skeletal hyperostosis (DISH), 29-30, 30f, Q29
Cardiac MRI, for myocardial inflammation in systemic lupus radiographic findings in, 9t
erythematosus, 45 Digital pitting. in systemic sclerosis, 80, 80f
Cardiac valves, abnormal, in systemic lupus erythematosus, 45 Discoid lupus erythematosus (OLE), 43-44, 43f
Cardiomyopathy, in systemic sclerosis, Q91 Disease Activity Score 28 (DAS28), 26
Cartilage calcification, 57, 58f Disease-modifying antirheumatic drugs (DMARDs), 11
Cerebral angiography, in primary angiitis of central nervous system, 72 biologic, 13, 14f, 151
Cerebrospinal fluid analysis, in primary angiitis of central nervous system, 72 abatacept, 14
Certolizumab pegol, 13-14, 15t belimumab, 16
Cervical cancer risk, in systemic lupus erythematosus, 46 interleukin-I� inhibitors, 16
Cervical culture, in disseminated gonococcal infection, Q49 in pregnancy, 19t
Chest radiography, in polymyositis, Q14 rituximab, 14
Chikungunya arthritis, 62 tocilizumab, 14, 16
Chlamydia-induced reactive arthritis. Q45 tofacitinib, 16
Chondrocalcinosis, of knee, 57. SSf tumor necrosis factor (TNF)-a inhibitors, 13-14
Chondroitin sulfate, 18 ustekinumab, 16
Chronic graft versus host disease, 791 nonbiologic, 11-13, 12t
Chronic recurrent gout, 54-55 apremilast, 13
CH50 tests, 6 azathioprine, 13
Cigarette smoking, and rheumatoid arthritis. 20. 27 cyclophosphamide, 13
Classification Criteria for Psoriatic Arthritis (CASPAR), 37, 37t cyclosporine, 13
Colchicine, 10 hydroxychloroquine, 11
in acute gouty arthritis, 55 leflunomide, 13
in Beh,et syndrome, 83 methotrexate, 11
in calcium pyrophosphate deposition, 581 mycophenolate mofetil, 13
in familial Mediterranean fever, Q22 in pregnancy, 19t
in gout. 57, Q59 sulfasalazine, 11, 13
in hypersensitivity vasculitis, 77 in reactive arthritis, 42
in pregnancy, 19t in rheumatoid arthritis, 27
Complementary and alternative medicine (CAM). IS in spondyloarthritis, 41
Complement levels. 6 Disseminated gonococcal infection, 60-61, 60f, Q49. See also Infectious arthritis
in systemic lupus erythematosus. 48 DNA amplification urine test, for Chlamydia trachomatis, Q45
Contraction band necrosis, 82 Doppler echocardiography, in limited cutaneous systemic
C-reactive protein (CRP), 6 sclerosis, Ql3
in systemic lupus erythematosus, 48 Drug-induced lupus erythematosus (DILE), 48, 48t
Cryoglobulinemic vasculitis, 75-76, Q47 Drug-induced myopathies, 68
clinical manifestations of, 76 Duloxetine
diagnosis of. 76 in fibromyalgia, 34
epidemiology and pathophysiology of, 75-76 in osteoarthritis, 32. Q23
management of, 76
Cryoglobulins, St E
types of, 75-76, 76t Echocardiography, in Marfan syndrome, Q39
Crystal arthropathies, 53-59 Ehlers-Danlos syndrome (EDS), 86
basic calcium phosphate deposition, 58-59 Electromyography (EMG)
calcium pyrophosphate deposition. 57-58 in idiopathic inflammatory myopathies, 67
gout, 53-57 for mononeuritis multiplex in systemic lupus erythematosus, QSO
CT scan, in rheumatologic diseases, 7 Endothelin receptor antagonists, for pulmonary arterial hypertension in
Cyclooxygenase (COX) enzymes. 10 systemic sclerosis, 82
Cyclophosphamide. 12t, 13 Enzyme-linked immunosorbent assay (ELISA), 6
in cryoglobulinemic vasculitis, 76 Eosinophilic fasciitis, 79t
in eosinophilic granulomatosis with polyangiitis, 75 Eosinophilic granulomatosis with polyangiitis (EGPA), 75, Q27
in giant cell arteritis, 71 Episcleritis, 4t
in granulomatosis with polyangiitis, 74 Epoprostenol, for pulmonary arterial hypertension in systemic sclerosis, 82
in interstitial lung disease with diffuse cutaneous systemic sclerosis, Q24 Erosive osteoarthritis, 29. Q52
in microscopic polyangiitis. 75 Erythema infectiosum, 62
in mixed connective tissue disease. 52 Erythrocyte sedimentation rate (ESR), 3-6
in polyarteritis nodosa, 72 in giant cell arteritis, 70
in pregnancy, l9t in systemic lupus erythematosus, 48
in primary angiitis of central nervous system, 72 Escherichia coli, infectious arthritis by, 60. See also Infectious arthritis
in relapsing polychondritis, 84 Esophageal dysmotility/reflux, in systemic lupus erythematosus, 46
in systemic lupus erythematosus, 49, 49t Essential mixed cryoglobulinemia, 76
in systemic sclerosis with interstitial lung disease, 82 Etanercept, 13-14, 15t
Cyclosporine, 12t, 13 in rheumatoid arthritis. Q46
in pregnancy, 19t Evans syndrome, 46
in relapsing polychondritis. 84 Exercise
Cyclosporine drops, for dry eyes in Sj()gren syndrome. Q38 in fibromyalgia, Q67
Cystoscopy, in bladder cancer. Q90 in osteoarthritis, 32
Cytopenia, in systemic lupus erythematosus, 46 in spondyloarthritis, 41

164
Index

F Hepatitis B virus (HBV), 61-62


Familial cold autoinflammatory syndrome (FCAS), 85t Hepatitis B virus-associated arthritis, Q30
Familial Mediterranean fever (FMF), 85, 85t Hepatitis C virus (HCV), 61-62
colchicine in, Q22 Herbal products, for arthritis, 18
Febuxostat, 17 HIV antibody testing, in psoriatic arthritis, Q81
in gout, 56, Ql HLA-B27 testing, in spondyloarthritis, 35, Q21
in pregnancy, 19t Hodgkin lymphomas, in systemic lupus erythematosus, 46
Felty syndrome, 24-25 Hughes-Stovin syndrome, 83
Fibromyalgia, 33-35, Q9 Hyaluronic acid injection, in osteoarthritis, 33
aerobic exercise program in, Q67 Hydralazine, and drug-induced lupus erythematosus, 48t
diagnosis of, 34 Hydroxychloroquine, 11, 12t
epidemiology of, 33-34 in pregnancy, 19t
management of, 34 in psoriatic arthritis, 42
pathophysiology of, 34 in rheumatoid arthritis, 27
in systemic lupus erythematosus, 44 in risk of neonatal lupus erythematosus, 50
Fungal infections, infectious arthritis by, 61 in systemic lupus erythematosus, 49, 49t, Q41
Hyperimmunoglobulinemia D with periodic fever
G syndrome (HIDS), 85t
Gastrointestinal disease, in systemic lupus erythematosus, 46 Hypersensitivity vasculitis, 77, 77f, Q40
Giant cell arteritis (GCA), 70-71, Ql7 Hypopyon, in Behc;et syndrome, 83, 83f
clinical manifestations and diagnosis of, 70
epidemiology of, 70
management of, 70-71 Ibuprofen, in parvovirus Bl9 infection, Q82
pathophysiology of, 70 Idiopathic inflammatory myopathies (IIMs), 64-69
Glucocorticoids cancers associated with, 66
in acute gouty arthritis, 55-56 clinical manifestations of, 64-66
adverse events from, 10 cardiopulmonary involvement, 65-66
in basic calcium phosphate deposition, 59 cutaneous manifestations, 64-65, 65f
in calcium pyrophosphate deposition, 58t gastrointestinal involvement, 66
in cryoglobulinemic vasculitis, 76 muscular involvement, 64
in dermatomyositis/polymyositis, 68 diagnosis of, 66-68
in eosinophilic granulomatosis with polyangiitis, 75 autoantibodies, 66-67
in giant cell arteritis, 70 electromyography, 67
in granulomatosis with polyangiitis, 74 imaging studies, 67
in Henoch-Schonlein purpura, 77 muscle biopsy, 67-68
in inflammatory bowel disease-associated arthritis, 42 f
muscle-related enzymes, 66
in microscopic polyangiitis, 75 dif erential diagnosis for, 68, 68t
in mixed connective tissue disease, 52 epidemiology of, 64
in osteoarthritis, 33 features of, 67t
in pregnancy, I 9t management of, 68-69
in primary angiitis of central nervous system, 72 muscle weakness in, 64
in psoriatic arthritis, 42 pathophysiology of, 64
in rheumatoid arthritis, 27 prognosis of, 69
in rheumatologic diseases, 10 lgG4-related disease, 87-88, 87t, Q51
in sarcoidosis, 86 conditions associated with, 87t
in systemic lupus erythematosus, 49 diagnosis of, 87
in systemic sclerosis with interstitial lung disease, 82 therapy for, 88
Glucosamine sulfate, 18 lloprost, for pulmonary arterial hypertension in systemic sclerosis, 82
Glucose hydrogen breath test, Q92 Imaging studies
Golimumab, 13-14, 15t in idiopathic inflammatory myopathies, 67
Gottron sign/papules, 64, 65f in neuropsychiatric systemic lupus erythematosus, 45
Gout, 53-57, Q59 in osteoarthritis, 31, 3lf
acute gouty arthritis, 54 in rheumatoid arthritis, 23, 23f, 24f
differential diagnosis of, 55t in rheumatologic disease, 7, 9
American College of Rheumatology criteria for, 55, 55t CT scan, 7
chronic recurrent or tophaceous gout, 54-55, 54f joint aspiration, 9
clinical manifestations of, 54-55 MRI, 7
comorbidities associated with, 53 radiography, 7, 9t
diagnosis of, 55, 55t ultrasonography, 7, 9
epidemiology of, 53 in spondyloarthritis, 39-41, 41f
hyperuricemia in, 53, 54t lmmunofluorescence assay, 6
intercritical gout, 54 lmmunosuppressive therapy, recommendations for screening before, 16
low-fat dairy products in, Q66 Inclusion body myositis (IBM), 64, Q36. See also Idiopathic inflammatory
management of, 55-57 myopathies (IIMs)
acute gout flares, 55-56 Infectious arthritis, 59-64
urate-lowering therapy and prophylaxis, 56-57 causes of, 60-63
in older patients, 53 disseminated gonococcal infection, 60-61, 6lf
pathophysiology of, 53-54, 53f, 54t fungal infections, 61
radiographic findings in, 9t, 55, 56t gram-positive organisms, 60
Gram-negative bacterial joint infections, 60. See also Infectious arthritis Lyme arthritis, 61
Gram-positive bacterial joint infections, 60. See also Infectious arthritis mycobacterial infections, 61
Granulomatosis with polyangiitis (GPA), 74, 74f, Q31, Q83 nongonococcal gram-negative organisms, 60
previously damaged joints, 62
H prosthetic joint infection, 62
Haemophi/us injluenzae, infectious arthritis by, 60. See also Infectious viral infections, 61-62
arthritis clinical manifestations of, 59-60
Hand osteoarthritis, Q43 diagnosis of, 59
Heberden node, 30, 30f imaging in, 60
Heerfordt syndrome, 86 laboratory studies in, 60
Heliotrope rash, 65, 65f management of, 63-64
Hemophagocytic syndrome, 84 pathophysiology of, 59
Henoch-Schonlein purpura (HSP), 77 risk factors for, 59t

165
Index

Inf1ammatory bowel disease-associated arthritis, 3t, 361, 37-38, 42. Matrix metalloproteinases (MMPs), 28
See also Spondyloarthritis Mechanic's hands, 65, 6Sf
Inf1ammatory pain, l Meniscectomy, and left knee osteoarthritis, Q64
vs. noninf1ammatory pain, I, It Methotrexate, IJ, l2t
Inf1ammatory polyarthralgia, in systemic lupus erythematosus, 44 in adult-onset Still disease, 85
lnf1iximab, 13-14, !St in dermatomyositis/polymyositis, 69
lnt·erferon alfa, in Behc;et syndrome, 83 discontinuation of, in patient of childbearing age, QSO
lnterferon-y release assay, QJ8 in giant cell arteritis, 71
Interleukin-I� inhibitors, !St, 16 in granulomatosis with polyangiitis, 74
Interstitial lung disease {]LD) in lgG4-related disease, 88
in dermatomyositis/polymyositis, 65-66 in inf1ammatory bowel disease-associated arthritis, 42
polymyositis and, QL4 in microscopic polyangiitis, 75
in rheumatoid arthritis, 24 in mixed connective tissue disease, 52
in systemic lupus erythematosus, 45 in polyarteritis nodosa, 72
in systemic sclerosis, 81-82 in pregnancy, 191
Intra-articular glucocorticoids in psoriatic arthritis, 42
in gout, Q6 in relapsing polychondritis, 84
in osteoarthritis, QS in rheumatoid arthritis, 27, Q54
Intracerebral angiography with brain biopsy, in primary angiitis of the in sarcoidosis, 86
central nervous system, Q74 Methylprednisolone, in giant cell arteritis, 70
Intravenous immune globulin {]VIG) therapy Microscopic polyangiitis (MPA), 74-75
in dermatomyositis, 69 Milnacipran, in fibromyalgia, 34
in Kawasaki disease, 73 Milwaukee shoulder syndrome, 59
lschemic heart disease, in systemic lupus erythematosus, 45 Mind-body practices, 18
Minocycline, and drug-induced lupus erythematosus, 481, Q35
J Mixed connective tissue disease (MCTD), 52-53, QSS
Jaccoud arthropathy, 44, Q2 clinical manifestations of, 52
Jaw claudication, in giant cell arteritis, 70 diagnosis of, 52
Joint aspiration epidemiology of, 52
for basic calcium phosphate deposition. 59 management of, 52
in gout, 55, Q25 prognosis of, 53
in infectious arthritis, 60 Monoarticular arthritis, 2
in prosthetic joint infection, 62 Morphea, 791
in rheumatologic diseases, 9 MRI
Joint infection. 59. See also Infectious arthritis in idiopathic inflammatory myopathies, 67
Joint-space narrowing, in osteoarthritis, 31, 3Jf of large joints, for osteonecrosis, Q6L
in rheumatologic diseases, 7
K of sacroiliac joints, in spondyloarthritis, Ql2
Kawasaki disease (KD), 73 Muckle-Wells syndrome (MWS), 85t
Keratoconjunctivitis sicca, 24, 51. See also Sj/lgren syndrome Muscle biopsy, in idiopathic inf1ammatory myopathies, 67-68
Kidney biopsy Muscle enzyme elevation, in idiopathic inf1ammatory myopathies, 66
in granulomatosis with polyangiitis, 74 Myalgia, in systemic lupus erythematosus, 44
for lupus nephritis in systemic lupus erythematosus, 44, Q84 Mycobacrerium marinwn, infectious arthritis by, 61. See also Infectious
arthritis
L Mycobacrerium tuberculosis, infectious arthritis by, 61, Q79. See also
Laboratory studies Infectious arthritis
in osteoarthritis, 31 Mycophenolate mofetil, 121, 13
in rheumatoid arthritis, 22-23 in dermatomyositis/polymyositis, 69
in rheumatologic disease, 3-7 discontinuation of, in systemic lupus erythematosus, Q78
autoantibody tests, 6-7, Bt in granulomatosis with polyangiitis, 74
complement levels, 6 in IgG4-related disease, 88
C-reactive protein, 6 in microscopic polyangiitis, 75
erythrocyte sedimentation rate, 3-6 in polyarteritis nodosa, 72
in spondyloarthritis, 39 in pregnancy, 191
Lef1unomide, 12t. 13 in sarcoidosis, 86
in dermatomyositis/polymyositis, 69 in systemic lupus erythematosus, 49, 491
in pregnancy, l 9t Myocarditis, in systemic lupus erythematosus, 45
in rheumatoid arthritis, 27, QB Myositis-associated autoantibodies (MAA), in dermatomyositis/
Lef1unomide-induced hepatitis, Q75 polymyositis, 67
Leukemias, in systemic lupus erythematosus, 46 Myositis-specific autoantibodies (MSA), in dermatomyositis/polymyositis,
Leukopenia, in systemic lupus erythematosus, 46 66
Libman-Sacks endocarditis, 45
L.idocaine. l l N
Limited cutaneous systemic sclerosis (LcSSc), 78, 791, Q13, Q53. See also Naproxen
Systemic sclerosis (SSc) in acute gouty arthritis, 55
Linear scleroderma, 791 in Lofgren syndrome, Q57
Lip biopsy, in Sj/lgren syndrome, 51 Neisseria gonorrhoeae, infectious arthritis by, 60-61. See also Infectious
Lipid profile, in rheumatoid arthritis. QLS arthritis
L.Ofgren syndrome, 86, Q57 Neonalal lupus erythematosus, 50
Losartan, for hypertension in hyperuricemia, QBS Neonatal-onset multisystem inf1ammatory
Lung biopsy, in granulomatosis with polyangiitis, 74 disease (NOMID), 851
Lung cancer, in systemic lupus erythematosus, 46 Nephritis, in systemic lupus erythematosus, 44
Lupus erythematosus cells, 45 Nephrogenic systemic fibrosis, 791
Lyme arthritis, 61, Q34 Nerve conduction studies, for mononeuritis multiplex in systemic lupus
Lyme disease, dermatologic manifestations of, 4t eryrhematosus, QBO
Lymph node biopsy, in Sjogren syndrome, Q20 Neuropsychiatric systemic lupus erythematosus (NPSLE), 44-45, 441. See
Lymphoma, risk of, in Sjogren syndrome, SL also Systemic lupus erythematosus (SLE)
Nongonococcal gram-negative bacterial arthritis. 60. See also Infectious
M arthritis
Malar/butterf1y rash, of systemic lupus erythematosus, 43, 43f Non-Hodgkin lymphoma, in systemic lupus erythematosus, 46
Marfan syndrome (MFS), 86-87, Q39 Nonscarring alopecia in systemic lupus erythematosus, 44

166
Index

NSAIDs Poikiloderma, 65
in acute gouty arthritis,55, 56 Polyarteritis nodosa (PAN). 72, Q63
in adult-onset Still disease, 85 clinical manifestations and diagnosis of, 72
in ankylosing spondylitis, 41 epidemiology and pathophysiology of, 72
in basic calcium phosphate deposition, 59 management of, 72
in calcium pyrophosphate deposition, 58t Polyarthritis, 2
for gout prophylaxis, 57 Polymyalgia rheumatica (PMR), 71. Q94
in hypersensitivity vasculilis, 77 Polymyositis (PM), 64. See also Idiopathic inflammatory myopathies {]!Ms)
oral, 10 Porphyromonas gingivalis, and risk of rheumatoid arthritis, 20
in osteoarthritis,32 Prednisone
in pregnancy,19t in acute gouty arthritis, 56
in psoriatic arthritis,42 in adult-onset Still disease, 85
in reactive arthritis, 42 in Beh<;et syndrome, 83
in rheumatoid arthritis. 27 in giant cell arteritis, 70
in rheumatologic disease, LO in gout prophylaxis, 57
selection of,10 in Henoch-Schonlein purpura. Q7
side effects of, 10, lOt high-dose, in giant cell arteritis, Ql7
in spondyloarthritis, 41 in hypersensitivity vasculitis,77
in systemic lupus erythematosus, 49t, SO in lgG4-related disease,88
topical, 10 low-dose, in polymyalgia rheumatica,Q44
in polyarteritis nodosa, 72
0 in polymyalgia rheumatica, 71
Occupational therapy, 18 in polymyositis, Q28
Oligoarthrilis, 2, 3t in relapsing polychondritis, 84
Omega-3 fatty acids, 18 in systemic lupus erythematosus, 49, 49t
Opiates. 11 in Takayasu arteritis, 72
in pregnancy,19t Pregabalin,in fibromyalgia. 34
Osteoarthritis (OA),28-33, Q43 Pregnancy
with calcium pyrophosphate deposition, Q93 rheumatologic medications and, 18. 191
classification of, 29-30 systemic lupus erythematosus in, 50
clinical manifestations of,30-31, 30f systemic sclerosis and, 82
diagnosis of. 30-32 Primary angiitis of the central nervous system (PACNS), 72, Q74
differential diagnosis of, 31 Probenecid,17
diffuse idiopathic skeletal hyperostosis, 29-30, 30f in gout, 57
duioxetine in. Q23 in pregnancy. 19t
epidemiology of, 28 Procainamide, and drug-induced lupus erythematosus, 481
erosive, 29 Prostacyclin, 10
hand,30,30f Prostacyclin analogues, for pulmonary arterial hypertension in systemic
imaging studies in,31,3lf sclerosis, 82
knee/hip,30-31 Prostaglandin E2 (PGE,), lO
laboratory studies in, 31 Prosthetic joint infection, 62
management of, 32-33 blood and synovial fluid cultures in, Q65
glucocorticoid injection, 33 Proteinuria, during pregnancy in systemic lupus erythematosus,50
hyaluronic acid injection, 33 Proton pump inhibitors,in mixed connective tissue disease, 52
intra-articular injection,33 Pseudogout. 58
nonpharmacologic therapy,32 Pseudomonas aerugi11osa. infectious artluitis by. 60. See also Infectious arthritis
pharmacologic therapy, 32-33 Pseudo-rheumatoid arthritis, 58
surgical therapy, 33 Psoriatic arthritis, 3t, 36t, 37, 37f, 37t. 38f, 42, Q26. See also
in older patients,topical NSAID therapy for, Q68 Spondyloarthritis
pathophysiology of,28-29 dennatologic manifestations of, 4t
primary,29 radiographic findings in, 9t
radiography in,9t,Q37 Pulmonary arterial hypertension (PAH), in systemic sclerosis, 82
risk factors for, 29 Pyoderma gangrenosum, in rheumatoid arthritis, 24
secondary,29
of spine,31 R
Osteogenesis imperfecta (01), 87 Radiography
Osteonecrosis,in systemic lupus erythematosus,44 in gout, 55, 56t
Ovarian cancer,risk of, in dermatomyositis, Q62 in osteoarthritis, Q37
in rheumatologic diseases. 7. 9t
p Raynaud phenomenon. 80, Q73
p-ANCA,in rheumatic disease, St Reactive arthritis, 3t, 36t, 38, 42, Q58. See also Spondyioarthritis
Parvovirus B19 arthritis, 62 dermatologic manifestations of,4t
Pathergy, in Beh<;et syndrome, 83, 83f Relapsing poiychondritis (RP). 83-84
Pegloticase, 17 diagnosis of, 84
in gout. 57 floppy ears in, 84f
in pregnancy, l 9t McAdams criteria for, 84t
Pegloticase, in tophaceous gout, Q86 Retinal disease, 41
Pencil-in-cup deformity,in psoriatic arthritis, 37, 38f Rheumatic fever. dermatologic manifestations of, 4t
Pericarditis Rheumatoid arthritis (RA),3t, 20-28, Q3, Q69
rheumatoid arthritis and, 24, Q95 American College of Rheumatology classification criteria for, 211
in systemic lupus erythematosus. 45 clinical manifestations of,21-22,22f, 22t
Phosphodiesterase inhibitors diagnosis of. 21-24
in 1nixed connective tissue disease. 52 epidemiology of. 20
for pulmonary arterial hypertension in systemic sclerosis. 82 extra-articular manifestations and complications of, 24-25
Physical therapy, 18 cardiac involvement, 24
Plasmapheresis, in cryoglobulinemic vasculitis, 76 eye manifestations. 24
Pleuritis Felty syndrome, 24-25
in rheumatoid arthritis. 24 joints, 24
in systemic lupus erythematosus. 45 pulmonary manifestation. 24
Pieuropericarditis,in mixed connective tissue disease,52 skin,24
Podagra, 54. See also Gout unusual complications, 25

167
Index

Rheumatoid arthritis (continued) Sjogren syndrome, 50-52, Q20


hands in, involvement of, 22f autoimmune diseases associated with. 51
imaging studies in, 23 clinical manifestations of, 51, Sit
MRI. 23 conditions mimicking with. 51
radiography. 23. 23f, 24f diagnosis of, 51
ultrasonography, 23 epidemiology of, 51
joints involved in, 21, 22 extraglandular clinical manifestations of, SI t
laboratory studies in, 22-23 laboratory findings in, 51
anti-CCP antibody testing, 23 management of, 51-52, 521
inflammatory markers, 23 pathophysiology of, 50-51
rheumatoid factor, 23 prognosis of, 52
management of, 25-28, 25f, 26f Skin manifestations, of mixed connective tissue disease. 52
disease-modifying antirheumatic drugs. 27 Soft tissue inflammation. in gout. 54
glucocorticoids, 27 Spondyloarthritis, 3t, 35-42
NSA!Ds. 27 classification of, 35, 36t
surgical therapy, 27-28 ankylosing spondylitis, 35-37
morning stiffness in, 20, 21 inflammatory bowel disease-associated arthritis, 37-38
musculoskeletal manifestations of, 22t psoriatic arthritis, 37, 37f, 371, 38f
pathophysiology and risk factors for reactive arthritis, 38
autoantibodies, 20 clinical features of, 361
environmental factors, 20 diagnosis of, 38-41, 391, 40f
genetic factors, 20 HLA-827 testing in, Q21
hormones, 20 imaging studies. 39-41, 4lf
infection. 20 CT, 40-41
pregnancy and. 28, QSO MRl.41
radiographic findings in. 9t radiography, 39-40, 4lf
Rheumatoid factor, 6, 8t. 20 ultrasonography.41
in rheumatoid arthritis. 20, 23, Q3 laboratory studies in. 39
Rheumatoid nodules, 24 management of. 41-42
Rheumatoid vasculitis, in rheumatoid arthritis, 24 MRI in, Ql2
Rheumatologic disease pathophysiology of
approach to patient with, 1-9 environmental factors. 35
arthritis, 2 genetic factors. 35
monoarthritis, 2 Staphylococcus aureus, infectious arthritis by, 60. See also Infectious arthritis
oligoarthritis. 2, 3t Statins, and drug-induced lupus erythematosus, 48t
polyarthritis, 2 Streptococcus pneumoniae, infectious arthritis by, 60. See also Infectious
extra-articular manifestations of, 2-3 arthritis
constitutional symptoms, 2 Subacute cutaneous lupus erythematosus (SCLE), 43, 43f
dermatologic manifestations, 2, 41 Subchondral sclerosis, in osteoarthritis, 31
eye involvement, 2-3, 41 Sulfasalazine, 11, 12t, 13
internal organ involvement, 3, St in inflammatory bowel disease-associated arthritis, 42
imaging studies in, 7. 9 in pregnancy, 19t
inflammatory us. noninflammatory pain, 1, lt in psoriatic arthritis, 42
laboratory studies in, 3-7 in rheumatoid arthritis, 27
medications used in. 9-18 Symptoms Severity Scale (SSS), fibromyalgia and, 34
analgesics. 11 Synovial fluid analysis, 9
anti-inflammatory agents, 10-11 in gout, 55
disease-modifying antirheumatic drugs, 11-16 for infection and crystal-related disease, Q96
and pregnancy, 18, l9t in mycobacterial infections, 61
urate-lowering therapy, 17-18 Synovial fluid leukocyte counts, 9
musculoskeletal examination, 1-2 Systemic lupus erythematosus (SLE), 3t, 42-50, Q61
nonpharmacologic management, 18, 20 American College of Rheumatology criteria for diagnosis of, 47, 47t
complementary and alternative medicine. 18 clinical manifestations of, 43-46
mind-body practices, 18 cardiovascular involvement, 45
natural products, 18 gastrointestinal involvement, 46
physical and occupational therapy, 18 hematologic involvement, 46
surgery, 18 kidney involvement, 44
rheumatologic and musculoskeletal manifestations in systemic dis­ malignancy, 46
ease. 3, 6t mucocutaneous involvement, 43-44, 43f
Rilonacept. !St, 16 musculoskeletal involvement, 44
in pregnancy, 19t neuropsychiatric involvement, 44-45. 44t
Rituximab, 14, !St pulmonary involvement, 45
in granulomatosis with polyangiitis, 74 dermatologic manifestations of, 4t
in lgG4-related disease. 88 diagnosis of, 46-49
in pregnancy, 19t differential diagnosis of, 48
in rheumatoid arthritis. 27 epidemiology of, 43
Rubella (German measles). 62 laboratory studies for, 47-48, 471
management of, 49-50
s mulliorgan involvement in, 42
Sarcoidosis. 86 pathophysiology of, 42-43
dermatologic manifestations of. 4t preconception counseling in, Q!O
Schirmer test, 51 pregnancy and childbirth issues, 50
Scleredema, 79t prognosis of, 50
Scleritis, 41, Q31 serial monitoring of disease-responsive laboratory studies in, 49
Sclerodactyly, in systemic sclerosis. 78. 80f Systemic sclerosis (SSc), 77-82
Scleroderma renal crisis (SRC). 81. Q88 classification of, 78
Scleromyxedema. 791 clinical manifestations and management of, 78-82
Serotonin syndrome. 11 cardiac involvement. 82
Shawl sign, 65, 65f cutaneous involvement, 78, 80, 80f
Shrinking lung syndrome. 45 gastrointestinal involvement, 81
Sildenafil, for pulmonary arterial hypertension in systemic sclerosis, 82 kidney involvement, 81

168
Index

musculoskeletal involvement, 80 u
pulmonary involvement, 81-82 Ulceration, aral/nasopharyngeal, in systemic lupus erythematosus,44
vascular involvement, 80-81, 8lf Ultrasonography, in rheumatologic diseases,7, 9
common manifestations/features of, 78,791 Undifferentiated connective tissue disease (UCTD), 48,481
dermatologic manifestations of, 4t Urate-lowering therapy (ULT), 17-18
diagnosis of, 78 allopurinol, 17
diffuse cutaneous, 78 febuxostat,17
epidemiology of, 78 in gout,56-57
limited cutaneous, 78 pegloticase, 17
pathophysiology of,78 in pregnancy,19t
pregnancy and,82 probenecid, 17
Systemic sclerosis sine scleroderma,79t Uric acid,53, 53f. See also Gout
Ustekinumab, 15t,16
T in pregnancy, l9t
Takayasu arteritis (TA), 71-72,Q77 in psoriatic arthritis, 42
clinical manifestations of, 71 Uveitis,4t
diagnosis of,71, 7lf Uveoparotid fever, 86
epidemiology and pathophysiology of, 71
Thiazide diuretics, and drug-induced lupus erythematosus,481 V
Thrombocytopenia, in systemic lupus erythematosus,46 Vaccination, prior to immunosuppressive therapy, 16
Thromboxane A,,10 Vardenafil, for pulmonary arterial hypertension in systemic
Tidal lavage, for basic calcium phosphate deposition,59 sclerosis, 82
Tocilizumab,14, 15t, 16 Vasculitis, 69-77
in giant cell arteritis, 71 dermatologic manifestations of, 4t
in pregnancy,19t differential diagnosis of, 701
in relapsing polychondritis, 84 large-vessel, 69-72
in rheumatoid arthritis, 27 giant cell arteritis, 70-71
Tofacitinib, 15t, 16 polymyalgia rheumatica, 71
and abnormal lipid profile,Ql5 Takayasu arteritis, 71-72,7lf
in pregnancy,l9t medium-vessel, 72-73
in rheumatoid arthritis, 27 Kawasaki disease, 73
Tophi, 54-55. See also Gout polyarteritis nodosa, 72
Total joint arthroplasty, 18 primary angiitis of central nervous system, 72
for knee and hip osteoarthritis, 33 secondary, 69,69t
Tramadol, 11 small-vessel, 73-77
in osteoarthritis, 32 ANCA-associated vasculitis,73-75,73f
in pregnancy, l9t cryoglobulinemic vasculitis,75-76
Transferrin saturation, measurement of,Qll eosinophilic granulomatosis with polyangiitis, 75
Transvaginal pelvic ultrasonography,in dermatomyositis,Q62 granulomatosis with polyangiitis, 74
Treprostinil, for pulmonary arterial hypertension in systemic sclerosis, 82 Henoch-Schonlein purpura, 77
Trigeminal neuralgia,in mixed connective tissue disease,52 hypersensitivity vasculitis, 77
Trimethoprim-sulfamethoxazole,discontinuation of,in hypersensitivity immune complex-mediated vasculitis, 75-77, 76f
vasculitis, Q40 microscopic polyangiitis, 74-75
Tuberculin skin tests,61 Viral infections, infectious arthritis by,61-62. See also Infectious arthritis
Tuberculosis, screening for, before biologic therapy, Ql8 Chikungunya, 62
Tumor necrosis factor (TNF)-a inhibitors, 13-14 hepatitis,61-62
in ankylosing spondylitis, 41 Parvovirus Bl9, 62
in Behc;et syndrome, 83 rubella, 62
and drug-induced lupus erythematosus, 48t
in inflammatory bowel disease-associated arthritis,42 w
in pregnancy, 19t Widespread Pain Index (WP[), fibromyalgia and,34
in psoriatic arthritis,42
in rheumatoid arthritis, 27 X
Tumor necrosis factor receptor-associated periodic syndrome (TRAPS), 85t Xerostomia,51. See also Sjogren syndrome

169

You might also like